BRS/Gray's Block 3 Review

Lakukan tugas rumah & ujian kamu dengan baik sekarang menggunakan Quizwiz!

19. A 22-year-old man has a gonorrheal infection that has infiltrated the space between the inferior fascia of the urogenital diaphragm and the superficial perineal fascia. Which of the following structures might be inflamed? (A) Bulb of the penis (B) Bulbourethral gland (C) Membranous part of the male urethra (D) Deep transverse perineal muscle (E) Sphincter urethrae

19. The answer is A. The bulb of the penis is located in the superficial perineal space between the inferior fascia of the urogenital diaphragm and the membranous layer of the superficial perineal fascia (Colles fascia). All of the other structures are found in the deep perineal pouch.

A 34-year-old man is lifting heavy weights while doing squats. Unfortunately, while making a maximal effort, he drops the weight and immediately grabs at his upper thigh, writhing in pain. The man is admitted to the emergency department and during physical examination is diagnosed with a femoral hernia. What reference structure would be found immediately lateral to the herniated structures? A. Femoral vein B. Femoral artery C. Pectineus muscle D. Femoral nerve E. Adductor longus muscle

A. In a femoral hernia, abdominal contents are forced through the femoral ring, which is just lateral to the lacunar ligament (of Gimbernat) and just medial to the femoral vein. The femoral vein would be found immediately lateral to the femoral hernia. This is correct in most cases because in the majority of people, the femoral vein is found more medial to both the femoral artery and nerve in the femoral triangle. The adductor longus muscle as well as the pectineus muscle would be found deep and medial to the hernia.

A 15-year-old is admitted to the emergency department 2 days after crashing his bicycle. MRI examination reveals severe edema of the boy's scrotum and abdominal wall and extravasated urine. Which of the following structures is most likely ruptured? A. Spongy urethra B. Preprostatic urethra C. Prostatic urethra D. Urinary bladder E. Ureter

A. Rupture of the spongy urethra leads to accu- mulation of fluid (edema) in the superficial perineal cleft. The continuity of Colles' fascia (superficial mem- branous layer of the superficial perineal fascia) with Scarpa's fascia of the abdominal wall allows for fluid spread upward upon the body wall. Rupture of the preprostatic urethra, prostatic urethra, or urinary blad- der would lead to internal fluid accumulation within the pelvis because they are not located in the perineum. Damage to the ureter would manifest within the abdo- men or pelvis, depending upon the level of rupture.

Into which structure does the azygos vein drain venous blood?

A. SVC

Lies on the right side of the aortic arch and ascending aorta?

A. SVC

A 22-year-old male martial arts competitor was examined by the clinician because of pain and serious disability suffered from a kick to the side of his knee. Physical examination revealed a dark bruise just distal to the head of the fibula. Which of the following muscles will most likely be paralyzed? A. Tibialis anterior and extensor digitorum longus B. Tibialis posterior C. Soleus and gastrocnemius D. Plantaris and popliteus E. Flexor digitorum longus and flexor hallucis longus

A. The common fibular (peroneal) nerve passes around the head of the fibula and gives off deep (L4-5) and superficial fibular (peroneal) nerve (L5, S1-2) branches. The two nerves supply the dorsiflexors and evertors of the foot, respectively. In this case, the tibialis anterior and extensor digitorum longus are the only muscles listed that are supplied by either of these nerve branches, and both are innervated by the deep fibular (peroneal) nerve. The fibularis (peroneus) brevis and longus are innervated by the superficial fibular (peroneal) nerve and are evertors of the foot. The tibial nerve supplies each of the other muscles listed.

The quadriceps femoris muscle group is formed by the rectus femoris, vastus lateralis, vastus medialis, and vastus intermedius. Which of the following nerves innervates this group of muscles? A. Sciatic nerve B. Femoral nerve C. Obturator nerve D. Saphenous nerve E. Tibial nerve

A: Only the femoral nerve innervates the quadriceps femoris. The sciatic nerve, specifically the tibial part, supplies the posterior thigh muscles not the anterior compartment. The obturator nerve supplies the medial compartment and the saphenous nerve does not supply any thigh muscles.

10. During a domestic dispute, a 16-year-old boy receives a deep stab wound around the superior angle of the scapula near the medial border, which injures both the dorsal scapular and spinal accessory nerves. Such an injury could result in paralysis or weakness of which of the following muscles? (A) Trapezius and serratus posterior superior (B) Rhomboid major and trapezius (C) Rhomboid minor and latissimus dorsi (D) Splenius cervicis and sternocleidomastoid (E) Levator scapulae and erector spinae

B

Can be removed in a surgical resection of a lobe to remove lung cancer on the diaphragmatic surface?

B. Right inferior lobar bronchus

C right ventricle

Becomes hypertrophied as result of the pulmonary stenosis?

A 34-year-old woman is admitted to the hospital due to severe lower abdominal pain. Radiographic ex- amination reveals tumors in both of her ovaries. A bi- opsy is ordered and confirms the initial diagnosis of ovarian cancer. Which of the following lymph nodes are the first to receive lymph from the diseased ovaries? A. Superficial and deep inguinal lymph nodes B. External iliac nodes C. Paraartotic nodes at the level of the renal vessels D. Node of Cloquet E. Internal iliac nodes accompanying the uterine artery and vein

C. Ovarian lymph first drains into the paraaortic nodes at the level of the renal vessels. The superficial and deep inguinal nodes drain the body wall below the umbilicus, the lower limbs, and the cutaneous portion of the anal canal and parts of the perineum. The exter- nal iliac nodes receive the lymph from the inguinal nodes. The node of Cloquet is located in the femoral ring, adjacent to the external iliac vein and beneath the inguinal ligament. The node of Cloquet drains into the common iliac nodes. The internal iliac nodes accom- pany the uterine artery and vein, receiving lymph from much of the uterus but not the ovaries.

Becomes hypertrophied as result of the pulmonary stenosis?

C. Right ventricle

A 32-year-old man is admitted to the emergency department after an injury to his foot while playing football with his college friends. An MRI examination reveals multiple tendinous tears (Fig. 5-8). Which of the following bones is associated with the muscle tears? A. Navicular B. Cuboid C. Calcaneus D. Sustentaculum tali E. Talus

C. The bone to which the injured ligament attaches is the calcaneus. The navicular bone, located medially in the foot, articulates posteriorly with the head of the talus and anteriorly with the cuneiform bones. The cuboid bone of the lateral longitudinal arch articulates posteriorly with the calcaneus. The talus articulates with the tibia and fibula in the ankle joint mortise.

How much DNA does a primary spermatocyte contain? (A) 1N (B) 2N (C) 4N (D) 6N (E) 8N

C. Type B spermatogonia give rise to primary spermatocytes by undergoing DNA replication, thereby doubling the amount of DNA (2 x 2N = 4N) within the cell.

B right inferior lobar bronchus

Can be removed in a surgical resection of a lobe to remove lung cancer on the diaphragmatic surface?

C right ventricle

Contains the septomarginal trabecula?

1. During an outbreak of meningitis at a local college, a 20-year-old student presents to a hospital emergency department complaining of headache, fever, chills, and stiff neck. On examination, it appears that he may have meningitis and needs a lumbar puncture or a spinal tap. Cerebrospinal fluid (CSF) is normally withdrawn from which of the following spaces? (A) Epidural space (B) Subdural space (C) Space between the spinal cord and the pia mater (D) Subarachnoid space (E) Space between the arachnoid and dura maters

D

Which structure is crossed superiorly by the aortic arch and left pulmonary artery?

D. Left primary bronchus

A 10-year-old girl is admitted to the emergency department after falling from a tree in which she was playing with her friends. Radiologic and physical examinations reveal Osgood-Schlatter disease (Fig. 5-4). Which of the following bony structures is chiefly affected? A. Medial condyle of tibia B. Posterior intercondylar area C. Intercondylar eminence D. Tibial tuberosity E. Anterolateral tibial tubercle (Gerdy's tubercle)

D. Osgood-Schlatter disease is also called tibial tuberosity apophysitis and affects the area of the tibial tuberosity. It is not a disease but a problem of overuse, typically in boys of 12 to 14 years or girls 10 to 12 years of age. Very active boys and girls, usually during a growth spurt, are subject to the pain and swelling that occur at the site of attachment of the patellar ligament. The ligament can tear, resulting in a long period of healing following treatment. The medial femoral condyle is the area of attachment of the medial collateral ligament and medial meniscus of the knee joint. The posterior intercondylar eminence is the location of origin of the posterior cruciate ligament. The intercondylar eminence is a bony protuberance on the tibial plateau to which the cruciate ligaments and menisci are attached. The anterolateral tibial tubercle, or Gerdy's tubercle, is the attachment of the iliotibial band or tract; thus it connects the femur and tibia laterally.

A 34-year-old male runner visits the outpatient clinic complaining of pain in his foot for the past week. Physical examination reveals inflammation of the tough band of tissue stretching from the calcaneus to the ball of the foot. Which of the following conditions is characteristic of these symptoms? A. Pott's fracture B. Dupuytren fracture C. Tarsal tunnel D. Plantar fasciitis E. Rupture of spring ligament

D. Plantar fasciitis is a common clinical condition that results from tearing or inflammation of the tough band of tissue stretching from the calcaneus to the ball of the foot (the plantar aponeurosis). It usually happens to people who are on their feet frequently or engaged in athletics, especially running and jumping. Plantar fasciitis is usually most painful in the morning, just after getting up from bed and beginning to walk. Rest, orthotics, night splints, and antiinflammatory medications are employed in treatment. A Pott's fracture is a bimalleolar fracture, specifically a fracture of the distal end of the fibula (lateral malleolus) and medial malleolus, with outward displacement of the foot. Dupuytren's fracture involves fracture of the distal fibula with dislocation of the foot. Each of these fractures occurs due to sudden and forceful eversion of the foot.

A 61-year-old female immigrant had been diagnosed with spinal tuberculosis. The woman had developed a fluctuant, red, tender bulge on one flank, with a similar bulge in the groin on the same side. This presentation is likely due to spread of disease process within the fascia of a muscle with which of the following actions at the hip? A. Abduction B. Adduction C. Extension D. Flexion E. Internal rotation

D. Spinal tuberculosis can spread within the sheath of the psoas major to its insertion with the iliacus upon the lesser trochanter, presenting there also with painful symptoms. The iliopsoas muscle is the principal flexor of the hip joint. Abduction of the hips is performed by the gluteus medius and minimus with assistance from short lateral rotator muscles. Extension of the hip is a function of the gluteus maximus, together with the hamstring muscles. Internal rotation is performed by the adductor muscle group.

Which of the following events is involved in cleavage of the zygote during week 1 of development? (A) A series of meiotic divisions forming blastomeres (B) Production of highly differentiated blastomeres (C) An increased cytoplasmic content of blastomeres (D) An increase in size of blastomeres (E) A decrease in size of blastomeres

E. Cleavage is a series of mitotic divisions whereby the large amount of zygote cytoplasm is successively partitioned among the newly formed blastomeres. Although the number of blastomeres increases during cleavage, the size of individual blastomeres decreases until they resemble adult cells in size.

Which structure branches into the bronchial arteries?

E. Descending (thoracic) aorta

72. The patient is unable to adduct her little finger because of paralysis of this structure.

G

A SVC

Into which structure does the azygos vein drain venous blood?

A SVC

Lies on the right side of the aortic arch and ascending aorta?

E left atrium

Receives oxygenated blood via pulmonary veins?

C ascending aorta

The left coronary artery arises from which structure?

E descending (thoracic) aorta

Which structure branches into the bronchial arteries?

D left primary bronchus

Which structure is crossed superiorly by the aortic arch and left pulmonary artery?

B right superior secondary (eparterial) bronchus

Which structure is most likely to be removed by a pulmonary surgeon in a surgical resection of a lobe (lobectomy) to remove lung cancer in the apex of the right lung?

1 A 4-month-old male infant is admitted to the pediatric clinic because he was passing urine near the anus rather than from the tip of his penis. Physical examination reveals that the patient has perineal hypospadias. Which of the following embryologic structures failed to fuse properly? A. Labioscrotal folds B. Cloacal membrane C. Urogenital folds D. Genital tubercle E. Urogenital membrane

1 A. Perineal hypospadias is due to a failure of fusion of labioscrotal folds so that the external urethral orifice is between the unfused halves of the scrotum. The cloacal membrane is formed from endoderm of the cloaca and ectoderm of the proctodeum and forms the future anus. The urogenital folds normally fuse along the ventral side of the penis to form the spongy urethra. Epispadias is a condition in which the urethra opens on the dorsal surface of the penis resulting from the genital tubercle developing more dorsally during development. The urogenital membrane is bounded by the urogenital folds and ruptures to form the urogenital orifice. GAS 466-468, 512; N 366; McM 265

1. A 68-year-old woman with uterine carcinoma undergoes surgical resection. This cancer can spread directly to the labia majora in lymphatics that follow which of the following structures? (A) Pubic arcuate ligament (B) Suspensory ligament of the ovary (C) Cardinal (transverse cervical) ligament (D) Suspensory ligament of the clitoris (E) Round ligament of the uterus

1. The answer is E. The round ligament of the uterus runs laterally from the uterus through the deep inguinal ring, inguinal canal, and superficial inguinal ring and becomes lost in the subcutaneous tissues of the labium majus. Thus, carcinoma of the uterus can spread directly to the labium majus by traveling in lymphatics that follow the ligament.

Due to torsioning of the lower limb and merging of separate premuscle masses during development, certain adult lower limb muscles are supplied by two separate nerves. These interneural fusions are termed "hybrid muscles". Which of the following is a hybrid muscle? (A) Adductor magnus (B) Gastrocnemius (C) Rectus femoris (D) Semimembranosus (E) Tibialis anterior

(A) Adductor magnus

A 15-year-old girl is struck by a car while crossing the street. She suffers numerous pelvic injuries, including tearing of the sacrotuberous ligament. The damage to this ligament will most likely cause direct trauma to which of the following muscles? (A) Gluteus maximus (B) Gluteus medius (C) Gluteus minimus (D) Gemelli (E) Obturator externus

(A) Gluteus maximus

Two 11-year-old boys sneak up on their friend from behind. In surprising their friend, they shove him suddenly, which forcefully pushes his hips forward while he is standing relaxed talking to other friends. Which of the following ligaments best resists anterior dislocation of the head of the femur? (A) Iliofemoral ligament (B) Pubofemoral ligament (C) Ischiofemoral ligament (D) Lacunar ligament (E) Ligament of the head of the femur

(A) Iliofemoral ligament

A 16-year-old boy was fishing barefoot in a muddy river when the plantar surface of his foot was cut by unseen debris. He suffers a large transverse cut, penetrating the first two layers of his plantar musculature, in the area of the first cuneiform bone. In the emergency room, his physician notes a complete inability to flex and abduct the big toe and numbness on the plantar aspect of the three medial toes. Which of the following nerves is most likely damaged? (A) Medial plantar nerve (B) Lateral plantar nerve (C) Sural nerve (D) Deep fi bular nerve (E) Superfi cial fi bular nerve

(A) Medial plantar nerve

A 21-year-old female basketball player lands on her opponent's foot after jumping to rebound the basketball. Her foot is forcefully inverted, and when leaving the court, she tells her trainer that she twisted or sprained her ankle. After getting her ankle taped for support, she reenters the game. What ligament was most likely damaged? (A) Calcaneofibular ligament (B) Anterior talofibular ligament (C) Posterior talofibular ligament (D) Plantar calcaneonavicular ligament (E) Medial (deltoid) ligament

(B) Anterior talofibular ligament

Following surgery to repair a broken right tibia, a 22-year-old patient is placed in a short leg cast. Several hours later, she complains of extreme pain, numbness with a "pins and needles sensation," infl ammation, and abnormal pressure on the anterior and lateral aspects of the affected lower leg. The cast is removed, and the physician notes weakness in dorsifl exion of the foot and toes, a weak dorsalis pedis arterial pulse, and sensory loss between the first and second toes. What nerve is most likely damaged? (A) Tibial nerve (B) Deep fibular nerve (C) Superficial fi bular nerve (D) Medial plantar nerve (E) Lateral plantar nerve

(B) Deep fibular nerve

A patient presents with extreme pain due to arterial insufficiency in the posterior femoral compartment. This compartment of the thigh receives its blood supply mainly from the perforating arteries. An arteriogram confi rms partial occlusion of the artery that gives rise to these perforating arteries. What artery is occluded in the arteriogram? (A) Femoral artery (B) Profunda femoris artery (C) Obturator artery (D) Popliteal artery (E) Medial femoral circumfl ex artery

(B) Profunda femoris artery

A 22-year-old soldier is injured from shrapnel from an improvised explosive device in the right upper thigh, below the midpoint of the inguinal ligament. Though he received fi eld dressings from a medic, he arrives at the military hospital having lost copious amounts of blood. What sign and/or symptom would accompany this patient's presentation? (A) Increased pulse in the right dorsalis pedis artery (B) Tachycardia (C) Warm right foot (D) Increased hematocrit (E) Hypertension

(B) Tachycardia

A roofi ng installer falls off a high ladder and lands with the sole of his right foot hitting the ground first. He suffers a fracture and inferior displacement of the sustentaculum tali of the calcaneus. Which of the following structures is most likely torn? (A) Tendon of the tibialis posterior muscle (B) Tendon of the fl exor hallucis longus (FHL) muscle (C) Tendons of the fl exor digitorum brevis muscle (D) Small saphenous vein (E) Plantar arterial arch

(B) Tendon of the flexor hallucis longus (FHL) muscle

The relative positions of blood vessels can sometimes be explained in terms of rotation of the limbs during development. For example, upon entering the femoral triangle the femoral artery resides ____ to the femoral vein, whereas in the popliteal fossa the popliteal artery lies ____ to the popliteal vein. (A) medial...posterior (B) lateral...deep (C) lateral...posterior (D) medial...superfi cial (E) anterior...lateral

(B) lateral...deep

A 65-year-old man with a history of heavy smoking visits his physician complaining of intermittent pain in his feet, accompanied by pallor and coldness of his feet. The physician suspects vascular insuffi ciency and takes a pulse of the dorsalis pedis artery. That pulse is best palpated at which of the following locations? (A) Immediately anterior to the medial malleolus (B) Immediately anterior to the lateral malleolus (C) Between the tendons of the extensor hallucis longus and extensor digitorum longus muscles (D) Between the tendons of the extensor digitorum longus and fibularis (peroneus) tertius muscles (E) Immediately lateral to the tendon of the fibularis (peroneus) tertius muscle

(C) Between the tendons of the extensor hallucis longus and extensor digitorum longus muscles

A 25-year-old woman is brought to the emergency room in severe pain due to an ankle injury. She tells the attending physician she was wearing high-heel shoes, stepped off the curb into the street, lost her balance and landed awkwardly, causing her foot to turn extremely outward (eversion). Which of the following ligaments is most likely damaged? (A) Plantar calcaneonavicular (spring) ligament (B) Calcaneofi bular ligament (C) Deltoid ligament (D) Anterior talofi bular ligament (E) Plantar calcaneocuboid (long plantar) ligament

(C) Deltoid ligament

After spending 2 days moving heavy furniture out of her house, a 56-year-old woman goes to an emergency room in acute pain. The patient reports nausea, vomiting, and severe abdominal pain. On examination, the doctor discovers a painful, globular mass located inferior and lateral to the pubic tubercle. Given her presentation and history, what is the most likely diagnosis? (A) Indirect inguinal hernia (B) Direct inguinal hernia (C) Femoral hernia (D) Lymphadenitis of superfi cial inguinal nodes (E) Fractured hip

(C) Femoral hernia

A 72-year-old woman slips and falls on a wet floor, fracturing the neck of her right femur. Subsequent physical examination in the ER shows her right foot is laterally rotated, and the right lower extremity appears slightly shorter than the left. Which of the following muscles is mainly responsible for the rotated posture of the right limb? (A) Semitendinosus (B) Adductor longus (C) Gluteus maximus (D) Rectus femoris (E) Gracilis

(C) Gluteus maximus

A 23-year-old female medical student notices she was gaining weight due to a sedimentary lifestyle and compulsive studying. Therefore, she decides to run in a marathon and starts her training by running 6 miles each morning before class. After 2 weeks, she presents with right lateral knee pain and infl ammation, specifi cally in the area of the lateral femoral epicondyle. This pain intensifi es throughout her morning jogs, especially when her right foot strikes the ground or when she is running downhill. What is most likely diagnosis in this patient? (A) Sprained fibular (lateral) collateral ligament (FCL) (B) Patellofemoral pain syndrome (C) Iliotibial band syndrome (ITBS) (D) Torn lateral meniscus (E) Pes anserinus bursitis

(C) Iliotibial band syndrome (ITBS)

When standing upright, the femur moves into the locked position by slightly hyperextending, gliding posteriorly, and medially rotating on the tibial plateaus. Which of the following muscles acts to initiate the unlocking of the knee? (A) Biceps femoris (B) Gastrocnemius (C) Popliteus (D) Sartorius (E) Plantaris

(C) Popliteus

A man working in a junkyard trips and falls into a pile of scrap metal, suffering a deep cut immediately posterior to the lateral malleolus. Which of the following is most likely to be injured? (A) Saphenous nerve (B) Tendon of the fibularis (peroneus) tertius (C) Tendon of the fibularis (peroneus) longus (D) Posterior tibial artery (E) Great saphenous vein

(C) Tendon of the fibularis (peroneus) longus

An overweight woman participates in her fi rst rugby match without proper training and conditioning. Upon catching the opening kickoff, she awkwardly twists her right knee, screams in pain, and falls to the ground. The team manager notes her patella is dislocated, residing on the lateral side of her knee. After straightening the woman's knee, the patellar dislocation is reduced (goes back into place). To prevent future dislocation of the patella, what specific muscle should be targeted during rehabilitation? (A) Vastus intermedius (B) Vastus lateralis (C) Vastus medialis (D) Rectus femoris (E) Tibialis anterior

(C) Vastus medialis

A 6-year-old boy playing barefooted in his backyard steps on a piece of broken glass and suffers a large transverse cut on his sole, at the level of the midfoot. In the emergency room, the examining physician determines the cut is to the depth of the first layer of the plantar muscles. Which of the following structures is most likely damaged in this injury? (A) Plantar arterial arch (B) Tendons of the flexor digitorum longus muscle (C) Tendon of the fibularis (peroneus) longus muscle (D) Abductor hallucis muscle (E) Superficial fibular (peroneal) nerve

(D) Abductor hallucis muscle

Human feet are everted so that their soles lie fully on the ground during ambulation. What muscle is developmentally unique to humans, inserts into the base of the fi fth metatarsal, and assists in eversion (or pronation)? (A) Extensor digitorum longus (B) Extensor digitorum brevis (C) Fibularis longus (D) Fibularis tertius (E) Flexor digiti minimi brevis

(D) Fibularis tertius

A rising second-year medical student spends his summer hiking the Appalachian Trial with a 100-lb backpack in tow. After a month of hiking 30-mi/day (48-km/day), he notices numbness, tingling, and burning sensations in the lateral aspect of his right upper thigh. His symptoms are exacerbated by his backpack pressing on the area surrounding his right anterior superior iliac spine. What nerve is most likely affected? (A) Femoral branch of genitofemoral nerve (B) Saphenous nerve (C) Anterior cutaneous branches of femoral nerve (D) Lateral femoral cutaneous nerve (E) Iliohypogastric nerve

(D) Lateral femoral cutaneous nerve

A 36-year-old man arrives at the ER unconscious and with a suspected spinal cord injury after being involved in a motor vehicle accident. If the physician wants to localize the spinal cord lesion, what reflex would test the integrity of the L1-2 spinal cord segment? (A) Anocutaneous reflex (B) Patellar reflex (C) Plantar reflex (D) Calcaneal reflex (E) Cremasteric reflex

(E) Cremasteric reflex

A 25-year-old man suffers a gunshot wound to the calf that severs the posterior tibial artery at its origin. Which of the following vessels will not receive blood flow immediately following the injury? (A) Anterior tibial artery (B) Inferior medial genicular artery (C) Dorsalis pedis artery (D) Popliteal artery (E) Fibular (peroneal) artery

(E) Fibular (peroneal) artery

A 55-year-old woman recently had pelvic surgery during which cancerous lymph nodes were removed from the lateral wall of her pelvis. During a postoperative examination, she says she has been having painful muscle spasms in her thigh. Which of the following muscles is most likely involved? (A) Sartorius (B) Biceps femoris (C) Tensor muscle of fascia lata (D) Vastus medialis (E) Gracilis

(E) Gracilis

A 70-year-old man reports an inability to climb stairs and stand up from a sitting position. Further examination also shows weakness when laterally rotating the thigh against resistance. What nerve is most likely compromised in this patient? (A) Femoral nerve (B) Obturator nerve (C) Sciatic nerve (D) Superior gluteal nerve (E) Inferior gluteal nerve

(E) Inferior gluteal nerve

As part of a physical examination to evaluate lower limb function, a physician asks a patient to abduct her second through fifth toes. What specific nerve is the doctor testing? (A) Sural nerve (B) Deep fibular nerve (C) Superficial fibular nerve (D) Medial plantar nerve (E) Lateral plantar nerve

(E) Lateral plantar nerve

A 15-year-old girl, unaware that she is pregnant, borrows her friend's Accutane (retinoic acid; vitamin A) to combat an acne problem. She uses the Accutane for about 2 months, which corresponds to weeks 7 to 15 of the embryonic development of her fetus. Which of the following skeletal elements is most likely to be absent in the newborn infant? (A) Ilium (B) Femur (C) Patella (D) Tibia (E) Phalanges

(E) Phalanges

A veteran infantry soldier develops painful fl at feet after several years of service including hundreds of miles of marches. The pain is particularly acute on the medial aspect of his sole. Which of the following structures is most likely strained in this condition? (A) Calcaneal (Achilles) tendon (B) Plantar calcaneocuboid (long plantar) ligament (C) Extensor retinaculum (D) Tendon of the fi bularis (peroneus) longus muscle (E) Plantar calcaneonavicular (spring) ligament

(E) Plantar calcaneonavicular (spring) ligament

A 35-year-old male prisoner received a right gluteal intramuscular (IM) injection during a visit to the infi rmary. Following the injection, the man experienced a painful, swollen right leg. Within a month, he complained that his right leg started to shrink. Examination revealed muscle wasting with fasciculations in the L4-S1 distribution and marked weakness in dorsiflexion, inversion, and eversion at the ankle joint. He also exhibited a typical high-steppage gait indicating right foot drop. What nerve was most likely damaged during the gluteal IM injection? (A) Superior gluteal nerve (B) Common fi bular nerve (C) Tibial nerve (D) Inferior gluteal nerve (E) Sciatic nerve

(E) Sciatic nerve

A 32-year-old male farmer cuts the medial aspect of his midthigh when climbing over a barbwire fence. Though he bandages the wound, he reports to the ER 5 days later with an infected wound, high fever (102.7°), and lymphadenitis (swollen lymph nodes). Given the location of the injury, which groups of nodes would be the first to receive drainage from the infected wound? (A) Popliteal (B) External iliac (C) Deep inguinal (D) Horizontal group of superfi cial inguinal (E) Vertical group of superficial inguinal

(E) Vertical group of superficial inguinal

10 A 1-year-old male infant is admitted to the pediatric clinic because he is passing urine on the underside of the penis as shown in Fig. 4-1. Which embryologic structure failed to fuse? A. Spongy urethra B. Labioscrotal folds C. Urethral folds D. Urogenital folds E. Genital tubercle

10 C. Hypospadias is a developmental defect in the urethra resulting in urine being expelled from the ventral side of the penis as seen in this patient. This ectopic malformation may present when the urethral folds fail to completely fuse. Failure of fusion of the spongy urethra would result in epispadias. A failure of the labioscrotal folds to fuse will cause the external urethral orifice to be situated between the two scrotal halves. This is referred to as penile hypospadias. Failure of the urogenital folds to fuse would lead to agenesis of the external urethral folds. GAS 466-468, 512; N 366; McM 265

10. A 46-year-old woman has a history of infection in her perineal region. A comprehensive examination reveals a tear of the superior boundary of the superficial perineal space. Which of the following structures would most likely be injured? (A) Pelvic diaphragm (B) Colles fascia (C) Superficial perineal fascia (D) Deep perineal fascia (E) Perineal membrane

10. The answer is E. The superior (deep) boundary of the superficial perineal space is the perineal membrane (inferior fascia of the urogenital diaphragm). Colles fascia is the deep membranous layer of the superficial perineal fascia. The deep perineal fascia essentially divides the superficial perineal space into a superficial and deep compartment. The pelvic diaphragm consists of the levator ani and coccygeus muscles.

100 A 4-month-old boy is admitted to the clinic because urine is leaking through an opening on the dorsal surface of the penis. What is the name of this condition? A. Epispadias B. Hermaphrodism C. Hydrocele D. Hypospadias E. Ectopic ureter

100 A. Epispadias is the correct option. Epispadias describes the abnormal opening of the urethra on the dorsal side of the penis. During the development of the genital tubercle defective or inadequate interaction between the ectoderm and mesoderm leads to the genital tubercle developing more dorsally. As a consequence, when the more dorsally located urogenital membrane ruptures, the urogenital sinus will open on the dorsal aspect of the penis. Exstrophy of the urinary bladder is a condition in which there is a defect in the anterior abdominal wall with the bladder and part of the urethra are protruding. GAS 466-468, 512; N 359; McM 365

101 A 6-year-old girl is admitted to the hospital with fever, malaise, and painful voiding. She is treated with antibiotics for a urinary tract infection but the symptoms persist. An abdominal CT scan reveals a suprapubic cystic mass attached to the umbilicus and the apex of the bladder. What is the most likely diagnosis? A. Hydrocele B. Meckel's cyst C. Meckel's diverticulum D. Omphalocele E. Urachal cyst

101 E. Urachal cysts are embryological remnants of the epithelium of the urachus occurring between the umbilicus and bladder. It presents as an extraperitoneal mass in the umbilical region. Urachal cysts are generally asymptomatic; however, if infected, they present with abdominal pain and fever. If fistulation occurs, it may drain through the umbilicus or rupture, leading to peritonitis. Hydroceles are the accumulation of fluids between the two layers of the tunica vaginalis surrounding the testicles. Meckel's cyst and Meckel's diverticulum are formed on the distal ileum and do not have the anatomical relations described in the CT findings. Omphaloceles result from defective development of the muscles of the anterior abdominal wall where abdominal viscera herniate into the umbilical cord where they are outside of the abdomen. GAS 463; N 341; McM 223, 272

102 A 72-year-old man undergoes radical prostatectomy for a cancerous prostatic mass. Following the operation, he is unable to achieve erection. The nerve fibers that were injured during the procedure are carried by which nerve(s)? A. Superior hypogastric B. Pelvic splanchnics C. Sacral splanchnics D. Lumbar splanchnics E. Pudendal

102 B. The sacral components of the craniosacral outflow (parasympathetics) are the pelvic splanchnic nerves. These nerves take their origin from the ventral rami of the S2 to S4 spinal segments and enter the sacral plexus. They carry both preganglionic parasympathetic fibers as well as visceral afferent fibers. The pelvic splanchnics provide innervation to pelvic viscera and genitals regulating the voiding of the bladder, controlling the internal urethral sphincter, rectal motility, as well as sexual functions such as erection. The pelvic splanchnic should not be confused with the lumbar splanchnics or sacral splanchnics which carry preganglionic sympathetics and general visceral afferents. GAS 492-494, 515; N 394; McM 261, 269

103 A 60-year-old woman with a known history of colon cancer presents to the clinic with abdominal pain. A CT scan reveals spread of her cancer to multiple areas, with subsequent fluid accumulation in her peritoneal cavity. In order to collect this fluid in a minimally invasive manner to test for cancer markers a needle is passed through the posterior vaginal fornix into the peritoneal cavity while the patient is seated upright. From which space is the fluid drained? A. Rectouterine pouch B. Pararectal fossa C. Paravesical space D. Uterovesical pouch E. Superficial perineal pouch

103 A. The rectouterine pouch (or pouch of Douglas) is the extension of the peritoneal cavity situated between the rectum and the posterior uterine wall of the uterus in women. It is the most dependent or lowest part of the abdominopelvic cavity when in the supine position making it a typical site of collection of fluids. Due to its intimacy with the posterior wall of the uterus it can be examined transvaginally or transrectally by digital palpation. Fluids collected there may also be drained through the vagina or rectum without entering the abdominopelvic cavity transabdominally. The pararectal fossa is a feature found only in men. The paravesical space is made up of paired subdivisions of the extraperitoneal space found lateral to the prevesical space. The uterovesical pouch is formed by the peritoneum over the uterus and bladder, continued over the intestinal surface and fundus of the uterus onto its vesical surface. Its more anterior location dictates that when the body is in a supine position, fluid that accumulates there will immediately travel down to deeper parts of the peritoneal cavity (the rectouterine pouch). The superficial perineal pouch can also be ruled out as an option because it is located in the perineum where it is completely enclosed by the deep perineal fascia and the perineal membranes that make up its borders (GAS Fig. 5-58). GAS 483; N 340; McM 272

104 A 36-year-old man is admitted to the hospital after a car crash. An MRI scan reveals rupture of the penile urethra and deep (Buck's) fascia. Where is the most likely place to which extravasated urine will flow? A. Ischioanal fossa B. Rectovesical pouch C. Deep perineal pouch D. Retropubic space E. Superficial perineal pouch

104 E. The inferior border of the superficial perineal pouch is the deep fascia (Gallaudet's fascia or Buck's fascia). Rupture of the penile urethra leads to extravasation and collection of urine in the superficial perineal pouch. The retrovesical pouch is a peritoneal cavity extension found between the posterior wall of the uterus and the bladder. The deep perineal pouch is the region between the perineal membrane and pelvic diaphragm. The deep perineal pouch is bordered inferiorly by the perineal membrane. It is bordered superiorly by the superior fascia of the urogenital diaphragm. It shares no communication with the deep fascia. The retropubic space, also known as the cave or space of Retzius, is an extraperitoneal space between the pubic symphysis and urinary bladder that bears no relation to the penile urethra. GAS 506; N 374; McM 365

105 A 66-year-old woman with multiple sclerosis suffers from neurologic urinary incontinence. Involvement of which nerve causes dysfunction of her external urethral sphincter, leading to this incontinence? A. Pelvic splanchnic B. Sacral splanchnic C. Pudendal D. Superior gluteal E. Inferior gluteal

105 C. The pudendal nerve branches innervate the external urethral sphincter. The pudendal nerve takes its origin from the S2 to S4 spinal segments via the sacral plexus. Its motor branches supply the external urethral sphincter, external anal sphincter, and skeletal muscle of the deep and superficial pouches. Its cutaneous branches supply the skin of the anal triangle and external genitalia. Pelvic and sacral splanchnics carry autonomic fibers, as the external urethral sphincter is a somatic structure. The superior gluteal nerve arises from the dorsal divisions of the fourth and fifth lumbar and the first sacral nerves. Its fibers go on to innervate the gluteus medius, the gluteus minimus, and the tensor fasciae latae, which are unrelated to urinary incontinence. Likewise the inferior gluteal nerve innervates the gluteus maximus muscle. GAS 486-494; N 389; McM 277

106 A 25-year-old man was brought to the emergency department after being thrown from his horse. Radiological studies revealed separation of the pubic symphysis with injury to the pubovesical and puboprostatic ligaments. The urologist also suspects tearing of the cavernous nerves surrounding the prostate. The cavernous nerves provide efferent fibers to which of the following structures? A. Erectile bodies of the penis B. Glands penis C. External urethral sphincter D. Internal urethral sphincter E. Seminal vesicles

106 A. Cavernous nerves are postganglionic parasympathetic fibers that mediate penile erection. Its preganglionic fibers derive from the pelvic splanchnic nerve (S2 to S4). The glans penis receives its supply from the pudendal nerve. The external urethral sphincter is innervated by a voluntary, somatic nerve called the pudendal nerve. The internal urethral sphincter receives parasympathetic fibers also from the inferior hypogastric plexus, which are very different from the cavernous nerves. The seminal vesicles are innervated by the sympathetic nervous system via the inferior hypogastric plexus (GAS Fig. 5-16). GAS 492-494, 515; N 394; McM 278, 279

107 A 34-year-old man has a painful, infected abscess on the posterior aspect of the scrotum. Which of the following lymph nodes is most likely to be tender and swollen? A. External iliac B. Internal iliac C. Lateral aortic/lumbar D. Sacral E. Superficial inguinal

107 E. The scrotal skin is drained by superficial lymphatics that will first empty into the horizontal group of superficial inguinal nodes. The external iliac nodes are deep lymph nodes receiving lymphatics from deep abdominal wall structures below the umbilicus, the pelvic viscera as well as from inguinal lymph nodes. The internal iliac nodes are also deep nodes that drain pelvic viscera, deeper parts of the perineum, and parts of the urethra. The sacral nodes drain the rectum and posterior pelvic wall. GAS 501, 519; N 386; McM 368

108 A 48-year-old woman has the sudden onset of abdominal pain 1 day after undergoing abdominal hysterectomy to remove leiomyomata uteri. Abdominal examination shows tenderness over the right lower quadrant. A retrograde pyelogram shows contrast material in the left renal pelvis and none in the right. These findings indicate that a structure was probably damaged during surgery. That structure passes inferior to which of the following? A. Ovarian artery B. Ovarian ligament C. Mesosalpinx D. Round ligament of the uterus E. Uterine artery

108 E. During hysterectomy, hemostasis may be ensured by proper ligation of the arteries that supply the uterus. The uterine artery, a branch of the internal iliac artery, is anterior to the ureter in the broad ligament of the uterus. During surgeries, the ureter may be ligated mistakenly in the place of the uterine artery. The ovarian ligament attaches the ovary to the lateral and superior part of the uterus. It is a fibrous cord that is contained in the broad ligament of the uterus and has no direct relationship with the ureters. The mesosalpinx is the part of the broad ligament that stretches from the ovary to the uterine tube above. The round ligament of the uterus is an embryologic remnant of the gubernaculum ovary and has no direct relationship with the ureters (GAS Fig. 5-12A). GAS 478; N 378; McM 274, 275

109 A 55-year-old woman presents to the clinic complaining of blood in her stools. Physical exam reveals firm, enlarged inguinal lymph nodes. A contrast CT scan is performed and a possibly cancerous mass is visualized in the lower part of the gastrointestinal tract. In which location is the mass most likely located? A. Anal canal inferior to the pectinate line B. Distal rectum C. Sigmoid colon D. Proximal rectum E. Anal canal superior to the pectinate line

109 A. The lymph from the inferior part of the anal canal drains to the superficial inguinal nodes. The lymph from the distal rectum drains to the sacral nodes. The lymph from the sigmoid colon drains to the inferior mesenteric nodes and then to the internal iliac nodes. The lymph from the proximal rectum drains to the sacral nodes, and the lymph from the anal canal above the pectinate line drains to the internal iliac nodes. GAS 501, 519; N 384; McM 368

11 A 4-month-old male infant is admitted to the pediatric clinic because urine can be observed passing through an opening on the dorsum of the penis. Which of the following embryologic structures most likely failed to fuse? A. Spongy urethra B. Labioscrotal folds C. Cloacal membrane D. Urogenital folds E. Genital tubercle

11 A. Epispadias is a developmental defect in the spongy urethra resulting in urine being expelled from the dorsal aspect of the penis. A failure of the labioscrotal folds to fuse will cause the external urethral orifice to be situated between the two scrotal halves. This is referred to as penile hypospadias. Failure of the urogenital folds to fuse would lead to agenesis of the external urethral folds. The genital tubercle would not directly cause epispadias, as the tubercle still continues to develop, but it is located more dorsally. GAS 466-468, 512; N 366; McM 266

11. A 58-year-old man is diagnosed as having a slowly growing tumor in the deep perineal space. Which of the following structures would most likely be injured? (A) Bulbourethral glands (B) Crus of penis (C) Bulb of vestibule (D) Spongy urethra (E) Great vestibular gland

11. The answer is A. The deep perineal space contains the bulbourethral (Cowper) glands. The crus of the penis, bulb of the vestibule, spongy urethra, and great vestibular gland are found in the superficial perineal space.

110 During laparoscopic removal of an ovarian tumor the surgeon notes firm, immobile lymph nodes that are highly suspicious for cancer as they are the first nodes to receive drainage from the ovary. Which nodes are these? A. Superficial inguinal B. External iliac C. Lumbar/lateral aortic D. Deep inguinal E. Internal iliac

110 C. Embryologically, the ovaries develop from the posterolateral abdominal wall and descend to their adult location in the pelvis. In doing this, they retain their embryological nerve supply and lymphatic drainage. Therefore, lymphatic drainage from the ovaries drains to the lumbar/lateral aortic nodes. Superficial inguinal nodes receive lymphatics from the lower limb, perineum, anal canal below the pectinate line, and the abdominal wall below the umbilicus. The external iliac nodes drain lymph from deep abdominal wall structures below the umbilicus, some pelvic viscera. Deep inguinal nodes are deep lymph nodes that receive lymph from the popliteal nodes, the clitoris in women and the glans penis in men. The internal iliac nodes receive lymph from the pelvic viscera, deep part of the perineum, and the urethra. GAS 501, 5019; N 367; McM 367

111 A 22-year-old woman is involved in a motor vehicle collision. Her complaints are consistent with a pelvic fracture and a CT scan is ordered to rule out internal bleeding. The CT shows no bleeding and that her uterus is in the most typical position. What is this position? A. Anteflexed and retroverted B. Retroflexed and anteverted C. Anteflexed and anteverted D. Retroflexed and retroverted E. Posteriorly retroflexed and anteverted

111 C. The normal position of the uterus in most women is anteflexion and anteversion. Anteflexion is the forward bending of the long axis of the uterus in relation to the cervix at the level of the internal os. Anteversion is the forward bending of the long axis of the uterus in relation to the long axis of the vagina (GAS Fig. 5-54). GAS 477-479; N 340; McM 272

112 A 35-year-old woman presents to the clinic with a malignancy involving the lower part of her vagina. In the image above (Fig. 4-5) which of the following nodes would first receive drainage from this area? A. A B. B C. C D. D E. E

112 E. Fig. 4-5 depicts a lymphangiogram showing several lymphatic groups at the pelvic and inguinal region. The lymphatic groups are as follows A=Lumbar, B=Common iliac, C=External iliac, D=Deep iliac, and E=Superficial inguinal. Malignancies involving the lower part of the vagina typically involve the superficial inguinal lymph nodes, so E is the correct answer (GAS Fig. 5-78). GAS 501, 519; N 384; McM 368

12 A 25-year-old man is admitted to the hospital with testicular pain. Physical examination reveals a swollen and inflamed right testis. A computed tomography (CT) scan examination reveals abnormal accumulation of fluid in the cavity of the tunica vaginalis (Fig. 4-2). Which of the following is the most likely diagnosis for this patient? A. Varicocele B. Rectocele C. Cystocele D. Hydrocele E. Hypospadias

12 D. Hydrocele results from an excess amount of fluid within a persistent processus vaginalis. Hydrocele can result from injury to the testis or by retention of a processus that fills with fluid in infants. The tunica vaginalis consists of parietal and visceral layers, the latter of which is closely attached to the testis and epididymis. The fluid buildup occurs within the cavity between these layers. A varicocele consists of varicosed veins of the pampiniform plexus and is associated with increased venous pressure in the testicular vein, followed by the accumulation and coagulation of venous blood. GAS 470; N 368; McM 266

12. An elderly man with a benign enlargement of his prostate experiences difficulty in urination, urinary frequency, and urgency. Which of the following lobes of the prostate gland is commonly involved in benign hypertrophy that obstructs the prostatic urethra? (A) Anterior lobe (B) Middle lobe (C) Right lateral lobe (D) Left lateral lobe (E) Posterior lobe

12. The answer is B. The middle lobe of the prostate gland is commonly involved in benign prostatic hypertrophy, resulting in obstruction of the prostatic urethra, whereas the posterior lobe is commonly involved in carcinomatous transformation. The anterior lobe contains little glandular tissue, and the two lateral lobes on either side of the urethra form the major part of the gland.

13 A 54-year-old man is admitted to the hospital with severe back pain. Radiographic examination suggests carcinoma of the left kidney blocking the drainage of the testicular vein. Which of the following conditions will be most likely associated with these signs? A. Varicocele B. Rectocele C. Cystocele D. Hydrocele E. Hypospadias

13 A. Varicose veins occur with loss of elasticity within the walls of the vessels. As the veins weaken, they consequently dilate under pressure. A varicocele often occurs with a varicosity of the veins of the pampiniform venous plexus, resulting in a swelling of the veins. This condition can arise from a tumor in the left kidney, which occludes the testicular vein due to an anatomic constriction and increased pressure in veins draining the testis. A hydrocele is an accumulation of fluid within the cavity of the tunica vaginalis. Hypospadias occurs from failure of fusion of the urethral and labioscrotal folds, resulting in an external urethral opening on the ventral surface of the penis or in the perineum. GAS 516, 527; N 379; McM 265

13. A 59-year-old man is diagnosed with prostate cancer following a digital rectal examination. For the resection of prostate cancer, it is important to know that the prostatic ducts open into or on which of the following structures: (A) Membranous part of the urethra (B) Seminal colliculus (C) Spongy urethra (D) Prostatic sinus (E) Prostatic utricle

13. The answer is D. Ducts from the prostate gland open into the prostatic sinus, which is a groove on either side of the urethral crest. The prostate gland receives the ejaculatory duct, which opens into the prostatic urethra on the seminal colliculus (a prominent elevation of the urethral crest) just lateral to the prostatic utricle, which is a small blind pouch. The bulbourethral gland lies on the lateral side of the membranous urethra within the deep perineal space, but its duct opens into the bulbous portion of the spongy (penile) urethra.

14 A 54-year-old man is admitted to the hospital with severe back pain. Upon radiographic examination (Fig. 4-3) the scrotum resembles a "bag of worms." Which of the following conditions will be most likely associated with this radiographic picture? A. Varicocele B. Rectocele C. Cystocele D. Hydrocele E. Hypospadias

14 A. When veins lose their elasticity, they can become weak and often dilate. This causes the veins to become swollen and oftentimes tortuous, as a result of incompetent valves. The appearance of a "bag of worms" on the radiograph is characteristic of a varicosity of the pampiniform venous plexus. A hydrocele is an accumulation of fluid within the tunica vaginalis cavity. Hypospadias occurs from failure of fusion of the urethral and labioscrotal folds, resulting in an external urethral opening on the ventral surface of the penis or in the perineum. GAS 516, 527; N 381; McM 265

14. A 29-year-old woman with a ruptured ectopic pregnancy is admitted to a hospital for culdocentesis. A long needle on the syringe is most efficiently inserted through which of the following structures? (A) Anterior fornix of the vagina (B) Posterior fornix of the vagina (C) Anterior wall of the rectum (D) Posterior wall of the uterine body (E) Posterior wall of the bladder

14. The answer is B. A needle should be inserted through the posterior fornix just below the posterior lip of the cervix while the patient is in the supine position to aspirate abnormal fluid in the cul-de-sac of Douglas (rectouterine pouch). Rectouterine excavation is not most efficiently aspirated by puncture of other structures.

15 A 16-year-old woman is brought to the emergency department with severe abdominal pain and fever. Laboratory examination is remarkable for an elevated white blood cell count and a positive test for pregnancy. Colpocentesis is performed to ascertain the presence of blood in the pelvis from a ruptured ectopic pregnancy. Through which of the following structures does the needle need to be inserted to perform colpocentesis? A. Through the perineal body into the vesicouterine space B. Through the posterior fornix of the vagina into the rectouterine pouch C. Through the anterior fornix into the endocervical canal D. Through the introitus into the vestibular gland E. Through the perineal membrane into the urogenital diaphragm

15 B. It is very likely that the ectopic pregnancy ruptured into the rectouterine pouch, also known as the pouch of Douglas. The most direct route to the rectouterine pouch is through the posterior vaginal fornix. It is unlikely that the pregnancy would have occurred in the vesicouterine space because the transfer of ova from the ovary to the fimbriae occurs on the posterior side of the broad ligament. Therefore, it would not be advisable to attempt initially to insert a needle into the vesicouterine space. Inserting a needle through the anterior fornix into the endocervical canal would lead one into the uterine cavity, with the probability of other undesirable consequences. The urogenital diaphragm is a closed space in the perineum. Entering a vestibular gland with a needle would not be near the location of ectopic pregnancy. GAS 530; N 340; McM 272

15. A 37-year-old man is suffering from carcinoma of the skin of the penis. Cancer cells are likely to metastasize directly to which of the following lymph nodes? (A) External iliac nodes (B) Internal iliac nodes (C) Superficial inguinal nodes (D) Aortic (lumbar) nodes (E) Deep inguinal nodes

15. The answer is C. The superficial inguinal nodes receive lymph from the penis, scrotum, buttocks, labium majus, and the lower parts of the vagina and anal canal. These nodes have efferent vessels that drain primarily into the external iliac and common iliac nodes and ultimately to the lumbar (aortic) nodes. The internal iliac nodes receive lymph from the upper part of the rectum, vagina, uterus, and other pelvic organs, and they drain into the common iliac nodes and then into the lumbar (aortic) nodes. Lymph vessels from the glans penis drain initially into the deep inguinal nodes and then into the external iliac nodes.

16 A 46-year-old woman is admitted to the hospital with a noticeable bulge of tissue through her vaginal opening. During physical examination a rectocele is identified. Which of the following is most likely responsible for this condition? A. Compromised rectovaginal septum B. Weakened superficial and deep transverse perineal muscles C. Paralyzed ischiocavernosus muscle D. Loose sacrospinous ligament E. Ruptured sphincter urethra

16 A. A break or tear in the rectovaginal septum (fascia of Denonvilliers) can allow small intestine (in an enterocele) or rectum (in a rectocele) to herniate into the posterior vaginal wall, even to the point of protrusion through the vaginal introitus. The muscles listed are all in the anterior region of the perineum and have no association with an enterocele or rectocele. The sacrospinous ligament is unrelated to this condition. GAS 481; N 340; McM 272

16. A 42-year-old woman who has had six children develops a weakness of the urogenital diaphragm. Paralysis of which of the following muscles would cause such a symptom? (A) Sphincter urethrae (B) Coccygeus (C) Superficial transversus perinei (D) Levator ani (E) Obturator internus

16. The answer is A. The urogenital diaphragm consists of the sphincter urethrae and deep transverse perineal muscles. Weakness of the muscles, ligaments, and fasciae of the pelvic floor, such as the pelvic diaphragm, urogenital diaphragm, and cardinal (transverse cervical) ligaments, occurs as a result of multiple child delivery, advancing age, and menopause. The pelvic diaphragm is composed of the levator ani and coccygeus muscles. The superficial transversus perinei is one of the superficial perineal muscles, and the obturator internus forms the lateral wall of the ischiorectal fossa.

17 A 68-year-old man is admitted to the hospital with painful urination and nocturia (urination during the night). MRI examination reveals enlargement and irregularity of the uvula of the urethra. This enlargement resulted in difficulty with urinary voiding and inadequate emptying of the bladder. Which of the following lobes of the prostate gland will most likely be hypertrophied? A. Anterior B. Median C. Lateral D. Posterior E. Lateral and posterior

17 B. When the internal urethral orifice is obstructed, it is most likely due to an enlargement of the median (or middle) lobe of the prostate gland. The prostate gland is located at the base of the urinary bladder and is often described as possessing five ill-defined lobes, although this is not accepted by most urologists. The middle lobe consists of glandular tissue dorsal to the uvula of the urethral meatus of the urinary bladder, adjacent to the beginning of the urethra. This glandular tissue is most frequently involved in benign hypertrophy. GAS 466-468, 473; N 362; McM 268

17. A 43-year-old man has a benign tumor located near a gap between the arcuate pubic ligament and the transverse perineal ligament. Which of the following structures is most likely compressed by this tumor? (A) Perineal nerve (B) Deep dorsal vein of the penis (C) Superficial dorsal vein (D) Posterior scrotal nerve (E) Deep artery of the penis

17. The answer is B. The deep dorsal vein, dorsal artery, and dorsal nerve of the penis pass through a gap between the arcuate pubic ligament and the transverse perineal ligament. The perineal nerve divides into a deep branch, which supplies all of the perineal muscles, and superficial branches as posterior scrotal nerves, which supply the scrotum. The superficial dorsal vein of the penis empties into the greater saphenous vein. The deep artery of the penis runs in the corpus cavernosum of the penis.

18 Radiographic studies of a 42-year-old woman reveal that she has a vulvar malignancy involving the clitoris. Removal of all affected lymph nodes would be indicated to avoid spread of this cancer. Which are the first lymph nodes to filter the lymphatic drainage of the involved area? A. Superficial and deep inguinal lymph nodes B. Internal iliac nodes C. Para-aortic lymph nodes D. Presacral lymph nodes E. Axillary lymph nodes

18 A. The deep inguinal lymph nodes drain the glans clitoris and receive lymph also from superficial nodes. The internal iliac nodes drain the inferior pelvic structures, deep perineal structures, and sacral nodes. The para-aortic lymph nodes, or lumbar nodes, receive lymph from the common iliac nodes. The drainage of presacral lymph nodes can pass to the common or internal iliac nodes. Axillary nodes drain body wall structures above the T10 dermatome (or the umbilicus). GAS 501; N 385; McM 368

18. An obstetrician performs a median episiotomy on a woman before parturition to prevent uncontrolled tearing. If the perineal body is damaged, the function of which of the following muscles might be impaired? (A) Ischiocavernosus and sphincter urethrae (B) Deep transverse perineal and obturator internus (C) Bulbospongiosus and superficial transverse perineal (D) External anal sphincter and sphincter urethrae (E) Bulbospongiosus and ischiocavernosus

18. The answer is C. The perineal body (central tendon of the perineum) is a fibromuscular node at the center of the perineum. It provides attachment for the bulbospongiosus, the superficial and deep transverse perineal muscles, and the sphincter ani externus muscles. Other muscles (ischiocavernosus, sphincter urethrae, and obturator internus) are not attached to the perineal body.

19 While performing a voiding cystourethrogram on a 45-year-old man, the urology resident was too forceful when he inserted the catheter and accidentally damaged the wall of the membranous portion of the urethra in the deep perineal compartment (urogenital diaphragm). Which of the following structures would most likely be traumatized at this location? A. Bulbospongiosus muscle B. Sphincter urethra (compressor urethra) C. Corpus cavernosus penis (crus) D. Ischiocavernosus muscle E. Opening of the bulbourethral duct

19 B. If the membranous portion of the urethra is injured, urine and blood can leak upward into the retropubic space (of Retzius) limited inferiorly by the urogenital diaphragm and the muscle within (compressor urethra or external urethral sphincter), which would be injured. The bulbospongiosus muscle and other perineal muscles, the corpus cavernosum, and the openings of the bulbourethral ducts are inferior and anterior to the region of injury. GAS 466-468; N 363; McM 266

2 A 2-day-old infant girl is diagnosed with tracheoesophageal fistula. In addition, physical examination reveals an imperforate anus. A magnetic resonance imaging (MRI) examination reveals that the rectum, vagina, and colon are joined into a single channel. Which of the following structures is directly involved in this malformation? A. Labioscrotal folds B. Persistent cloaca C. Urogenital folds D. Genital tubercle E. Urogenital membrane

2 B. Most anorectal anomalies result from abnormal development of the urorectal septum, ultimately resulting in nondivision of the cloaca into urogenital and anorectal parts. The common outlet of the intestinal, urinary, and reproductive tracts is specifically associated with a persistent cloaca. The labioscrotal folds are involved in forming the external urethral orifice only. The urogenital folds normally fuse along the ventral side of the penis to form the spongy urethra. Epispadias is an anomaly in the development of the genital tubercle and involves the urethral orifice. The urogenital membrane is bounded by the urogenital folds and ruptures to form the urogenital orifice. GAS 322, 430, 460; N 366; McM 266

2. A 17-year-old boy suffers a traumatic groin injury during a soccer match. The urologist notices tenderness and swelling of the boy's left testicle that may be produced by thrombosis in which of the following veins? (A) Left internal pudendal vein (B) Left renal vein (C) Inferior vena cava (D) Left inferior epigastric vein (E) Left external pudendal vein

2. The answer is B. A tender swollen left testis may be produced by thrombosis in the left renal vein because the left testicular vein drains into the left renal vein. The right testicular vein drains into the inferior vena cava. The left internal pudendal vein empties into the left internal iliac vein. The left inferior epigastric vein drains into the left external iliac vein, and the left external pudendal vein empties into the femoral vein.

20 A 22-year-old man complained to the urologist of pain that he experiences from bladder fullness after drinking large quantities of fluid. What is the location of the neural cell bodies responsible for pain sensation from the urinary bladder? A. Dorsal root ganglia of spinal cord levels S2, S3, and S4 B. The intermediolateral cell column of spinal cord levels S2, S3, and S4 C. The sensory ganglia of spinal nerves T5 to T9 D. The preaortic ganglia at the site of origin of the testicular arteries E. Dorsal root ganglia of spinal levels T10 to L2

20 A. Conscious pain due to urinary bladder fullness results from the excitation of stretch receptors in the bladder wall. These pain fibers are carried through the pelvic nerve plexuses and into the pelvic splanchnic nerves. The sensory fibers enter the dorsal root ganglia of spinal nerves S2, S3, and S4. Sensory fibers enter the spinal cord via these ganglia. The intermediomedial cell column of spinal cord levels S2, S3, and S4 contains parasympathetic soma. The levels T5 to T9, T10 to L2, and preaortic ganglia are well above where sensory fibers from the bladder are located. GAS 463, 494; N 395; McM 261

20 A 3-day-old newborn was born with ectopia cordis. Despite the efforts of doctors at the pediatric intensive care unit the infant died from cardiac failure and hypoxemia. Which of the following embryologic events is most likely responsible for the development of such conditions? A. Faulty development of the sternum and pericardium, secondary to incomplete fusion of the lateral folds B. Interruption of third pharyngeal arch development C. Interruption of fourth pharyngeal arch development D. Interruption of fifth pharyngeal arch development E. Faulty development of sinus venosus

20 A. Ectopia cordis is a condition in which the heart is located abnormally outside the thoracic cavity, commonly resulting from a failure of fusion of the lateral folds in forming the thoracic wall. This is incompatible with life because of the occurrence of infection, cardiac failure, or hypoxemia. Faulty development of the sinus venosus is related to atrial septal defects that result from deficient absorption of the sinus venosus into the right atrium and/or unusual development of the septum secundum.

20. A 39-year-old man is unable to expel the last drops of urine from the urethra at the end of micturition because of paralysis of the external urethral sphincter and bulbospongiosus muscles. This condition may occur as a result of injury to which of the following nervous structures? (A) Pelvic plexus (B) Prostatic plexus (C) Pudendal nerve (D) Pelvic splanchnic nerve (E) Sacral splanchnic nerve

20. The answer is C. The perineal branch of the pudendal nerve supplies the external urethral sphincter and bulbospongiosus muscles in the male. All other nervous structures do not supply skeletal muscles but supply smooth muscles in the perineal and pelvic organs. The pelvic and prostatic plexuses contain both sympathetic and parasympathetic nerve fibers. The pelvic splanchnic nerve carries preganglionic parasympathetic fibers, whereas the sacral splanchnic nerve transmits preganglionic sympathetic fibers.

21 A 55-year-old woman complains of fecal incontinence. The most likely contributing factor to such a problem is atrophy, paralysis, or dysfunction of which of the following structures? A. Pubococcygeus muscle B. Iliococcygeus muscle C. Coccygeus muscle D. Pubovesicocervical fascia E. Urogenital diaphragm

21 A. The pubococcygeus muscle, especially its most medial portion, the puborectalis, is of prime importance in fecal continence. The levator ani consists of two major portions, the pubococcygeus and iliococcygeus, which help support pelvic viscera and resist increases in intraabdominal pressure. The puborectalis muscle is the most medial and inferior portion of the pubococcygeus. The puborectalis forms a loop around the anorectal junction, which should keep the anorectal angle to around 90 degrees; the integrity of this muscle is critical in the maintenance of fecal continence. The coccygeus and pubovesicocervical fascia are not in direct contact with the rectum. Damage to the urogenital diaphragm can contribute to urinary incontinence but not fecal incontinence (GAS Fig. 5-7). GAS 454-456; N 336; McM 264

21 A 2-day-old newborn male is admitted to the pediatric intensive care unit with cyanosis and tachypnea. Cardiac ultrasound and MRI examinations reveal totally anomalous pulmonary connections. Which of the following embryologic events is responsible for this malformation? A. Abnormal septation of the sinus venosus B. Abnormal development of the septum secundum C. Abnormal development of the left sinus horn D. Abnormal development of the coronary sinus E. Abnormal development of common cardinal vein

21 A. The right horn of the sinus venosus has two divisions: One develops into the sinus venarum, the smooth interior aspect of the right atrial wall; the other half develops into the pulmonary veins. Abnormal septation of the sinus venosus can lead to inappropriate pulmonary connections. Abnormal development of the left sinus horn would present with abnormalities in the coronary sinus, whereas incorrect development of the septum secundum can result in an atrial septal defect but would not be involved with anomalous pulmonary veins. The left sinual horn develops into the coronary sinus, and the right sinual horn is incorporated into the right atrial wall.

21. A 21-year-old marine biologist asks about her first bimanual examination, and it is explained to her that the normal position of the uterus is (A) Anteflexed and anteverted (B) Retroflexed and anteverted (C) Anteflexed and retroverted (D) Retroverted and retroflexed (E) Anteverted and retroverted

21. The answer is A. The normal position of the uterus is anteverted (i.e., angle of 90 degrees at the junction of the vagina and cervical canal) and anteflexed (i.e., angle of 160 to 170 degrees at the junction of the cervix and body).

22 A 3-day-old newborn has difficulties breathing. A CT scan of his chest and abdomen reveals the absence of the central tendon of the diaphragm. Which of the following structures failed to develop normally? A. Pleuroperitoneal folds B. Pleuropericardial folds C. Septum transversum D. Cervical myotomes E. Dorsal mesentery of the esophagus

22 C. The septum transversum is a thickened layer of mesoderm that gives origin to the central tendon of the diaphragm. It is situated between the thoracic cavity and the omphaloenteric duct. As the lungs grow into the pericardio-peritoneal canal, they give rise to two folds: the pleuroperitoneal and pleuropericardial folds. The pleuroperitoneal folds are responsible for formation of the posterolateral aspect of the diaphragm, and the pleuropericardial folds develop into the fibrous pericardium. The crura provide origin of the dorsal mesentery of the esophagus, whereas the cervicalyotomes are responsible for the musculature of the diaphragm. (Note that these are cervical myotomes C3 to C5, the levels of origin of the phrenic nerve.)

22 A 45-year-old obese woman was admitted to the hospital because of "pains in her leg." Physical examination led to a diagnosis of "meralgia paresthetica." In her condition, excessive adipose tissue bulging over the inguinal ligament exerted traction upon it, compressing a nerve that passed through, or beneath, the ligament, just medial to the anterior superior iliac spine. Which of the following nerves was most likely affected in this patient? A. Femoral branch of the genitofemoral nerve B. Femoral nerve C. Iliohypogastric nerve D. Ilioinguinal nerve E. Lateral femoral cutaneous nerve

22 E. The lateral femoral cutaneous nerve (L2, L3) emerges from the lateral side of the psoas muscle and runs in front of the iliacus and through, or behind, the inguinal ligament and innervates the skin of the lateral aspect of the thigh to the level of the knee. This nerve has been constricted in this case of "Calvin Klein syndrome" (in this case from the patient's obesity, not their too tight jeans) causing pain, tingling, or burning sensations in the lateral thigh. The femoral branch of the genitofemoral nerve (L1, L2) supplies a small area of skin (over the femoral triangle), just inferior to the midpoint of the inguinal ligament. The femoral nerve (L2 to L4) is motor to the quadriceps and sartorius muscles and sensory to the anterior thigh and the medial thigh and leg. The ilioinguinal supplies the suprapubic region; part of the genitalia and anterior perineum; and the upper, medial thigh. Cutaneous branches of the iliohypogastric nerve innervate skin of the anterolateral gluteal area and suprapubic region. GAS 398; N 391; McM 261

22. After his bath but before getting dressed, a 4-year-old boy was playing with his puppy. The boy's penis was bitten by the puppy, and the deep dorsal vein was injured. The damaged vein (A) Lies superficial to Buck fascia (B) Drains into the prostatic venous plexus (C) Lies lateral to the dorsal artery of the penis (D) Is found in the corpus spongiosum (E) Is dilated during erection

22. The answer is B. The deep dorsal vein of the penis lies medial to the dorsal artery of the penis on the dorsum of the penis and deep to Buck fascia, drains into the prostatic plexus of veins, and is compressed against the underlying deep fascia of the penis during erection.

23 On digital examination of the vagina, the portion of the uterus that one anticipates palpating with the examining finger is the cervix and its external os. Which of the following is the most common position of the uterus? A. Anteflexed and retroverted B. Retroflexed and anteverted C. Anteflexed and anteverted D. Retroflexed and retroverted E. Anteflexed and anteverted

23 C. Normally, the uterus is anteflexed at the junction of the cervix and the body and anteverted at the junction of the vagina and the cervical canal. GAS 477; N 350; McM 273

23 A 30-year-old man is diagnosed with a blockage of arterial flow in the proximal part of the thoracic aorta. Brachial arterial pressure is markedly increased, femoral pressure is decreased, and the femoral pulses are delayed. The patient shows no external signs of inflammation. Which of the following structures failed to develop normally? A. Second aortic arch B. Third aortic arch C. Fourth aortic arch D. Fifth aortic arch E. Ductus venosus

23 C. The fourth aortic arch develops into the aortic arch on the left side and the brachiocephalic and subclavian arteries on the right side of the embryo. Improper development of the arch of the aorta will cause an increased pressure in the subclavian artery and, subsequently, the brachial artery. Similarly, decreased flow through the aorta will lead to a decreased pressure in the femoral artery. The second aortic arch, specifically the dorsal aspect, develops into aspects of the small stapedial artery. The proximal part of the third aortic arch gives rise to the common carotid arteries, which supply the head. The fifth aortic arch is said not to usually develop in human embryos. The proximal part of the sixth aortic arch develops into the left pulmonary artery.

23. A 62-year-old man is incapable of penile erection after rectal surgery with prostatectomy. The patient most likely has a lesion of which of the following nerves? (A) Dorsal nerve of the penis (B) Perineal nerve (C) Hypogastric nerve (D) Sacral splanchnic nerve (E) Pelvic splanchnic nerve

23. The answer is E. The pelvic splanchnic nerve contains preganglionic parasympathetic fibers, whereas the sacral splanchnic nerve contains preganglionic sympathetic fibers. Parasympathetic fibers are responsible for erection, whereas sympathetic fibers are involved with ejaculation. The right and left hypogastric nerves contain primarily sympathetic fibers and visceral sensory fibers. The dorsal nerve of the penis and the perineal nerve provide sensory nerve fibers.

24 A 1-year-old child was admitted to the pediatric clinic due to severe dyspnea. ECG reveals cardiac arrhythmia and right ventricular hypertrophy. An angiogram reveals a patent ductus arteriosus (PDA). From which of the following embryologic arterial structures does the PDA take origin? A. Left sixth arch B. Right sixth arch C. Left fifth arch D. Right sixth arch E. Left fourth arch

24 A. The left sixth aortic arch is responsible for the development both of the pulmonary arteries and the ductus arteriosus. Without regression of the ductus arteriosus, a patent connection remains between aorta and the pulmonary trunk. The ductus arteriosus often reaches functional closure within 24 hours after birth, whereas anatomic closure and subsequent formation of the ligamentum arteriosum often occur by the twelfth postnataleek.

24 A 42-year-old woman is admitted to the emergency department because of pelvic discomfort. During physical examination the gynecologist discovers that the patient has suffered a complete uterine prolapse. Which of the following ligaments provides direct support to the uterus? A. Mesosalpinx and mesometrium B. Infundibulopelvic ligament C. Round ligament of the uterus D. Lateral cervical (cardinal) ligament E. Broad ligament of the uterus

24 D. The cardinal ligament, also known as Mackenrodt's ligament or transverse cervical ligament, is composed of condensations of fibromuscular tissues that accompany the uterine vessels. These bands of pelvic fascia provide direct support to the uterus. The other ligaments listed do not play a direct role in uterine stability. GAS 481; N 351; McM 274

24. A 23-year-old massage therapist who specializes in women's health attends a lecture at an annual conference on techniques of massage. She asks, "What structure is drained by the lumbar (aortic) lymph nodes?" Which of the following structures is the correct answer to this question? (A) Perineum (B) Lower part of the vagina (C) External genitalia (D) Ovary (E) Lower part of the anterior abdominal wall

24. The answer is D. The lymphatic vessels from the ovary ascend with the ovarian vessels in the suspensory ligament and terminate in the lumbar (aortic) nodes. Lymphatic vessels from the perineum, external genitalia, and lower part of the anterior abdominal wall drain into the superficial inguinal nodes.

25 A 34-year-old woman is admitted to the hospital due to severe lower abdominal pain. Radiographic examination reveals tumors in both of her ovaries. A biopsy is ordered and confirms the initial diagnosis of ovarian cancer. Which of the following lymph nodes are the first to receive lymph from the diseased ovaries? A. Superficial and deep inguinal lymph nodes B. External iliac nodes C. Para-aortic nodes at the level of the renal vessels D. Node of Cloquet E. Internal iliac nodes accompanying the uterine artery and vein

25 C. Ovarian lymph first drains into the para-aortic nodes at the level of the renal vessels. The superficial and deep inguinal nodes drain the body wall below the umbilicus, the lower limbs, and the cutaneous portion of the anal canal and parts of the perineum. The external iliac nodes receive the lymph from the inguinal nodes. The node of Cloquet is located in the femoral ring, adjacent to the external iliac vein and beneath the inguinal ligament. The node of Cloquet drains into the common iliac nodes. The internal iliac nodes accompany the uterine artery and vein, receiving lymph from much of the uterus but not the ovaries. GAS 501; N 384; McM 362

25 A 4-year-old girl is admit ed to the hospital with high fever. Staphylococcus aureus is isolated from her blood cultures and antibiotic therapy is initiated. A loud, harsh murmur is heard on auscultation. A chest radiograph shows prominent pulmonary arteries. Echocardiography shows all the valves to be normal. Which congenital heart disease most likely explains these findings? A. Atrial septal defect B. Tetralogy of Fallot C. Coarctation of the aorta D. Patent ductus arteriosus E. Aortic atresia

25 D. With a patent ductus arteriosus, an abnormal connection persists between the aorta and the pulmonary trunk. Blood leaving the left ventricle of the heart and into the aorta is reshunted back into the left pulmonary artery. This is responsible for the murmur heard during auscultation of the heart. The diversion of blood to the pulmonary arteries causes increased atrial pressure, leading to enlarged, and therefore noticeable, pulmonary arteries on the chest radiograph. The tetralogy of Fallot often presents with a right-to-left shunt of blood flow through the ventricles. It is also associated with pulmonary artery stenosis, right ventricular hypertrophy, interventricular septal defect, and an overriding of the aorta. This condition would not present with a murmur, however. Atrial septal defects are often characterized by a left-to-right shunt of blood, which often presents with dyspnea and abnormal heart sounds. A chest radiograph would not reveal prominent pulmonary arteries in such cases. Both aortic atresia and coarctation of the aorta result in a narrowing of the aorta but would not lead to noticeable prominent pulmonary arteries on the radiograph.

25. A sexually active adolescent presents with an infection within the ischiorectal fossa. Which of the following structures is most likely injured? (A) Vestibular bulb (B) Seminal vesicle (C) Greater vestibular gland (D) Inferior rectal nerve (E) Internal pudendal artery

25. The answer is D. The ischiorectal fossa contains the inferior rectal nerves and vessels and adipose tissue. The bulb of the vestibule and the great vestibular gland are located in the superficial perineal space, whereas the bulbourethral gland is found in the deep perineal space. The internal pudendal artery runs in the pudendal canal, but its branches pass through the superficial and deep perineal spaces.

26 A 29-year-old pregnant woman is admitted to the hospital to deliver her baby. During a vaginal delivery the obstetrician performs a median episiotomy in which the area of the perineal body is cut deeply. Two weeks after the delivery the woman complains that she has had fecal incontinence since the delivery. Which of the following structures was also most likely damaged during the episiotomy? A. Superficial and deep transverse perineal muscles B. External anal sphincter C. Ischiocavernosus muscle D. Sacrospinous ligament E. Sphincter urethra

26 B. The external anal sphincter is important for maintaining fecal continence. The external anal sphincter is located immediately posterior to the perineal body (central tendon) and would be susceptible to damage during a median episiotomy. The other structures listed play no role in maintaining fecal continence (GAS Fig. 5-17). GAS 460; N 347; McM 277

26 A 3-day-old infant is admitted to the cardiology unit with severe cyanosis. During echocardiographic examination a right-to-left shunt is identified. Which of the following conditions will most likely produce this type of shunt? A. Interatrial septal defect B. Interventricular septal defect C. Patent ductus arteriosus D. Corrected transposition of the great arteries E. Common truncus arteriosus

26 E. A common truncus arteriosus results from failure of separation of the pulmonary trunk and aorta. Without proper perfusion of the child by oxygenated blood, severe cyanosis will result.

26. A first-year resident in the urology department reviews pelvic anatomy before seeing patients. Which of the following statements is correct? (A) The dorsal artery of the penis supplies the glans penis. (B) The seminal vesicles store spermatozoa. (C) The duct of the bulbourethral gland opens into the membranous urethra. (D) The duct of the greater vestibular gland opens into the vagina. (E) The anterior lobe of the prostate gland is prone to carcinomatous transformation.

26. The answer is A. The dorsal artery of the penis supplies the glans penis. The seminal vesicles store no spermatozoa. The duct of the bulbourethral gland opens into the bulbous portion of the spongy urethra, whereas the greater vestibular gland opens into the vestibule between the labium minora and the hymen. The anterior lobe of the prostate is devoid of glandular substance, the middle lobe is prone to benign hypertrophy, and the posterior lobe is prone to carcinomatous transformation.

27 A 4-day-old infant was admitted to the pulmonary unit suffering from dyspnea and cyanosis. Radiographic examination revealed a left hypoplastic lung and herniation of abdominal intestines into the left thoracic cavity. Which of the following embryologic structures most likely failed to develop properly? A. Septum transversum B. Pleuroperitoneal membrane C. Tracheoesophageal septum D. Laryngotracheal groove E. Foregut

27 B. The pleuroperitoneal membrane forms the posterolateral aspect of the diaphragm. A defect in this membrane would allow for communication between the upper left abdominal cavity and thoracic cavity and could result in a congenital diaphragmatic hernia. The septum transversum provides origin to the central tendon of the diaphragm but is not involved in herniation of the intestines. The tracheoesophageal septum, laryngotracheal groove, and oligohydramnios are not associated with development of the diaphragm.

27 After having given birth to five children, a 41-year-old woman seeks correction of chronic urinary incontinence. While relating her history the patient reveals that she has leakage of urine with increased intraabdominal pressure (stress incontinence). An MRI examination reveals injury to the pelvic floor that has altered the position of the neck of the bladder and the urethra. Which of the following structures has most probably been injured at some time during the multiple deliveries? A. Tendinous arch of levator ani B. Coccygeus C. Tendinous arch of fascia pelvis D. Obturator internus E. Rectovaginal septum

27 C. The tendinous arch of fascia pelvis is a dense band of connective tissue that joins the fascia of the levator ani to the feltlike pubocervical fascia that covers the anterior wall of the vagina. If this fascial band is torn, the ipsilateral side of the vagina falls, carrying with it the bladder and urethra, often leading to urinary incontinence. The tendinous arch of the levator ani is a thickened portion of the fascia of the obturator internus and provides part of the origin of the levator ani muscle, but it plays no direct role in incontinence. The coccygeus muscle supports and raises the pelvic floor but is not directly associated with urinary incontinence. The obturator internus is involved with lateral rotation of the thigh. If the rectovaginal septum is torn, the patient can be subject to the occurrence of rectocele or enterocele, as the lower portion of the GI tract prolapses into the posterior wall of the vagina (GAS Fig. 5-34). GAS 466, 459; N 356; McM 275

27. A 43-year-old woman presents with a prolapsed uterus. Repair of a prolapsed uterus requires knowledge of the supporting structures of the uterus. Which of the following structures plays the most important role in the support of the uterus? (A) Levator ani (B) Sphincter urethrae (C) Uterosacral ligament (D) Ovarian ligament (E) Arcuate pubic ligament

27. The answer is A. The pelvic diaphragm, particularly the levator ani, provides the most important support for the uterus, although the urogenital diaphragm and the uterosacral and ovarian ligaments support the uterus. The arcuate pubic ligament arches across the inferior aspect of the pubic symphysis.

28 A 58-year-old postmenopausal woman is diagnosed with carcinoma of the distal gastrointestinal tract. During surgery lymph nodes from the sacral, internal iliac, and inguinal lymph node groups were removed and sent for histopathologic examination. The pathology report revealed positive cancerous cells only at the inguinal lymph nodes. Which of the following parts of the gastrointestinal tract were most likely affected? A. Cutaneous portion of anal canal B. Distal rectum C. Mucosal zone of anal canal D. Pectinate line of anal canal E. Proximal rectum at the inferior valve (of Houston)

28 A. Lymph from the cutaneous portion of the anal canal (below the pectinate line) drains into the inguinal nodes. Lymph from most parts of the rectum and from the mucosal zone of the anal canal (above the pectinate line) drains into the internal iliac nodes. Lymph from some parts of the rectum also drains into the sacral nodes. GAS 501; N 384; McM 368

28 A 3-month-old infant is diagnosed with a ventricular septal defect (VSD) at the area of the subpulmonary infundibulum. Which of the following structures must be avoided carefully by the surgeon when the sutures are placed at the site of the defect? A. Right bundle branch B. Right coronary artery C. Tricuspid valve D. Left anterior descending coronary artery E. Aortic valve

28 A. When closing a ventricular septal defect, it is important not to suture over the right bundle branch because it carries the stimulating impulse from the atrioventricular node to the apex of the heart through the right bundle of His. Following the course of the right bundle branch on the interventricular septum, the impulses travel along the septomarginal trabeculation (moderator band) and Purkinje fibers, leading to ventricular contraction. The right coronary artery passes dorsally in the atrioventricular groove; therefore, it does not pass through the interventricular septum. The anterior interventricular (left anterior descending) coronary artery is superficial to the IV septum on the anterior surface of the heart. The tricuspid valve and aortic valve are not directly associated with the interventricular septum.

28. A 16-year-old boy presents to the emergency department with rupture of the penile urethra. Extravasated urine from this injury can spread into which of the following structures? (A) Scrotum (B) Ischiorectal fossa (C) Pelvic cavity (D) Testis (E) Thigh

28. The answer is A. Extravasated urine from the penile urethra below the perineal membrane spreads into the superficial perineal space, scrotum, penis, and anterior abdominal wall. However, it does not spread into the testis, ischiorectal fossa, pelvic cavity, and thigh because Scarpa fascia ends by firm attachment to the fascia lata of the thigh.

29 A 2-day-old infant is diagnosed with incomplete division of the foregut into respiratory and digestive portions. Which is the most common congenital condition characteristic of this description? A. Esophageal atresia B. Esophageal achalasia C. Tracheoesophageal fistula D. Congenital diaphragmatic hernia E. Esophageal fistula

29 A. Esophageal atresia is often the result of an incomplete division of the tracheoesophageal septum, thus causing an absence of, or blind ending of, the esophagus. Though similar to an esophageal atresia, a tracheoesophageal fistula is an atypical connection between the trachea and the esophagus.

29 A 62-year-old man is admitted to the emergency department due to increasing difficulty in urinating over a period of several months. Physical examination reveals prostatic hypertrophy. After several unsuccessful attempts to catheterize the penile urethra, the urologist orders drainage of the urinary bladder by the least invasive procedure, avoiding entry into the peritoneal cavity or the injury of any major vessels or organs. Which of the following spaces needs to be traversed by the needle to reach the bladder? A. Ischioanal fossa B. Perineal body C. Retropubic space (of Retzius) D. Superficial perineal cleft E. Deep perineal pouch

29 C. The retropubic space (of Retzius) is the extraperitoneal space between the pubic symphysis and the bladder. A needle placed over the pubic bone, through the body wall, and into the space of Retzius will enter the full bladder but avoids entry into the peritoneum and there is little risk of damaging major organs or vessels. Entry through the ischioanal fossa would not provide a direct route to the bladder. With entry through the superficial perineal cleft, perineal body, and deep perineal pouch there is a high risk of damaging important structures. GAS 463, 466; N 374; McM 266

29. A 23-year-old woman visits her obstetrician for an annual checkup. During vaginal examination, which of the following structures may be palpated? (A) Apex of the urinary bladder (B) Fundus of the uterus (C) Terminal part of the round ligament of the uterus (D) Body of the clitoris (E) Uterine cervix

29. The answer is E. In addition to the uterine cervix, the uterus, uterine tubes, ovaries, and ureters can be palpated. The apex of the urinary bladder is the anterior end of the bladder; thus, it cannot be palpated. The fundus of the uterus is the anterosuperior part of the uterus. The terminal part of the round ligament of the uterus emerges from the superficial inguinal ring and becomes lost in the subcutaneous tissue of the labium majus.

3 A 6-year-old boy is admitted to the hospital because of a palpable mass located external to the aponeurosis of the external oblique. Radiographic examination reveals that the mass is an ectopic testis, classified as interstitial. Failure of normal development of which of the following embryologic structures is responsible for ectopic testis? A. Gubernaculum B. Processus vaginalis C. Genital tubercle D. Seminiferous cords E. Labioscrotal swellings

3 A. The gubernaculum arises in the upper abdomen from the lower end of the gonadal ridge and helps guide the testis in its descent through the abdominal wall and then into the scrotum. Ectopic testes occur when a portion of the gubernaculum passes to an abnormal position or otherwise fails to descend or become fixed to the skin of the scrotum. The processus vaginalis is a tube of peritoneum that follows the same oblique course through the body wall as the testis, ventral to the gubernaculum. The distal part of the processus is retained as the tunica vaginalis. If part of the remainder of the processus remains patent, it can fill with fluid as a hydrocele of the testis or spermatic cord. The genital tubercle forms the primordial phallus and is associated with epispadias. The seminiferous cords form the primordia of the seminiferous tubules. The labioscrotal swellings approach each other and fuse to form the scrotum. GAS 293-302, 470; N 364; McM 266

3. On a busy Saturday night in Chicago, a 16-year-old boy presents to the emergency department with a stab wound from a knife that entered the pelvis above the piriformis muscle. Which of the following structures is most likely to be damaged? (A) Sciatic nerve (B) Internal pudendal artery (C) Superior gluteal nerve (D) Inferior gluteal artery (E) Posterior femoral cutaneous nerve

3. The answer is C. The superior gluteal nerve leaves the pelvis through the greater sciatic foramen, above the piriformis. The sciatic nerve, internal pudendal vessels, inferior gluteal vessels and nerve, and posterior femoral cutaneous nerve leave the pelvis below the piriformis.

3. A 29-year-old man comes in with a stab wound, cannot raise his arm above horizontal, and exhibits a condition known as "winged scapula." Which of the following structures of the brachial plexus would most likely be damaged? (A) Medial cord (B) Posterior cord (C) Lower trunk (D) Roots (E) Upper trunk

3. The answer is D. Winged scapula is caused by paralysis of the serratus anterior muscle that results from damage to the long thoracic nerve, which arises from the roots of the brachial plexus (C5-C7).

30 An unconscious 2-month-old infant is admitted to the emergency department after an automobile collision. An emergency tracheostomy is performed. Which of the following structures is most commonly at high risk of injury during this procedure? A. Left brachiocephalic vein B. Left common carotid artery C. Vagus nerve D. Phrenic nerve E. Thoracic duct

30 A. In a tracheotomy, an incision is made at the level of the sixth cervical vertebra, near the cricoid cartilage. The left brachiocephalic vein passes across the trachea immediately anterior to the brachiocephalic trunk. This vein is the most superficial structure and thus the most likely to be damaged. The left common carotid artery, the vagus nerve, and the phrenic nerve are not situated near the midline incision of the tracheotomy. The thoracic duct is located posterior and lateral to the esophagus and the trachea and is not likely to be damaged during a tracheotomy, other than the intentional opening made in it.

30 A 13-year-old female is brought to the emergency department with a complaint of severe, deep pelvic discomfort. Physical examination reveals that the patient has an intact hymen. Incision of the hymen reveals hematocolpos. Which of the following conditions is associated with hematocolpos? A. Cyst of Bartholin's gland B. Bleeding from an ectopic pregnancy C. Imperforate hymen D. Indirect inguinal hernia with cremasteric artery bleeding E. Iatrogenic bleeding from the uterine veins

30 C. Hematocolpos is characterized by filling of the vagina with menstrual blood. This commonly occurs due to the presence of an imperforate hymen. Bartholin's gland ducts open into the vestibule of the vagina; therefore, a cyst in Bartholin's gland would not cause hematocolpos. Blood from a ruptured ectopic pregnancy most often drains into the rectouterine pouch (of Douglas). Women often have a diminutive cremaster muscle and cremasteric artery and vein, but none of these are associated with hematocolpos. The cremasteric artery provides a small branch to the round ligament of the uterus (sometimes called "Samson's artery"), which must be kept in mind during a hysterectomy, with division of the round ligament. Bleeding from the uterine veins would not flow into the vagina. GAS 480; N 354; McM 272

30. A 53-year-old bank teller is admitted to a local hospital for surgical removal of a benign pelvic tumor confined within the broad ligament. There is a risk of injuring which of the following structures that lies in this ligament? (A) Ovary (B) Proximal part of the pelvic ureter (C) Terminal part of the round ligament of the uterus (D) Uterine tube (E) Suspensory ligament of the ovary

30. The answer is D. The uterine tubes lie in the broad ligament. The anterior surface of the ovary is attached to the posterior surface of the broad ligament of the uterus. The ureter descends retroperitoneally on the lateral pelvic wall but is crossed by the uterine artery in the base (in the inferomedial part) of the broad ligament. The terminal part of the round ligament of the uterus becomes lost in the subcutaneous tissue of the labium majus. The suspensory ligament of the ovary is a band of peritoneum that extends superiorly from the ovary to the pelvic wall.

31 A 45-year-old female is admitted to the hospital with difficulty breathing. Radiographic examination reveals a tumor invading the lung surface anterior to the hilum. Which nerve is most likely compressed by the tumor to result in dyspnea? A. Phrenic B. Vagus C. Intercostal D. Recurrent laryngeal E. Cardiopulmonary

31 A. The phrenic nerve has a path between the anterior medial aspect of the lung and the mediastinum. Along the path of the nerve, it courses over the hilum of the lung. The vagus nerves run posterior to the heart as they give off branches to the cardiac plexus upon the trachea near the carina. The recurrent laryngeal nerves arise from the vagus nerves before the vagus nerves pass behind the hila of the lungs.

31 A 42-year-old woman is admitted to the emergency department with a complaint of dull, poorly localized pain in the deep pelvis. An MRI examination reveals a prolapsing of abdominal viscera, probably due to a tear of the rectovaginal septum. Which of the following conditions will most likely result from a defect in the rectovaginal septum? A. Cystocele B. Urethrocele C. Enterocele D. Urinary incontinence E. Prolapsed uterus

31 C. An enterocele (herniation of small intestine into the posterior wall of the vagina) is caused by a tear of the rectovaginal septum that weakens the pelvic floor. An urethrocele is characterized by prolapse of the urethra into the vagina. It is usually associated with a cystocele (prolapse of the bladder into the urethra). Cystocele or urethrocele are associated with defects in the pubocervical fascia that covers the anterior wall of the vagina and assists in supporting the bladder. Urinary incontinence can result from weakening of the muscles that surround the urethra but would not be caused by a tear of the rectovaginal septum. Prolapse of the uterus is caused by weakening or tearing of the ligaments that support the uterus (especially the cardinal and/or uterosacral ligaments). GAS 480-481; N 343; McM 272

31. A 72-year-old man comes to his physician for an annual checkup. Which of the following structures is most readily palpated during rectal examination? (A) Prostate gland (B) Epididymis (C) Ejaculatory duct (D) Ureter (E) Testis

31. The answer is A. The prostate gland may be palpated on rectal examination. The ejaculatory duct runs within the prostate gland and cannot be felt. In the male, the pelvic part of the ureter lies lateral to the ductus deferens and enters the posterosuperior angle of the bladder, where it is situated anterior to the upper end of the seminal vesicle, and thus, it cannot be palpated during rectal examination. However, in the female, the ureter can be palpated during vaginal examination because it runs near the uterine cervix and the lateral fornix of the vagina to enter the posterosuperior angle of the bladder. The testes are examined during a routine annual checkup but obviously not during a rectal examination.

32 A 34-year-old woman is admitted to the hospital complaining of urinary incontinence. MRI examination reveals that one of the skeletal muscles of the pelvis has a significant tear. Which of the following muscles is the most significant in terms of maintaining continence? A. Pubococcygeus B. Obturator internus C. Piriformis D. Coccygeus E. Iliococcygeus

32 A. Of the answer choices listed, the pubococcygeus is the muscle that is most directly associated with the arcus tendineus fascia pelvis and connective tissues of the vagina and the support of the bladder. The obturator internus, piriformis, and coccygeus do not form parts of the levator ani and provide no direct support to the urogenital organs, nor do they have any role in urinary incontinence. The iliococcygeus does form part of the levator ani, but it is located lateral to the pubococcygeus and therefore does not play a direct role in maintaining urinary continence. GAS 454-456; N 336; McM 264

32 A 62-year-old male patient expresses concern that his voice has changed over the preceding months. Imaging reveals a growth located within the aortic arch, adjacent to the left pulmonary artery. Which neural structure is most likely being compressed to cause the changes in the patient's voice? A. Left phrenic nerve B. Esophageal plexus C. Left recurrent laryngeal nerve D. Left vagus nerve E. Left sympathetic trunk

32 C. The left recurrent laryngeal nerve passes beneath the ligamentum arteriosum and then loops superiorly toward the tracheoesophageal groove, medial to the arch of the aorta.

32. A 48-year-old college football coach undergoes a radical prostatectomy for a malignant tumor in his prostate. Following surgery, he is incapable of achieving an erection. Which of the following nerves is most likely damaged during the surgery? (A) Sacral splanchnic nerve (B) Pelvic splanchnic nerve (C) Pudendal nerve (D) Dorsal nerve of the penis (E) Posterior scrotal nerve

32. The answer is B. Parasympathetic preganglionic fibers in the pelvic splanchnic nerve are responsible for erection of the penis. Sympathetic preganglionic fibers in the sacral splanchnic nerve are responsible for ejaculation. The pudendal nerve supplies the external anal sphincter and perineal muscles and supplies GSA fibers to the perineal region. The dorsal nerve of the penis is a terminal branch of the pudendal nerve and supplies sensation of the penis. The posterior scrotal nerves are superficial branches of the perineal nerve and supply sensory fibers to the scrotum.

33 A 42-year-old woman is admitted to the emergency department with severe abdominal pain. MRI examination reveals a tumor at her left ovary. A frozen biopsy during the time of surgery reveals an ovarian carcinoma. Which of the following actions can be performed to reduce the pain from the ovarian carcinoma? A. Cut the infundibulopelvic ligament B. Cut the pelvic sympathetic trunk C. Cut the clunial nerves D. Cut the pudendal nerve E. Cut the broad ligament

33 A. The ovarian vessels and nerves lie within the infundibulopelvic ligament (suspensory ligament of the ovary); therefore, cutting this ligament interrupts pain fibers from the ovary. Cutting the sympathetic trunk might help to reduce some of the pain from the ovary, but the results of such a procedure are rather unpredictable, plus locating the lumbar sympathetic trunk is more of a surgical challenge. The clunial nerves are cutaneous nerves that innervate parts of the buttocks. They are not associated with the ovaries. The pudendal nerve innervates the perineum and does not carry afferent pain fibers from the ovary. The broad ligament contains only the uterovaginal vessels and nerve plexus and does not carry any nerve fibers from the ovary (GAS Fig. 5-65). GAS 498; N 342; McM 274

33 A 39-year-old woman visits the outpatient clinic and complains of inability to reach a pantry shelf just above her head. History reveals that 2 months ago she underwent a mastectomy procedure and she did not have this complaint prior to the surgery. Which nerve was most likely damaged during surgery to result in the patient's complaint? A. Axillary B. Spinal accessory C. Long thoracic D. Radial E. Thoracodorsal

33 C. During mastectomy procedures, three superficial nerves are susceptible to ligation or laceration: the longhoracic nerve, intercostobrachial nerve, and thoracodorsal nerve. In the event of injury to the long thoracic nerve, the patient complains of an inability to fully abduct the humerus above the horizontal. The serratus anterior (supplied by the long thoracic nerve) is necessary to elevate, rotate, and abduct the scapula, to facilitate abduction of the humerus above the shoulder. Because the patient does not indicate any loss of medial rotation or adduction of the humerus, ligation or injury of the thoracodorsal nerve can be eliminated.

33. While performing a pelvic exenteration, the surgical oncologist notices a fractured or ruptured boundary of the pelvic inlet. Which of the following structures is most likely damaged? (A) Promontory of the sacrum (B) Anterior-inferior iliac spine (C) Inguinal ligament (D) Iliac crest (E) Arcuate pubic ligament

33. The answer is A. The pelvic inlet (pelvic brim) is bounded by the promontory and the anterior border of the ala of the sacrum, the arcuate line of the ilium, the pectineal line, the pubic crest, and the superior margin of the pubic symphysis.

34 A 41-year-old female is admitted to the emergency department with a complaint of severe, sharp, but poorly localized pain on the chest wall. Radiographic examination gives evidence of pleural effusion. What is the location of the neuronal cell bodies responsible for the nerve fibers that carry this pain to the central nervous system (CNS)? A. Dorsal root ganglia B. Sympathetic chain ganglia C. Dorsal horn of the spinal cord D. Lateral horn of the spinal cord E. Ventral horn of the spinal cord

34 A. The dorsal root ganglia contain nerve cell bodies for general somatic afferent and general visceral afferent neuronal processes. Pain localized on the chest wall is transmitted back to the CNS via sensory fibers.

34 A 45-year-old man is admitted to the emergency department after a violent car crash. Physical examination reveals that the patient suffers from a "straddle" injury to the perineum. A MRI examination reveals that extravasating urine and blood from a torn bulbar urethra are present in the superficial perineal cleft. Which of the following fasciae provide boundaries for this space? A. Camper's fascia and Scarpa's fascia B. Perineal membrane and external perineal fascia of Gallaudet C. Colles fascia and external perineal fascia of Gallaudet D. Perineal membrane and the superior fascia of the urogenital diaphragm E. The urogenital diaphragm and the apex of the prostate gland

34 C. The superficial perineal space or cleft lies between the external perineal fascia of Gallaudet (fascia of inferior perineal muscles in the superficial perineal compartment) and the membranous layer of Colles' fascia. Camper's fascia is the superficial fatty layer of the anterior abdominal wall and the perineum; Scarpa's fascia is the deep membranous layer of the abdominal wall. The perineal membrane is the inferior fascia of the urogenital diaphragm that forms the inferior boundary of the deep perineal compartment. The superior fascia of the urogenital diaphragm bounds the inferior border of the anterior recess of the ischioanal fossa. There is no space between the urogenital diaphragm and the apex of the prostate gland (GAS Fig. 5-69). GAS 506-512; N 374; McM 266

34. A 32-year-old patient with multiple fractures of the pelvis has no cutaneous sensation in the urogenital triangle. The function of which of the following nerves is most likely to be spared? (A) Ilioinguinal nerve (B) Iliohypogastric nerve (C) Posterior cutaneous nerve of the thigh (D) Pudendal nerve (E) Genitofemoral nerve

34. The answer is B. The iliohypogastric nerve innervates the skin above the pubis. The skin of the urogenital triangle is innervated by the pudendal nerve, perineal branches of the posterior femoral cutaneous nerve, anterior scrotal or labial branches of the ilioinguinal nerve, and the genital branch of the genitofemoral nerve.

35 Several days after a 34-year-old woman was admitted to the hospital after her automobile collided with a lamppost, her urinary bladder gives evidence of paralysis. A CT scan reveals multiple fractures of her pelvis with a significant pelvic hematoma, either of which could have interrupted or injured the nerve supply of the organ. Which of the following nerves was most likely traumatized? A. Superior hypogastric B. Pelvic splanchnic C. Sacral splanchnic D. Lumbar splanchnic E. Pudendal

35 B. The urinary bladder wall includes the detrusor muscle, and it receives both its motor and sensory innervation from parasympathetic nerve fibers transmitted by way of the pelvic splanchnic nerves from S2 to S4. GAS 492-494; N 395; McM 268

35 A 23-year-old man is admitted to the emergency department after an automobile collision. Physical examination reveals tachycardia. What is the location of the preganglionic neural cell bodies involved in increasing the heart rate? A. Deep cardiac plexus B. Dorsal motor nucleus of vagus C. Lateral horn T5 to T9 D. Lateral horn T1 to T4 E. Superior, middle, and inferior cervical ganglia

35 D. The lateral horns, or intermediolateral cell columns, contain the cell bodies of preganglionic neurons of the sympathetic system. Spinal cord segments T1 to T4 are often associated with the upper limbs and thoracic organs; the autonomic neurons in spinal cord segments T5 to T9 usually correlate with innervation of organs in the abdominal cavity, specifically organs derived from the foregut.

35. A 22-year-old victim of an automobile accident has received destructive damage to structures that form the boundary of the perineum. Which of the following structures is spared? (A) Pubic arcuate ligament (B) Tip of the coccyx (C) Ischial tuberosities (D) Sacrospinous ligament (E) Sacrotuberous ligament

35. The answer is D. The sacrospinous ligament forms a boundary of the lesser sciatic foramen. The pubic arcuate ligament, tip of the coccyx, ischial tuberosities, and sacrotuberous ligament all form part of the boundary of the perineum.

36 A 34-year-old woman is hospitalized because of an enlarged, painful abdomen. An ultrasound examination is performed and the presence of ascites (fluid) in the peritoneal cavity is confirmed. A needle is placed through the posterior vaginal fornix to drain the fluid. Which space must the needle enter to drain the fluid? A. Rectouterine pouch B. Pararectal fossa C. Paravesical space D. Uterovesical pouch E. Superficial perineal pouch

36 A. The rectouterine pouch (of Douglas) is the lowest point of the woman's peritoneal cavity. Therefore, a fluid collection within the peritoneal cavity accumulates here when the patient is standing or sitting. It is accessible transvaginally through the posterior fornix, with the patient positioned upright. GAS 483; N 340; McM 272

36 A 55-year-old male is admitted to the emergency department with a diagnosis of possible myocardial infarction. Which nerves carry pain fibers from the heart to the CNS? A. Vagus B. Greater thoracic splanchnic C. Least thoracic splanchnic D. Cardiopulmonary (thoracic visceral) E. T5 to T9 ventral rami

36 D. The cardiopulmonary splanchnic (or thoracic visceral) nerves are responsible for carrying the cardiac sympatheticfferent fibers from the sympathetic ganglia to the thoracic viscera and afferent fibers for pain from these organs. The vagus nerve is responsible for carrying parasympathetic fibers. The greater and lesser splanchnic nerves carry sympathetic preganglionic fibers to the abdomen. T1 to T4 ventral rami receive sensory fibers forpain, carried initially by the cardiopulmonary nerves, en route to their respective final destination.

36. A 32-year-old man undergoes vasectomy as a means of permanent birth control. A physician performing the vasectomy by making an incision on each side of the scrotum should remember which of the following statements most applicable to the scrotum? (A) It is innervated by the ilioinguinal and genitofemoral nerves. (B) It receives blood primarily from the testicular artery. (C) Its venous blood drains primarily into the renal vein on the left. (D) Its lymphatic drainage is primarily into upper lumbar nodes. (E) Its dartos tunic is continuous with the perineal membrane.

36. The answer is A. The scrotum is innervated by branches of the ilioinguinal, genitofemoral, pudendal, and posterior femoral cutaneous nerves. The scrotum receives blood from the posterior scrotal branches of the internal pudendal arteries and the anterior scrotal branches of the external pudendal arteries, but it does not receive blood from the testicular artery. Similarly, the scrotum is drained by the posterior scrotal veins into the internal pudendal vein. The lymph vessels from the scrotum drain into the superficial inguinal nodes, whereas the lymph vessels from the testis drain into the upper lumbar nodes. The dartos tunic is continuous with the membranous layer of the superficial perineal fascia (Colles fascia).

37 A 17-year-old girl is admitted to the hospital with severe dyspnea. Physical examination reveals that the patient is suffering from an asthma attack, with associated bronchospasm. Which of the following nerves is responsible for the innervation of the bronchial smooth muscle cells? A. Greater thoracic splanchnic B. Phrenic C. Vagus D. Intercostal E. Lesser thoracic splanchnic

37 C. The vagus nerve is the only nerve responsible for parasympathetic innervation of the lungs. The phrenic nerve and intercostal nerves are somatic nerves and are not involved in innervation of the heart or lungs. The greater thoracic splanchnic and lesser thoracic splanchnic nerves are responsible for carrying preganglionic sympathetic fibers for the innervation of the abdomen. They also carry afferents for pain from the abdomen.

37 A 36-year-old male rodeo rider is admitted to the hospital after being thrown violently from a Brahma bull. An MRI scan reveals rupture of the penile urethra and deep (Bucks) fascia. Where is the most likely place to which extravasated urine will flow? A. Ischioanal fossa B. Rectovesical pouch C. Deep perineal pouch D. Retropubic space E. Superficial perineal cleft

37 E. Because the penile urethra and deep (Buck's) fascia are both located in the superficial perineal pouch, rupture will occur here, with extravasation of fluids into the superficial perineal cleft. The ischioanal fossa is located posterior to the urogenital triangle, behind the area of injury. The other listed spaces are deep to the superficial compartment or within the pelvis and are not associated with the area of injury. GAS 466-468, 512; N 374; McM 266

37. A 37-year-old woman complains of a bearing- down sensation in her womb and an increased frequency of and burning sensation on urination. On examination by her gynecologist, she is diagnosed with a uterine prolapse. Which of the following structures provides the primary support for the cervix of the uterus? (A) External anal sphincter (B) Broad ligament of the uterus (C) Cardinal (transverse cervical) ligament (D) Round ligament of the uterus (E) Suspensory ligament of the ovary

37. The answer is C. The cardinal (transverse cervical) ligament provides the major ligamentous support for the uterus. The sphincter ani externus does not support the uterus. The broad and round ligaments of the uterus provide minor supports for the uterus. The suspensory ligament of the ovary does not support the uterus.

38 A 68-year-old man complains of pain upon urination. A CT scan and a biopsy provide evidence of an enlarged, cancerous prostate gland. Subsequently, he undergoes a radical prostatectomy. Postoperatively, he suffers from urinary incontinence because of paralysis of the external urethral sphincter. Which of the following nerves was injured during the operation? A. Pelvic splanchnic B. Sacral splanchnic C. Pudendal D. Superior gluteal E. Inferior gluteal

38 C. The perineal branch of the pudendal nerve is responsible for the innervation of the external urethral sphincter, and injury to this nerve can result in paralysis of the sphincter and urinary incontinence. Pelvic splanchnic and sacral splanchnic nerves are autonomic nerves that do not supply skeletal muscle. The gluteal nerves innervate gluteal muscles. GAS 491; N 389; McM 266

38 A 42-year-old woman is admitted to the hospital with an inability to speak. The patient's personal history reveals that she has experienced hoarseness for the past month. A chest radiograph reveals a mass at the aortopulmonary window. Which of the following nerves is most likely compressed? A. Vagus B. Phrenic C. Left recurrent laryngeal D. Right recurrent laryngeal E. Greater thoracic splanchnic

38 C. There is close proximity between the aortopulmonary window and the left recurrent laryngeal nerve. A mass within or adjacent to this window is thus likely to compress the left recurrent laryngeal nerve, resulting in the hoarseness for the patient. The greater and lesser thoracic splanchnic nerves arise inferior and posterior to the aortopulmonary window and are thus unlikely to be compressed. The thoracic splanchnic nerves are not involved in the innervation of the larynx. Though the vagus is responsible for innervation of the larynx, it passes dorsal to the area of the aortopulmonary window and is not likely to be compressed.

38. A woman is delivering a breech baby. The obstetrician decides that it is best to perform a mediolateral episiotomy. Which of the following structures should the obstetrician avoid incising? (A) Vaginal wall (B) Superficial transverse perineal muscle (C) Bulbospongiosus (D) Levator ani (E) Perineal membrane

38. The answer is D. An obstetrician should avoid incising the levator ani and the external anal sphincter. The levator ani is the major part of the pelvic diaphragm, which forms the pelvic floor and supports all of the pelvic organs. None of the other choices applies here.

39 Following the diagnosis of breast cancer, a 42-year-old woman underwent a total mastectomy, including excision of the axillary tail (of Spence). Postoperatively, the patient complains of dysesthesia in the inner aspect of the arm and axilla. Which of the following nerves was most likely injured during the procedure? A. Ulnar B. Long thoracic C. Intercostobrachial D. Lateral cutaneous nerve of T4 E. Axillary nerve

39 C. The intercostobrachial nerve is responsible for innervation of the skin on the medial surface of the arm. The ulnar nerve is responsible for cutaneous sensation on the medial aspect of the hand, and the axillary nerve innervates the lateral aspect of the shoulder. The lateral cutaneous branch of T4 innervates the dermatome corresponding to the nipple and areola and also supplies the medial aspect of the axilla. The long thoracic nerve provides motor supply to the serratus anterior and is not involved in cutaneous innervation of the axillary region. Only the intercostobrachial nerve is responsible for sensory supply of the lateral aspect of the axilla.

39 A 65-year-old man with a history of heavy smoking and hypercholesterolemia is diagnosed with severe atherosclerosis, affecting most of the arteries of his body. During the taking of the patient's history he complains also of erectile dysfunction. Compromise of which of the following arteries would most likely be the cause of this condition? A. External iliac B. Inferior epigastric C. Umbilical D. Internal pudendal E. Superficial and deep circumflex

39 D. The internal pudendal artery gives rise to both the dorsal artery and deep artery of the penis. The deep artery is the main supply for erectile tissue; therefore, significant atherosclerosis of the internal pudendal artery may result in impotence (erectile dysfunction). GAS 496-497; N 383; McM 267

39. During pelvic surgery, a surgeon notices severe bleeding from the artery that remains within the true pelvis. Which of the following arteries is most likely to be injured? (A) Iliolumbar artery (B) Obturator artery (C) Uterine artery (D) Internal pudendal artery (E) Inferior gluteal artery

39. The answer is C. Of all the arteries listed, the uterine artery remains within the pelvic cavity.

4 A 2-month-old male infant has epispadias and the bladder mucosa is exposed to the outside. Which of the following is the most likely cause of this condition? A. Failure of the primitive streak mesoderm to migrate around the cloacal membrane B. Failure of urethral folds to fuse C. Insufficient androgen stimulation D. Klinefelter syndrome E. Persistent allantois

4 A. When the urinary bladder mucosa is open to the outside in the fetus or newborn, the condition is referred to as extrophy of the bladder. The extrophy results from failure of the primitive streak mesoderm to migrate around the cloacal membrane, and it occurs often in combination with epispadias. Penile hypospadias is characterized by a failure of fusion of the labioscrotal folds, with the external urethral orifice located between the two unfused halves of the scrotum. Androgens are responsible for development of the testes. Klinefelter syndrome is a condition in which the male has 47,XXY chromosomes. A persistent allantois is associated with a patent urachus and an allantoic cyst. GAS 463; N 348; McM 266

4. A 22-year-old woman receives a deep cut in the inguinal canal lateral to the pubic tubercle. Which of the following ligaments is lacerated within the inguinal canal? (A) Suspensory ligament of the ovary (B) Ovarian ligament (C) Mesosalpinx (D) Round ligament of the uterus (E) Rectouterine ligament

4. The answer is D. The round ligament of the uterus is found in the inguinal canal along its course. The other ligaments do not pass through the inguinal canal.

40 A 58-year-old postmenopausal woman complains of pelvic discomfort and dull pain. Lymph nodes from the sacral, internal iliac, and inguinal lymph node groups are surgically removed for histopathologic examination. The pathology report reveals positive cancerous cells only in the inguinal lymph nodes. Which pelvic organ would most likely be involved in the cancer? A. The body of the uterus B. Distal rectum C. One or both of her ovaries D. Proximal rectum E. Anal canal superior to the pectinate line

40 A. Cancer present in the inguinal nodes can be indicative of cancer of the uterus at the level of the round ligaments, by which the cancerous cells drain to the inguinal region. Uterine cancer must be especially suspected if the tissues of the lower limb, vulva, and anal canal appear normal. The pectinate line marks the end of the mucosal lining of the anal canal, below which the canal is lined with nonkeratinized, stratified squamous epithelium. The pectinate line is also associated with the distal ends of the anal columns and anal valves. Lymphatic vessels inferior to the pectinate line of the anal canal will drain into the superficial inguinal nodes, but those above the pectinate line flow to internal iliac nodes. Lymph from the ovaries flows to the para-aortic nodes at the level of the kidneys. Lymph from the rectum flows to pelvic lymph nodes. GAS 501, 519; N 384; McM 274

40 A 39-year-old male is admitted to the hospital with a complaint of severe retrosternal pain that radiates to the left shoulder. The pain is relieved by leaning forward. Auscultation reveals a pericardial friction rub, leading to a diagnosis of pericarditis. Which of the following nerves is responsible for the radiating pain? A. Intercostobrachial B. Phrenic C. Long thoracic D. Greater thoracic splanchnic E. Cardiopulmonary

40 B. Pericarditis is an inflammation of the pericardium and often causes a pericardial friction rub, with the surface of the pericardium becoming gradually coarser. Because the phrenic nerve is solely responsible for innervation of the pericardium, it would transmit the pain fibers radiating from the pericardial friction rub. The phrenic nerve contains sensory nerve fibers from C3 to C5, spinal nerve levels that also supply the skin of the shoulder area; therefore, pain carried by the phrenic nerve may be referred to the shoulder.

40. A neurosurgeon performs a surgical resection of a rare meningeal tumor in the sacral region. He tries to avoid an injury of the nerve that arises from the lumbosacral plexus and remains within the abdominal or pelvic cavity. To which of the following nerves should he pay particular attention? (A) Ilioinguinal nerve (B) Genitofemoral nerve (C) Lumbosacral trunk (D) Femoral nerve (E) Lateral femoral cutaneous nerve

40. The answer is C. The lumbosacral trunk is formed by part of the ventral ramus of the fourth lumbar nerve and the ventral ramus of the fifth lumbar nerve. This trunk contributes to the formation of the sacral plexus by joining the ventral ramus of the first sacral nerve in the pelvic cavity and does not leave the pelvic cavity. All other nerves leave the abdominal and pelvic cavities.

41 A 72-year-old male is admit ed to the hospital with complaints of severe chest pain radiating to his left arm. ECG examination provides evidence of significant myocardial infarction of the posterior wal of the left ventricle. Which of the fol owing nerves is responsible for the radiation of pain to the arm during myocardial infarction? A. Phrenic B. Vagus C. Intercostobrachial D. Greater splanchnic E. Suprascapular

41 C. The intercostobrachial nerve is the lateral cutaneous branch of the second intercostal nerve. It serves a sensory function both in the thoracic wall and medial aspect of the arm. The phrenic nerve arises from spinal nerves C3 to C5 and innervates the diaphragm. This nerve has no branches that pass into the arm. The vagus nerve is CN X and is a major supplier of autonomic function to the gut, up to the left colic flexure, and also provides some autonomic motor and sensory supply to organs in the head, neck, and thorax. The greater thoracic splanchnic nerve originates in the thorax from the sympathetic chain at the levels of T5 to T9 and innervates abdominal structures. The suprascapular nerve originates from the upper trunk of the brachial plexus and receives fibers primarily from C5 and C6. It innervates the supraspinatus and the infraspinatus.

41 A 34-year-old woman is admitted to the hospital due to severe lower abdominal pain. Radiographic examination reveals an ovarian tumor. Which of the following lymph nodes will most likely become invaded by cancerous cells? A. Superficial inguinal B. External iliac C. Lumbar/lateral aortic D. Deep inguinal E. Internal iliac

41 C. The lymphatic vessels of the ovaries join with lymphatics from the uterine tubes and the fundus of the uterus. These ascend to the right and left lumbar (caval/aortic) lymph nodes. These lymph nodes are the first to receive cancerous cells from the ovaries. Superficial inguinal nodes drain the lower limb, the anterior abdominal wall inferior to the umbilicus, and superficial perineal structures. The external iliac nodes drain the deep inguinal nodes that drain the clitoris and superficial inguinal nodes. The internal iliac nodes drain inferior pelvic structures and deep perineal structures. GAS 501, 519; N 384; McM 362

41. After repair of a ruptured diverticulum, a 31-year-old patient begins to spike with fever and complains of abdominal pain. An infection in the deep perineal space would most likely damage which of the following structures? (A) Ischiocavernosus muscles (B) Superficial transverse perineal muscles (C) Levator ani (D) Sphincter urethrae (E) Bulbospongiosus

41. The answer is D. The sphincter urethrae are found in the deep perineal space, whereas the other structures are located in the superficial perineal space.

42 A 42-year-old woman has a malignancy involving the vestibule of her vagina. Which are the first lymph nodes to filter the lymph drainage from this area and therefore the most likely to become involved in the spread of the tumor? A. Superficial inguinal B. Internal iliac C. Lumbar/lateral aortic D. Presacral lymph E. Axillary lymph

42 A. The superficial inguinal nodes drain superficial perineal structures, including the superolateral uterine body near attachment of the round ligament, skin of the perineum (including the vulva), and the introitus of the vagina inferior to the hymen. The internal iliac nodes drain the middle and upper vagina, cervix, and body of the uterus. The lumbar/lateral aortic lymph nodes drain the ovaries. The axillary lymph nodes drain the upper limb and chest wall, including the breasts. GAS 501, 519; N 385; McM 368

42 A 43-year-old male hunter is admitted to the emergency department after falling over a barbed wire fence, as a result of which he suffered several deep lacerations along the left midaxillary line. When the patient is examined in the outpatient clinic several days later, numbness and anhydrosis are observed anterior to the area of the cuts. Which structures were most likely damaged to result in these signs? A. Dorsal roots B. Ventral roots C. Cutaneous branches of dorsal rami D. Cutaneous branches of ventral rami E. Rami communicans

42 D. Ventral rami contain both sensory and motor fibers and also sympathetics to the body wall, supplying all areas of the body wall except for tissues of the back. In this case both sensory fibers (numbness) and sympathetics (anhydrosis) are disrupted at the midaxillary line; therefore, cutaneous ventral rami is the only correct choice. The dorsal roots carry somatic and visceral sensory information from the periphery. Because only cutaneous sensation is lost the deficit cannot be the dorsal roots. The ventral roots of the spinal cord carry only somatic and visceral efferents. Because no motor functions are disrupted, this is not theorrect answer. The branches of dorsal rami provide cutaneous and postural muscle innervation to the back and thus have no relation to the midaxillary line. The rami communicans are components of the sympathetic nervous system and are not involved with general somatic afferent sensation.

42. A radiologist interprets a lymphangiogram for a 29-year-old patient with metastatic carcinoma. Upper lumbar nodes most likely receive lymph from which of the following structures? (A) Lower part of the anal canal (B) Labium majus (C) Clitoris (D) Testis (E) Scrotum

42. The answer is D. Lymphatic vessels from the testis and epididymis ascend along the testicular vessels in the spermatic cord through the inguinal canal and continue upward in the abdomen to drain into the upper lumbar nodes. The lymph from the other structures drains into the superficial inguinal lymph nodes.

43 A 34-year-old pregnant woman is prepared in the hospital for delivery. The gynecologist decides to perform a pudendal nerve block using a transvaginal approach. Which bony structure would be the most reliable as a landmark to block the pudendal nerve? A. Ischial spine B. Posterior inferior iliac spine C. Ischial tuberosity D. Posterior superior iliac spine E. Coccyx

43 A. The ischial spine is the correct bony landmark used to administer a pudendal nerve block. The pudendal nerve crosses the sacrospinous ligament, which attaches to the ischial spine. Accessing the ischial spine and thus the pudendal nerve is done most easily using a transvaginal approach. The posterior superior and inferior iliac spines are located on the posterior aspect of the pelvis and articulate with the lateral aspect of the sacrum. They do not relate to the course of the pudendal nerve. The ischial tuberosities are the most inferior aspect of the bony pelvis. The skin and soft tissues around the ischial tuberosities receive sensory supply from the pudendal nerve and perineal branches of the posterior femoral cutaneous nerve. Injections into the area around the tuberosities are less certain, however, than injections at the sacrospinous ligament and often fail to anesthetize the anal triangle well. The coccyx is a poor target for locating and anesthetizing the pudendal nerve. GAS 441-445; N 391; McM 92

43 A 62-year-old patient is admitted to the hospital with a complaint of suddenly occurring, tearing pain radiating to his back. A CT examination reveals that the patient has an aortic aneurysm. An urgent placement of an endovascular stent-graft is ordered. Which of the following nerves are most likely responsible for the tearing sensation radiating to his back? A. Somatic afferent B. Thoracic visceral afferent C. Sympathetic postganglionics D. Sympathetic preganglionics E. Parasympathetic afferent

43 D. General visceral afferents are nerve fibers that carry sensation from organs, in this case pain from the abdominal aorta. These fibers get mixed with general somatic afferents in the dorsal roots. This is the phenomenon of "referred pain." The dorsal root ganglia (or their counterparts associated with sensory cranial nerves) contain the cell bodies associ ated with all sensory fibers from the body, including somatic and visceral sensation.

43. A 49-year-old woman has a large mass on the pelvic brim. Which of the following structures is most likely compressed by this mass when crossing the pelvic brim? (A) Deep dorsal vein of the penis (B) Uterine tube (C) Ovarian ligament (D) Uterine artery (E) Lumbosacral trunk

43. The answer is E. All of the listed structures do not cross the pelvic brim except the lumbosacral trunk, which arises from L4 and L5, enters the true pelvis by crossing the pelvic brim, and contributes to the formation of the sacral plexus. The deep dorsal vein of the penis enters the pelvic cavity by passing under the symphysis pubis between the arcuate and transverse perineal ligaments.

44 A 22-year-old woman had undergone elective breast enhancement, with the insertion of 250-ml saline bags bilaterally. This resulted, unfortunately, in loss of sensation bilaterally in the nipples and areolae and some reduction of sensation of the skin from the areolae laterally to the midaxillary lines. Which of the following nerves were most likely subject to iatrogenic injury? A. Anterior cutaneous branches of second and third intercostal nerves B. Anterior and lateral cutaneous branches of the fourth intercostal nerves C. Lateral pectoral nerves D. Cutaneous branches of the second thoracic spinal nerves (intercostobrachial nerves) E. Lateral cutaneous branches of the second and third intercostal nerves

44 B. The anterior and lateral cutaneous branches of the fourth intercostal nerves provide the sensory and sympathetic supply to the areolae and nipples. Anterior cutaneous branches of the second and third intercostal nerves innervate the skin above the nipples and areolae. Lateral pectoral nerves provide motor innervation to the pectoralis major and minor, not sensory supply. Ventral primary rami of the second thoracic spinal nerves provide muscle innervation and sensory innervation above the nipples and areolae and sensory fibers for the medial side of the arm.

44 A 55-year-old man complains of pain at his anus. Examination reveals external hemorrhoids. Which of the following nerves carries pain sensation from the anus? A. Sacral splanchnic B. Superior hypogastric C. Pelvic splanchnic D. Pudendal E. Ilioinguinal

44 D. Hemorrhoids are divided into two categories: internal and external. Pain due to external hemorrhoids is mediated by the pudendal nerve (somatosensory), which serves the majority of the perineum. The sacral splanchnic nerves are postganglionic sympathetic fibers from the sacral sympathetic trunk, and the pelvic splanchnic nerves carry preganglionic parasympathetic fibers and sensory fibers from within the pelvis. Superior hypogastric nerves are mixed nerves located anterior to the sacral promontory and do not mediate pain information from the perineum. The ilioinguinal nerve provides sensory innervation to the skin at the base of the penis; the scrotum; and upper, inner thigh. GAS 504; N 394; McM 266

44. A 26-year-old man comes to a hospital with fever, nausea, pain, and itching in the perineal region. On examination by a urologist, he is diagnosed as having infected bulbourethral (Cowper) glands. Which of the following structures is/are affected by this infection? (A) Superficial perineal space (B) Sphincter urethrae (C) Production of sperm (D) Testis (E) Seminal vesicles

44. The answer is B. The bulbourethral glands lie on either side of the membranous urethra, embedded in the sphincter urethrae. Their ducts open into the bulbous part of the penile urethra. Semen—a thick, yellowish-white, viscous, spermatozoa-containing fluid—is a mixture of the secretions of the testes, seminal vesicles, prostate, and bulbourethral glands. Sperm, or spermatozoa, are produced in the seminiferous tubules of the testis and mature in the head of the epididymis. The seminal vesicles are lobulated glandular structures, produce the alkaline constituent of the seminal fluid that contains fructose and choline, and lie inferior and lateral to the ampullae of the ductus deferens against the fundus (base) of the bladder.

45 A 32-year-old female is admitted to the emergency department with dyspnea, dysphagia, hoarseness, and severe anxiety. Her medical history reveals that she has lived on a liquid diet for some months and has lost more than 30 lb. Over the past several weeks, she has had bloody sputum during attacks of coughing. Fluoroscopy and a barium swallow reveal a 4-cm mass associated with a bronchus and associated compression of the esophagus. Which of the following nerves is most likely to be affected? A. Right recurrent laryngeal nerve B. Left vagus nerve, posterior to the hilum of the lung C. Left recurrent laryngeal nerve D. Greater thoracic splanchnic nerve E. Phrenic nerve

45 C. The left recurrent laryngeal nerve passes superiorly in the tracheoesophageal groove after looping around the aorta. The compression of this nerve and compression of the esophagus against the trachea would result in the presenting symptoms. The right recurrent laryngeal nerve loops around the right subclavian artery before passing toward the larynx and therefore does not descend into the thorax. The left vagus nerve courses posterior to the hilum of the lung, after it has already given off its left recurrent laryngeal branch at the level of the aortic arch; therefore, compression of this nerve would not result in the presenting symptoms. The greater thoracic splanchnic nerve arises from sympathetic chain ganglia at levels T5 to T9 and therefore would not cause the presenting symptoms. The phrenic nerve innervates the diaphragm; compression of this nerve would not result in the presenting symptoms.

45 A 42-year-old woman visits the outpatient clinic due to painful urination. A dipstick test reveals leukocytosis, which confirms the diagnosis of urinary tract infection. Which of the following is the best anatomic explanation for the fact that women are more susceptible to urinary tract infections than men? A. The vagina contains less bacterial flora than the penis B. The prostate gland produces antibacterial prostatic fluids C. The urethra is much shorter in women D. The urethra is located within the vagina E. The seminal vesicles produce fluids resistant to bacteria

45 C. The membranous urethra is shorter in women than in men. Because of its close proximity to the vestibule in women, it commonly leads to infections of the urinary tract. The vagina contains more bacterial flora than the penis. The prostate gland produces a clear, alkaline fluid, but it has not been proved that it protects against bacterial infections. The uterus has no known antibacterial functions. The seminal vesicles produce a fructose-containing fluid that provides nutrients to the sperm for the journey through the female genital tract. GAS 466-468, 512; N 348; McM 272

45. A 21-year-old man is involved in a highspeed motor vehicle accident. As a result, he has extensive damage to his sphincter urethra. Which of the following best describes the injured sphincter urethra? (A) Smooth muscle (B) Innervated by the perineal nerve (C) Lying between the perineal membrane and Colles fascia (D) Enclosed in the pelvic fascia (E) Part of the pelvic diaphragm

45. The answer is B. The sphincter urethra is a striated muscle that lies in the deep perineal space and forms a part of the urogenital diaphragm but not the pelvic diaphragm. It is not enclosed in the pelvic fascia. It is innervated by a deep (muscular) branch of the perineal nerve.

46 A 35-year-old man is admitted to the hospital with pain on swallowing. Imaging reveals a dilated left atrium. Which structure is most likely being compressed by the expansion of the left atrium to result in the patient's symptoms? A. Esophagus B. Root of the lung C. Trachea D. Superior vena cava E. Inferior vena cava

46 A. The patient's chief complaint is pain upon swallowing. With a dilated left atrium, the most probable structure being compressed is the esophagus. The esophagus descends into the abdomen immediately posterior to the left atrium below the level of the tracheal carina. The root of the lung is the site of junction at the hilum where the pulmonary arteries, veins, and bronchi enter or leave. The lung root is not so intimately associated with the esophagus and would not be associated with pain during swallowing. The trachea ends and bifurcates above the level of the left atrium and therefore would be unaffected by a dilated left atrium. The inferior vena cava ascends from the abdomen to the right atrium and the superior vena cava is quite anterior in position. Neither of these veins is closely related to the esophagus or the left atrium.

46 A 52-year-old woman visits the outpatient clinic due to a mass of tissue prolapsing through the vaginal introitus. Physical examination reveals that the patient has a cystocele. Loss of which of the following structures to the anterior part of the vagina is responsible for this problem? A. Pubovesical and vesicocervical fasciae B. Cardinal ligament C. Uterosacral ligament D. Levator ani muscle E. Median umbilical ligament

46 A. The uterus is stabilized and anchored to the bladder by the pubovesical and vesicocervical fasciae on its anterior surface. During pregnancy and childbirth this con nective tissue can be torn, allowing the bladder to herniate into the anterior vaginal wall, with prolapse possible through the vaginal introitus. The transverse cervical (cardinal) ligament is located within the base of the broad ligament and is a major ligament of the uterus but would offer no support if the bladder herniates through the vagina. The uterosacral ligament serves to anchor the uterus to the sacrum for support. Injury to the levator ani would not cause the bladder to herniate through the vagina. The median umbilical ligament contains the urachus and is located on the posterior aspect of the linea alba; the ligament is an embryologic remnant of the allantois. GAS 477; N 343; McM 275

to the children's hospital because he has no testis in his scrotum. During physical examination, the pediatrician palpated the testis in the inguinal canal. What is the diagnosis of this condition? (A) Male pseudohermaphroditism (B) Hypospadias (C) Epispadias (D) Cryptorchid testis (E) Chordee

46. The answer is D. Cryptorchid testis is called an undescended testis, which is located in the inguinal region. Male pseudohermaphroditism is a condition in which the affected individual is a genetic and gonadal male with genital anomalies. Hypospadias occurs when the spongy urethra opens on the underside of the penis, frequently associated with the chordee, which is a ventral curvature of the penis. Epispadias occurs when the urethra opens on the dorsal surface of the penis.

47 A 47-year-old woman with three children and a family history of breast cancer mutations in the BRCA1 and BRCA2 genes has made a decision to have an elective hysterectomy as a prophylactic treatment. During ligation of the uterine artery which of the following adjacent structures is most susceptible to iatrogenic injury? A. Ureter B. Internal iliac artery C. Internal iliac lymph nodes D. Obturator nerve E. Lumbosacral trunk

47 A. During a hysterectomy, ligation of or injury to the ureter can happen relatively easily because it is the most susceptible structure due to its location. The ureter is located immediatelly inferior to the uterine vessels ("water passes under the bridge") in the pelvic cavity approximately 1 cm lateral to the supravaginal cervix. The internal iliac artery bifurcates near the pelvic brim but is not in close proximity to the uterine vessels in the vicinity of the cervix. The obturator nerve travels along the pelvic sidewall and is not close to the site of ligation of the uterine vessels. The lumbosacral trunk is located on the lateral side of the sacrum and the pelvic sidewall, not in close proximity to the uterine vessels. GAS 478; N 378; McM 275

47 A 32-year-old female is admitted to the hospital in a comatose state. Physical examination reveals that the patient suffers from anorexia nervosa. A nasogastric tube is ordered to be inserted. What is the last site at which resistance would be expected as the tube passes from the nose to the stomach? A. Pharyngoesophageal junction B. Level of the superior thoracic aperture C. Posterior to the aortic arch D. Posterior to the left main bronchus E. Esophageal hiatus of the diaphragm

47 E. The esophageal hiatus in the diaphragm is one of four openings associated with the diaphragm. It is located at the level of T10 and allows the esophagus to pass through the thoracic cavity into the abdominal cavity. It is the most inferior of four esophageal constrictions. The pharyngoesophageal junction is the site at which the pharynx ends and the esophagus begins in the neck, at the level of the sixth cervical vertebra. It is the first and the most superior of the esophageal constrictions. There are no constrictions found at the level of the superior thoracic aperture; this is the opening for passage of the structures passing from the neck into the thorax. The esophagus descends posterior to the arch of the aorta. It is at this level that the second of the esophageal constrictions is found. The third constriction occurs as the esophagus passes posteriorly to the left main bronchus.

47. An obstetrician is about to perform a pudendal block so a woman can experience less pain when she delivers her child. He recalls what he learned in medical school about this nerve. Which of the following statements is correct? (A) It passes superficial to the sacrotuberous ligament (B) It innervates the testis and epididymis in a male (C) It provides motor fibers to the coccygeus (D) It can be blocked by injecting an anesthetic near the inferior margin of the ischial spine (E) It arises from the lumbar plexus

47. The answer is D. The pudendal nerve, which arises from the sacral plexus, provides sensory innervation to the labium majus (or scrotum in a male). It leaves the pelvis through the greater sciatic foramen and enters the perineum through the lesser sciatic foramen near the inferior margin of the ischial spine. Therefore, it can be blocked by injection of an anesthetic near the inferior margin of the ischial spine.

48 A 32-year-old woman is admitted to the hospital with a complaint of painful spasms of her vagina. Physical examination reveals several involuntary contractions of the vaginal musculature. The patient also complains of painful intercourse. Which of the following conditions will most likely describe the signs of this patient? A. Vaginismus B. Pudendal nerve compression in the pudendal (Alcock's) canal C. Disruption of the perineal body D. Endometriosis E. Fibroma of the uterus

48 A. Vaginismus is a painful, psychosomatic gynecologic disorder; it is described as involving distension of the cavernous tissues and the bulbospongiosus and transverse perineal muscles, the stimulation of which triggers the involuntary spasms of the perivaginal and levator ani muscles. This can in turn lead to dyspareunia. GAS 480; N 350; McM 276

48 A 59-year-old man is admitted to the hospital with severe chest pain. During examination a slight rhythmic pulsation on the chest wall at the left fifth intercostal space is noted in the midclavicular line. What part of the heart is responsible for this pulsation? A. Right atrium B. Left atrium C. Aortic arch D. Apex of the heart E. Mitral valve

48 D. The apex of the heart is located in the left fifth intercostal space, about 3 1/2 inches to the left of the sternum. When this area of the heart is palpated, any pulsations would be generated by throbbing of the apex of the heart against the thoracic wall. This is also the location for performing auscultation (listening) of the mitral valve, not associated with palpation. The right atrium is located to the right of the sternum. The left atrium is located on the posterior aspect of the heart, thus no direct palpation is realized. The aortic arch would be located posterior to the manubrium of the sternum, above the second intercostal space.

48. A trauma surgeon in the emergency department at a local center examines a 14-year-old boy with extensive pelvic injuries after a hit and run accident. The surgeon inspects the ischiorectal fossa because it (A) Accumulates urine leaking from rupture of the bulb of the penis (B) Contains the inferior rectal vessels (C) Has a pudendal canal along its medial wall (D) Is bounded anteriorly by the sacrotuberous ligament (E) Contains a perineal branch of the fifth lumbar nerve

48. The answer is B. The ischiorectal fossa is bounded posteriorly by the gluteus maximus and the sacrotuberous ligament. It contains fat, the inferior rectal nerve and vessels, and perineal branches of the posterior femoral cutaneous nerve. The pudendal canal runs along its lateral wall. Urine leaking from a ruptured bulb of the penis does not spread into the ischiorectal fossa because Scarpa fascia ends by firm attachment to the fascia lata of the thigh.

49 A 42-year-old male was admitted to the hospital after a head-on vehicular collision in which he received severe blunt trauma to his sternum from the steering wheel. What part of the heart would be most likely to be injured by the impact? A. Right ventricle B. Apex of left ventricle C. Left ventricle D. Right atrium E. Anterior margin of the left atrium

49 A. These components of the heart are readily viewed in a plain radiograph of the thorax. It is important to understand the spatial arrangement of the heart as it rests in the thorax. The conus region of the right ventricle is located on the most anterior aspect of the heart, thus it is the most anterior portion of the heart within the thorax. The apex of the left ventricle is also located anteriorly, but it is located lateral to the sternum and occupies little area compared with the right ventricle. The left ventricle is positioned on the left lateral side and slightly posterior position in the thorax. The right atrium is located on the right lateral side of the heart. The anterior margin of the left atrium is positioned posteriorly in the thorax.

A 42-year-old male sign painter is admitted to the emergency department after falling to the sidewalk from his ladder. Radiologic examination reveals a fracture of the proximal femur. Which of the following arteries supplies the proximal part of the femur? A. Deep circumflex iliac B. Acetabular branch of obturator C. Lateral circumflex femoral D. A branch of profunda femoris E. Medial circumflex femoral

49 D. The second perforating branch of the profunda femoris (deep femoral) artery commonly provides the nutrient artery to the femur, a vessel that passes through a rather large foramen to enter the proximal part of the shaft. The deep circumflex branch of the external iliac passes around the medial aspect of the iliac crest, also supplying the lower lateral part of the anterior abdominal wall. The acetabular branch of the obturator artery supplies tissues in the hip socket, usually including a branch to the ligament of the head of the femur. The lateral circumflex femoral branch of the deep femoral artery supplies the vastus lateralis muscle. The medial circumflex femoral branch of the deep femoral artery supplies proximal adductor musculature and the region of the hip joint, including the neck and head of the femur

49 A 46-year-old man is admitted to the emergency department after a car crash. An MRI examination reveals a hematoma of the perineum spreading to his abdominal wall beneath the superficial fascia. Where should the initial extravasation be located? A. Between the superior aspect of the urogenital diaphragm and the pelvic diaphragm B. Between the perineal membrane and the fascia of Gallaudet C. Between Camper's fascia and Scarpa's fascia D. Between Colles' fascia and Gallaudet's fascia E. Between Buck's fascia and the dartos layer

49 E. This is a classic example of extravasation of blood and urine from the superficial perineal pouch. This usually is a result of rupture of the spongy urethra. The extravasation of the fluid (urine) will begin to invade the layer between the Buck's fascia and the dartos layer. This extravasation example is evident due to the fluid invading up to the abdomen between the subcutaneous tissues and muscle fascia. If the fluid collects between the other layers of the perineum, the clinical evidence will present differently in the perineum and abdominal area. GAS 466-468, 502-504; N 374; McM 266

49. An elderly man with prostatitis is seen at an internal medicine clinic. The seminal colliculus of his prostate gland is infected, and its fine openings are closed. Which of the following structures is/are most likely to be disturbed? (A) Ducts of the prostate gland (B) Prostatic utricle (C) Ducts of the bulbourethral glands (D) Ejaculatory ducts (E) Duct of the seminal vesicles

49. The answer is D. The ejaculatory ducts, which open onto the seminal colliculus, may be injured. The prostate ducts open into the urethral sinus, the bulbourethral ducts open into the bulbous part of the penile urethra, and the ducts of the seminal vesicle join the ampulla of the ductus deferens to form the ejaculatory duct. The prostatic utricle is a minute pouch on the summit of the seminal colliculus.

5 A bifid ureter or paired unilateral ureters result from partial or complete division of which of the following embryologic structures? A. Ureteric bud/metanephric diverticulum B. Mesonephric duct C. Paramesonephric duct D. Metanephric mesoderm E. Pronephros

5 A. The ureteric bud, or metanephric diverticulum, is an outgrowth from the mesonephric duct. It is the primordium of the ureter, renal pelvis, the calyces, and the collecting tubules. Incomplete division results in a divided kidney with a bifid ureter. Complete division results in a double kidney with a bifid ureter, or separate ureters. GAS 379, 462; N 314; McM 267

5. A 29-year-old carpenter sustains severe injuries of the pelvic splanchnic nerve by a deep puncture wound, which has become contaminated. The injured parasympathetic preganglionic fibers in the splanchnic nerve are most likely to synapse in which of the following ganglia? (A) Ganglia in or near the viscera or pelvic plexus (B) Sympathetic chain ganglia (C) Collateral ganglia (D) Dorsal root ganglia (E) Ganglion impar

5. The answer is A. The pelvic splanchnic nerves carry preganglionic parasympathetic general visceral efferent fibers that synapse in the ganglia of the inferior hypogastric plexus and in terminal ganglia in the muscular walls of the pelvic organs. The sympathetic preganglionic fibers synapse in the sympathetic chain (paravertebral) ganglia or in the collateral (prevertebral) ganglia. The dorsal root ganglia contain cell bodies of general somatic afferent (GSA) and general visceral afferent (GVA) fibers and have no synapsis. The two sympathetic trunks unite and terminate in the ganglion impar (coccygeal ganglion), which is the most inferior, unpaired ganglion located in front of the coccyx.

50 A 54-year-old male is admit ed to the hospital with dyspnea. Imaging and physical examination and echocardiographic studies reveal severe mitral valve prolapse. Auscultation of this valve is best performed at which location? A. Left fifth intercostal space, just below the nipple B. Right lower part of the body of the sternum C. Right second intercostal space near the lateral border of the sternum D. Directly over the middle of the manubrium E. Left second intercostal space near the lateral border of the sternum

50 A. The left fifth intercostal space, just below the left nipple, is typically the location to listen to the mitral valve. Although the mitral valve is located at the fourth intercostalpace just to the left of the sternum, the sound is best realized "downstream" from the valve. The right lower part of the body of the sternum is the location of the tricuspid valve. The right second intercostal space near the lateral border of the sternum is the typical location of auscultationof the aortic valve. It is difficult to hear valvular sounds through bone, so auscultating directly over the middle of the manubrium is not a good choice. The left second intercostal space near the lateral border of the sternum is the site chosen typically for auscultation of the pulmonary valve.

50 A 45-year-old woman is admitted to the hospital with lower abdominal and pelvic discomfort. Combined laparoscopic and MRI examinations reveal tears of the ligaments supporting the uterus with moderate uterine prolapse. Which of the following ligaments provides direct support to the uterus and thereby resists prolapse? A. Uterosacral B. Round ligament of the uterus C. Broad ligament D. Arcus tendineus fascia pelvis E. Levator ani muscle

50 A. The uterosacral ligaments and the transverse cervical (cardinal) ligaments are the two main ligaments stabilizing the uterus. They help to inhibit the uterus from prolapsing into the vagina. The round ligament of the uterus is related to the descent of the ovaries in embryologic development and continues into the inguinal canal. This ligament is the remnant of the gubernaculum ovary. The broad ligament is the peritoneal covering over the uterine tubes, uterus, and vessels. The arcus tendineus fascia pelvis joins the muscle fascia of the levator ani to the pubocervical fascia on the vagina and is not directly associated with the uterus or its ligaments. The levator ani muscles contribute to the floor of the pelvis and support all of the pelvic viscera indirectly; it does not, however, prevent prolapse of the uterus into the vagina (GAS Fig. 5-57). GAS 481; N 352; McM 275

50. A general surgeon is giving a lecture to a team of surgery residents. She describes characteristics of structures above the pectinate line of the anal canal, which include (A) Stratified squamous epithelium (B) Venous drainage into the caval system (C) Lymphatic drainage into the superficial inguinal nodes (D) Visceral sensory innervation (E) External hemorrhoids

50. The answer is D. The pectinate line is a point of demarcation between visceral and somatic portions of the anal canal. Characteristics above the pectinate line include columnar epithelium, venous drainage into the portal system, lymphatic drainage into the internal iliac nodes, visceral sensory innervation, and internal hemorrhoids.

51 A 68-year-old man is admitted to the hospital with dysuria, nocturia, urgency, and painful urination. MRI examination reveals enlargement and irregularity of the uvula of the urethra due to prostatic hypertrophy. Laboratory investigation reveals high levels of PSA (prostate-specific antigen) suggesting prostatic carcinoma, and a biopsy is positive for cancer. A prostatectomy is performed. Which of the following lymph nodes should be removed during prostatectomy? A. Internal iliac and sacral B. External iliac C. Superficial inguinal D. Deep inguinal E. Gluteal

51 A. The internal iliac lymph nodes and sacral nodes would be involved in a pelvic lymphadenectomy, which often would be desired in surgical resection for prostate cancer. Sacral lymphatics can communicate with lymphatics within the vertebral canal and thus metastasize cranially. The external iliac nodes drain all of the anterosuperior pelvic structures, the lower limb and perineum, and the body wall to the level of the umbilicus. The superficial inguinal nodes drain all of the superficial structures below the umbilicus. The deep inguinal nodes drain the glans penis in the male. GAS 501, 519; N 386; McM 268

51 A 48-year-old male patient is admitted with chronic angina. Coronary angiography reveals nearly total blockage of the circumflex artery near its origin from the left coronary artery. When this artery is exposed to perform a bypass procedure, what accompanying vein must be protected from injury? A. Middle cardiac B. Great cardiac C. Small cardiac D. Anterior cardiac E. Posterior cardiac

51 B. The great cardiac vein (anterior interventricular vein) takes a pathway initially beside the anterior interventricular coronary artery (left anterior descending: LAD) in its course, finally terminating in the coronary sinus when it is joined by the oblique vein of the left atrium (of Marshall). This vein must be protected when performing bypass procedures. The middle cardiac vein is located on the posterior aspect of the heart, but it also drains into the coronary sinus. The small cardiac vein drains blood along the same path as the right marginal branch. The anterior cardiac veins drain the blood from the right ventricle anteriorly and drain directly into the right atrium, and are not associated with the anterior interventricular artery.

51. A 5-month-old boy is admitted to the children's hospital because of urine being expelled from the dorsal aspect of the penis. Which of the following embryologic structures failed to fuse in this patient? (A) Labioscrotal swellings or folds (B) Urogenital sinus (C) Spongy urethra (D) Phallus (E) Urethral folds

51. The answer is C. A developmental defect in the spongy urethra results in epispadias, causing the patient to pass urine through an opening on the dorsum of the penis. Labioscrotal swellings form the scrotum in males and the labia majora in females. Urogenital sinus forms the urinary bladder, urethra, prostate, and bulbourethral glands in males, and the bladder, urethra, lower vagina, and greater vestibular glands in females. The phallus (genital tubercle) forms the penis in males and the clitoris in females. Urethral (urogenital) folds form the spongy urethra and a portion of the shaft of the penis in males and the labia minora in females.

52 A 55-year-old patient is to undergo a coronary bypass operation. The artery of primary concern is the vessel that supplies much of the left ventricle and the right and left bundle branches of the cardiac conduction system. Which artery is the surgeon most concerned with? A. Right marginal B. Anterior interventricular C. Circumflex D. Artery to the sinoatrial (SA) node E. Posterior interventricular

52 B. The anterior interventricular artery supplies the right and left ventricles and anterior two thirds of the IV septum. The right marginal artery supplies the right ventricle and apex of the heart; therefore, it does not supply the left ventricle. The left coronary circumflex artery supplies the left atrium and left ventricle; it courses posteriorly in, or near to, the coronary sulcus and supplies the posterior portion of the left ventricle and left atrium. The artery to the SA node is a branch of the right coronary artery and does not supply the left ventricle. The posterior interventricular (posterior descending) artery arises from the right coronary artery in 67% of people (this is referred to as a right dominant pattern) and supplies the posterior aspect of both ventricles and the posterior third of the interventricular septum.

52 A 22-year-old man is admitted to the hospital with groin pain and blood in the semen. An MRI examination and biopsy reveal testicular cancer. Which of the following lymph nodes will be first involved in case of metastasis? A. Internal iliac B. External iliac C. Superficial inguinal D. Deep inguinal E. Para-aortic and lumbar

52 E. The para-aortic and lumbar nodes at the level of the kidneys will most likely be infiltrated by metastasis of testicular cancer because testicular lymphatics run in close association with the testicular vessels and drain the testicles and epididymis. Testicular cancer is a disease that is especially dangerous for young men, as intrabdominal lymph node swelling is often a late clinical presentation. The internal iliac nodes drain the inferior pelvis and deep perineal structures. The external iliac nodes drain all anterosuperior pelvic structures. The superficial inguinal nodes drain all of the superficial structures below the umbilicus. Finally, the deep inguinal nodes receive more superficial vessels and drain the glans penis in males. GAS 501, 519; N 386; McM 362

52. A 78-year-old man has carcinoma of the rectum. The cancer is likely to metastasize via the veins into which of the following structures? (A) Spleen (B) Kidney (C) Liver (D) Duodenum (E) Suprarenal gland

52. The answer is C. Cancer cells from rectal cancer are likely to metastasize to the liver via the superior rectal, inferior mesenteric, splenic, and portal veins. Cancer cells are not directly spread to the other organs listed. The spleen and duodenum drain their venous blood to the portal venous system, and the kidney and suprarenal gland empty into the caval (inferior vena cava) system.

53 A 68-year-old man underwent a radical prostatectomy. Six months postoperatively the patient complains of being incapable of penile erection without the use of sildenafil (Viagra). Which nerve was most probably damaged during the operation? A. Pudendal B. Perineal C. Pelvic splanchnic D. Sacral splanchnic E. Dorsal nerve to the penis

53 C. Penile erection is a parasympathetic mediated response that is delivered via the pelvic splanchnic nerves that pass through nerve bundles on the posterolateral aspect of the prostate gland. (In prostatectomy, these bundles should be left intact, if at all possible, to avoid erectile dysfunction, also known as impotence.) The pudendal nerve and its terminal branch, the dorsal nerve of the penis, carry the primary skeletal motor and sensory innervation to the external genitalia, and also sympathetic fibers. Sacral splanchnic nerves contain sympathetic fibers. GAS 492-494, 515; N 394; McM 269

53 A 58-year-old patient presents himself to the emergency department with severe angina. Upon cardiac catheterization, it is found that he has a significant occlusion in his right coronary artery, just distal to the right sinus of the aortic valve. His collateral cardiac circulation is minimal. Assuming the patient is right coronary dominant, which of the following arteries would be most likely to still have normal blood flow? A. Right (acute) marginal artery B. Atrioventricular nodal artery C. Posterior interventricular artery D. Sinoatrial nodal artery E. Anterior interventricular artery

53 E. The anterior interventricular artery arises from the left coronary artery. If there is occlusion in the right coronary artery, the anterior interventricular artery will still have normal blood flow. The right marginal artery branches from the right coronary artery; therefore, if there is occlusion of the right coronary artery, flow from the marginal artery will be compromised. The AV nodal artery is supplied by the coronary artery that crosses the crux of the heart posteriorly. If this artery arises from the right coronary, supply to the AV node might be reduced, depending upon collateral supply. The SA nodal artery is supplied by the right coronary artery in 55% of the population (only 35% from the left); inasmuch as it is stated that the patient is right coronary dominant, it would be predicted that the SAodal artery will not have normal blood flow.

53. During a partial hysterectomy leaving the ovaries in tact, the surgeon detaches the ovary from the uterus by transecting the ovarian ligament. This ligament developed from which embryonic structure? (A) Mesonephric duct (B) Urogenital folds (C) Gubernaculum (D) Processus vaginalis (E) Paramesonephric ducts

53. The answer is C. The ovarian ligament and the round ligament of the uterus are formed by the gubernaculum. The mesonephric duct gives rise only to the vestigial epoophoron in the female. The urogenital folds form the labia minora. The processus vaginalis forms no adult female structures, while the paramesonephric ducts form the uterine tubes, uterus, cervix, and upper vagina.

54 A 15-year-old boy is admitted to the emergency department 2 days after crashing his bicycle. MRI examination reveals severe edema of the boy's scrotum and abdominal wall and extravasated urine. Which of the following structures is most likely ruptured? A. Spongy urethra B. Preprostatic urethra C. Prostatic urethra D. Urinary bladder E. Ureter

54 A. Rupture of the spongy urethra leads to accumulation of fluid (edema) in the superficial perineal cleft. The continuity of Colles' fascia (superficial membranous layer of the superficial perineal fascia) with Scarpa's fascia of the abdominal wall allows for fluid spread upward upon the body wall. Rupture of the preprostatic urethra, prostatic urethra, or urinary bladder would lead to internal fluid accumulation within the pelvis because they are not located in the perineum. Damage to the ureter would manifest within the abdomen or pelvis, depending upon the level of rupture (GAS Fig. 5-46). GAS 466-468, 512; N 369; McM 266

54 A 55-year-old male is admitted to the emergency department with severe chest pain. Coronary angiography reveals that the patient's right coronary artery is free of pathology. The left coronary artery is found to be 70% to 80% occluded at three points proximal to its bifurcation into the circumflex and left anterior descending arteries. Having a left dominant coronary circulation, and without surgery, what is the most likely explanation for a poor prognosis for recovery of this patient to a normally active life? A. All the branches of the coronary artery are end arteries, precluding the chance that anastomotic connections will occur. B. It is probable that the anterior and posterior papillary muscles of the tricuspid valve have been damaged. C. The blood supply of the SA node is inadequate. D. The development of effective collateral circulation between anterior and posterior interventricular arteries will not be possible. E. The blood supply of the AV node will be inadequate.

54 D. Because the patient is left coronary artery dominant, if there is 70% to 80% occlusion of the left coronary, there will be deficiencies in flow both in the anterior descending and circumflex coronary arteries. No possibility is available for collateral flow from the posterior descending interventricular artery, for it too would be derived from the left coronary, by way of the circumflex artery. If the patient does not undergo surgery to remove or bypass the occlusion, he will be unable to have any substantial type of collateral circulation between the two major branches of the left coronary. The branches of the coronary arteries are not end arteries, and there are anastomoses between them. The papillary muscles of the tricuspid valve would not be affected with left coronary artery occlusion. The blood supply to the SA node would not be inadequate. The blood supply of the region of the AV node might or might not be adequate, for it could still be supplied by a branch of the right coronary artery.

Questions 54 to 58: Choose the appropriate lettered structure in this magnetic resonance image (see Figure below) of the female perineum and pelvis. 54. Which structure extends between the vestibule and the cervix of the uterus and serves as the excretory channel for the products of menstruation?

54. The answer is D. The vagina is the genital canal in the female, extending from the vestibule to the uterine cervix. The vagina transmits the products of menstruation and receives the penis in copulation.

55 A 35-year-old woman is admitted to the hospital with dyspnea. During physical examination her S1 heart sound is very loud. Which of the following valves is most likely defective? A. Mitral valve B. Aortic C. Pulmonary D. Aortic and pulmonary E. Tricuspid

55 A. The mitral valve corresponds to the S1 heart sound produced during systole. The aortic and pulmonary valves correspond to the S2 heart sound produced during diastole. The tricuspid valve also corresponds with the S1 heart sound. The aortic valve, however, corresponds with the S2 sound, so this answer would be incorrect.

55 A 35-year-old man is admitted to the surgery ward for correction of an inguinal hernia. During physical examination an indirect hernia is noted, together with an absent cremasteric reflex. Which of the following nerves carries the efferent component of the cremasteric reflex? A. Ilioinguinal nerve B. Genital branch of genitofemoral C. Iliohypogastric nerve D. Pudendal nerve E. Obturator nerve

55 B. The cremasteric reflex afferents are carried by the ilioinguinal nerve; the motor (efferent) output is by the genitofemoral nerve. The sensory fibers of the genitofemoral nerve are to skin over the femoral triangle. The ilioinguinal nerve is sensory to parts of the suprapubic region, anterior perineum, and inner thigh. The iliohypogastric nerve provides sensation for the abdominal wall and suprapubic area. The pudendal and obturator nerves do not travel through the inguinal canal and would not be damaged by the hernia. In addition, they play no role in the cremasteric reflex. GAS 492-494, 515; N 387; McM 262

55. Which structure in the female is much shorter than the corresponding structure in the male?

55. The answer is A. In females, the urethra extends from the bladder, runs above the anterior vaginal wall, and pierces the urogenital diaphragm to reach the urethral orifice in the vestibule behind the clitoris. It is approximately 4 cm long. In males, the urethra is approximately 20 cm long.

56 A 19-year-old woman is admitted to the hospital with low blood pressure and intense pelvic pain. Physical examination reveals heavy blood loss during this menstrual period. Speculum examination reveals irritation of the cervix of the uterus. Which of the following nerves conveys sensory fibers from the cervix of the uterus? A. Pudendal B. Superior hypogastric C. Pelvic splanchnic D. Sacral splanchnic E. Lesser splanchnic

56 C. Pain from the cervix is transmitted via the pelvic splanchnic nerves because the cervix is below the inferior limit of the peritoneum, which is also known as the pelvic pain line. Pain above the pelvic pain line is carried via nerves that are primarily sympathetic in function. The superior hypogastric nerves carry pain fibers from the upper portions of the uterus. Sacral splanchnic nerves are principally sympathetic in function. The pudendal nerve contains skeletal motor, sensory, and sympathetic fibers and provides primary sensory innervation to external genitalia, including the lower third of the vagina. GAS 492-494, 515; N 393; McM 269

56 A 72-year-old male is admitted to the hospital with severe chest pain. ECG examination provides evidence of severe myocardial infarction of the lower part of the muscular interventricular septum. The function of which of the following valves will be most severely affected? A. Pulmonary B. Aortic C. Tricuspid D. Mitral E. Eustachian

56 C. The interventricular septum is intimately involved with the tricuspid valve on the right side, via the muscular connections of the septomarginal trabeculum (moderator band) to the anterior papillary muscle. Therefore, if the electrical system of the heart is disrupted, as with a myocardial infarction in the upper portion of the muscular septum, the innervation of the interventricular septum will be compromised and the tricuspid valve will be directly affected. None of the other valves are directly involved with the interventricular septum.

56. Into which structure does hemorrhage occur after injury to the inferior rectal vessels?

56. The answer is E. The ischiorectal fossa lies in the anal triangle and is bound laterally by the obturator internus with its fascia and superomedially by the levator ani and external anal sphincter. It contains the inferior rectal vessels. Thus, hemorrhage occurs in the ischiorectal fossa when it is ruptured.

57 A 38-year-old woman visits her gynecologist for a routine Pap smear examination. During the collection of cells from her uterine cervix she feels a mild pain. Which of the following areas is most likely to experience "referred pain" during this procedure? A. Perineum and lateral portion of the thigh B. Suprapubic region C. Umbilical region D. Inguinal region E. Epigastric region

57 A. Pain from this area is mediated via parasympathetic responses and would thus travel to the S2 to S4 levels through the pelvic splanchnic nerves. The S2, S3, and S4 spinal cord levels also provide sensory innervation of the perineum and posterior thigh. The suprapubic and inguinal regions are supplied by ilioinguinal and iliohypogastric nerves (L1). The umbilical region receives sensory innervation from the T10 level. In the epigastric region the sensory innervation is provided by T7 to T10. GAS 492-494, 515; N 393; McM 272

57 A 35-year-old woman is admitted to the hospital with a complaint of shortness of breath. During physical examination it is noted that there is wide splitting in her S2 heart sound. ECG reveals a right bundle branch block. Which of the following valves is most likely defective? A. Mitral valve B. Pulmonary C. Aortic and mitral D. Tricuspid E. Tricuspid and aortic

57 B. The pulmonary valve is associated with the S2 heart sound produced in diastole. A splitting in the S2 sound indicates that the aortic and pulmonary valves are not closing simultaneously and would correlate with a possible defect in this valve. The mitral valve is associated with the S1 heart sound, produced in systole; therefore, it cannot be defective if only the S2 sound is involved. The aortic valve is associated with the S2 heart sound, but the mitral valve is not (as stated earlier); therefore, this answer cannot be correct. The tricuspid valve is associated with the S1 heart sound and therefore is not associated with the occurrence of an abnormal S2 heart sound.

57. Which structure has a Houston valve or fold, with its venous blood drained by the portal venous system?

57. The answer is C. The mucous membrane and the circular smooth-muscle layer of the rectum form three transverse folds; the middle one is called Houston valve. The venous blood returns to the portal venous system via the superior rectal vein.

58 A 35-year-old man is admitted to the hospital after being kicked in the groin while playing football. During physical examination it is noted that the left testicle of the patient is swollen. An MRI examination reveals coagulation of blood in the veins draining the testis. Into which of the following veins would a thrombus most likely pass first from the injured area? A. Inferior vena cava B. Left renal vein C. Left inferior epigastric D. Left internal pudendal E. Left iliac vein

58 B. The left testicular vein drains directly into the left renal vein, which then crosses over the midline to enter the inferior vena cava. The left inferior epigastric, left internal pudendal, and left iliac veins are not involved in the drainage of the testes. GAS 495-500; N 379; McM 261

58 A 3-month-old infant is diagnosed with a membranous ventricular septal defect. A cardiac operation is performed, and the septal defect is patched inferior to the noncoronary cusp of the aorta. Two days postoperatively the infant develops severe arrhythmias affecting both ventricles. Which part of the conduction tissue was most likely injured during the procedure? A. Right bundle branch B. Left bundle branch C. Bundle of His D. Posterior internodal pathway E. Atrioventricular node

58 C. The bundle of His is a collection of specialized cardiac muscle cells that carry electrical activity to the right and left bundle branches. Because both ventricles are affected, this is the logical site of injury, for this bundle leads to the bundle branches supplying both ventricles. An injury either to the right or left bundle branch would affect only one ventricle. Terminal Purkinje fibers transmit the electrical activity to the greater sections of the ventricles, yet dysfunction in the terminal part of the conduction system would affect only a small section of one ventricle, not both. The atrioventricular node is a group of specialized cardiac muscle cells that serve to decrease the rate of conduction to the ventricles and is located in the region deep to the septal wall of the right atrium. The posterior internodal pathway is in the roof of the right atrium and is not involved here.

58. Which structure is innervated by the nerve passing through both the greater and lesser sciatic foramina?

58. The answer is B. The obturator internus muscle and its fascia form the lateral wall of the ischiorectal fossa. This muscle is innervated by the nerve to the obturator internus, which passes through the greater and lesser sciatic foramen.

59 A 32-year-old woman is admitted to the hospital due to a palpable mass superior to the inguinal ligament. It is demonstrated by physical examination that she suffers from an indirect inguinal hernia. Due to the severity of the hernia, an open hernia repair is performed. Which of the following structures would the surgeon expect to find in the canal? A. Round ligament of uterus B. Urachus C. Suspensory ligament of the ovary D. Uterine tube E. Mesosalpinx

59 A. During repair, the round ligament of the uterus may be seen within the inguinal canal, although it is often a small, fibrous strand that is easily overlooked. The remaining choices are not found in this region. The ovarian ligament connects the ovary to the uterus, whereas the suspensory (infundibulopelvic) ligament contains the ovarian vessels, nerves, and lymphatic. The uterine tubes are lateral projections of the uterus toward the ovaries. The mesosalpinx is a portion of the peritoneum of the broad ligament that attaches to the uterine tubes. GAS 476-479; N 350, 351; McM 272

59 A 62-year-old male was admitted to the hospital with intense left chest pain. ECG and echocardiography reveal myocardial infarction and pulmonary valve regurgitation. Emergency coronary angiography is performed and provides evidence that the artery supplying the upper portion of the anterior right ventricular free wall is occluded. Which of the following arteries is most likely to be occluded? A. Circumflex B. Anterior interventricular artery C. Posterior interventricular artery D. Artery of the conus E. Acute marginal branch of the right coronary artery

59 D. The artery of the conus is given off from the right coronary artery and winds around the conus arteriosus. The conus region is the superior part of the right ventricle that tapers into a cone (infundibulum) where the pulmonary valve leads into the pulmonary trunk. This conus artery supplies the upper portion of the anterior right ventricle and usually has a small anastomotic connection with the anterior interventricular (left anterior descending) branch of the left coronary artery. The circumflex artery supplies the left atrium and ventricle and does not supply the right ventricle except when the posterior interventricular (posterior descending) artery arises from the circumflex, or in unusual cases in which the circumflex passes to the surface of the right ventricle. The anterior interventricular artery supplies the right and left ventricles and the anterior two thirds of the IV septum. It is given off by the left coronary artery and does not specifically supply the upper portion of the right ventricle. The posterior interventricular artery supplies the right and left ventricles and the posterior third of the IV septum. It does not supply the upper portion of the right ventricle.

Questions 59 to 63: Choose the appropriate lettered structure in this computed tomography scan (see Figure below) of the male perineum and pelvis. 59. Which structure, when fractured, results in paralysis of the obturator internus muscles?

59. The answer is E. The greater trochanter provides an insertion site for the obturator internus muscle.

6 A 16-year-old woman is visiting her gynecologist for her first checkup. On ultrasound examination it is noted that the woman has a double uterus. Failure of which of the following processes is responsible for the double uterus? A. Fusion of the inferior parts of the paramesonephric ducts B. Fusion of the superior parts of the mesonephric ducts C. Development of the hymen D. Development of the sinovaginal bulbs E. Fusion of the inferior parts of the mesonephric ducts

6 A. Failure of fusion of the inferior parts of the paramesonephric (Müllerian) ducts results in a double uterus. A bicornuate uterus is the result of failure of fusion of the superior parts of the paramesonephric ducts. A failure of the sinovaginal bulbs to form the vaginal plate causes agenesis of the vagina. The mesonephric ducts are important embryologic structures involved in the development of male urogenital structures. GAS 379, 462; N 367; McM 274

6. A 59-year-old woman comes to a local hospital for uterine cancer surgery. As the uterine artery passes from the internal iliac artery to the uterus, it crosses superior to which of the following structures that is sometimes mistakenly ligated during such surgery? (A) Ovarian artery (B) Ovarian ligament (C) Uterine tube (D) Ureter (E) Round ligament of the uterus

6. The answer is D. The ureter runs under the uterine artery near the cervix; thus, the ureter is sometimes mistakenly ligated during pelvic surgery. The other structures mentioned are not closely related to the uterine artery near the uterine cervix.

60 A 32-year-old woman is admitted to the hospital with intense lower abdominal pain and an elevated temperature. Upon physical examination it is readily observed that the abdominal wall is tender to the touch. Blood tests reveal leukocytosis and a diagnosis of peritonitis is made. An exploratory laparotomy reveals a ruptured ectopic pregnancy. Which of the following is the most common site of an ectopic pregnancy? A. Over the internal cervical os B. Wall of the bowel C. Uterine tube D. Mesentery of the bowel E. Surface of the ovary

60 C. The most common site of ectopic pregnancy is the uterine (fallopian) tube. Implantation at the internal cervical os would be within the uterus and lead to placenta previa. The other choices are less common sites of ectopic pregnancies. GAS 530; N 352; McM 274

60 A 3-month-old male infant died unexpectedly in his sleep. The pathologist examined the histologic slides of tissue samples taken from the heart of the infant and observed that a portion of the conduction tissue that penetrates the right fibrous trigone had become necrotic. As a result, a fatal arrhythmia probably developed, leading to the death of the infant. Which of the following parts of the conduction tissue was most likely interrupted? A. Right bundle branch B. The bundle of Bachmann C. The left bundle branch D. The atrioventricular bundle of His E. The posterior internodal pathway

60 D. The atrioventricular bundle of His is a strand of specialized cardiac muscle fibers that arises from the atrioventricular node and passes through the right fibrous trigone. The right fibrous trigone (central fibrous body) is a dense area of connective tissue that interconnects the mitral, tricuspid, and aortic valve rings. After reaching the upper portion of the muscular interventricular septum, the bundle of His splits into right and left bundle branches. The bundle of Bachmann is a collection of fibers running from the sinoatrial node to the left atrium and is the only collection of conducting fibers to innervate the left atrium. Finally, the posterior internodal pathway, also known as Thorel's pathway, is the principal pathway of electrical activation between the sinoatrial node and atrioventricular node in humans.

60. Which structure secretes fluid containing fructose, which allows for forensic determination of rape?

60. The answer is B. The seminal vesicle is a lobulated glandular structure and produces the alkaline constituent of the seminal fluid, which contains fructose and choline. Fructose, which is nutritive to spermatozoa, also allows forensic determination of rape, whereas choline crystals are the preferred basis for the determination of the presence of semen.

61 A 42-year-old woman is admitted to the hospital after blunt trauma to her sternum by the steering wheel during a car crash. Radiographic examination reveals a cardiac tamponade. ECG data indicate that the heart has been severely injured. Which of the following cardiac structures will most likely be injured? A. Right ventricle B. Obtuse margin of the left ventricle C. Right atrium D. Left atrium E. Apex of the left ventricle

61 A. The sternocostal surface of the heart consists mostly of the right ventricle. Therefore, an anterior injury to the thorax would mostly likely first affect the right ventricle because it is adjacent to the deep surface of the sternum.

61 A 32-year-woman is admitted to the emergency department as a rape case. Fluids from her vagina are collected for DNA and fructose examination. Which of the following male organs is responsible for fructose production? A. Prostate gland B. Seminal vesicles C. Kidneys D. Testis E. Bulbourethral (Cowper) glands

61 B. Seminal vesicles produce the alkaline portion of the ejaculate. This includes fructose and choline. The prostate gland secretes prostaglandins, citric acid, and acid phosphatase. The kidneys are the sites of urine production. The testes produce spermatozoa and sex hormones. The bulbourethral glands (Cowper's glands) produce mucous secretions that enter the penile bulb. GAS 473; N 362; McM 268

61. In which structure would ligation of the external iliac artery reduce blood pressure?

61. The answer is A. The external iliac artery becomes the femoral artery immediately after passing the inguinal ligament. Therefore, ligation of the external iliac artery reduces blood pressure in the femoral artery.

62 A 1-year-old male infant is admitted to the pediatric clinic because his parents could not palpate his testes in the scrotum. The pediatrician examines the infant and palpates the testes in the inguinal canal. Which of the following best describes this condition? A. Pseudohermaphroditism B. True hermaphroditism C. Cryptorchidism D. Congenital adrenal hyperplasia E. Chordee

62 C. In cryptorchidism, the testis has failed to descend into its proper location in the scrotum and may be found within the abdomen or in the inguinal canal. GAS 470; N 364; McM 266

62 A 69-year-old male is admitted to the hospital with intense left chest pain. ECG reveals hypokinetic ventricular septal muscle, myocardial infarction in the anterior two thirds of the interventricular septum, and left anterior ventricular wall. The patient's ECG also exhibited left bundle branch block. Which of the following arteries is most likely occluded? A. Circumflex B. Proximal right coronary C. Proximal left coronary D. Proximal left anterior interventricular artery E. Posterior interventricular artery

62 D. The tissues affected in this case, the interventricular septum and anterior ventricular wall, are mostly supplied by the proximal portion of the left anterior interventricular artery. If the circumflex artery were blocked, the left atrium and left ventricle would be affected (in a right coronary dominant heart). If the right coronary artery were occluded, again assuming right coronary dominance, it would affect the right atrium, the sinoatrial and atrioventricular nodes, part of the posterior left ventricle, and the posterior part of the interventricular septum. If the left coronary artery (LCA) were blocked, most of the left atrium and left ventricle, the anterior two thirds of the interventricular septum, and the area of bifurcation of the bundle of His would be affected. If the posterior interventricular artery were occluded, it would affect the right and left ventricles and the posterior third of the interventricular septum. The circumflex and the anterior interventricular arteries are branches of the LCA, and the posterior interventricular artery is most commonly a branch of the terminal segment of the right coronary artery.

A 17-year-old boy is injured in an automobile accident. He has a fracture of the shaft of the humerus. 62. After this accident, supination is still possible through contraction of which of the following muscles? (A) Supinator (B) Pronator teres (C) Brachioradialis (D) Biceps brachii (E) Supraspinatus

62. The answer is D. A lesion of the radial nerve causes paralysis of the supinator and brachioradialis. The biceps brachii muscle is a fl exor of the elbow and also a strong supinator; thus, supination is still possible through action of the biceps brachii muscle. Other muscles cannot supinate the forearm.

62. A knife wound to the obturator foramen might injure which structure?

62. The answer is D. The obturator foramen transmits the obturator nerve and vessels. Therefore, the knife wound in this foramen injures the obturator nerve and vessels.

63 A 49-year-old woman is admit ed to the hospital complaining of severe, crushing, retrosternal pain during the preceding hour. An ECG reveals that she is suffering from acute myocardial infarction in the posterior aspect of her left ventricle and posteromedial papillary muscle. A coronary angiogram is performed and the patient is found to have left coronary dominant circulation. Which of the following arteries is the most likely to be occluded? A. Artery of the conus B. Right coronary artery C. Circumflex D. Right acute marginal E. Left diagonal

63 C. A "left coronary dominant" circulation means, most simply, that the left coronary artery (LCA) provides the posterior interventricular artery as a terminal branch of the coronary circumflex. The posterior aspect of the heart is composed primarily of the left ventricle and is supplied by the posterior interventricular branch. The artery of the conus supplies the right ventricular free wall. If the right coronary artery were occluded (in a right coronary dominant heart), it would affect the right atrium, right ventricle, the sinoatrial and atrioventricular nodes, the posterior part of the interventricular septum, and part of the posterior aspect of the left ventricle. The right acute marginal artery supplies the inferior margin of the right ventricle. The left diagonal arteries arise most commonly from the anterior interventricular (left anterior descending) artery but can also arise as branches of the left coronary or the circumflex.

63 A 38-year-old pregnant woman is admitted to the emergency department with severe vaginal bleeding. Ultrasound examination confirms the initial diagnosis of placenta previa. What is the site of implantation in placenta previa? A. Uterine (fallopian) tubes B. Cervix C. Mesentery of the abdominal wall D. Lower part of uterine body, overlapping the internal cervical os E. Fundus of the uterus

63 D. By definition, the site of implantation in placenta previa overlaps the internal cervical os. Ectopic pregnancy in the uterine (fallopian) tubes results in tubal pregnancy. The fundus of the uterus is the normal site of implantation. Implantation within the mesenteries of the abdomen will result in an abdominal pregnancy. The cervix is not a notable site of ectopic implantation. GAS 530; N 352; McM 272

63. A stab wound immediately superior to the pubic symphysis on the anterior pelvic wall would most likely injure which visceral organ first?

63. The answer is C. The bladder is situated in the anterior part of the pelvic cavity. Thus, a stab wound superior to the pubic symphysis would injure the bladder.

64 A 75-year-old man is scheduled for his routine annual medical examination. During echocardiographic examination a large, mobile structure resembling a thrombus is identified in the right atrium near the opening of the inferior vena cava. After careful examination the doctor identifies the large mobile structure as a normal component of the heart. Which of the following structures could most likely resemble a thrombus in this location? A. Tricuspid valve B. Eustachian valve C. Thebesian valve D. Septum primum E. Fossa ovalis

64 B. The eustachian valve is an embryologic remnant of the valve of the inferior vena cava and is not a functional valve. The tricuspid valve is located below the inferior vena cava between the right atrium and right ventricle. The fossa ovalis is an embryonic remnant of the septum primum of the interatrial septum, located interatrially. The Thebesian valve is a semicircular fold at the orifice of the coronary sinus.

64 A 63-year-old man is admitted to the hospital with ascites, rectal bleeding, and an enlarged cirrhotic liver. He is overweight and has a history of alcohol abuse. Upon clinical examination it is found he is suffering from internal hemorrhoids. Which of the following best describes the nerves containing the pain fibers from his hemorrhoids? A. The pain fibers are carried by the inferior rectal nerve B. The pain fibers are carried by the perineal nerve C. The pain fibers are carried by the obturator nerve D. The patient would not experience pain because this area is innervated by visceral afferent fibers E. The patient would not experience well localized pain because this area is innervated by pelvic splanchnic nerves

64 D. Internal hemorrhoids are located above the pectinate line. This tissue is derived from the hindgut and innervated by visceral nerves. Pain is not a common symptom of internal hemorrhoids. GAS 504; N 377; McM 268

64. If the surgical neck of the humerus is fractured, which of the following structures is most likely injured? (A) Musculocutaneous nerve (B) Radial nerve (C) Deep brachial artery (D) Posterior humeral circumfl ex artery (E) Scapular circumfl ex artery

64. The answer is D. Fracture of the surgical neck of the humerus occurs commonly and damages the axillary nerve and the posterior humeral circumfl ex artery.

65 A 4-year-old male is operated on for a correction of a small, muscular interventricular septal defect. To access the right side of the intraventricular septum, a wide incision is first made in the anterior surface of the right atrium. Instruments are then inserted through the tricuspid valve to correct the ventricular septal defect. Which of the following structures is the most crucial to protect during the opening of the right atrium? A. Crista terminalis B. Pectinate muscles C. Tricuspid valve D. Eustachian valve E. Coronary sinus

65 A. The crista terminalis is a muscular ridge that runs from the opening of the superior vena cava to the inferior vena cava. This ridge provides the path taken by the posterior internodal pathway (of Thorel) between the sinoatrial and atrioventricular nodes. The crista also provides the origin of the pectinate muscles of the right auricle. The tricuspid valve is located below the inferior vena cava, between the right atrium and right ventricle. The eustachian valve is an embryologic remnant of the valve of the inferior vena cava. The ostium of the coronary sinus is located between the right atrioventricular orifice and the inferior vena cava.

65 A 34-year-old man is admitted to the emergency department after a traumatic landing into a swimming pool from a high diving platform. The patient has multiple traumas in his abdominal cavity. After a reconstructive operation of his abdominal organs the patient develops a high fever. Radiographic examination reveals that the lower portion of the descending colon and rectum has become septic and must be excised. Six months postoperatively the patient complains of impotence. Which of the following structures was most likely injured during the second operation? A. Pudendal nerve B. Sacral splanchnic nerves C. Pelvic splanchnic nerves D. Sympathetic trunk E. Vagus nerve

65 C. The pelvic splanchnic nerves (nervi erigentes) contain parasympathetic efferent fibers that mediate erection. These same nerves innervate the hindgut, the portions of the large intestine that were removed in this patient. GAS 492-494, 515; N 394; McM 269

66 A 32-year-old woman visits her gynecologist for a routine examination. The Pap smear reveals atypical cervical cells, indicating the possible existence of cervical cancer. Which of the following lymph nodes need to be biopsied to confirm the existence of initial metastasis from the suspected cancerous tumor? A. Internal iliac B. External iliac C. Superficial inguinal D. Deep inguinal E. Sacral

66 A. The internal iliac nodes are the first in a trunk of lymph nodes that receive lymph from the uterine cervix. Cancerous cells from the cervix are likely to involve the internal iliac nodes first. If these nodes do not have cancerous cells, this indicates that the tumor has not spread, at least through lymphatic channels. Presacral nodes are also sometimes involved and can be felt during a digital rectal examination. GAS 501, 519; N 384; McM 366

66 A 52-year-old patient is admitted to the hospital with severe chest pain. ECG and radiographic examinations provide evidence of a significant myocardial infarction and cardiac tamponade. An emergency pericardiocentesis is ordered. At which of the following locations will the needle best be inserted to relieve the tamponade? A. Right seventh intercostal space in the midaxillary line B. Left fifth intercostal space in the midclavicular line C. Right third intercostal space, 1 inch lateral to the sternum D. Left sixth intercostal space, 1/2 inch lateral to the sternum E. Triangle of auscultation

66 B. During pericardiocentesis, the needle is inserted below the xiphoid process, or in the left fifth intercostal space in the midclavicular line. The most effective way of draining the pericardium is by penetrating the thoracic wall at its lowest point anatomically, hence the third intercostal space would be too cranial in position. The sixth and seventh intercostal spaces are locations that are not used clinically because of the increased likelihood of injury to the pleura or lungs and other complications.

67 A 55-year-old man is brought to the emergency department after his motorcycle collided with an automobile. He is hypotensive, his pulse is irregular, and he shows other signs of substantial blood loss. MRI and CT scan evaluations reveal profuse abdominal bleeding. A decision is made to enter the chest so that the descending thoracic aorta can be clamped to minimize blood loss and to preserve cerebral blood flow. After surgical entrance into the thorax, the fibrous pericardium is elevated with a forceps and punctured. A midline, longitudinal incision of the pericardium would best be made to prevent injury to which of the following structures? A. Auricular appendage of the left atrium B. Coronary sinus C. Left anterior descending artery D. Left phrenic nerve E. Left sympathetic trunk

67 C. The anterior interventricular (left anterior descending) artery lies anteriorly and to the left and descends vertically to the left toward the apex. It can be more easily injured by a transverse incision of the pericardium, which would cross perpendicular to this artery. The auricular appendage of the left atrium is located posteriorly; therefore, it would not be injured in an anterior longitudinal incision. The coronary sinus is between the right atrioventricular orifice and the inferior vena cava and would not be affected. The left phrenic nerve lies between the heart and the left lung and is too deep to be injured in this incision. The left sympathetic trunk is also too posterior to be injured.

67 A 2-year-old male toddler is admitted to the hospital with testicular pain. Physical examination reveals an enlarged scrotum. An otoscope is placed beneath the lateral side of the scrotum and the testis is transilluminated through the scrotal sac (Fig. 4-4). Which of the following best describes the signs observed in this patient? A. Varicocele B. Rectocele C. Cystocele D. Hydrocele E. Hypospadias

67 D. As seen in the photograph, the swollen scrotum contains mostly a clear fluid. Since hydrocele is the accumulation of fluid between the visceral and parietal layers of the tunica vaginalis, this condition best accounts for the findings in this patient. GAS 470-475; N 365; McM 265

68 A 34-year-old woman is admitted to the hospital with perineal pain. Laboratory blood tests reveal marked leukocytosis. Physical examination reveals abscesses in the anterior recess of the ischioanal fossa. A surgical procedure is performed to debride and drain the abscesses. Which of the following nerves will most likely need to be anesthetized to specifically numb the surgical area? A. Dorsal nerve to the clitoris B. Superficial perineal branch of perineal nerve C. Perineal nerve D. Inferior rectal nerve E. Pudendal nerve

68 C. The perineal nerve would need to be anesthetized because it supplies the area described. The dorsal nerve to the clitoris pierces the perineal membrane and innervates the clitoris and not the anterior recess of the ischioanal fossa. The superficial perineal branch of the perineal nerve supplies only the labia majora. The inferior rectal nerve innervates the skin around the anus and the external anal sphincter muscle. The pudendal nerve is the main nerve of the perineum and gives rise to all of the aforementioned nerves; therefore, anesthetizing it would result in widespread effects that would be superfluous to what is actually needed for drainage of the abscess. GAS 486-494, 575; N 391; McM 279

68 During cardiac surgery of a 45-year-old male the cardiac surgeon can place her fingers in the transverse pericardial sinus, if necessary. This allows the surgeon to easily place a vascular clamp upon which of the following vessels? A. Right and left pulmonary veins B. Superior and inferior vena cava C. Right and left coronary arteries D. Pulmonary trunk and ascending aorta E. Pulmonary trunk and superior vena cava

68 D. A finger passing through the transverse pericardial sinus passes directly behind the great arteries exiting the heart, allowing the surgeon to rather easily place a vascular clamp upon the pulmonary trunk and ascending aorta. The other vessels listed are not readily accessible by way of the transverse sinus.

69 A 48-year-old male patient is scheduled to have a coronary arterial bypass because of chronic angina. Coronary arteriography reveals nearly total blockage of the posterior descending interventricular artery. In exposing this artery to perform the bypass procedure, which accompanying vessel is most susceptible to injury? A. Middle cardiac vein B. Great cardiac vein C. Small cardiac vein D. Anterior cardiac vein E. Coronary sinus

69 A. The middle cardiac veins run parallel with the posterior interventricular (posterior descending) artery and drains directly into the coronary sinus. The great cardiac vein parallels the anterior interventricular artery and the small cardiac veins pass parallel with the right marginal artery. The anterior cardiac veins are several small veins that drain directly into the right atrium. The coronary sinus is a wide venous channel that runs from left to right in the posterior part of the coronary groove.

69 A 34-year-old woman is admitted to the hospital with perineal pain. Laboratory blood tests reveal marked leukocytosis. Physical examination reveals perforation of the wall of the anal canal at the level of the anal valves. A horseshoe abscess extends from one ischioanal fossa to the other in the posterior recess. Which of the following nerves will most likely need to be anesthetized? A. Dorsal nerve to the clitoris B. Superficial perineal branch of perineal nerve C. Perineal nerve D. Inferior rectal nerve E. Pudendal nerve

69 D. The inferior rectal nerve supplies the external anal sphincter muscle and the skin around the anus. Therefore this would be the best nerve to anesthetize for abscess drainage in this area. The dorsal nerve to the clitoris does not innervate the posterior recess of the perineum. The superficial perineal branch of the perineal nerve supplies the labia majora and would not need to be anesthetized in the event of a horseshoe anal abscess. The perineal nerve supplies all the perineal muscles and the labia majora, but for the area in question it does not have as direct a supply as the inferior rectal nerve. The pudendal nerve gives off all the branches above and thus anesthetizing it would result in additional unwanted side effects (GAS Fig. 5-75). GAS 492-494, 515; N 391; McM 279

7 A 6-year-old boy has a large intraabdominal mass in the midline just above the pubic symphysis. During surgery a cystic mass is found attached to the umbilicus and to the apex of the bladder. Which of the following is the most likely diagnosis? A. Hydrocele B. Meckel cyst C. Meckel diverticulum D. Omphalocele E. Urachal cyst

7 E. The persistence of the epithelial lining of the urachus can give rise to a urachal cyst. This swelling is found in the midline in the umbilical region. Hydrocele is fluid accumulation between the visceral and parietal layers of the tunica vaginalis of the testis. A Meckel diverticulum is located in the ileum of the small intestine. When it becomes inflamed, it can cause symptoms of appendicitis. A diverticulum can form a cyst (Meckel cyst). An omphalocele is the persistence of the herniation of the abdominal contents into the umbilical cord. GAS 463; N 348; McM 223

7. A 29-year-old woman is admitted to a hospital because the birth of her child is several days overdue. Tearing of the pelvic diaphragm during childbirth leads to paralysis of which of the following muscles? (A) Piriformis (B) Sphincter urethrae (C) Obturator internus (D) Levator ani (E) Sphincter ani externus

7. The answer is D. The pelvic diaphragm is formed by the levator ani and coccygeus, whereas the urogenital diaphragm consists of the sphincter urethrae and deep transverse perinei muscles. The piriformis passes through the greater sciatic notch and inserts on the greater trochanter of the femur. The obturator internus forms the lateral wall of the ischiorectal fossa. The sphincter ani externus is composed of three layers, including the subcutaneous (corrugator cutis ani), superficial, and deep portions, and maintains a voluntary tonic contracture.

70 A 35-year-old woman is undergoing a tension free vaginal tape procedure to repair an inguinal hernia. Two days postoperatively the patient has a high fever and shows signs of hypovolemic shock. A radiographic examination reveals that a vessel crossing the pectineal (Cooper's) ligament as it descends into the pelvis was injured by a staple, confirming the presence of the so called "arterial circle of death." Which of the following arteries is most likely injured? A. Obturator artery B. Aberrant obturator artery C. Superior vesical artery D. Middle rectal E. Inferior vesical

70 B. A variant obturator artery arising from the inferior epigastric arteries can be found in 20% to 30% of the population. Patients with this variation are more susceptible to inadvertent damage during certain surgeries if the surgeon is not aware of presence of the aberrant artery. This artery is alternatively called the "corona mortis," which translates as "crown of death," relating to the result of cutting this artery without realizing the seriousness of the error. The other arteries listed would be less likely to be injured because the surgeon would assume they are present and will thus take great care in making sure not to staple them. GAS 495-500; N 378; McM 272

70 A 54-year-old male is admitted to the hospital with severe chest pain. ECG examination reveals a myocardial infarction. If the posterior interventricular branch in the patient arises from the right coronary artery, which part of the myocardium will most likely have its blood supply reduced if the circumflex branch of the left coronary artery becomes occluded from an atherosclerotic plaque? A. Anterior part of the interventricular septum B. Diaphragmatic surface of the right ventricle C. Infundibulum D. Lateral wall of the left ventricle E. Posterior part of the interventricular septum

70 D. The left coronary artery bifurcates into the anterior interventricular artery (left anterior descending: LAD) and the coronary circumflex branch. The circumflex branch gives off the left marginal branch, which supplies the lateral wall (obtuse margin) of the left ventricle. The anterior part of the interventricular septum is supplied by the LAD. The diaphragmatic surface of the right ventricle is supplied by the posterior descending artery and the right marginal, a branch of the right coronary artery. The infundibulum, also known as the conus arteriosus, is the outflow portion of the right ventricle. The posterior part of the interventricular septum is supplied by the posterior descending artery, in most cases a branch of the right coronary artery.

71 A 70-year-old male with a history of two previous myocardial infarctions is admitted to the hospital with severe chest pain. ECG reveals a new myocardial infarction and ventricular arrhythmia. Coronary angiography reveals that the right coronary artery is blocked just distal to the origin of the right marginal artery in a right coronary dominant circulation. Which of the following structures would most likely be affected after such a blockade? A. Right atrium B. SA node C. AV node D. Lateral wall of the left ventricle E. Anterior interventricular septum

71 C. The atrioventricular (AV) node is most commonly supplied by a branch of the right coronary artery. This branch arises at the crux of the heart (the point of junction of all four cardiac chambers posteriorly); this is the location of the occlusion. The right atrium is supplied by the right coronary artery, which additionally supplies the sinoatrial node. The left marginal artery supplies the lateral wall of the left ventricle. The anterior portion of the interventricular septum is supplied by the anterior interventricular artery.

71 A 41-year-old man visits the outpatient urology clinic for a vasectomy procedure. Two months later the patient revisits the outpatient clinic complaining of pain in his testis. The diagnosis was made for postvasectomy pain syndrome. Which of the following nerves was most likely injured? A. Sympathetic fibers to ductus deferens B. Ilioinguinal C. Iliohypogastric D. Genital branch of genitofemoral E. Visceral afferent T10 to L2

71 E. The afferents of the testis and most of the ductus accompany sympathetics to enter the trunk at T10 to L2, with cell bodies in the dorsal root ganglia of those spinal nerves. (Which is why a forceful kick to the testes seems to hurt so severely in the periumbilical region of the abdomen.) The more proximal portion of the ductus has sensory fibers in the pelvic splanchnics. The ilioinguinal nerve is somatic and innervates the upper and medial thigh as well as the anterior scrotum and skin at the root of the penis, not the ductus deferens. The iliohypogastric nerve is an anterior abdominal wall nerve that innervates transverse and oblique abdominal muscles, supplies skin above the pubis, and has cutaneous supply to the lateral buttocks. The genital branch of the genitofemoral nerve supplies the cremaster muscle and the scrotum. GAS 492-494, 515; N 394; McM 265

72 A 41-year-old woman is admitted to the obstetrics and gynecology department for a scheduled tubal ligation procedure. Two days postoperatively the patient has a high fever and shows signs of hypovolemic shock. A radiographic examination reveals a large hematoma adjacent to the external iliac artery. Which of the following vessels was most likely injured? A. Ovarian arteries B. Ascending branch of uterine arteries C. Descending branch of uterine arteries D. Superior vesical artery E. Inferior vesical artery

72 A. The ovarian arteries arise from the abdominal aorta, descend retroperitoneally along the posterior abdominal wall, and cross just anterior to the external iliac vessels. The ovarian arteries are the most likely source of blood from a hematoma following a tubal ligation. The ascending and descending uterine arteries and superior and inferior vesical arteries branch from the internal iliac arteries and are not likely to be the source of blood in this situation. GAS 495-500; N 378; McM 272

72 A 43-year-old woman is diagnosed with mitral valve stenosis. During physical examination the first heart sound is abnormally loud. Which of the following heart valves are responsible for the production of the first heart sound? A. Aortic and mitral B. Aortic and tricuspid C. Tricuspid and mitral D. Mitral and pulmonary E. Tricuspid and pulmonary

72 C. The first heart sound is caused by the closure of the tricuspid and mitral valves. The second heart sound is caused by the closure of the aortic and pulmonary valves.

73 A 75-year-old woman is admitted to the hospital with anginal pain. ECG reveals myocardial infarction and a right bundle branch block. During physical examination the patient has a loud second heart sound. Which of the following heart valves are responsible for the production of the second heart sound? A. Aortic and pulmonary B. Aortic and tricuspid C. Tricuspid and mitral D. Mitral and pulmonary E. Tricuspid and pulmonary

73 A. The second heart sound is caused by the closure of the aortic and pulmonary valves. The first sound by the heart is caused by the closure of the tricuspid and mitral valves.

73 A 37-year-old woman is admitted to the hospital with pelvic pain. Radiographic examination reveals a benign tumor on the left ovary. An ovariectomy is performed and the ovarian vessels are ligated. Which of the following structures is most at risk of injury when the ovarian vessels are ligated? A. Uterine artery B. Vaginal artery C. Ureter D. Internal pudendal artery E. Pudendal nerve

73 C. When an ovariectomy (also called oophorectomy) is performed, the ovarian vessels must be ligated. The ovarian vessels lie anterior to the ureter just proximal to the bifurcation of the aorta. The ureter is the structure that is at the most risk when ligating the ovarian vessels. The vaginal artery is a branch of the uterine artery. The uterine artery does anastomose with the ovarian vessels via the ascending uterine artery; however, it lies too far distally to be at risk during ligation of the ovarian vessels. The internal pudendal artery and pudendal nerve mostly lie in the perineum and are not at risk. GAS 495-500; N 378; McM 272

74 A 23-year-old woman in her seventh month of pregnancy visits her gynecologist for a routine checkup. The patient is informed that a hormone called "relaxin" is responsible for the relaxation of the sacroiliac joint and pubic symphysis. Which of the following pelvic distances will most likely remain unaffected? A. Transverse diameter B. Interspinous distance C. True conjugate diameter D. Diagonal conjugate E. Oblique diameter

74 C. The conjugate diameter of the pelvis (anteroposterior) is not altered by relaxation of the pelvic joints. The transverse diameter is the longest distance extending from the middle of one pelvic brim to the other. The interspinous distance is the distance between the ischial spines and changes dramatically during pregnancy due to relaxation of the joints. The diagonal conjugate and oblique diameters are slightly increased during pregnancy due to the effects of the hormone relaxin. GAS 446-453; N 332; McM 277

74 Ten days after a surgical procedure to correct her cardiac malformation, a 3-month-old infant died unexpectedly in her sleep. After an autopsy, the pathologist reported as follows: "A significant portion of the conduction tissue was found to be necrotic. The area of the necrotic tissue was located inferior to the central fibrous body, membranous septum, and septal leaflet of the tricuspid valve. Further examination revealed infarction of the surrounding tissue. The rest of the heart was unremarkable." Which of the following arteries was most likely occluded? A. Artery of the conus B. SA node artery C. AV node artery D. First septal perforator of the anterior interventricular artery E. All of the above

74 D. The first septal perforating branch of the anterior interventricular artery (left anterior descending: LAD) is the first branch of the LAD that supplies the conducting tissue of the heart; it passes directly to the point of bifurcation of the common atrioventricular bundle of His. The other vessels listed have no anatomic relation to the area of ischemia.

75 A 55-year-old male is undergoing an aortic valve replacement. During the procedure the heart is connected to the heart lung machine. As the surgeon explores the oblique pericardial sinus, which of the following is not directly palpable with the tips of the fingers? A. Inferior vena cava B. Superior vena cava C. Posterior wall of the left atrium D. Inferior right pulmonary vein E. Right atrium

75 B. The superior vena cava empties into the right atrium on the superior aspect of the heart; it is not directly palpable from the oblique sinus. The oblique sinus is a blind cul-de-sac providing access to the inferior vena cava, the posterior wall of the left atrium, right atrium, and the right and left pulmonary veins.

75 A 42-year-old woman is admitted to the hospital with a mass on her right ovary. An ovariectomy is performed and the lymphatics of the lateral pelvic wall are also removed. Four days postoperatively the patient complains of painful spasms of the adductor muscles of the thigh and sensory deficit in the distal medial thigh. Which of the following nerves is most likely injured? A. Genitofemoral B. Ilioinguinal C. Iliohypogastric D. Obturator E. Lumbosacral trunk

75 D. The obturator nerve runs a course along the lateral pelvic wall and innervates the adductors of the thigh and the skin on the medial aspect of the distal thigh. Damage to the obturator nerve is the most likely cause for the sensory and motor deficit experienced by the patient. The genitofemoral nerve is motor to the cremaster muscle and sensory to the skin over the femoral triangle. The ilioinguinal nerve innervates the skin over the labium majus and upper, inner thigh. The iliohypogastric nerve supplies skin over the anterolateral gluteal region and a strip to the area above the pubis. The lumbosacral trunk contains motor and sensory fibers from L4 and L5 and is the lumbar contribution to the lumbosacral plexus. GAS 486-494, 515; N 388; McM 272

76 A 69-year-old man is admitted to the hospital for a scheduled radical prostatectomy. Six months postoperatively the patient visits the outpatient clinic complaining of impotence. Where are the nerve cell bodies located that are responsible for erection? A. Sacral parasympathetic nucleus B. Sacral sympathetic trunk ganglia C. Inferior mesenteric ganglion D. Superior hypogastric plexus E. Intermediolateral column of L1, L2

76 A. The neural cell bodies responsible for erection are located in the sacral parasympathetic nucleus (intermediomedial cell column). The parasympathetic nervous system is responsible for producing an erection. The sacral sympathetic trunk ganglia would not be responsible for the action of erection but rather the action of ejaculation. (Mnemonic: "Point and Shoot": parasympathetics for erection, sympathetics for ejaculation.) The inferior mesenteric ganglion would not contain parasympathetic neural cell bodies responsible for erection because they go to the hindgut. The superior hypogastric plexus contains few if any parasympathetic fibers and is not the primary location for the parasympathetic neural cell bodies. The intermediolateral column of L1 and L2 contains nerve cell bodies of preganglionic sympathetic neurons and therefore would not contribute to producing an erection. GAS 486-494, 515; N 395; McM 265

76 A 42-year-old female is admitted urgently to the emergency department after suffering a penetrating wound to her chest from an ice pick during a violent domestic dispute. Physical and ultrasound examinations reveal that the patient has cardiac tamponade. Which of the following will most likely be found during physical examination? A. There will be a visible or palpable decrease in the dimensions of the external jugular and internal jugular vein. B. There will be gradual enlargement of the ventricles in diastole. C. The difference between systolic and diastolic arterial pressures will increase significantly. D. There will be diminished heart sounds. E. The pulses in the internal carotid arteries will become increasingly distinct, as detected behind the angles of the mandible.

76 D. Cardiac tamponade is characterized by hypotension, tachycardia, muffled heart sounds, and jugular vein distention. Bleeding into the pericardial cavity would muffle the heart sounds because of the increased distance between the chest wall and the heart, leading to "distant" heart sounds. When the effusion is particularly severe, the heart may take on a "water bottle" appearance on an anterior-posterior radiograph.

77 A 34-year-old woman is at her third stage of labor. The obstetrician is concerned that the pelvic canal is too narrow for a vaginal delivery. Which of the following dimensions is the most reliable determinant of the capacity for a vaginal birth? A. Transverse diameter B. Interspinous distance C. True conjugate diameter D. Diagonal conjugate E. Oblique diameter

77 B. The interspinous distance is the distance between the ischial spines. The interspinous distance is usually the shortest distance, therefore being the minimum dimension along the birth canal. The true conjugate diameter is the anteroposterior distance and does not change. The transverse diameter, oblique diameter, and diagonal conjugate diameter can change slightly during pregnancy, but the interspinous distance changes the most during birth; it is also more easily measured. GAS 446-453; N 332; McM 276

77 During surgical repair of a congenital cardiac anomaly in a 15-year-old boy with a right dominant coronary arterial system, the surgeon accidentally injured a vessel that usually supplies part of the conduction system. This results in intermittent periods of atrioventricular block and severe arrhythmia. The injured artery was most likely a direct branch of which of the following arteries? A. Distal anterior interventricular artery B. Circumflex artery C. Left coronary artery D. Marginal artery E. Right coronary artery

77 E. "Right coronary dominant circulation" refers simply to the fact that the right coronary artery provides origin for the posterior interventricular (posterior descending) coronary artery. In such cases, it provides supply for the sinoatrial and atrioventricular nodes. It might be anticipated that right coronary blockage could result in dysfunction of the atrioventricular node, if collateral supply is poor or absent. The LAD, circumflex, and left marginal are all branches of the left coronary artery. The right marginal artery marginal is a branch of the right coronary artery.

78 A 32-year-old man visits the outpatient clinic and says he has had a painless mass at his right scrotum for several months. Ultrasonographic examination reveals a homogeneous hypoechoic intratesticular mass. Biopsy reveals a seminoma. Cancer of the testis metastasizes first to which of the following lymph nodes? A. Deep inguinal B. External iliac C. Internal iliac D. Lumbar E. Superficial inguinal

78 D. Lymph vessels from the testes follow the path of the testicular blood supply (abdominal aorta), and therefore lymph from the testicles drains into the lumbar nodes. The superficial inguinal lymph nodes drain lymph from the lower limb. The deep inguinal nodes drain lymph from the superficial inguinal nodes, the glans of the penis, and spongy urethra. The external iliac nodes drain lymph from anterosuperior pelvic structures and receive lymph from the deep inguinal nodes. Internal iliac nodes drain lymph from inferior pelvic structures and receive lymph from the sacral nodes. GAS 501, 519; N 386; McM 358

78 A 42-year-old female is admitted to the hospital with dyspnea. Imaging reveals severe mitral valve regurgitation. Which of the following structures prevents regurgitation of the mitral valve cusps into the left atrium during systole? A. Crista terminalis B. Crista supraventricularis C. Pectinate muscles D. Chordae tendineae E. Trabeculae carneae

78 D. The chordae tendineae are fibrous cords that connect papillary muscles to valve leaflets. The restraint provided by these cords on the valve leaflets prevents the prolapse of the mitral valve cusps into the left atrium. The crista terminalis is a ridge that runs from the opening of the inferior vena cava to the superior vena cava. Trabeculae carneae are irregular ridges of myocardium that are present within the ventricles.

79 A 42-year-old woman is admitted to the hospital with severe uterine bleeding. Radiologic examination reveals uterine fibroids. A uterine artery embolization is performed. Which of the following arteries will supply collateral supply to the uterus? A. External iliac B. Inferior mesenteric C. Ovarian D. Internal pudendal E. Superior mesenteric

79 C. Uterine artery embolization is performed to starve uterine fibroids of their blood supply resulting in a decrease in size of these benign tumors. Following the procedure, the uterus receives collateral blood supply from the ovarian artery (a direct branch of the abdominal aorta). The external iliac, inferior mesenteric, internal pudendal, and superior mesenteric arteries do not provide adequate collateral blood supply to the uterus. GAS 459; N 382; McM 270, 275

79 A 58-year-old female with cardiac arrhythmia has undergone a procedure to implant a pacemaker. The electrical conducting leads for the pacemaker must be passed into the heart from the pacemaker. Which of the following is the correct order of structures for passage of the leads into the right ventricle? A. Brachiocephalic vein, superior vena cava, mitral valve, right ventricle B. Superior vena cava, right atrium, mitral valve, right ventricle C. Superior vena cava, right atrium, tricuspid valve, right ventricle D. Brachiocephalic vein, superior vena cava, right atrium, tricuspid valve, right ventricle E. Brachiocephalic vein, superior vena cava, right atrium, mitral valve, right ventricle

79 D. The correct path that leads to the right ventricle for the lead of the pacemaker is the brachiocephalic vein (could be right or left; pacemakers are more commonly placed on the left so would be left brachiocephalic vein), superior vena cava, right atrium, tricuspid valve, and right ventricle.

8 A 26-year-old pregnant woman visits her gynecologist for a routine checkup. Ultrasound examination reveals that the patient has a normal pregnancy but that she also has two uteri. What is the most likely embryologic explanation of this condition? A. A complete fusion of the paramesonephric ducts B. An incomplete fusion of the paramesonephric ducts C. Hydronephrosis D. Cryptorchidism E. Regression of the pronephros

8 B. A double uterus is caused by failure of inferior parts of the paramesonephric ducts. A complete fusion results in abnormal development of the uterine tubes because the uterine tubes form from the unfused portions of the cranial parts of the paramesonephric ducts. Hydronephrosis, swelling of the renal pelvis and calyces with urine, can result from the obstruction of the ureter by a renal stone. Cryptorchidism is a condition characterized by an undescended testis, in which the testis can be localized in the abdominal cavity or in any place along the path of testicular descent. The pronephros is part of the primordial urinary system and generally degenerates in the first four weeks of development. GAS 379, 462; N 367; McM 274

8. A 37-year-old small business manager receives a gunshot wound in the pelvic cavity, resulting in a lesion of the sacral splanchnic nerves. Which of the following nerve fibers would primarily be damaged? (A) Postganglionic parasympathetic fibers (B) Postganglionic sympathetic (C) Preganglionic sympathetic fibers (D) Preganglionic parasympathetic fibers (E) Postganglionic sympathetic and parasympathetic fibers

8. The answer is C. The sacral splanchnic nerves consist primarily of preganglionic sympathetic neurons and also contain GVA fibers. None of the other fibers listed are contained in these nerves.

80 A 68-year-old male patient in the cardiology ward complains at each mealtime of difficulty in swallowing (dysphagia). Radiographic studies reveal significant cardiac hypertrophy. A barium swallow, followed by radiographic examination of the thorax, reveals esophageal constriction directly posterior to the heart. Which of the following is the most likely cause of the patient's dysphagia? A. Mitral valve stenosis B. Pulmonary valve stenosis C. Regurgitation of the aorta D. Occlusion of the anterior interventricular artery E. Occlusion of the posterior interventricular artery

80 A. Mitral stenosis leads to left atrial dilation, which can exert a compressive effect on the esophagus. The pulmonary valve is located between the outflow tract of the right ventricle and the pulmonary trunk. The aortic valve is located between the left ventricle and the aorta. Anterior interventricular (left anterior descending) and posterior interventricular (posterior descending) arterial occlusions can cause a myocardial infarction, but not dysphagia. In the normal position of the heart the left atrium lies most posteriorly. Therefore, a stenosis of the mitral valve (atrioventricular valve between left atrium and left ventricle) would lead to enlargement of the left atrium, which would in turn impinge upon the esophagus. A stenosis of the pulmonary valve would have no effect upon the esophagus because of the anterior position of the pulmonary trunk in the thorax. Regurgitation through any valve will ultimately decrease systemic blood flow. An occlusion of a coronary artery will lead to ischemia and possibly myocardial infarction.

80 A 34-year-old woman was admitted to the labor and delivery ward during the second stage of labor. The obstetrician noted that the descent of the fetus was arrested as the head seemed not to be able to pass the vaginal orifice. The decision was taken to perform a surgical incision of the perineum to enlarge the vaginal orifice, thereby decreasing the risk of tearing of the perineum and its muscles. An oblique incision was made to the right of the midline. Which structure is most likely maintained? A. External anal sphincter B. External urethral sphincter C. Right superficial transverse perineal muscle D. Right ischiocavernosus muscle E. Perineal body

80 E. The perineal body is a condensed mass of connective tissue to which the muscles of the pelvic floor and the muscles of the perineum attach. In many cases during childbirth, if the fetus' head is arrested, there is a high likelihood of perineal tears. In this situation an episiotomy is performed. This is usually done in an incision 45 degrees away from the midline in order to avoid damaging the perineal body. Damage to this tissue destabilizes the muscles that attach to it and can lead to urinary and fecal incontinence as well as uterine, bladder, and rectal prolapse. GAS 459; N 356; McM 276

81 A 35-year-old female is admitted to the emergency department because of cardiac arrhythmia. ECG examination reveals that the patient suffers from atrial fibrillation. Where is the mass of specialized conducting tissue that initiates the cardiac cycle located? A. At the junction of the coronary sinus and the right atrium B. At the junction of the inferior vena cava and the right atrium C. At the junction of the superior vena cava and the right atrium D. Between the left and right atria E. In the interventricular septum

81 C. The SA node, the primary pacemaker of the heart, is a mass of specialized cardiac cells within the myocardium at the upper end of the crista terminalis, near the opening of the superior vena cava into the right atrium. The AV node is at the junction of the coronary sinus and the right atrium upon the right fibrous trigone (central fibrous body). The eustachian valve directs blood from the inferior vena cava and through the right atrium toward the tricuspid valve ostium. The interatrial septum is located between the left and right atria. The septomarginal trabeculum (moderator band) arises from the muscular portion of the interventricular septum and passes to the base of the anterior papillary muscle in the right ventricle. The moderator band carries the right bundle branch of the conduction system just beneath its endocardial layer.

81 A 34-year-old woman presented to the emergency department at 36 weeks of pregnancy with complaints of severe abdominal pain every 3 minutes, each lasting more than 10 seconds. Each contraction was associated with the urge to "push." During physical examination, she was in painful distress, and strong uterine contractions were palpated. A vaginal examination revealed a fully dilated cervix and the fetal membranes were not intact. The obstetrician ordered a pudendal nerve block to decrease somatic pain during the vaginal delivery. Which of the following locations will be the most appropriate to place the anesthetic agent? A. Injection near ischial tuberosity anesthetizes the mons pubis and anterior labia majora B. Injection near ischial tuberosity anesthetizes and prevents pain from the uterus C. Injection superomedial to the anterior superior iliac spine anesthetizes most perineal skin D. Injection near ischial spines prevents pain from the uterus, cervix and upper 2/3 vagina E. Injection near ischial spines anesthetizes the posterior labia majora and inferior vagina

81 E. The pudendal nerve is a somatic nerve that innervates the perineum with exception of the anterior part of the labia majora (innervated by the ilioinguinal and genitofemoral nerves). The pudendal nerve forms from S2 to S4 ventral rami. It leaves the pelvis via the greater sciatic foramen and enters into the perineum via the lesser sciatic foramen by coursing around the sacrospinous ligament where it attaches to the ischial spine. This pudendal nerve block will anesthetize the area of distribution of the pudendal nerve. It will not anesthetize the uterus or cervix as these are visceral organs (GAS Fig. 5-15). GAS 486-494; N 391; McM 271, 276

82 A 75-year-old man presents to his family physician with complaints of generalized fatigue, weight loss, and anorexia for 5 months. He denies difficulty urinating, dysuria, hesitancy, and feelings of incomplete emptying with urination. During physical examination, the patient looked unwell. The abdomen was flat, soft, and was not tender, with no organomegaly, or inguinal lymphadenopathy. Digital rectal examination was performed and one of the lobes of the prostate was found to be enlarged. The patient was referred to the urologist. What structure was most likely affected in this patient's condition? A. The posterior lobe of the prostate B. The fibromuscular zone of the prostate C. The median zone of the prostate D. The lateral zone of the prostate E. The lateral capsule of the prostate

82 A. The prostate is a walnut shaped accessory organ of the reproductive system in men that lies immediately below the bladder and surrounds the exiting urethra. Anatomically, the prostate has four lobes: two lateral lobes, a posterior lobe, and a median lobe that directly surrounds the urethra. A tumor of the posterior lobe does not necessarily obstruct the urethra but it is frequently cancerous. Benign prostatic hyperplasia usually affects the median and lateral lobes. Micturition (urination) will be obstructed and patients present with a variety of symptoms including difficulty in initiating the urinary stream, a slow stream of urine, urinary frequency, and urinary urgency. GAS 473; N 362; McM 266, 268

82 A 45-year-old female is admitted to the hospital with swelling (edema) of the lower limbs. Ultrasound examination reveals an incompetent tricuspid valve. Into which area will regurgitation of blood occur in this patient? A. Pulmonary trunk B. Left atrium C. Ascending aorta D. Right atrium E. Left ventricle

82 D. The tricuspid valve is the atrioventricular valve located between the right atrium and right ventricle. An incompetent valve would allow blood to regurgitate into the right atrium during systole and subsequently raise pressure in the venous system, increasing capillary pressure and causing edema. A regurgitation of blood into the pulmonary trunk would be a result of an incompetent pulmonary valve. Regurgitation of blood from the left ventricle back into the left atrium is a result of prolapse of the mitral valve. There is no direct anatomic relationship between the tricuspid valve and the ascending aorta. Blood would pool in the left ventricle in the event of aortic valve incompetence.

83 A 34-year-old male with a complaint of sharp, localized pain over the thoracic wall is diagnosed with pleural effusion. Through which intercostal space along the midaxillary line is it most appropriate to insert a chest tube to drain the effusion fluid? A. Fourth B. Sixth C. Eighth D. Tenth E. Twelfth

83 C. To avoid damaging the lungs, a chest tube should be placed below the level of the lungs, in the costodiaphragmatic recess. Such a point of entrance for the tube would be the eighth or ninth intercostal space. At the midclavicular line, the costodiaphragmatic recess is localized between intercostal spaces 6 and 8, at the midaxillary line between 8 and 10, and at the paravertebral line between ribs 10 and 12.

83 A 5-year-old boy is complaining of pain of the penis after falling off his bicycle seat and striking himself between the legs with the bicycle bar. When he attempted to urinate, he could produce only small amounts of urine and his mother noticed that the penis and scrotum were swollen. During physical examination, there was blood at the external urethral meatus, and the penis, scrotum, and lower anterior abdominal wall were swollen. A CT scan showed that the prostate was in its normal position, and no bony fractures were observed. A MRI identified fluid around the penis and into the scrotum. Which of the following has occurred in this patient? A. Deep fascia of the penis was intact B. The membranous urethra was injured C. The attachment of superficial perineal fascia limited urine flow into the thigh D. The attachment of Colles' fascia prevented urine flow into the anorectal triangle E. There was urine between Camper's and Scarpa's fasciae over the lower abdomen

83 D. The patient has a tear in the spongy (penile) urethra. This commonly occurs when force is applied to the bulb of the penis. If there is an accompanying tear of the erectile bodies and the Buck's fascia that surrounds it, urine can spill into the superficial perineal pouch. This area is bound by Colles' fascia inferiorly, the perineal body posteriorly, and the perineal membrane superiorly. The attachment of Colles' fascia to the perineal body prevents the urine from spreading posteriorly to the anorectal triangle. The attachment of Colles' fascia to the ischiopubic ramus prevents urine from entering into the thigh. GAS 466-468; N 374; McM 266, 268

84 A 51-year-old male is admitted to the hospital with severe dyspnea. Radiographic examination reveals a tension pneumothorax. Adequate local anesthesia of the chest wall prior to insertion of a chest tube is necessary for pain control. Of the following layers, which is the deepest that must be infiltrated with the local anesthetic to achieve adequate anesthesia? A. Endothoracic fascia B. Intercostal muscles C. Parietal pleura D. Subcutaneous fat E. Visceral pleura

84 C. The parietal pleura is innervated by the intercostal nerves and is very sensitive to pain, in this case being somatic innervation. Therefore, the parietal pleura is the deepest layer that must be anesthetized to reduce pain during aspiration or chest tube placement.

84 After studying several hours straight in a seated position for an upcoming anatomy examination, a medical student experienced focal (nonradiating) pain in the buttocks. The pain immediately resolved when the student stood up. Which of the following bony features was most likely responsible for the student's pain? A. Inferior pubic ramus B. Posterior superior iliac spine C. Ischial tuberosity D. Pubic tubercle E. Ischial spine

84 C. The rough bony projection at the junction of the inferior end of the body of the ischium and its ramus is the large ischial tuberosity. Much of the upper body's weight rests on these tuberosities when sitting, and it provides the proximal, tendinous attachment of posterior thigh muscles (hamstring muscles and a portion of the adductor magnus). The small pointed posteromedial projection near the junction of the ramus and body is the ischial spine. The inferior pubic ramus is thin and flat and does not bear weight. The pubic tubercle is a prominent forward projecting tubercle on the upper border of the medial portion of the superior ramus of the pubis. The iliac crest extends posteriorly, terminating at the posterior superior iliac spine. GAS 441-445; N 334; McM 286, 292

85 A 5-year-old boy had been playing with his little race cars. Soon after he put a wheel from one of the cars in his mouth, he began choking and coughing. Where in the tracheobronchial tree is the most common site for a foreign object to lodge? A. The right primary bronchus B. The left primary bronchus C. The carina of the trachea D. The beginning of the trachea E. The left tertiary bronchus

85 A. The right main bronchus is the shorter, wider, and more vertical primary bronchus. Therefore, this is most often the location that foreign objects will likely be lodged. The left primary bronchus is not as vertical and therefore does not present the path of least resistance. (It must beunderstood, however, that in some cases of aspiration, the foreign body can pass into the left primary bronchus rather than the right bronchus!) The carina is a ridge separating the openings of left and right bronchi, the "fork in the road," so to speak. The trachea is a tubular structure supported by incomplete cartilaginous rings, and the likelihood that an object will be lodged there is minimal. It is unlikely that a foreign object would descend so far as to obstruct a tertiary bronchus, although this could happen.

85 A radiologist observed a pelvic MRI of a 22-yearold pregnant woman and measured the distance between the sacral promontory and the superior margin of the pubic symphysis. Which of the following was most likely measured by the radiologist? A. Bispinous outlet B. Maximum transverse diameter of inlet C. Sagittal inlet D. Sagittal outlet E. Bispinous inlet

85 C. The pelvic inlet or pelvic brim (sagittal inlet) is bounded posteriorly by the sacral promontory, laterally by the iliopectineal lines, and anteriorly by the symphysis pubis. The pelvic outlet (sagittal outlet) is bounded posteriorly by the coccyx, laterally by the ischial tuberosities, and anteriorly by the pubic arc. Bispinous (interspinous) outlet is the distance between the ischial spines. Maximum transverse diameter of inlet extends across the greatest width of the superior aperture, from the middle of the brim on one side to the same point on the opposite side. GAS 448-453; N 332; McM 92, 223

86 A 3-year-old child is admitted to the emergency department with a particularly severe attack of asthma. Which of the following is the most important factor in increasing the intrathoracic capacity in inspiration? A. "Pump handle movement" of the ribs—thereby increasing anterior-posterior dimensions of the thorax B. "Bucket handle movement" of the ribs— increasing the transverse diameter of the thorax C. Straightening of the forward curvature of the thoracic spine, thereby increasing the vertical dimensions of the thoracic cavity D. Descent of the diaphragm, with protrusion of the abdominal wall, thereby increasing vertical dimensions of the thoracic cavity E. Orientation and flexibility of the ribs in the baby, thus allowing expansion in all direction

86 D. Contraction of the diaphragm (descent) pulls the dome inferiorly, increasing the vertical dimension of the thorax. This is the most important factor in inspiration for increasing the internal pulmonary volume and concomitantly decreasing intrathoracic pressure. The contraction of intercostal muscles is usually involved in forced inspiration, resulting in increases in the transverse and anteroposterior dimensions of the thoracic cavity.

86 A 35-year-old woman gave birth to her third child. The birth was performed naturally and although the child was delivered without complications, the woman had severe postpartum bleeding. The obstetrician was unable to control the bleeding and an emergency hysterectomy had to be performed. Which of the following abdominal surgical incisions did the obstetrician most likely use to perform the hysterectomy? A. Right subcostal incision B. Median longitudinal incision C. Transverse incision just below the umbilicus D. McBurney's point incision E. Suprapubic incision

86 E. Suprapubic (Pfannenstiel) incisions are made 5 cm superior to the pubis symphysis. They are used when access to the pelvic organs is needed. When performing this incision, care must be taken not the perforate the bladder, as the fascia thins around the bladder area. A transverse incision just below the umbilicus is made just inferior and laterally to the umbilicus. This is a commonly used procedure, as it causes least damage to the nerve supply to the abdominal muscles, and heals well. McBurney's incision is performed at McBurney's point (1/3 of the distance between the ASIS and the umbilicus). It is mostly used in appendectomies. Subcostal incisions begin inferior to the xiphoid process, and extend inferior parallel to the costal margin. They are mainly used on the right side to operate on the gallbladder and the liver. Median incisions are made through the linea alba. They can be extended the whole length of the abdomen, by curving around the umbilicus. The linea alba is poorly vascularized, so blood loss is minimal, and major nerves are avoided. However, because it is poorly vascularized it heals slowly. GAS 466; N 340; McM 266

87 A 54-year-old female is admitted to the hospital with a stab wound of the thoracic wall in the area of the right fourth costal cartilage. Which of the following pulmonary structures is present at this site? A. The horizontal fissure of the left lung B. The horizontal fissure of the right lung C. The oblique fissure of the left lung D. The apex of the right lung E. The root of the left lung

87 B. The horizontal fissure of the right lung is a fissure separating the superior lobe from the middle lobe. It usually extends medially from the oblique fissure at the midaxillary line to the sternum, along the lower border of the fourth rib. The apex of the right lung reaches to a level above the clavicle and is therefore superior to the stab wound in the fourth costal cartilage. The other answers are related to features of the left lung, which is not addressed in the question.

87 During an elective urology rotation a medical student was asked by the urologist to name the structure that "slings" around the base of the body of penis and helps support it. Which of the following structures was the physician most likely referring to? A. Pubic symphysis B. Suspensory ligament of penis C. Midline raphe D. Fundiform ligament of penis E. Bulb of penis

87 D. The fundiform ligament of the penis is a thickening of the superficial fascia, and is superficial to the suspensory ligament that descends in the midline from the linea alba anterior to the pubic symphysis. The ligament splits to surround the penis and then unites and blends inferiorly with the dartos fascia forming the scrotal septum. GAS 508; N 344; McM 270

88 During the physical examination of a 35-year-old man, his urologist palpated the anterolateral surface of the scrotum. This part of the scrotum is most likely innervated by which of the following nerves? A. Ilioinguinal nerve (L1) B. Iliohypogastric nerve (L1) C. Ilioinguinal nerve (L2) D. Iliohypogastric nerve (L2) E. Pudendal nerve (S3)

88 A. The ilioinguinal nerve is a branch of the first lumbar nerve (L1). Its fibers are distributed to the skin of the upper and medial part of the thigh, and the skin over the root of the penis and upper part of the scrotum in men and the skin covering the mons pubis and labium majus in women. The iliohypogastric nerve is the superior branch of the ventral ramus of spinal nerve L1, which perforates the transversus abdominis, and divides into lateral and anterior cutaneous branches. The anterior cutaneous branch pierces the internal oblique, becomes cutaneous by perforating the aponeurosis of the external oblique about 2.5 cm above the superficial inguinal ring, and is distributed to the skin of the suprapubic region. The lateral femoral nerve innervates the skin on the lateral upper part of the thigh. The skin of the penis (or clitoris) is mainly supplied by the dorsal nerve (S2), a branch of the pudendal nerve. GAS 486-494; N 387; McM 218, 220

88 A 55-year-old female visited her doctor because of a painful lump in her right breast and a bloody discharge from her right nipple. Radiographic studies and physical examination reveal unilateral inversion of the nipple, and a tumor in the right upper quadrant of the breast is suspected. In addition, there is an orange-peel appearance of the skin (peau d'orange) in the vicinity of the areola. Which of the following best explains the inversion of her nipple? A. Retention of the fetal and infantile state of the nipple B. Intraductal cancerous tumor C. Retraction of the suspensory ligaments of the breast by cancer D. Obstruction of the cutaneous lymphatics, with edema of the skin E. Inflammation of the epithelial lining of the nipple and underlying hypodermis

88 C. The patient's symptoms are all indicative of inflammatory breast cancer. Common symptoms include inversion of the nipple and dimpling of the overlying skin, changes that are due to the retraction of the suspensory ligaments (of Cooper). Intraductal cancerous tumors show symptoms including breast enlargement, breast lump, breast pain, and nipple discharge. The other answers are not cancerous conditions.

89 A 58-year-old woman is admitted to the emergency department with severe dyspnea. Bronchoscopy reveals that the carina is distorted and widened. Enlargement of which group of lymph nodes is most likely responsible for altering the carina? A. Pulmonary B. Bronchopulmonary C. Inferior tracheobronchial D. Superior tracheobronchial E. Paratracheal

89 C. The inferior tracheobronchial nodes are also known as the carinal nodes and are located on the inferior aspect of the carina, the site of bifurcation of the trachea. The pulmonary nodes lie on secondary bronchi. The bronchopulmonary (hilar) nodes run along the primary bronchi. The superior tracheobronchial nodes are at the junction of the bronchi and the trachea. The paratracheal nodes run along the trachea.

89 A 16-year-old woman is admitted to the hospital with pelvic pain. History and physical examination reveal that the patient has a history of primary amenorrhea and an imperforate hymen. Which of the following is responsible for this condition? A. Cervical atresia B. Patent processus vaginalis C. Failure of the urorectal septum to develop D. Failure of the vaginal plate to canalize E. Incomplete fusion of the paramesonephric ducts

89 D. Failure of canalization of the vaginal plate results in atresia (blockage) of the vagina. A transverse vaginal septum occurs in approximately 1 in 80,000 females. Usually, the septum is located at the junction of the middle and superior thirds of the vagina. Failure of the inferior end of the vaginal plate to perforate results in an imperforate hymen. Variations in the appearance of the hymen are common. The vaginal orifice varies in diameter from very small to large, and there may be more than one orifice. The most common cause of indirect inguinal hernia is patent processus vaginalis. Most anorectal anomalies result from abnormal development of the urorectal septum, resulting in incomplete separation of the cloaca into urogenital and anorectal parts. Growth of one paramesonephric duct is retarded and does not fuse with the other one, a bicornuate uterus with a rudimentary horn (cornu) will develop. GAS 480; N 350; McM 272, 273

9 A 35-year-old woman is admitted to the emergency department with severe left abdominal and back pain. Radiographic evaluation reveals that the left ureter is blocked with a kidney stone. Because the ureter is completely obstructed, an emergency surgical procedure must be performed. Which of the following landmarks is most reliable for the identification of the ureter? A. The left ureter is located anterior to the left common iliac artery. B. The left ureter is located medial to the left inferior epigastric artery. C. The left ureter is located anterior to the left gonadal artery. D. The left ureter is located anterior to the left renal vein. E. The left ureter is located anterior to the left inferior epigastric artery.

9 A. The ureters cross the pelvic brim anterior to the bifurcation of the common iliac artery bilaterally. Because of the proximity of this artery to the ureter, it is in danger of being damaged during surgery. GAS 466-468, 512; N 378; McM 261

9. A young couple is having difficulty conceiving a child. Their physician at a reproduction and fertility clinic explains to them that (A) The ovary lies within the broad ligament (B) The glans clitoris is formed from the corpus spongiosum (C) Erection of the penis is a sympathetic response (D) Ejaculation follows parasympathetic stimulation (E) Fertilization occurs in the infundibulum or ampulla of the uterine tube

9. The answer is E. Fertilization takes place in the infundibulum or ampulla of the uterine tube. The glans clitoris is derived from the corpora cavernosa, whereas the glans penis is the expanded terminal part of the corpus spongiosum. Erection of the penis is caused by parasympathetic stimulation, whereas ejaculation is mediated via the sympathetic nerve. The ovaries are not enclosed in the broad ligament, but their anterior surface is attached to the posterior surface of the broad ligament.

90 A 60-year-old woman suffers from left sided pelvic pain that has been getting worse over the last 2 months. She reports that the pain is present every day and that its intensity does not vary based upon her menstrual cycle. She has also noticed an increase in abdominal size despite a recent decrease in her appetite. After a series of imaging studies, a right sided ovarian mass was found and a surgical procedure was planned to excise the mass. Which of the following structures should be ligated in order to avoid excessive bleeding during the surgery? A. Round ligament B. Suspensory ligament C. Ovarian ligament D. Transverse cervical ligament E. Mesosalpinx

90 B. The ovaries are supplied by ovarian vessels that originate from the abdominal aorta. Ovarian vessels cross the pelvic brim and descend in the suspensory ligament of ovary to supply the ovaries, fimbrial end of the uterine (fallopian) tube, and portions of the abdominal/pelvic ureter. The ovarian ligament also contains nerves and lymphatics. The remaining ligaments are devoid of major vessels. GAS 476; N 352; McM 274

90 A 72-year-old patient vomited and then aspirated some of the vomitus while under anesthesia. On bronchoscopic examination, partially digested food is observed blocking the origin of the right superior lobar bronchus. Which of the following groups of bronchopulmonary segments will be affected by this obstruction? A. Superior, medial, lateral, medial basal B. Apical, anterior, posterior C. Posterior, anterior, superior, lateral D. Apical, lateral, medial, lateral basal E. Anterior, superior, medial, lateral

90 B. The superior lobar bronchus is one of the divisions of the right main bronchus. This bronchus branches into apical, anterior, and posterior tertiary bronchi.

91 A 49-year-old woman suffers from left sided pelvic pain that has been getting worse over the last 2 months. She reports that the pain is present every day and that its intensity does not vary based upon her menstrual cycle. She has also noticed an increase in abdominal size despite a recent decrease in her appetite. After a series of imaging studies a left-sided ovarian mass was found and a surgical procedure was planned to excise the mass. During the procedure the surgeon can palpate the left ureter immediately medial to which of the following structures? A. Gonadal vein B. External iliac artery C. Inferior vena cava D. Internal iliac artery E. Uterine artery

91 A. The ureter, which is located retroperitoneally, originates from the renal pelvis and runs inferiorly crossing the pelvic brim close to the bifurcation of the common iliac artery. At this important landmark the ureter is medial to the gonadal vessels as it enters into the pelvis. It is lateral to the internal iliac artery as it descends past the pelvic brim. Although not immediately adjacent, the ureter is medial in relation to the external iliac artery but not close to the inferior vena cava or uterine artery at this location. GAS 379, 462; N 378; McM 270, 275

91 A 35-year-old woman is admitted to a surgical ward with a palpable mass in her right breast and swollen lymph nodes in the axilla. Radiographic studies and biopsy reveal carcinoma of the breast. Which group of axillary lymph nodes is the first to receive lymph drainage from the secretory tissue of the breast and therefore most likely to contain metastasized tumor cells? A. Lateral B. Central C. Apical D. Anterior (pectoral) E. Posterior (subscapular)

91 D. Lymphatic drainage of the breast is typically to the axillary nodes, more specifically to the anterior (pectoral) nodes. Lymphatic vessels from the pectoral nodes continue into the central axillary nodes, the drainage of which passes farther into the apical node, just inferior to the clavicle in the deltopectoral triangle. From these nodes lymph passes to the "sentinel," or scalene, nodes and the subclavian lymph trunk. The lateral and posterior axillary nodes do not normally receive lymph drainage from the breast but do receive lymph from the upper limb. (This is the reason for the edema of the upper limb that occurs after a mastectomy, in which there may be a total removal of axillary lymph nodes.)

92 A 30-year-old man is admitted to the emergency department because of significant nosebleeding and a headache that has worsened over several days. He also complains of fatigue. Upon examination it is noted that brachial artery pressure is markedly increased, femoral pressure is decreased, and the femoral pulses are delayed. The patient shows no external signs of inflammation. Which of the following is the most likely diagnosis? A. Coarctation of the aorta B. Cor pulmonale C. Dissecting aneurysm of the right common iliac artery D. Obstruction of the superior vena cava E. Pulmonary embolism

92 A. Increased arterial pressure in the upper limbs (as demonstrated in the brachial artery) and decreased pressure in the lower limbs (as demonstrated in the femoral artery) are common symptoms of coarctation of the aorta. Other symptoms include tortuous and enlarged blood vessels above the coarctation and an increased risk of cerebral hemorrhage. This condition of coarctation occurs when the aorta is abnormally constricted during development. The patient does not complain of respiratory distress, so cor pulmonale would not likely be the underlying condition. Dissection of the right common iliac artery would not result in nosebleed or headache. Obstruction of the superior vena cava would not account for decreased femoral pulse. A pulmonary embolism will not present with these findings.

92 An infant boy born to a 40-year-old mother fails to pass to meconium during the first 24 hours after his birth. On physical examination a "dimple" is found instead of an anal opening in the perineum. Which of the following additional defects are most likely to be found in the patient? A. Tracheoesophageal fistula B. Vertebral abnormalities C. Urinary tract defects D. Cardiac abnormalities E. Ileal atresia

92 C. Most anorectal abnormalities including imperforate anus occur from abnormal development of the urorectal septum leading to abnormal or incomplete cloacal separation. This results in abnormal urogenital and anorectal compartments and as such imperforate anus is usually associated with urinary tract defects. VATER syndrome may include tracheoesophageal, vertebral, renal, limb, and cardiac defects but this association is not as common as that between urogenital and anorectal defects. There is no association with ileal atresia. GAS 460-461; N 372; McM 279

93 An obstetrician administered anesthesia to a 30-year-old pregnant woman in labor. The physician palpated the ischial spine transvaginally and then injected a local anesthetic. Injection of the anesthetic at this location blocked a nerve that most likely contained fibers from which of the following ventral rami? A. S2, S3, S4 B. L4, L5, S1 C. L5, S1, S2 D. S1, S2, S3 E. S3, S4, S5

93 A. Pudendal or saddle block is less common than an epidural anesthesia but still used clinically to obtain perineal anesthesia during the final stages of labor or preceding an episiotomy. It is done by locating the pudendal nerve in or as it exits the pudendal canal and anesthetizing either transcutaneously or transvaginally by locating the ischial spine or tuberosity, respectively. The source for the pudendal nerve is S2 to S4 ventral rami. GAS 486-494; N 391; McM 276, 277

93 A 22-year-old man is diagnosed with signs of reduced aortic flow. Upon examination it is noted that brachial artery pressure is markedly increased, femoral pressure is decreased, and the femoral pulses are delayed. The patient shows no external signs of inflammation. Which of the following conditions will most likely be observed in a radiographic examination? A. Flail chest B. Pneumothorax C. Hydrothorax D. Notching of the ribs E. Mediastinal shift

93 D. The diagnosis for these symptoms is coarctation of the aorta. This condition occurs when the aorta is abnormally constricted. One of the cardinal signs is a characteristic rib notching. "Notching" of the ribs is due to the reversal of direction of blood flow through the anterior intercostal branches of the internal thoracic artery, as these usually small arteries carry collateral arterial blood flow to the lower thoracic portion of the aorta inferior to the coarctation. Enlargement and vibration of the intercostal arteries against the rib results in erosion ("notching") of the subcostal grooves, which is visible on radiography.

94 During an obstetrics and gynecology rotation, a medical student was reading a pelvic MRI of a 24-yearold woman with a physician. The physician pointed to a blood vessel that coursed between the lumbosacral trunk and ventral ramus of S1 and exited the pelvic cavity through the greater sciatic foramen. Which of the following vessels was the physician most likely pointing to? A. Inferior gluteal artery B. Superior vesical artery C. Iliolumbar artery D. Lateral sacral artery E. Superior gluteal artery

94 E. The superior gluteal artery, which is the largest branch of the posterior division of internal iliac artery, provides spinal branches as it passes between the lumbosacral trunk and ventral ramus of S1 and exits the pelvis via the greater sciatic foramen superior to the piriformis and supplying piriformis, the gluteal muscles, and tensor fasciae latae. Inferior gluteal artery, an anterior division of the internal iliac, runs inferior to the first sacral nerve and it exits the pelvis through the infrapiriform part of the greater sciatic foramen. None of the remaining vessels traverses the greater sciatic foramen. GAS 495; N 380; McM 268, 276

95 A 69-year-old woman complained of fecal incontinence that was found to be due to dysfunction of a muscle. Dysfunction of which of the following muscles most likely caused this patient's condition? A. Puborectalis B. Pubococcygeus C. Iliococcygeus D. Coccygeus E. Obturator internus

95 A. Puborectalis, a part of the levator ani group, is crucial to maintaining the anorectal angle and thereby maintaining fecal continence. Relaxation or injury to this muscle decreases the angle between the ampulla of rectum and the upper portion of the anal canal thus aiding defecation or incontinence respectively. None of the remaining muscles functions in maintaining the anorectal angle. GAS 454-457; N 339; McM 268

95 A 22-year-old woman sustained a chest injury upon impact with the steering wheel during a car crash. Upon admission of the patient to the hospital, physical examination revealed profuse swelling, inflammation, and deformation of the chest wall. A radiograph revealed an uncommon fracture of the manubrium at the sternomanubrial joint. Which of the following ribs would be most likely to also be involved in such an injury? A. First B. Second C. Third D. Fourth E. Fifth

95 B. The superior margin of the manubrium is characterized by the jugular notch. Laterally are the sternoclavicular joints and the articulations of the first ribs with the manubrium. The second pair of ribs articulates with the sternum at the sternal angle, the junction of the manubrium with the body of the sternum. GAS 144-147; GA 58-61

96 A 28-year-old pregnant woman delivers her baby before reaching the hospital and incurs a posterior vaginal tear. A pudendal nerve block is necessary to adequately anesthetize the area to facilitate proper closure of the wound. If this block is performed transvaginally, which landmark can be palpated in order to determine the proper site of anesthetic injection? A. Ischial spine B. Posterior inferior iliac spine C. Ischial tuberosity D. Posterior superior iliac spine E. Coccyx

96 A. The pudendal nerve passes between the sacrospinous and sacrotuberous ligaments. When performing this block with an internal approach, the ischial spine is used as a landmark as it is easily palpated transvaginally. The ischial tuberosity is used when the procedure is performed externally as it can be easily palpated in the ischioanal fossa. The posterior inferior and the posterior superior iliac spines are posterior; they cannot be palpated transvaginally and are not associated with the pudendal nerve. The coccyx is the most inferior portion of the vertebral column, cannot be palpated transvaginally, and is not near the pudendal nerve. GAS 486-494; N 389; McM 276, 277

96 A 47-year-old male is admitted to the emergency department, due to severe dysphagia. Edema of the lower limbs is apparent upon physical examination. A barium sulfate swallow imaging procedure reveals esophageal dilation, with severe inflammation, due to constriction at the esophageal hiatus. What is the most likely cause of the severe edema of the lower limbs? A. Thoracic aorta constriction B. Thoracic duct blockage C. Superior vena caval occlusion D. Aortic aneurysm E. Femoral artery disease

96 B. The thoracic duct is important in lymph drainage of the entire body with the exception of the upper right quadrant. The thoracic duct ascends between the aorta and azygos vein behind the esophagus. Dilation of the esophagus here in the lower thorax can compress the thoracic duct, leading to impairment of lymphatic drainage and resultant edema. GAS 219-220; GA 71

97 In coronary bypass graft surgery of a 49-year-old female, the internal thoracic artery is used as the coronary artery bypass graft. The anterior intercostal arteries in intercostal spaces three to six are ligated. Which of the following arteries will be expected to supply these intercostal spaces? A. Musculophrenic B. Superior epigastric C. Posterior intercostal D. Lateral thoracic E. Thoracodorsal

97 C. The anterior intercostal arteries anastomose with the posterior intercostal arteries. Ligation of the anterior arteries would not affect the supply of the intercostal spaces because the posterior arteries would provide collateral arterial supply. Branches of the musculophrenic artery provide supply for the lower seventh, eighth, and ninth intercostal spaces. The superior epigastric artery passes into the rectus sheath of the anterior abdominal wall. The lateral thoracic artery arises from the second part of the axillary artery, and the thoracodorsal artery is a branch of the subscapular artery, a branch of the third part of the axillary artery.

97 Anal fissures are slitlike openings in the anal mucosa, mainly found in the adult population and which result in excruciating pain on defecation. Which nerve innervates the area of the anal fissures? A. Sacral splanchnic B. Superior hypogastric C. Pelvic splanchnic D. Pudendal E. Ilioinguinal

97 D. Nerve supply to the perineum including the anal canal is via the pudendal nerve. The sacral splanchnic nerves carry postganglionic sympathetic nerve fibers to the pelvic organs. The pelvic splanchnic nerves carry preganglionic parasympathetic nerves to the pelvic organs. The superior hypogastric plexus is an autonomic plexus carrying sympathetic and parasympathetic innervation to the lower abdominal region. The ilioinguinal nerve supplies the skin of the upper medial thigh and anterior perineal area. GAS 492-494, 515; N 394; McM 276, 277

98 After radical resection of a pelvic tumor, a 45-yearold woman complains of postoperative bowel incontinence. Nerve injury leading to dysfunction of which muscle is responsible for her condition? A. Puborectalis B. Iliococcygeus C. Coccygeus D. Obturator internus E. Piriformis

98 A. The puborectalis muscle is a sling that acts as an internal sphincter around the rectum. The iliococcygeus runs from the obturator fascia to the midline and forms a raphe to which the anal aperture is anchored. The obturator externus and piriformis muscles are located on the lateral walls of the pelvis. The coccygeus muscle runs between the ischial spine and the coccyx and sacrum. GAS 454-457; N 339; McM 268

98 A 10-year-old boy is admitted to the hospital with retrosternal discomfort. A CT scan reveals a midline tumor of the thymus gland. Which of the following veins would most likely be compressed by the tumor? A. Right internal jugular B. Left internal jugular C. Right brachiocephalic D. Left brachiocephalic E. Right subclavian

98 D. The thymus lies in the superior mediastinum and extends upward into the neck, especially in the young. A midline tumor of this gland can compress the left brachiocephalic vein. The subclavian vein is distal or lateral to this location, and the thymus gland would not likely impinge upon it. The internal jugular veins are located superior and lateral to the position of the thymus gland. A midline tumor is more likely to cause compression of the left brachiocephalic vein, which crosses the midline, than the right brachiocephalic vein, which is not located in the midline.

99 A 69-year-old man is admitted to the hospital with a painful mass on the glans penis. A biopsy of the mass reveals carcinoma. Which lymph nodes will most likely be affected first if metastatic spread of the cancer occurs? A. External iliac B. Internal iliac C. Deep inguinal D. Lumbar/lateral aortic E. Axillary

99 C. In the perineum lymph drains according to arteries that supply the area, and so the glans penis drains to the deep inguinal nodes. Superficial skin of the penis and scrotum would drain to the superficial inguinal nodes. External iliac nodes receive lymph from the deep inguinal, and the internal iliac from the pelvic organs. The lumbar/lateral aortic nodes receive lymph from the abdominal organs, and the axillary drains the upper limb and external thorax. GAS 454-457; N 339; McM 268

1 35. A 23-year-old woman is participating in a dry ski-slope competition. The woman is admitted to the emergency department after falling and catching her thumb in the matting. Radiographic and physical examinations reveal rupture of the ulnar collateral ligament of the metacarpophalangeal joint of the thumb. The thumb is extremely painful and an injection of lidocaine is performed. What is the most likely diagnosis in this case? ⃣ A. Gamekeeper's thumb ⃣ B. Scaphoid fracture ⃣ C. Bennett's fracture ⃣ D. Smith's fracture ⃣ E. Boxer's fracture

A

100 Physical examination reveals weakness of medial deviation of the wrist (adduction), loss of sensation on the medial side of the hand, and clawing of the fi ngers. Where is the most likely place of injury? ⃣ A. Compression of a nerve passing between the humeral and ulnar heads of origin of fl exor carpi ulnaris ⃣ B. Compression of a nerve passing at Guyon's canal between the pisiform bone and fl exor retinaculum ⃣ C. Compression of a nerve passing through the carpal tunnel ⃣ D. Compression of a nerve passing between the ulnar and radial heads of origin of fl exor digitorum superfi cialis ⃣ E. Compression of a nerve passing deep to brachioradialis muscle

A

105 A 36-year-old patient is admitted to the emergency department with a dull ache in the shoulder or axilla. During physical examination the pain worsens by activity, and, conversely, rest and elevation relieve the pain. History reveals that the patient was hospitalized the past week and a central venous line was used. What is the most likely diagnosis? A. Axillary-subclavian vein thrombosis B. Compression of C5 to C8 spinal nerve C. Disk herniation of C4 to C8 D. Impingement syndrome E. Injury to radial, ulnar, and median nerves

A

107 In a penetrating wound to the forearm of a 24-year-old male, the median nerve is injured at the entrance of the nerve into the forearm. Which of the following would most likely be apparent when the patient's hand is relaxed? ⃣ A. The MCP and IP joints of the second and third digits of the hand will be in a condition of extension. ⃣ B. The third and fourth digits will be held in a slightly fl exed position. ⃣ C. The thumb will be fl exed and slightly abducted. ⃣ D. The fi rst, second, and third digits will be held in a slightly fl exed position. ⃣ E. The MCP and IP joints of the second and third digits of the hand will be in a condition of fl exion.

A

109 A 69-year-old man has numbness and pain in the middle three digits of his right hand at night. He retired9 years ago after working as a carpenter for 30 years. He has atrophy of the thenar eminence (see F ig. 6-2 on page 172). Which of the following conditions will be the most likely cause of this atrophy? ⃣ A. Compression of the median nerve in the carpal tunnel ⃣ B. Formation of the osteophytes that compress the ulnar nerve at the ulnar condyle ⃣ C. Hypertrophy of the triceps muscle compressing the brachial plexus ⃣ D. Osteoarthritis of the cervical spine ⃣ E. Repeated trauma to the ulnar nerve

A

112 A 23-year-old woman arrives at the emergency department with a swollen, painful forearm. An MRI examination reveals a compartment syndrome originating at the interosseous membrane between the radius and ulna. Which of the following type of joint will most likely be affected? A. Synarthrosis B. Symphysis C. Synchondrosis D. Trochoid E. Ginglymus

A

118 A 34-year-old woman is admitted to the emergency department after a car crash. Radiographic studies show marked edema and hematoma of the arm, but there are no fractures. During physical examination the patient presents with inability to abduct her arm without first establishing lateral momentum of the limb, and inability to flex the elbow and shoulder. Which of the following portions of the brachial plexus is most likely injured? A. Superior trunk B. Middle trunk C. Inferior trunk D. Lateral cord E. Medial cord

A

120 A 56-year-old woman is admitted to the hospital after a severe car crash. A large portion of her chest wall needed to be surgically removed and replaced with a musculoosseous scapular graft involving the medial border of the scapula. Which of the following arteries will most likely recompensate the blood supply to the entire scapula? A. Suprascapular B. Dorsal scapular artery C. Posterior humeral circumflex artery D. Lateral thoracic E. Supreme thoracic artery

A

124 A 34-year-old man is admitted to the hospital after a car collision. Radiographic examination reveals a fracture at his wrist. Physical examination reveals paralysis of the muscles that act to extend the interphalangeal joints (F ig. 6-10 ). Which of the following nerves is most likely injured? A. Ulnar B. Recurrent branch of median C. Radial D. Musculocutaneous E. Anterior interosseous

A

129 A 45-year-old woman is bitten by a dog on the lateral side of her hand. Two days later the woman develops fever and swollen lymph nodes. Which of the following group of lymphatics will most likely be involved? ⃣ A. Central ⃣ B. Humeral ⃣ C. Pectoral ⃣ D. Subscapular ⃣ E. Parasternal

A

132 A 43-year-old man visits the outpatient clinic with a painful shoulder. Physical examination reveals a painful arc syndrome due to supraspinatus tendinopathy. Which of the following conditions will be present during physical examination? ⃣ A. Painful abduction 0 ° to 15 ° ⃣ B. Painful abduction 0 ° to 140 ° ⃣ C. Painful abduction 70 ° to 140 ° ⃣ D. Painful abduction 15 ° to 140 ° ⃣ E. Painful abduction 40 ° to 140 °

A

133 A 54-year-old woman is admitted to the hospital after falling from a tree with an outstretched hand. Radiographic examination reveals a wrist dislocation. Which of the following carpal bones will most likely be involved? ⃣ A. Scaphoid-lunate ⃣ B. Trapezoid-trapezium ⃣ C. Hamate-lunate ⃣ D. Pisiform-triquetrum ⃣ E. Hamate-capitate

A

141 A 54-year-old man is admitted to the emergency department with severe chest pain. Electrocardiographic evaluation reveals a myocardial infarction. Due to the severity of the infarction, a coronary artery bypass surgery using a radial artery graft is proposed. Which of the following tests should be performed during physical examination prior to the bypass graft operation? A. Allen test B. Triceps reflex C. Tinel test D. Brachioradialis reflex E. Biceps reflex

A

29 A 45-year-old man is admitted to the hospital after a car crash. Radiographic examination reveals mild disk herniations of C7, C8, and T1. The patient presents with a sensory defi cit of the C8 and T1 spinal nerve dermatomes. The dorsal root ganglia of C8 and T1 would contain cell bodies of sensory fi bers carried by which of the following nerves? ⃣ A. Medial antebrachial cutaneous nerve ⃣ B. Long thoracic nerve ⃣ C. Lateral antebrachial cutaneous nerve ⃣ D. Deep branch of ulnar nerve ⃣ E. Anterior interosseous nerve

A

29. Which structure may herniate through the annulus fibrosus, thereby impinging on the roots of the spinal nerve?

A

34 A 21-year-old female softball pitcher is examined in the emergency department after she was struck in the arm by a line drive (a ball hit very hard and low). Radiographic and MRI studies show soft tissue injury to the region of the spiral groove, with trauma to the radial nerve. Which of the following muscles would be intact after this injury? ⃣ A. Flexor carpi ulnaris ⃣ B. Extensor indicis ⃣ C. Brachioradialis ⃣ D. Extensor carpi radialis ⃣ E. Supinator

A

35 Examination of a 21-year-old female athlete with an injury of the radial nerve in the spiral groove would typically demonstrate which of the following physical signs? ⃣ A. Weakness of thumb abduction and thumb extension ⃣ B. Weakness of thumb opposition ⃣ C. Inability to extend the elbow ⃣ D. Paralysis of pronation of the hand ⃣ E. Paralysis of abduction and adduction of the arm

A

39 The fact that the kidneys of a 32-year-old female patient were failing required that she be placed on dialysis. However, the search in her upper limb for a suitable vein was unexpectedly diffi cult. The major vein on the lateral side of the arm was too small; others were too delicate. Finally, a vein was found on the medial side of the arm that passed through the superfi cial and deep fascia to join veins beside the brachial artery. Which of the following veins was this? ⃣ A. Basilic ⃣ B. Lateral cubital ⃣ C. Cephalic ⃣ D. Medial cubital ⃣ E. Medial antebrachial

A

40 A 29-year-old female had sustained a deep laceration in the proximal part of the forearm. After the wound is closed, the following functional defi cits are observed by the neurologist on the service: The fi rst three digits are in a position of extension and cannot be fl exed. Digits 4 and 5 are partially fl exed at the metacarpophalangeal joints and noticeably more fl exed at the distal interphalangeal joints. Sensation is absent in the lateral side of the palm and the palmar surfaces of digits 1 to 3 and half of the fourth digit. Which of the following nerves has (have) most likely been injured? ⃣ A. Median nerve ⃣ B. Ulnar and median nerve ⃣ C. Ulnar nerve ⃣ D. Radial and ulnar nerve ⃣ E. Radial nerve

A

42 The 35-year-old female patient has a hard nodule about 1 cm in diameter slightly above and lateral to her right areola. A specifi c dye is injected into the tissue around the tumor, and an incision is made to expose the lymphatic vessels draining the area, for the lymphatic vessels take up the dye—which is visible to the eye. The vessels can then be traced to surgically expose the lymph nodes receiving the lymph from the tumor. Which of the following nodes will most likely fi rst encounter the lymph from the tumor? ⃣ A. Anterior axillary (pectoral) nodes ⃣ B. Rotter interpectoral nodes ⃣ C. Parasternal nodes along the internal thoracic artery and vein ⃣ D. Central axillary nodes ⃣ E. Apical or infraclavicular nodes

A

48 A 68-year-old female is examined by the senior resident in emergency medicine after her fall on a wet bathroom fl oor in the shopping center. Physical examination reveals a posterior displacement of the left distal wrist and hand. Radiographic examination reveals an oblique fracture of the radius. Which of the following is the most likely fracture involved in this case? ⃣ A. Colles' fracture ⃣ B. Scaphoid fracture ⃣ C. Bennett's fracture ⃣ D. Volkmann's ischemic contracture ⃣ E. Boxer's fracture

A

50 A 22-year-old male construction worker is admitted to the hospital after he suffers a penetrating injury to his upper limb from a nail gun. Upon physical examination the patient is unable to fl ex the distal interphalangeal joints of digits 4 and 5. What is the most likely cause of his injury? ⃣ A. Trauma to the ulnar nerve near the trochlea ⃣ B. Trauma to the ulnar nerve at the wrist ⃣ C. Median nerve damage proximal to the pronator teres ⃣ D. Median nerve damage at the wrist ⃣ E. Trauma to spinal nerve root C8

A

56 A 61-year-old man was hit by the cricket bat in the midhumeral region of his left arm. Physical examination reveals an inability to extend the wrist and loss of sensation on a small area of skin on the dorsum of the hand proximal to the fi rst two fi ngers. What nerve supplies this specifi c region of the hand? ⃣ A. Radial ⃣ B. Posterior interosseous ⃣ C. Lateral antebrachial cutaneous ⃣ D. Medial antebrachial cutaneous ⃣ E. Dorsal cutaneous of ulnar

A

63 A 35-year-old male body builder has enlarged his shoulder muscles to such a degree that the size of the quadrangular space is greatly reduced. Which of the following structures would most likely be compressed in this condition? ⃣ A. Axillary nerve ⃣ B. Anterior humeral circumfl ex artery ⃣ C. Cephalic vein ⃣ D. Radial nerve ⃣ E. Subscapular artery

A

67 A 41-year-old woman is scheduled for a latissimus dorsi muscle fl ap to cosmetically augment the site of her absent left breast, postmastectomy. Part of the latissimus dorsi muscle is advanced to the anterior thoracic wall, based upon arterial supply provided in part by the artery that passes through the triangular space of the axilla. What artery is forming the vascular base of this fl ap? ⃣ A. Circumfl ex scapular artery ⃣ B. Dorsal scapular artery ⃣ C. Transverse cervical artery ⃣ D. Lateral thoracic artery ⃣ E. Thoracoacromial artery

A

70 A 12-year-old male had received a laceration in the palmar surface of the wrist while playing with a very sharp knife. The cut ends of a tendon could be seen within the wound in the exact midline of the wrist. Which tendon lies in this position in most people? ⃣ A. Palmaris longus ⃣ B. Flexor carpi radialis ⃣ C. Abductor pollicis longus ⃣ D. Flexor carpi ulnaris ⃣ E. Flexor pollicis longus

A

74 A 26-year-old male power lifter visits the outpatient clinic with a painful shoulder. Radiographic examination reveals tendinopathy of the long head of the biceps. Which of the following conditions will most likely be present during physical examination? ⃣ A. Pain is felt in the anterior shoulder during forced contraction. ⃣ B. Pain is felt in the lateral shoulder during forced contraction. ⃣ C. Pain is felt during abduction and fl exion of the shoulder joint. ⃣ D. Pain is felt during extension and adduction of the shoulder joint. ⃣ E. Pain is felt in the lateral shoulder during fl exion of the shoulder joint.

A

81 A 31-year-old female fi gure skater is examined in the emergency department following an injury that forced her to withdraw from competition. When her male partner missed catching her properly from an overhead position, he grasped her powerfully, but awkwardly, by the forearm. Clinical examination demonstrated a positive Ochsner test, inability to fl ex the distal interphalangeal joint of the index fi nger on clasping the hands. In addition, she is unable to fl ex the terminal phalanx of the thumb and has loss of sensation over the thenar half of the hand. What is the most likely nature of her injury? ⃣ A. Median nerve injured within the cubital fossa ⃣ B. Anterior interosseous nerve injury at the pronator teres ⃣ C. Radial nerve injury at its entrance into the posterior forearm compartment ⃣ D. Median nerve injury at the proximal skin crease of the wrist ⃣ E. Ulnar nerve trauma halfway along the forearm

A

89 A 67-year-old female had a bad fall while walking her dog the evening before. She states that she fell on her outstretched hand. Radiographs do not demonstrate any bony fractures. The clinician observes the following signs of neurologic injury: There is weakness of fl exion of her wrist in a medial direction, there is a loss of sensation on the medial side of the hand, and there is clawing of the fi ngers. Where is the most likely place of nerve trauma? ⃣ A. Behind the medial epicondyle ⃣ B. Between the pisiform bone and the fl exor retinaculum ⃣ C. Within the carpal tunnel ⃣ D. At the cubital fossa, between the ulnar and radial heads of origin of fl exor digitorum superfi cialis ⃣ E. At the radial neck, 1 cm distal to the humerocapitellar joint

A

95 A 24-year-old female basketball player is admitted to the emergency department after an injury to her shoulder. Radiographic examination reveals a shoulder dislocation. What is the most commonly injured nerve in shoulder dislocations? ⃣ A. Axillary ⃣ B. Radial ⃣ C. Median ⃣ D. Ulnar ⃣ E. Musculocutaneous

A

96 A 45-year-old male is admitted to the hospital with a painful arm after a "strongest man in the world" contest. Physical examination gives evidence of a rupture of the long tendon of the biceps brachii ( Fig. 6-5 ). Which of the following is the most likely location of the rupture? ⃣ A. Intertubercular groove ⃣ B. Midportion of the biceps muscle ⃣ C. Junction with the short head of the biceps muscle ⃣ D. Proximal end of the combined biceps muscle ⃣ E. Bony insertion of the muscle

A

A 27-year-old man was admitted to the emergency department after an automobile collision in which he suffered a fracture of the lateral border of the scapula. Six weeks after the accident, physical examination reveals weakness in medial rotation and adduction of the humerus. Which nerve was most likely injured? A. Lower subscapular B. Axillary C. Radial D. Spinal accessory E. Ulnar

A

A 45-year-old male is admitted to the hospital after accidentally walking through a plate glass door in a bar while intoxicated. Physical examination shows multiple lacerations to the upper limb, with inability to fl ex the distal interphalangeal joints of the fourth and fi fth digits. Which of the following muscles is most likely affected? ⃣ A. Flexor digitorum profundus ⃣ B. Flexor digitorum superfi cialis ⃣ C. Lumbricals ⃣ D. Flexor digitorum profundus and fl exor digitorum superfi cialis ⃣ E. Interossei

A

A 69-year-old man has numbness in the middle three digits of his right hand and finds it difficult to grasp objects with that hand. He states that he retired 9 years earlier, after working as a carpenter for 50 years. He has atrophy of the thenar eminence ( Fig. 6-2 ). Which of the following conditions is the most likely cause of the problems in his hand? ⃣ A. Compression of the median nerve in the carpal tunnel ⃣ B. Formation of the osteophytes that compress the ulnar nerve at the medial epicondyle ⃣ C. Hypertrophy of the triceps muscle compressing the brachial plexus ⃣ D. Osteoarthritis of the cervical spine ⃣ E. Repeated trauma to the ulnar nerve

A

A mother tugs violently on her male child's hand to pull him out of the way of an oncoming car and the child screams in pain. Thereafter, it becomes obvious that the child cannot straighten his forearm at the elbow. When the child is seen in the emergency department, radiographic examination reveals a dislocation of the head of the radius. Which of the following ligaments is most likely directly associated with this injury? ⃣ A. Anular ⃣ B. Joint capsular ⃣ C. Interosseous ⃣ D. Radial collateral ⃣ E. Ulnar collateral

A

Laboratory studies in the outpatient clinic on a 24-year-old female included assessment of circulating blood chemistry. Which of the following arteries is most likely at risk during venipuncture at the cubital fossa? ⃣ A. Brachial ⃣ B. Common interosseous ⃣ C. Ulnar ⃣ D. Anterior interosseous ⃣ E. Radial

A

Stenosis of the aorta

A 12-year-old boy was admitted to a local hospital with a known history of heart problems. His left ventricular hypertrophy could result from which of the following conditions? -A constricted pulmonary trunk -An abnormally small left AV opening -Improper closing of the pulmonary valves -An abnormally large right AV opening -Stenosis of the aorta

Dorsal root ganglion and lateral horn of the spinal cord

A 17-year-old boy was involved in a gang fight, and a stab wound severed the white rami communicantes at the level of his sixth thoracic vertebra. This injury would result in degeneration of nerve cell bodies in which of the following structures? -Dorsal root ganglion and anterior horn of the spinal cord -Sympathetic chain ganglion and dorsal root ganglion -Sympathetic chain ganglion and posterior horn of the spinal cord -Dorsal root ganglion and lateral horn of the spinal cord -Anterior and lateral horns of the spinal cord

Middle cardiac vein

A 19-year-old man came to the emergency department, and his angiogram exhibited that he was bleeding from the vein that is accompanied by the posterior interventricular artery. Which of the following veins is most likely to be ruptured? -Great cardiac vein -Middle cardiac vein -Anterior cardiac vein -Small cardiac vein -Oblique veins of the left atrium

Dilation of the bronchial lumen

A 21-year-old patient with a stab wound reveals a laceration of the right vagus nerve proximal to the origin of the recurrent laryngeal nerve. Which of the following conditions would most likely result from this lesion? -Contraction of bronchial muscle -Stimulation of bronchial gland secretion -Dilation of the bronchial lumen -Decrease in cardiac rate -Constriction of coronary artery

GVA and preganglionic sympathetic fibers

A 23-year-old man received a gunshot wound, and his greater splanchnic nerve was destroyed. Which of the following nerve fibers would be injured? -General somatic afferent (GSA) and preganglionic sympathetic fibers -General visceral afferent (GVA) and postganglionic sympathetic fibers -GVA and preganglionic sympathetic fibers -General somatic efferent (GSE) and postganglionic sympathetic fibers -GVA and GSE fibers

In the left fifth intercostal space at the midclavicular line

A 27-year-old cardiac patient with an irregular heartbeat visits her doctor's office for examination. Where should the physician place the stethoscope to listen to the sound of the mitral valve? -Over the medial end of the second left intercostal space -Over the medial end of the second right intercostal space -In the left fourth intercostal space at the midclavicular line -In the left fifth intercostal space at the midclavicular line -Over the right half of the lower end of the body of the sternum

Left recurrent laryngeal nerve

A 27-year-old patient with Marfan syndrome has an aneurysm of the aortic arch. This may compress which of the following structures? -Right vagus nerve -Left phrenic nerve -Right sympathetic trunk -Left recurrent laryngeal nerve -Left greater splanchnic nerve

Is longer than the right primary bronchus

A 31-year-old man was involved in a severe automobile accident and suffered laceration of the left primary bronchus. The damaged primary bronchus: -Has a larger diameter than the right primary bronchus -Often receives more foreign bodies than the right primary bronchus -Gives rise to the eparterial bronchus -Is longer than the right primary bronchus -Runs under the arch of the azygos vein

Above the upper border of the ribs

A 32-year-old patient has a tension pneumothorax that can be treated with needle aspiration. To avoid an injury of the intercostal neurovascular bundle, the needle may be inserted in which of the following locations? -Above the upper border of the ribs -Deep to the upper border of the ribs -Beneath the lower border of the ribs -Between the external and internal intercostals -Through the transversus thoracis muscle

In the left fifth intercostal space

A 32-year-old patient who weighs 275 lb comes to the doctor's office. On the surface of the chest, the physician is able to locate the apex of the heart: At the level of the sternal angle In the left fourth intercostal space In the left fifth intercostal space In the right fifth intercostal space At the level of the xiphoid process of the sternum

Lower six posterior

A 33-year-old patient is suffering from a sudden occlusion at the origin of the descending (thoracic) aorta. This condition would most likely decrease blood flow in which of the following intercostal arteries? -Upper six anterior -All of the posterior -Upper two posterior -Lower anterior -Lower six posterior

Body of the second thoracic vertebra

A 37-year-old house painter fell from a ladder and fractured his left third rib and the structures with which it articulated. Which of the following structures would most likely be damaged? -Manubrium of the sternum -Body of the second thoracic vertebra -Spinous process of the third thoracic vertebra -Body of the fourth thoracic vertebra -Transverse process of the second thoracic vertebra

Left superior intercostal

A 37-year-old man is brought to the emergency department complaining of severe chest pain. His angiogram reveals thromboses of both brachiocephalic veins just before entering the superior vena cava. This condition would most likely cause a dilation of which of the following veins? -Azygos -Hemiazygos -Right superior intercostal -Left superior intercostal -Internal thoracic

Right ventricle

A 37-year-old patient with palpitation was examined by her physician, and one of the diagnostic records included a posterior-anterior chest radiograph. Which of the following comprises the largest portion of the sternocostal surface of the heart seen on the radiograph? -Left atrium -Right atrium -Left ventricle -Right ventricle -Base of the heart

Mitral valve insufficiency

A 37-year-old patient with severe chest pain, shortness of breath, and congestive heart failure was admitted to a local hospital. His coronary angiograms reveal a thrombosis in the circumflex branch of the left coronary artery. Which of the following conditions could result from the blockage of blood flow in the circumflex branch? -Tricuspid valve insufficiency -Mitral valve insufficiency -Ischemia of AV node -Paralysis of pectinate muscle -Necrosis of septomarginal trabecula

Elastic tissue in the lungs and thoracic wall

A 43-year-old female patient has been lying down on the hospital bed for more than 4 months. Her normal, quiet expiration is achieved by contraction of which of the following structures? -Elastic tissue in the lungs and thoracic wall -Serratus posterior superior muscles -Pectoralis minor muscles -Serratus anterior muscles -Diaphragm

Loss of sensation in fibrous pericardium and mediastinal pleura

A 44-year-old man with a stab wound was brought to the emergency department, and a physician found that the patient was suffering from a laceration of his right phrenic nerve. Which of the following conditions has likely occurred? -Injury to only GSE fibers -Difficulty in expiration -Loss of sensation in the fibrous pericardium and mediastinal pleura -Normal function of the diaphragm -Loss of sensation in the costal part of the diaphragm

Descending aorta

A 45-year-old woman presents with a tumor confined to the posterior mediastinum. This could result in compression of which of the following structures? -Trachea -Descending aorta -Arch of the aorta -Arch of the azygos vein -Phrenic nerve

Arch of the azygos vein

A 46-year-old patient comes to his doctor's office and complains of chest pain and headache. His computed tomography (CT) scan reveals a tumor located just superior to the root of the right lung. Blood flow in which of the following veins is most likely blocked by this tumor? -Hemiazygos vein -Arch of the azygos vein -Right subclavian vein -Right brachiocephalic vein -Accessory hemiazygos vein

Contains the SA node

A 47-year-old man with a known atrial fibrillation returns to see his cardiologist for follow-up of his cardiac health. The right atrium is important in this case because it: -Receives blood from the oblique cardiac vein -Is associated with the apex of the heart -Contains the SA node -Receives the right pulmonary vein -Is hypertrophied by pulmonary stenosis

Foramen ovale

A 5-year-old girl is brought to the emergency department because of difficulty breathing (dyspnea), palpitations, and shortness of breath. Doppler study of the heart reveals an atrial septal defect (ASD). This malformation usually results from incomplete closure of which of the following embryonic structures? -Ductus arteriosus -Ductus venosus -Sinus venarum -Foramen ovale -Truncus arteriosus

Sinoatrial (SA) node

A 54-year-old patient is implanted with an artificial cardiac pacemaker. Which of the following conductive tissues of the heart had a defective function that required the pacemaker? -Atrioventricular (AV) bundle -AV node -Sinoatrial (SA) node -Purkinje fiber -Moderator band

Anterior interventricular artery

A 56-year-old patient recently suffered a myocardial infarction in the area of the apex of the heart. The occlusion by atherosclerosis is in which of the following arteries? -Marginal artery -Right coronary artery at its origin -Anterior interventricular artery -Posterior interventricular artery -Circumflex branch of the left coronary artery

Chordae tendineae

A 57-year-old patient has a heart murmur resulting from the inability to maintain constant tension on the cusps of the AV valve. Which of the following structures is most likely damaged? -Crista terminalis -Septomarginal trabecula -Chordae tendineae -Pectinate muscle -Anulus fibrosus

Left fifth intercostal space

A 62-year-old patient with pericardial effusion comes to a local hospital for aspiration of pericardial fluid by pericardiocentesis. The needle is inserted into the pericardial cavity through which of the following intercostal spaces adjacent to the sternum? -Right fourth intercostal space -Left fourth intercostal space -Right fifth intercostal space -Left fifth intercostal space -Right sixth intercostal space

Right superior intercostal vein

A 62-year-old woman who is a heavy smoker has an advanced lung cancer that spread into her right third posterior intercostal space posterior to the midaxillary line. If cancer cells are carried in the venous drainage, they would travel first to which of the following veins? -SVC -Right superior intercostal vein -Right brachiocephalic vein -Azygos vein -Hemiazygos vein

Mesoderm

A 7-day-old baby is diagnosed as having congenital neonatal emphysema, which is caused by collapsed bronchi because of failure of bronchial cartilage development. Bronchial cartilages are derived from which of the following derivations? -Ectoderm -Mesoderm -Endoderm -Proctodeum -Neuroectoderm

Superior lobe of the left lung

A 75-year-old patient has been suffering from lung cancer located near the cardiac notch, a deep indentation on the lung. Which of the following lobes is most likely to be excised? -Superior lobe of the right lung -Middle lobe of the right lung -Inferior lobe of the right lung -Superior lobe of the left lung -Inferior lobe of the left lung

Lingula

A 75-year-old woman was admitted to a local hospital, and bronchograms and radiographs revealed a lung carcinoma in her left lung. Which of the following structures or characteristics does the cancerous lung contain? -Horizontal fissure -Groove for superior vena cava (SVC) -Middle lobe -Lingula -Larger capacity than the right

Hemiazygos vein

A 78-year-old patient presents with an advanced cancer in the posterior mediastinum. The surgeons are in a dilemma as to how to manage the condition. Which of the following structures is most likely damaged? -Brachiocephalic veins -Trachea -Arch of the azygos vein -Arch of the aorta -Hemiazygos vein

Left sixth arch

A 9-month-old girl was admitted to the children's hospital with tachypnea (fast breathing) and shortness of breath. Physical examination further exhibits tachycardia (fast heart rate), a bounding peripheral pulse, and her angiographs reveal a patent ductus arteriosus. Which of the following embryonic arterial structures is most likely responsible for the origin of the patent ductus arteriosus? -Right fourth arch -Left fifth arch -Right fifth arch -Left sixth arch -Right sixth arch

Blood flow in coronary arteries is maximal during diastole

A cardiologist is on clinical rounds with her medical students. She asks them, "During the cardiac cycle, which of the following events occurs?" -AV valves close during diastole -Aortic valve closes during systole -Pulmonary valve opens during diastole -Blood flow in coronary arteries is maximal during diastole -Aortic valve closes at the same time as AV valve

Ductus arteriosus

A mother with diabetes gives birth to a baby who is diagnosed as having dextroposition of the aorta and the pulmonary trunk with cyanosis and shortness of breath. Which of the following structures is required to remain patent until surgical correction of the deformity? -Umbilical arteries -Umbilical vein -Ductus arteriosus -Ductus venosus -Sinus venosus

AP septum failed to develop in a spiral fashion

A neonate appears severely cyanotic and breathing rapidly. Cardiac echocardiogram reveals that the aorta lies to the right of the pulmonary trunk. Which of the following is most likely occurred during development? -AP septum failed to develop in a spiral fashion -Excessive resorption of septum primum -Pulmonary valve atresia -Persistent truncus arteriosus -Coarctation of the aorta

Tetralogy of Fallot

A newborn baby is readmitted to the hospital with hypoxia and upon testing is found to have pulmonary stenosis, dextraposition of the aorta, interventricular septal defect, and hypertrophy of the right ventricle. Which of the following is best described by these symptoms? -ASD -Patent ductus arteriosus -Tetralogy of Fallot -Aortic stenosis -Coarctation of the aorta

Bifurcation of the trachea

A patient has a small but solid tumor in the mediastinum, which is confined at the level of the sternal angle. Which of the following structures would most likely be found at this level? -Bifurcation of the trachea -Beginning of the ascending aorta -Middle of the aortic arch -Articulation of the third rib with the sternum -Superior border of the superior mediastinum

SVC

A radiologist examines posterior-anterior chest radiographs of a 27-year-old victim of a car accident. Which of the following structures forms the right border of the cardiovascular silhouette? -Arch of the aorta -Pulmonary trunk -SVC -Ascending aorta -Left ventricle

Ribs 7 and 9

A thoracentesis is performed to aspirate an abnormal accumulation of fluid in a 37-year old patient with pleural effusion. A needle should be inserted at the midaxillary line between which of the following two ribs so as to avoid puncturing the lung? -Ribs 1 and 3 -Ribs 3 and 5 -Ribs 5 and 7 -Ribs 7 and 9 -Ribs 9 and 11

Medial and lateral

A thoracic surgeon removed the right middle lobar (secondary) bronchus along with lung tissue from a 57-year-old heavy smoker with lung cancer. Which of the following bronchopulmonary segments must contain cancerous tissues? -Medial and lateral -Anterior and posterior -Anterior basal and medial basal -Anterior basal and posterior basal -Lateral basal and posterior basal

A 3-year-old male patient presents with a clinically significant atrial septal defect (ASD). The ASD usually results from incomplete closure of which of the following structures? A. Foramen ovale B. Ligamentum arteriosum C. Ductus arteriosus D. Sinus venarum E. Coronary sinus

A. An atrial septal defect (ASD) is a communication between the right and left atria. In the formation of the partition between the two atria, the opening in the foramen secundum, also known as the foramen ovale, typically closes at birth. If it remains patent, an ASD will result. The rest of the structures are not associated with atrial septal defects.

A 2-day-old is diagnosed with transposition of the great arteries. If this condition were to be left untreated for more than 4 months, it would be fatal. Which of the following structures must remain patent so that the infant can survive until surgical correction of the malformation? A. Ductus arteriosus B. Umbilical arteries C. Umbilical vein D. Coarctation of the aorta E. Pulmonary stenosis

A. In a case of transposition of the great arteries, oxygenated blood travels from the left ventricle into the pulmonary trunk, where it will eventually reach the lungs. In contrast, the aorta would be carrying deoxygenated blood into the systemic circulation. A patent ductus arteriosus acts as a shunt between the aorta and pulmonary trunk, allowing oxygenated and deoxygenated blood to mix and therefore allowing some oxygenated blood to reach the tissues. None of the other answer choices would correct this problem; with these structures remaining patent, the body would still not receive sufficient oxygenated blood for survival to be possible.

A 32-year-old woman in her third trimester of pregnancy is undergoing a routine ultrasound examination. The examination of the fetus reveals enlarged and echogenic lungs, inverted diaphragm, and fetal ascites. Which condition is best characterized by these signs? A. Laryngeal atresia B. Tracheal atresia C. Polyhydramnios D. Lung hypoplasia E. Oligohydramnios

A. Laryngeal atresia (congenital high airway obstruction syndrome) is a rare obstruction of the upper fetal airway. Distal to the site of the atresia, the airways dilate, lungs enlarge and become echogenic, the diaphragm flattens or inverts, and fetal ascites and/or hydrops develop. Tracheal atresia is a rare obstruction of the trachea, commonly found with a tracheoesophageal fistula, probably resulting from the unequal division of foregut into esophagus and trachea. Polyhydramnios is an excess of amniotic fluid, often associated with esophageal atresia or tracheoesophageal fistula. Lung hypoplasia is reduced lung volume, often seen in infants with a congenital diaphragmatic hernia. Oligohydramnios, or a decrease in amniotic fluid, is associated with stunted lung development and pulmonary hypoplasia.

A 2-day-old newborn female is diagnosed with pulmonary stenosis, overriding of the aorta, ventricular septal defect, and hypertrophy of the right ventricle. Which of the following embryologic mechanisms is most likely responsible for the development of this cluster of anomalies? A. Superior malalignment of the subpulmonary infundibulum B. Defect in the aorticopulmonary septum C. Endocardial cushion defect D. Total anomalous pulmonary venous connections E. Atrioventricular canal malformation

A. Superior malalignment of the subpulmonary infundibulum causes stenosis of the pulmonary trunk. This leads to the four symptoms mentioned and is known as tetralogy of Fallot. A defect in formation of the aorticopulmonary septum is characteristic of transposition of the great arteries. An endocardial cushion defect is associated with membranous ventricular septal defects.

A 3-month-old infant is diagnosed with a deletion at the 22q11 chromosome. A routine cardiovascular examination reveals severe congenital cardiac malformation. Which of the following malformations will most likely be associated with 22q11 syndrome? A. Tetralogy of Fallot and truncus arteriosus B. Transposition of the great arteries C. Atrial septal and ventricular septal defects D. Coarctation of the aorta E. Aortic atresia

A. Tetralogy of Fallot and truncus arteriosus are associated with DiGeorge syndrome (22q11). Transposition of the great arteries is associated with maternal diabetes. ASDs and VSDs are present in individuals with Down syndrome. Coarctation of the aorta is related to Turner syndrome. Marfan syndrome is present in individuals with aortic atresia.

A 2-day-old newborn female is diagnosed with pulmonary stenosis, overriding of the aorta, ventricular septal defect, and hypertrophy of the right ventricle. Which condition is best characterized by these signs? A. Tetralogy of Fallot B. Atrial septal defect C. Transposition of the great vessels D. Pulmonary atresia E. Ventricular septal defect

A. Tetralogy of Fallot is always characterized by four cardiac defects: pulmonary stenosis, ventricular septal defect (VSD), overriding aorta, and these in turn lead to right ventricular hypertrophy. An atrial septal defect (ASD) is characterized by the communication between the two atria. In a case of transposition of the great vessels, the aorta arises from the right ventricle and the pulmonary trunk arises from the left ventricle.

A 2-year-old child is seen in the pediatric cardiology unit for a congenital heart condition. Which of the following conditions occurs most often? A. Membranous ventricular septal defect B. Tetralogy of Fallot C. Muscular ventricular septal defect D. Ostium secundum defect E. Ostium primum defect

A. Ventricular septal defects account for 25% of congenital heart defects. The most common of these are defects in the membranous portion of the interventricular septum (membranous ventricular septal defects).

A 46-year-old woman is admitted to the hospital with a noticeable bulge of tissue through her vaginal opening. During physical examination a rectocele is identified. Which of the following is most likely responsible for this condition? A. Compromised rectovaginal septum B. Weakened superficial and deep transverse perineal muscles C. Paralyzed ischiocavernosus muscle D. Loose sacrospinous ligament E. Ruptured sphincter urethra

A. A break or tear in the rectovaginal septum (fascia of Denonvilliers) can allow small intestine (in an enterocele) or rectum (in a rectocele) to herniate into the posterior vaginal wall, even to the point of protrusion through the vaginal introitus. The muscles listed are all in the anterior region of the perineum and have no association with an enterocele or recto- cele. The sacrospinous ligament is unrelated to this condition.

A lateral blow to the knee during a tackle in a football game injures a 24-year-old woman. Field examination reveals an "anterior drawer sign." An MRI demonstrates injury to several structures of the knee, including her medial meniscus. Which structure might also have been injured by the tackle? A. Medial collateral ligament B. Lateral collateral ligament C. Lateral meniscus D. Posterior cruciate ligament E. Tendon of the semitendinosus

A. A lateral blow to the knee could result in injury to three structures in the knee: anterior cruciate ligament, medial collateral ligament, and the medial meniscus. When all three structures are involved it is collectively called the "unhappy triad." Anterior drawer sign is due to injury to the anterior cruciate ligament and denoted by anterior displacement of the tibia in relation to femur, similar to pulling out a drawer from a desk.

A lateral blow to the knee during a tackle in a football game injures a 24-year-old woman. Field examination reveals an "anterior drawer sign." An MRI demonstrates injury to several structures of the knee, including her medial meniscus. Which structure might also have been injured by the tackle? A. Medial collateral ligament B. Lateral collateral ligament C. Lateral meniscus D. Posterior cruciate ligament E. Tendon of the semitendinosus

A. A lateral blow to the knee often produces a trio of injuries referred to as the "unhappy triad." This involves damage to the anterior cruciate ligament, medial meniscus, and medial collateral ligament. The medial meniscus and medial collateral ligament are often damaged together, as they are tightly attached to each other. The lateral collateral ligament and lateral meniscus would not be damaged because a blow to the lateral knee would not put strain on these structures. Damage to the posterior cruciate ligament would produce a positive "posterior drawer sign" and is typically damaged during a blow to the medial side of the knee. The posterior cruciate ligament is stronger than the anterior and is only typically damaged when a person falls on the tibial tuberosity of a flexed knee. Tendon of semitendinosus is on the medial side of the knee but is not attached closely to the other structures or taut in this injury type.

A 20-year-old woman is trying to become pregnant. If she does, a blastocyst will form early in her pregnancy. Which of following statements accurately describes the formation of the blastocyst? A. Blastomere → morula → blastocyst B. Morula → zygote → blastocyst C. Blastomere → blastocyst D. Zygote → blastocyte → blastocyst E. Morula → blastocyte → blastocyst

A. After fertilization of the ovum, cleavage commences with the formation of blastomere and then a 16-cell morula. Fluid would then enter the cavity of the morula with the formation of a blastocyst. The blastocyst will then invade the endometrial wall.

A 45-year-old man after being diagnosed with a posterior acetabular fracture is taken to the operating room to repair the fracture. During the neurological examination the physician notices loss of sensation to the skin of the inferior half of the buttocks, posterior, and upper medial thigh. The patient had a normal neurovascular examination preoperatively. Which of the following nerves was mostly likely damaged during the operation? A. Posterior femoral cutaneous B. Obturator nerve C. Sciatic D. Femoral E. Lateral femoral cutaneous

A. All the branches of the posterior femoral cutaneous nerve are cutaneous. It arises from the dorsal divisions of the first and second and the ventral divisions of the second and third sacral nerves and travels through the greater sciatic foramen beneath the piriformis muscle to innervate the shin over the lower parts of the gluteus maximus muscle through the inferior clunial nerves and the posterior surface of the thigh and leg and perineum via its perineal branches. The lateral femoral cutaneous nerve, which innervates the lateral surface of the thigh, is an incorrect choice; the obturator, sciatic, and femoral nerves all have both motor and sensory branches and can be eliminated, since no motor deficits were described.

A 39-year-old woman who is a school teacher unwittingly sits on a thumbtack a student placed on her chair. Her left buttock becomes painful and inflamed. Which group of nodes will first receive lymph from the infected wound? A. Superficial horizontal group B. Superficial vertical group C. Superior and inferior gluteal nodes D. External iliac E. Deep inguinal

A. Any superficial inflammation in the gluteal region drains into the superficial horizontal group of inguinal nodes. The vertical group drains the lower limbs, whereas deep gluteal injuries drain into the superior and inferior gluteal nodes.

Very often the first indication that a woman has that she is pregnant is a missed menstrual period. In which week of embryonic development does a woman experience her first missed menstrual period? (A) Start of week 3 (B) Start of week 4 (C) Start of week 5 (D) Start of week 8 (E) End of week 8

A. Assuming a regular 28-day menstrual cycle, a woman who starts menses on February 1, for example, will ovulate on February 14, and the secondary oocyte is fertilized within 24 hours. So, the zygote undergoes week 1 of development from February 15 to 21. Week 2 of development is from February 22 to 28. On March 1, the woman should enter her next menstrual cycle, but because she is pregnant, she will not menstruate. Therefore, this first missed menstrual period corresponds with the start of week 3 of embryonic development. The embryonic period (week 3-8) is a time of high susceptibility to teratogens.

A 58-year-old postmenopausal woman complains of pelvic discomfort and dull pain. Lymph nodes from the sacral, internal iliac, and inguinal lymph node groups are surgically removed for histopathologic examination. The pathology report reveals positive cancerous cells only in the inguinal lymph nodes. Which pelvic organ would most likely be involved in the cancer? A. The body of the uterus B. Distal rectum C. One or both of her ovaries D. Proximal rectum E. Anal canal superior to the pectinate line

A. Cancer present in the inguinal nodes can be indicative of cancer of the uterus at the level of the round ligaments, by which the cancer passes to the inguinal region. Uterine cancer must be especially sus- pected if the tissues of the lower limb, vulva, and anal canal appear normal. The pectinate line marks the end of the mucosal lining of the anal canal, below which the canal is lined with nonkeratinized, stratified squa- mous epithelium. The pectinate line is also associated with the distal ends of the anal columns and anal valves. Lymphatic vessels inferior to the pectinate line of the anal canal will drain into the superficial inguinal nodes, but those above the pectinate line flow to inter- nal iliac nodes. Lymph from the ovaries flows to the paraaortic nodes at the level of the kidneys. Lymph from the rectum flows to pelvic lymph nodes.

A 22-year-old male complained to the urologist of pain that he experiences from bladder fullness after drinking large quantities of fluid. What is the location of the neural cell bodies responsible for pain sensation from the urinary bladder? A. Dorsal root ganglia of spinal cord levels S2, S3, and S4 B. The intermediolateral cell column of spinal cord levels S2, S3, and S4 C. The sensory ganglia of spinal nerves T5 to T9 D. The preaortic ganglia at the site of origin of the testicular arteries E. Dorsal root ganglia of spinal levels T10 to L2

A. Conscious pain due to bladder fullness results from the excitation of stretch receptors in the bladder wall. These pain fibers are carried through the pelvic nerve plexuses and into the pelvic splanchnic nerves. The sensory fibers enter the dorsal root ganglia of spinal nerves S2, S3, and S4. Sensory fibers enter the spinal cord via these ganglia. The intermediomedial cell column of spinal cord levels S2, S3, and S4 contains parasympathetic soma. The levels T5 to T9, T10 to L2, and preaortic ganglia are well above where sensory fibers from the bladder are located.

A 50-year-old diabetic man presents for a routine wellness checkup. During physical examination it is noted that he has paraesthesia in a classic glove and stocking distribution. The physician decides on a complete peripheral vascular system examination, which includes palpating the pulse of the dorsalis pedis. Where can the dorsalis pedis pulse be palpated? A. Between the tendons of extensor hallucis and extensor digitorum longus on the dorsum of the foot B. Superior to flexor hallucis longus just distal to the tarsal tunnel C. Inferolateral to the pubic symphysis and medial to the deep dorsal vein of the penis D. 2 cm anterior to the medial malleolus E. 2 cm posterior to the medial malleolus

A. Dorsalis pedis pulse is palpated at the prominent arch of the top of the foot between the first and second metatarsal bones between the tendon of the extensor hallucis longus and extensor digitorum longus for the second toe.

A 47-year-old woman with a family history of breast cancer mutations in the BRCA1 and BRCA2 genes has made a decision to have an elective hysterectomy as a prophylactic treatment. During ligation of the uterine artery which of the following adjacent structures must the surgeon be careful not to injure? A. Ureter B. Internal iliac artery C. Internal iliac lymph nodes D. Obturator nerve E. Lumbosacral trunk

A. During a hysterectomy, ligation of or injury to the ureter can happen relatively easily because it is the most susceptible structure due to its location. The ureter is located below the uterine vessels in the pelvic cavity approximately 1 cm lateral to the supra- vaginal cervix. The internal iliac artery bifurcates near the pelvic brim but is not in close proximity to the uterine vessels in the vicinity of the cervix. The obturator nerve travels along the pelvic sidewall and is not close to the site of ligation of the uterine ves- sels. The lumbosacral trunk is located on the lateral side of the sacrum and the pelvic sidewall, not in close proximity to the uterine vessels.

A 32-year-old woman is admitted to the hospital due to a palpable mass superior to the inguinal ligament. It is demonstrated by physical examination that she suffers from an indirect inguinal hernia. Due to the severity of the hernia, an open hernia repair is per- formed. Which of the following structures would the surgeon expect to find in the canal? A. Round ligament of uterus B. Urachus C. Suspensory ligament of the ovary D. Uterine tube E. Mesosalpinx

A. During repair, the round ligament of the uterus may be seen within the inguinal canal, al- though it is often a small, fibrous strand that is easily overlooked. The remaining choices are not found in this region. The ovarian ligament connects the ovary to the uterus, whereas the suspensory (infundibulo- pelvic) ligament contains the ovarian vessels, nerves, and lymphatic. The uterine tubes are lateral projec- tions of the uterus toward the ovaries. The mesosal- pinx is a portion of the peritoneum of the broad liga- ment that attaches to the uterine tubes.

During week 2 of development, the embryoblast receives its nutrients via (A) diffusion (B) osmosis (C) reverse osmosis (D) fetal capillaries (E) yolk sac nourishment

A. During week 2 of development, the embryoblast receives its nutrients from endometrial blood vessels, endometrial glands, and decidual cells via diffusion. Diffusion of nutrients does not pose a problem given the small size of the blastocyst during week 2. *Although the beginnings of a uteroplacental circulation are established during week 2, no blood vessels have yet formed in the extraembryonic mesoderm to carry nutrients directly to the embryoblast (this occurs in week 3).*

A 32-year-old woman visits an obstetrician complaining of significant bleeding in her first trimester and pain in the lower back. Ultrasound confirms the diagnosis of an abdominal pregnancy. Where is the most common place for an ectopic abdominal pregnancy? A. Pouch of Douglas B. Pouch of Morison C. Retropubic space D. Extraperitoneal space E. Retroperitoneal space

A. Ectopic pregnancy is the existence of a pregnancy outside the normal confines of the uterus or abnormally in the uterus. Although the uterine (fallopian) tube is the most common site of an ectopic pregnancy, it can also occur in the abdominal cavity, and when that is the case, the pouch of Douglas in the most common site.

A 4-month-old infant is admitted to the pediatric clinic because urine can be observed passing through an opening on the dorsum of the penis. Which of the following embryologic structures failed to fuse? A. Spongy urethra B. Labioscrotal folds C. Cloacal membrane D. Urogenital folds E. Genital tubercle

A. Epispadias is a developmental defect in the spongy urethra resulting in urine being expelled from the dorsal aspect of the penis. A failure of the labioscrotal folds to fuse will cause the external ure- thral orifice to be situated between the two scrotal halves. This is referred to as penile hypospadias. Failure of the urogenital folds to fuse would lead to agenesis of the external urethral folds. The genital tubercle would not directly cause epispadias, as the tubercle still continues to develop, but it is located more dorsally.

A 58-year-old female employee of a housecleaning business visits the outpatient clinic with a complaint of a constant burning pain in her knees. Clinical examinations reveal a "housemaid's knee" condition (Fig. 5-9). Which of the following structures is most likely affected? A. Prepatellar bursa B. Infrapatellar bursa C. Posterior cruciate ligament D. Patellar retinacula E. Lateral meniscus

A. Excessive compression of the prepatellar bursa, as in working on bended knees, can result in pain and swelling of the prepatellar bursa, the so called housemaid's knee. Prepatellar bursitis affects plumbers, carpet layers, and other people who spend a lot of time on their knees. The bursa normally enables the patella to move smoothly under the skin. The constant friction of these occupations irritates this small lubricating sac (bursa) located just in front of the patella, resulting in a deformable tense cushion of fluid. Treatment usually requires simple drainage, but this may need to be repeated and occasionally steroids introduced. Excessive irritation of the infrapatellar bursa in kneeling for frequent and long periods of time (as in prayer) can result in "parson's knee." The posterior cruciate ligament of the knee can be injured in sudden, strong flexion of the knee, with posterior displacement of the tibia upon the femur. The patellar retinacula are strong, tendinous bands of tissue that join the quadriceps tendon to the vastus lateralis and medialis muscles. The lateral meniscus is a cartilaginous structure between the lateral condyles of the femur and tibia.

A 16-year-old woman is visiting her gynecologist for her first checkup. On ultrasound examination it is noted that the woman has a double uterus. Failure of which of the following processes is responsible for the double uterus? A. Fusion of the inferior parts of the paramesonephric ducts B. Fusion of the superior parts of the mesonephric ducts C. Development of the hymen D. Development of the sinovaginal bulbs E. Fusion of the inferior parts of the mesonephric ducts

A. Failure of fusion of the inferior parts of the paramesonephric ducts results in a double uterus. A bicornuate uterus is the result of failure of fusion of the superior parts of the paramesonephric (Müllerian) ducts. A failure of the sinovaginal bulbs to form the vaginal plate causes agenesis of the vagina. The mesonephric ducts are important embryologic structures involved in the development of male uro- genital structures.

A 55-year-old man visits the outpatient clinic complaining that he cannot walk more than 5 minutes without feeling severe pain in his feet. An image of the feet of this patient is shown in Fig. 5-10. What is the most common cause of this condition? A. Collapse of medial longitudinal arch, with eversion and abduction of the forefoot B. Exaggerated height of the medial longitudinal arch of the foot C. Collapse of long plantar ligament D. Collapse of deltoid ligament E. Collapse of plantar calcaneonavicular ligament

A. Flat foot (pes planus) is due to flattening of the medial longitudinal arch. Often congenital, it may be associated with minor structural anomalies of the tarsal bones. This condition can be seen in wet footprints in which the medial surface of the sole (normally raised in an arch) is visible. Treatment may include intensive foot exercises or arch supports worn in the shoes. Occasionally, surgery is needed in the form of arthrodesis (fusion of the tarsal bones). Pes cavus is a deformity of the foot characterized by a very high medial arch and hyperextension of the toes. The long plantar ligament is a passive ligament of the longitudinal arch. The long plantar ligament connects the calcaneus and cuboid bones. It can be involved with the plantar aponeurosis in plantar fasciitis. The long plantar ligament converts the cuboid groove into a canal for the tendon of the fibularis (peroneus) longus. The deltoid ligament is a very strong ligament that interconnects the tibia with the navicular, calcaneus, and talus bones. The medial malleolus will usually fracture before this ligament will tear. The plantar calcaneonavicular, or spring, ligament is a key element in the medial longitudinal arch; it supports the head of the talus bone and thereby is subject to vertical forces exerted through the lower limbs. In the present case, the bilateral pes planus appears to be the res

A 22-year-old woman is admitted with high fever and vaginal discharge. Physical and laboratory examinations reveal gonorrheal infection. A series of intramuscular antibiotic injections are ordered. Into which of the following parts of the gluteal region should the antibiotic be injected to avoid nerve injury? A. Anterior and superior to a line between the posterior superior iliac spine and the greater trochanter B. In the middle of a line between the anterior superior iliac spine and the ischial tuberosity C. Inferolateral to a line between the posterior superior iliac spine and the greater trochanter D. Inferomedial to a line between the posterior superior iliac spine and the greater trochanter E. Halfway between the iliac tuberosity and the greater trochanter

A. Gluteal injections should be given anterior and superior to a line drawn between the posterior superior iliac spine and the greater trochanter to avoid the sciatic nerve and other important nerves and vessels. Occasionally, one can encounter the lateral cutaneous branch of the iliohypogastric nerve, but this usually causes no serious problem. Certainly, one must stay anterior to a vertical line dropped from the highest point of the ilium. If the injected material is too near the sciatic nerve or other motor nerves, it can infiltrate the connective tissue sheath of the nerve, following the nerve, and result in major insult to the neural elements. The needle can cause trauma to this, or other nerves, likewise. Precautions to avoid the sciatic nerve are especially important in injecting the gluteal area in babies. The reduced dimensions are less "forgiving" in babies.

A 43-year-old victim of a drunk driving car crash is undergoing reconstructive arm surgery. The surgeon performs an autograft using a weak adductor of the leg located superficially on the medial side of the thigh. Which muscle is most likely being harvested to perform this reconstruction? A. Gracilis B. Sartorius C. Rectus femoris D. Vastus lateralis E. Vastus medialis

A. Gracilis due to its shape, size, and more importantly the nature of neurovascular supply is used very commonly in reconstructive surgery as a free functioning autograft. Also, the other adductors of the thigh compensate for the absence of the gracilis. For similar reasons the remaining muscles are not good candidates during reconstructive surgery of the upper limb.

A 49-year-old man is admitted to the emergency department complaining that he has difficulties walking. Physical examination reveals that the patient suffers from peripheral vascular disease. An ultrasound examination reveals an occlusion of his femoral artery at the proximal portion of the adductor canal. Which of the following arteries will most likely provide collateral circulation to the thigh? A. Descending branch of the lateral circumflex femoral B. Descending genicular C. Medial circumflex femoral D. First perforating branch of deep femoral E. Obturator artery

A. If the femoral artery is occluded, the descending branch of the lateral circumflex femoral will provide collateral circulation to the thigh. The descending genicular artery is a branch of the femoral and therefore would also be occluded. The medial circumflex femoral artery is a proximal branch of the deep femoral artery and supplies part of the head of the femur. The first perforating branch of the deep femoral artery supplies a small portion of the muscles of the posterior thigh. Finally, the obturator artery supplies a very small artery and vascularizes only the most proximal part of the head of the femur and usually only during the early years of life.

A 15-year-old boy falls and injures his ankle while skateboarding. Examination in the emergency department leads to the conclusion that the ankle is mildly sprained, and it is wrapped with an elastic bandage. The boy still complains of pain in his ankle. Which of the following peripheral nerves is involved in carrying pain sensation from the ankle? A. Deep fibular (peroneal) B. Femoral C. Obturator D. Posterior femoral cutaneous E. Sural

A. Knowledge of Hilton's Law would lead to this correct answer. This law in a modified form, can be remembered as "a joint is innervated by the same nerves that innervate the muscles that move that joint." A complete explanation of this law can be found in an article by Hebert-Blouin et al., Clinical Anatomy 27:548-555, 2013. The deep fibular (peroneal) nerve is the only nerve listed that innervates muscles that move the ankle joint.

A 58-year-old postmenopausal woman is diagnosed with carcinoma of the distal gastrointestinal tract. During surgery lymph nodes from the sacral, internal iliac, and inguinal lymph node groups were removed and sent for histopathologic examination. The pathology report revealed positive cancerous cells only at the inguinal lymph nodes. Which of the following parts of the gastrointestinal tract were most likely affected? A. Cutaneous portion of anal canal B. Distal rectum C. Mucosal zone of anal canal D. Pectinate line of anal canal E. Proximal rectum at the inferior valve (of Houston)

A. Lymph from the cutaneous portion of the anal canal (below the pectinate line) drains into the inguinal nodes. Lymph from most parts of the rectum and from the mucosal zone of the anal canal (above the pectinate line) drains into the internal iliac nodes. Lymph from some parts of the rectum also drains into the sacral nodes.

A 39-year-old woman is admitted to the emergency department with a painful foot. Radiologic examination reveals a Morton's neuroma. What is the most typical location of this neuroma? A. Between the third and fourth metatarsophalangeal joints B. Between the second and third metatarsophalangeal joints C. Between the first and second metatarsophalangeal joints D. Between the fourth and fifth metatarsophalangeal joints E. In the region of the second, third, and fourth metatarsophalangeal joints

A. Morton's neuroma most commonly involves compression (and possible enlargement) of an anastomosing branch that connects the medial and lateral plantar nerves between the third and fourth toes. The pain can be severe. The medial plantar nerve provides sensation for the medial three and a half toes; the lateral plantar nerve supplies the little toe and half of the fourth toe. The neural interconnection can be compressed between the transverse metatarsal ligament and the floor. Women are 10 times more likely than men to be afflicted with this problem, most likely due to wearing shoes that put excessive stress on the forefoot. In about 80% of cases the pain can be eased with different (less confining) shoes or cortisone injections.

A 34-year-old woman is admitted to the hospital complaining of urinary incontinence. MRI examination reveals that one of the skeletal muscles of the pelvis has a significant tear. Which of the following muscles is the most significant in terms of maintaining continence? A. Pubococcygeus B. Obturator internus C. Piriformis D. Coccygeus E. Iliococcygeus

A. Of the answer choices listed, the pubococcygeus is the muscle that is most directly associated with the arcus tendineus fascia pelvis and connective tissues of the vagina and the support of the bladder. The obturator internus, piriformis, and coccygeus do not form parts of the levator ani and provide no direct support to the urogenital organs, nor do they have any role in urinary incontinence. The iliococcygeus does form part of the levator ani, but it is located lateral to the pubococcygeus and therefore does not play a direct role in maintaining urinary continence.

A 38-year-old woman visits her gynecologist for a routine Pap smear examination. During the collection of cells from her uterine cervix she feels a mild pain. Which of the following areas is most likely to experience "referral of pain" during this procedure? A. Perineum and lateral portion of the thigh B. Suprapubic region C. Umbilical region D. Inguinal region E. Epigastric region

A. Pain from this area is mediated via parasym- pathetic responses and would thus travel to the S2 to S4 levels through the pelvic splanchnic nerves. The S2, S3, and S4 spinal cord levels also provide sensory innervation of the perineum and posterior thigh. The suprapubic and inguinal regions are supplied by ilio- inguinal and iliohypogastric nerves (L1). The umbili- cal region receives sensory innervation from the T10 level. In the epigastric region the sensory innervation is provided by T7 to T10.

A 4-month-old male infant is admitted to the pediatric clinic because he was passing urine near the anus rather than from the tip of the penis. Physical examination reveals that the patient has perineal hypospadias. Which of the following embryologic structures failed to fuse? A. Labioscrotal folds B. Cloacal membrane C. Urogenital folds D. Genital tubercle E. Urogenital membrane

A. Perineal hypospadias is due to a failure of fu- sion of labioscrotal folds so that the external urethral orifice is between the unfused halves of the scrotum. The cloacal membrane is formed from endoderm of the cloaca and ectoderm of the procotodeum and forms the future anus. The urogenital folds normally fuse along the ventral side of the penis to form the spongy urethra. Epispadias is a condition in which the urethra opens on the dorsal surface of the penis resulting from the genital tubercle developing more dorsally during development. The urogenital mem- brane is bounded by the urogenital folds and ruptures to form the urogenital orifice.

A 28-year-old woman visits an obstetrician for in vitro fertilization. FSH-analogs are injected to stimulate follicles. Then hCG is injected to induce final oocyte maturation. Lastly, the oocyte is retrieved via a procedure called transvaginal oocyte retrieval. The retrieved oocyte is arrested at what stage of development? A. Prophase of meiosis 1 B. Metaphase of meiosis 1 C. Prophase of meiosis 2 D. Metaphase of meiosis 2 E. Prophase of meiosis 2

A. Primary oocytes begin the first meiotic divisions before birth, but completion of prophase in meiosis 1 does not occur until adolescence. The follicular cells surrounding the primary oocytes secrete a substance, oocyte maturation inhibitor, which arrests the meiotic process of the oocyte.

In the production of male gametes, which of the following cells remains dormant for 12 years? (A) Primordial germ cell (B) Primary spermatocyte (C) Secondary spermatocyte (D) Spermatid (E) Sperm

A. Primordial germ cells migrate from the wall of the yolk sac during week 4 of embryonic life and enter the gonad of a genetic male, where they remain dormant until puberty (about age 12) when hormonal changes in the young man stimulate the production of sperm

A 55-year-old cowboy is admitted to the emergency department after he was knocked from his feet by a young longhorn steer. MRI examination reveals a large hematoma in the knee joint. Physical examination reveals that the patient suffers from the "unhappy triad" (of O'Donoghue). Which of the following structures are involved in such an injury? A. Medial collateral ligament, medial meniscus, and anterior cruciate ligament B. Lateral collateral ligament, lateral meniscus, and posterior cruciate ligament C. Medial collateral ligament, lateral meniscus, and anterior cruciate ligament D. Lateral collateral ligament, medial meniscus, and anterior cruciate ligament E. Medial collateral ligament, medial meniscus, and posterior cruciate ligament

A. The "unhappy triad" (of O'Donoghue) is composed of the medial collateral ligament, medial meniscus, and anterior cruciate ligament. Sudden, forceful thrusts against the lateral side of the knee put tension on the medial collateral ligament, which can then rupture. The medial meniscus is attached to the medial collateral ligament so that it then tears. The anterior cruciate ligament resists hyperextension of the knee; thus, it is the third structure that ruptures in the "unhappy triad" of the knee.

In an accident during cleanup of an old residential area of the city, the Achilles tendon of a 32-year-old worker was cut through by the blade of a brush cutter. The patient is admitted to the hospital and a laceration of the Achilles tendon is diagnosed. Which of the following bones serves as an insertion for the Achilles tendon? A. Calcaneus B. Fibula C. Cuboid D. Talus E. Navicular

A. The Achilles tendon inserts upon the calcaneus bone. This tendon represents a combination of the tendons of gastrocnemius and soleus muscles. The tendon of the plantaris can insert with this tendon.

A 58-year-old female dancer presented to the orthopedic clinic with a complaint of pain during her work because of bilateral bunions. She was referred to a podiatric surgeon who scheduled her for surgery. The protruding bony and soft tissues of the toe were excised, and a muscle was reflected from the lateral side of the proximal phalanx, together with a sesamoid bone, upon which the muscle also inserted. What muscle was this? A. Adductor hallucis B. Abductor hallucis C. First dorsal interosseous D. First lumbrical E. Quadratus plantae

A. The adductor hallucis muscle inserts upon the lateral side of the proximal phalanx of the great toe, and also the lateral sesamoid bone, by way of its oblique and transverse heads. It is supplied by the lateral plantar nerve. The abductor hallucis inserts upon the medial side of the proximal phalanx and the medial sesamoid bone of the great toe. The sesamoid bones are within the tendon of the flexor hallucis brevis and assist it in its function at the first metatarsophalangeal joint. The abductor and flexor hallucis brevis are innervated by the medial plantar nerve. The first dorsal interosseous muscle and the first lumbrical both insert on the medial side of the extensor mechanism of the second toe. The quadratus plantae arises from the calcaneus and inserts on the tendon of the flexor digitorum longus muscle. The first lumbrical is supplied by the medial plantar nerve. The quadratus plantae, the lumbricals 2 to 4, and all interossei are innervated by the lateral plantar nerve

A 29-year-old man is brought to the physician for removal of a cast from his left leg. He had sustained a fracture of the left lower extremity 6 weeks prior which was immobilized in a cast that extended from just below the knee to the foot. At the time of injury, there was severe pain but normal strength in the extremity. When the cast was removed, physical examination showed a pronounced left foot drop with paresthesia and sensory loss over the dorsum of the left foot and lateral leg. Injury to which nerve is the most likely cause? A. Common fibular (peroneal) B. Superficial fibular (peroneal) C. Deep fibular (peroneal) D. Sciatic E. Tibial

A. The common fibular (peroneal) nerve is a branch of the sciatic nerve. It descends on the lateral side of the popliteal fossa before winding around the head of the fibula. It then divides into superficial and deep nerves that supply the lateral and anterior compartments of the leg respectively. Due to its superficial course, it is easily injured in patients with long leg casts (which run from just below the knee). The nerve supplies the dorsiflexors of the leg, the skin of the first web space (via the deep fibular), the evertors of the foot, and the skin of the lateral side of the leg and dorsum of the foot (via the superficial fibular).

Radiographic studies of a 42-year-old woman reveal that she has a vulvar malignancy involving the clitoris. Removal of all affected lymph nodes would be indicated to avoid spread of this cancer. Which are the first lymph nodes to filter the lymphatic drainage of the involved area? A. Superficial and deep inguinal lymph nodes B. Internal iliac nodes C. Paraaortic lymph nodes D. Presacral lymph nodes E. Axillary lymph nodes

A. The deep inguinal lymph nodes drain the glans clitoris and receive lymph also from superficial nodes. The internal iliac nodes drain the inferior pel- vic structures, deep perineal structures, and sacral nodes. The paraaortic lymph nodes, or lumbar nodes, receive lymph from the common iliac nodes. The drainage of presacral lymph nodes can pass to the common or internal iliac nodes. Axillary nodes drain body wall structures above the T10 dermatome (or the umbilicus).

A 49-year-old male construction worker is admitted to the emergency department with a painful lump on the proximal medial aspect of his thigh. Radiologic and physical examinations reveal that the patient has a herniation of abdominal viscera beneath the inguinal ligament into the thigh. Through which of the following openings will a hernia of this type initially pass to extend from the abdomen into the thigh? A. Femoral ring B. Superficial inguinal ring C. Deep inguinal ring D. Fossa ovalis E. Obturator canal

A. The femoral ring is the abdominal opening of the femoral canal. A femoral hernia passes through the femoral ring into the femoral canal deep and inferior to the inguinal ligament. It can appear as a bulging at the saphenous hiatus (fossa ovalis) of the deep fascia of the thigh, the hiatus through which the saphenous vein passes to the femoral vein. The superficial inguinal ring is the triangular opening in the aponeurosis of the external abdominal oblique and lies lateral to the pubic tubercle. The deep inguinal ring lies in the transversalis fascia lateral to the inferior epigastric vessels. Herniation into either of these two openings is associated with an inguinal hernia. The obturator canal, a bony opening between the superior and inferior ramus of the pubic bone, is the site of an obturator hernia.

A 25-year-old man, an intravenous drug abuser, had been injecting himself with temazepam (a powerful intermediate acting drug in the same group as diazepam (Valium) and heroin for 5 years, leaving much residual scar tissue over points of vascular access. The patient is admitted to the emergency department for a detoxification program requiring an intravenous infusion. The femoral veins in his groin are the only accessible and patent veins for intravenous use. Which of the following landmarks is the most reliable to identify the femoral veins? A. The femoral vein lies medial to the femoral artery. B. The femoral vein lies within the femoral canal. C. The femoral vein lies lateral to the femoral artery. D. The femoral vein lies directly medial to the femoral nerve. E. The femoral vein lies lateral to the femoral nerve.

A. The femoral vein lies medial to the femoral artery in the femoral sheath. The femoral sheath is broken into three compartments: lateral, intermediate, and medial. The lateral compartment contains the femoral nerve. The medial compartment encloses the femoral canal and consists of lymphatic tissue and a lymph node, plus areolar tissue. The intermediate contains the femoral vein.

The first indication of gastrulation in the embryo is (A) formation of the primitive streak (B) formation of the notochord (C) formation of the neural tube (D) formation of extraembryonic mesoderm (E) formation of tertiary chorionic villi

A. The formation of the primitive streak on the dorsal surface of the bilaminar embryonic disk is the first indication of gastrulation.

A 43-year-old woman receives deep intramuscular injections for the past week for treatment of a sexually transmitted disease. She complains to her doctor that she has difficulty walking. During physical examination her right hip drops every time she raises her right foot. Which of the following injection locations will most likely correspond with the clinical presentation of this patient? A. Superomedial quadrant of the buttock B. Superolateral quadrant of the buttock C. Inferomedial quadrant of the buttock D. Inferolateral quadrant of the buttock E. Posterior thigh

A. The gluteal region (buttocks) is a common site for intramuscular injection of drugs, particularly if the volume of the injection is large. To avoid injury to the underlying sciatic nerve, the injection should be given well forward on the upper outer quadrant of the buttock (superolateral quadrant). The patient is showing the Trendelenburg gait pattern (or gluteus medius lurch), which is caused by weakness of the gluteus medius and minimus muscles. These muscles are supplied by the superior gluteal nerve (L4, L5, S1), which emerges from the greater sciatic notch above the upper border of the piriformis and immediately disappears beneath the posterior border of the gluteus medius and runs forward between the gluteus medius and minimus. Intramuscular injection in the upper inner quadrant (superomedially) is most likely to damage this nerve. The sciatic nerve is most likely damaged in the inferomedial quadrant of the buttock.

A 32-year-old patient received a badly placed intramuscular injection to the posterior part of his gluteal region. The needle injured a motor nerve in the area. Later, he had great difficulty rising to a standing position from a seated position. Which muscle was most likely affected by the injury? A. Gluteus maximus B. Gluteus minimus C. Hamstrings D. Iliopsoas E. Obturator internus

A. The gluteus maximus is innervated by the inferior gluteal nerve, and this muscle is responsible for extension and lateral rotation of the thigh. It is the primary muscle that extends the flexed hip and is used to rise from a seated position. The gluteus minimus is innervated by the superior gluteal nerve and is responsible for abduction of the thigh. Hamstring muscles are innervated by the tibial portion of the sciatic nerve, and these are responsible for extension of the thigh and flexion of the leg. The iliopsoas muscle is innervated by L1 and L2 and the femoral nerve, and flexes the thigh. The obturator internus is innervated by the nerve to the obturator internus and is a lateral rotator of the thigh.

A 6-year-old boy is admitted to the hospital because of a palpable mass located external to the aponeurosis of the external oblique. Radiographic examination reveals that the mass is an ectopic testis, classified as interstitial. Failure of normal development of which of the following embryologic structures is re- sponsible for ectopic testis? A. Gubernaculum B. Processus vaginalis C. Genital tubercle D. Seminiferous cords E. Labioscrotal swellings

A. The gubernaculum arises in the upper abdo- men from the lower end of the gonadal ridge and helps guide the testis in its descent to the inguinal region and then through the abdominal wall. Ectopic testes occur when a portion of the gubernaculum passes to an abnormal position or otherwise fails to descend or become fixed to the skin of the scrotum. The processus vaginalis is a tube of peritoneum that follows the same oblique course through the body wall as the testis, ventral to the gubernaculum. The distal part of the processus is retained as the tunica vaginalis. If part of the remainder of the processus remains patent, it can fill with fluid as a hydrocele of the testis or spermatic cord. The genital tubercle forms the primordial phallus and is associated with epispadias. The seminiferous cords form the primor- dia of the seminiferous tubules. The labioscrotal swellings approach each other and fuse to form the scrotum.

A 68-year-old male is admitted to the hospital with dysuria, nocturia, urgency, and painful urination. MRI examination reveals enlargement and irregularity of the uvula of the urethra due to prostatic hypertrophy. Laboratory investigation reveals high levels of PSA (prostate- specific antigen) suggesting prostatic carcinoma, and a prostatectomy is performed. Which of the following lymph nodes should be removed during prostatectomy? A. Internal iliac and sacral B. External iliac C. Superficial inguinal D. Deep inguinal E. Gluteal

A. The internal iliac lymph nodes and sacral nodes would be involved in a pelvic lymphadenectomy, which often would be desired in surgical resection for prostate cancer. Sacral lymphatics can communicate with lymphatics within the vertebral canal and thus metastasize cranially. The external iliac nodes drain all of the anterosuperior pelvic structures, the lower limb and perineum, and the body wall to the level of the umbilicus. The superficial inguinal nodes drain all of the superficial structures below the umbilicus. The deep inguinal nodes drain the glans penis in the male.

A 32-year-old woman visits her gynecologist for a routine examination. The Pap smear reveals atypical cervical cells, indicating the possible existence of cervical cancer. Which of the following lymph nodes need to be biopsied to confirm the existence of initial metastasis from the suspected cancerous tumor? A. Internal iliac B. External iliac C. Superficial inguinal D. Deep inguinal E. Sacral

A. The internal iliac nodes are the first in a chain of lymph nodes that receive lymph from the uterine cervix. Cancerous cells from the cervix are likely to involve the internal iliac nodes first. If these nodes do not have cancerous cells, this indicates that the tumor has not spread, at least through lymphatic channels.

A 34-year-old pregnant woman is prepared in the hospital for delivery. The gynecologist decides to perform a pudendal nerve block using a transvaginal approach. Which bony structure would be the most reliable as a landmark to block the pudendal nerve? A. Ischial spine B. Posterior inferior iliac spine C. Ischial tuberosity D. Posterior superior iliac spine E. Coccyx

A. The ischial spine is the correct bony land- mark used to administer a pudendal nerve block. The pudendal nerve crosses the sacrospinous ligament, which attaches to the ischial spine. Accessing the is- chial spine and thus the pudendal nerve is done most easily using a transvaginal approach. The posterior superior and inferior iliac spines are located on the posterior aspect of the pelvis and articulate with the lateral aspect of the sacrum. They do not relate to the course of the pudendal nerve. The ischial tuber- osities are the most inferior aspect of the bony pelvis. The skin and soft tissues around the ischial tuberosi- ties receive sensory supply from the pudendal nerve and perineal branches of the posterior femoral cuta- neous nerve. Injections into the area around the tu- berosities are less certain, however, than injections at the sacrospinous ligament and often fail to anesthe- tize the anal triangle well. The coccyx is a poor target for locating and anesthetizing the pudendal nerve.

A 42-year-old man is admitted to the emergency department after his automobile hit a tree. He is treated for a pelvic fracture and several deep lacerations. Physi- cal examination reveals that dorsiflexion and inversion of the left foot and extension of the big toe are very weak. Sensation from the dorsum of the foot, skin of the sole, and the lateral aspect of the foot has been lost and the patellar reflex is normal. The foot is everted and plantar flexed. Which of the following structures is most likely injured? A. The lumbosacral trunk at the linea terminalis B. L5 and S1 spinal nerves torn at the intervertebral foramen C. Fibular (peroneal) division of the sciatic nerve at the neck of the fibula D. Sciatic nerve injury at the greater sciatic foramen ("doorway to the gluteal region") E. Tibial nerve in the popliteal fossa

A. The lumbosacral trunk consists of fibers from a portion of the ventral ramus of L4 and all of the ventral ramus of L5 and provides continuity between the lumbar and sacral plexuses. The deep fibular (peroneal) nerve receives supply from segments of L4, L5, and S1. It supplies the extensor hallucis longus and extensor digitorum longus, the main functions of which are extension of the toes and dorsiflexion of the ankle. L5 is responsible for cutaneous innervation of the dorsum of the foot. Injury to L4 would affect foot inversion by the tibialis anterior. Injury to L4 in the lumbosacral trunk would not affect the patellar tendon reflex, for these fibers are delivered by the femoral nerve. Therefore, an injury to the lumbosacral trunk would result in all of the patient's symptoms. Nerve root injury at L5 and S1 would result in loss of sensation of the plantar aspect of the foot and motor loss of plantar flexion, with weakness of hip extension and abduction. The fibularis (peroneus) longus and brevis are supplied by the superficial fibular (peroneal) nerve, which is composed of fibers from segments L5, S1, and S2; these are responsible for eversion of the foot (especially S1). Transection of the fibular (peroneal) division of the sciatic nerve would result in loss of function of all the muscles of the anterior and lateral compartments of the leg. Injury to the sciatic nerve will affect hamstring muscles and all of the muscles below the knee. Injury to the tibial nerve causes loss of plantar flexion and impaired inversion.

A 19-year-old football player was hit on the lateral side of his knee just as he put that foot on the ground. Unable to walk without assistance, he is taken to the hospital. An MRI examination reveals a torn medial collateral ligament. Which structure would most likely also be injured due to its attachment to this ligament? A. Medial meniscus B. Anterior cruciate ligament C. Lateral meniscus D. Posterior cruciate ligament E. Tendon of the semitendinosus

A. The medial meniscus is firmly attached to the medial (tibial) collateral ligament. Damage to the medial collateral ligament often causes concomitant damage to the medial meniscus because of this relationship. The anterior cruciate ligament lies inside the knee joint capsule but outside the synovial cavity. It is taut during extension of the knee and may be torn when the knee is hyperextended. If this were damaged along with the medial meniscus and medial cruciate ligament, an "unhappy triad" (of O'Donoghue, or Donahue, both spellings are correct; also called a "blow knee") injury would result. The lateral meniscus is not attached to the medial collateral ligament but receives muscular attachment to the popliteus muscle. The posterior cruciate ligament also lies outside of the synovial cavity and limits hyperflexion of the knee. The tendon of the semitendinosus forms one third of the pes anserinus, with the tendons of the sartorius and gracilis making up the other two thirds. The pes anserinus (goose foot) is located at the medial border of the tibial tuberosity, and a portion can be used for surgical repair of the anterior cruciate ligament.

A 69-year-old man is admitted to the hospital for a scheduled radical prostatectomy. Six months postoperatively the patient visits the outpatient clinic complaining of impotence. Where are the nerve cell bodies located that are responsible for erection? A. Sacral parasympathetic nucleus B. Sacral sympathetic chain ganglia C. Inferior mesenteric ganglion D. Superior hypogastric plexus E. Intermediolateral column of L1, L2

A. The neural cell bodies responsible for erection are located in the sacral parasympathetic nucleus (in- termediomedial cell column). The parasympathetic nervous system is responsible for producing an erec- tion. The sacral sympathetic chain ganglia would not be responsible for the action of erection but rather the action of ejaculation. The inferior mesenteric ganglion would not contain parasympathetic neural cell bodies responsible for erection because they go to the hindgut. The superior hypogastric plexus contains few if any parasympathetic fibers and is not the primary location for the parasympathetic neural cell bodies. The inter- mediolateral column of L1 and L2 contains nerve cell bodies of preganglionic sympathetic neurons and there- fore would not contribute to producing an erection.

A 29-year-old woman is involved in a car crash and is taken to the emergency department. Radiographs reveal a fracture of her pelvis. During healing of the pelvic fracture, a nerve becomes entrapped in the bone callus. Musculoskeletal examination reveals an inability to adduct the thigh. Which of the following nerves is most likely affected? A. Obturator B. Femoral C. Inferior gluteal D. Superior gluteal E. Tibial

A. The obturator nerve is a branch of the lumbar plexus that originates from L2 to L4. It descends medial to the psoas on the posterior abdominal wall into the pelvis where it runs along the lateral wall of the lesser pelvis, above and anterior to the obturator vessels. It enters into the medial thigh via the obturator canal (an opening above the obturator membrane) to supply the obturator externus muscle and the adductors of the thigh. The femoral nerve innervates the anterior compartment of the thigh. The inferior gluteal innervates the gluteus maximus muscle, while the superior gluteal innervates the gluteus minimus and medius. The tibial nerve innervates the posterior compartment of the lower limb.

A 41-year-old woman is admitted to the obstetrics and gynecology department for a scheduled tubal ligation procedure. Two days postoperatively the patient has a high fever and shows signs of hypovolemic shock. A radiographic examination reveals a large hematoma adjacent to the external iliac artery. Which of the following vessels was most likely injured? A. Ovarian arteries B. Ascending branch of uterine arteries C. Descending branch of uterine arteries D. Superior vesicle artery E. Inferior vesicle artery

A. The ovarian arteries arise from the abdominal aorta, descend retroperitoneally along the posterior abdominal wall, and cross just anterior to the external iliac vessels. The ovarian arteries are the most likely source of blood from a hematoma follow- ing a tubal ligation. The ascending and descending uterine arteries and superior and inferior vesicle arteries branch from the internal iliac arteries and are not likely to be the source of blood in this situation.

A 42-year-old woman is admitted to the emer- gency department with severe abdominal pain. MRI examination reveals a tumor at her left ovary. A fro- zen biopsy during the time of surgery reveals an ovar- ian carcinoma. Which of the following actions can be performed to reduce the pain from the ovarian carcinoma? A. Cut the infundibulopelvic ligament. B. Cut the pelvic sympathetic chain. C. Cut the cluneal nerves. D. Cut the pudendal nerve. E. Cut the broad ligament.

A. The ovarian vessels and nerves lie within the infundibulopelvic ligament (suspensory ligament of the ovary); therefore, cutting this ligament interrupts pain fibers from the ovary. Cutting the sympathetic chain might help to reduce some of the pain from the ovary, but the results of such a procedure are rather unpredictable, plus locating the lumbar sympathetic chain is more of a surgical challenge. The cluneal nerves are cutaneous nerves that innervate parts of the buttocks. They are not associated with the ova- ries. The pudendal nerve innervates the perineum and does not carry afferent pain fibers from the ovary. The broad ligament contains only the uterovaginal vessels and nerve plexus and does not carry any nerve fibers from the ovary.

The patellar reflex appears to be markedly reduced in a 33-year-old diabetic female patient, due to deficient vascular supply of the nerves of her lower limb. The tendon of which of the following muscles is stretched during the patellar reflex? A. Quadriceps femoris B. Quadratus femoris C. Sartorius D. Pectineus E. Biceps femoris

A. The patellar ligament is a very heavy ligament that connects the patella to the tibial tuberosity; it provides the insertion of the quadriceps femoris tendon upon the tibia. The patella can be thought of as a bone (a sesamoid bone) that develops within the tendon of the quadriceps femoris muscle. When the reflex hammer strikes the patellar ligament, it stretches the ligament slightly for a brief time, resulting in reflex contraction of the quadriceps femoris muscles. This reflex arc is elicited by the femoral nerve (L4 sensory input component and L2, L3 motor output). The quadriceps femoris includes the rectus femoris and the vastus lateralis, intermedius, and medialis. The patella is the largest sesamoid bone in the body. A sesamoid bone is a bone that develops within a tendon. The quadratus femoris muscle of the gluteal area arises from the ischial tuberosity and inserts on the femur proximally. The sartorius arises from the anterior superior iliac spine and inserts on the proximal, medial aspect of the tibia as one of the three tendinous components of the pes anserinus (goose foot). The biceps femoris of the posterior thigh has a long head that arises from the ischial tuberosity and a short head that arises from the femur; they insert on the head of the fibula.

A 58-year-old man visited his physician for his annual check-up. Physical examination reveals a hyperreflexia patellar reflex. Which muscle(s) contribute(s) to the tendon that is struck when testing this reflex? A. Quadriceps femoris B. Quadratus femoris C. Sartorius D. Pectineus E. Biceps femoris

A. The patellar reflex causes extension of the knee and is produced by the quadriceps muscle group which consist of; biceps femoris and vastus lateralis, medialis and intermedius. Quadratus femoris is a lateral rotator of the thigh. The sartorius is a flexor of the hip and knee, and the pectineus is an adductor and flexor of the hip.

An 81-year-old man is admitted to the emergency department with severe pain in his knees. The patient has a long history of osteoarthritis. Radiologic examination reveals degeneration of the joints of his lower limbs. The degeneration is more severe on the medial side of the knees, which causes his knees to be bowed outward when he stands upright. Which of the following terms best describes the condition of his knees? A. Genu varus B. Genu valgus C. Coxa varus D. Coxa valgus E. Hallux valgus

A. The patient has bowlegs, or genu varus. The opposite of this is genu valgus, or knock knee. The normal angle between the femoral shaft and femoral neck is between 120 and 135 degrees. In coxa vara the angle between the shaft and neck of the femur is less than 120 degrees. This can result from fractures, other injuries, or congenital softness of the bone of the femoral neck. This defect results in limb shortening and limping. In coxa valga there is an increase in femoral shaft neck angulation, which can lead to hip subluxation or dislocation. Coxa valga results from weakness of the adductor musculature. Hallux valgus is commonly known as bunion. In this deformity the big toe points toward the little toe and may override the second toe; the base of the first metatarsal points medially, with a swollen bursal sac at the metatarsophalangeal joint. Excess bony growth of the distal protruding part of the metatarsal bone can also occur. Bunions occur only rarely in people who do not routinely wear shoes.

The baby was quite large, and the pelvis of the mother-to-be was somewhat narrow, causing her considerable difficulty and pain during the delivery. At her specific request, it was decided to inject local anesthetic into the perineum. The genitofemoral and ilioinguinal nerves were infiltrated anteriorly, and a deep injection was made medial to the ischial tuberosity to anesthetize the pudendal nerve, which supplies much of the perineum in most cases. A few minutes later, it became very obvious to those in attendance that the injection had not been effective enough in the central and posterior parts of the perineum. A separate injection was therefore inserted lateral to the ischial tuberosity. What other nerve(s) can provide much of the sensory supply to the perineum in some individuals? A. Posterior femoral cutaneous B. Inferior cluneal nerves C. Iliohypogastric nerve D. Inferior gluteal nerve E. Middle cluneal nerves

A. The perineal cutaneous branch of the posterior femoral cutaneous nerve provides a significant portion of the cutaneous innervation of the perineum in some individuals and can require separate anesthetic blockade in childbirth or perineal surgery, if other types of anesthesia are not used. The inferior cluneal branches of the posterior femoral cutaneous nerve supply the lower part of the gluteal skin. The lateral cutaneous branch of the iliohypogastric nerve provides sensation for the anterior superior aspect of the gluteal area. The inferior gluteal nerve innervates the gluteus maximus muscle. The middle cluneal nerves arise from the dorsal primary rami of S1 to S3 and supply skin over the middle of the gluteal region.

A 41-year-old man is admitted to the emergency department with a swollen and painful foot. Radiologic examination reveals that the head of the talus has become displaced inferiorly, thereby causing the medial longitudinal arch of the foot to fall. What would be the most likely cause in this case? A. Tearing of the plantar calcaneonavicular (spring) ligament B. Fracture of the cuboid bone C. Interruption of the plantar aponeurosis D. Sprain of the anterior talofibular ligament E. Sprain of the deltoid ligament

A. The plantar calcaneonavicular (spring) ligament supports the head of the talus and maintains the longitudinal arch of the foot. A fracture of the cuboid bone would not disrupt the longitudinal arch of the foot. Interruption of the plantar aponeurosis is not the best answer because this aponeurosis provides only passive support, unlike the spring ligament. A sprain of the anterior talofibular ligament would result from an inversion injury of the ankle and would not disrupt the longitudinal arch of the foot. A sprain of the deltoid ligament results from eversion of the ankle joint and would not disrupt the longitudinal arch of the foot.

A 16-year-old boy received a superficial cut on the lateral side of his foot while playing football and is admitted to the emergency department where the wound is sutured. Four days later the patient returns to the hospital with high fever and swollen lymph nodes. Which group of nodes will first receive lymph from the infected wound? A. Popliteal B. Vertical group of superficial inguinal C. Deep inguinal D. Horizontal group of superficial inguinal E. Internal iliac

A. The popliteal lymph nodes are the first to receive lymph from the foot. These nodes will then drain into the deep inguinal nodes and then to the external iliac nodes. The superficial inguinal and internal iliac nodes do not receive lymph from the foot

A 55-year-old woman complains of fecal incontinence. The most likely contributing factor to such a problem is atrophy, paralysis, or dysfunction of which of the following structures? A. Pubococcygeus muscle B. Iliococcygeus muscle C. Coccygeus muscle D. Pubovesicocervical fascia E. Urogenital diaphragm

A. The pubococcygeus muscle, especially its most medial portion, the puborectalis, is of most im- portance in fecal continence. The levator ani consists of two major portions, the pubococcygeus and ilio- coccygeus, which help support pelvic viscera and re- sist increases in intraabdominal pressure. The pu- borectalis muscle is the most medial and inferior portion of the pubococcygeus. The puborectalis forms a loop around the rectoanal junction, and the integrity of this muscle is critical in maintenance of fecal con- tinence. The coccygeus and pubovesicocervical fascia are not in direct contact with the rectum. Damage to the urogenital diaphragm can contribute to urinary incontinence but not fecal incontinence.

A 34-year-old woman is hospitalized because of an enlarged, painful abdomen. An ultrasound examination is performed and the presence of ascites (fluid) in the peritoneal cavity is confirmed. A needle is placed through the posterior vaginal fornix to drain the fluid. Which space must the needle enter to drain the fluid? A. Rectouterine pouch B. Pararectal fossa C. Paravesical space D. Uterovesical pouch E. Superficial perineal pouch

A. The rectouterine pouch (of Douglas) is the lowest point of the female peritoneal cavity. There- fore, fluid buildup within the peritoneal cavity accu- mulates here when the patient is standing or sitting. It is accessible transvaginally through the posterior fornix, with the patient positioned appropriately.

A 58-year-old woman presents to the outpatient surgery clinic for removal of varicose veins on the medial aspect of her foot. The operation was successful however, one month later she reports loss of sensation over the medial aspect of her leg and foot. Which of the following nerves was most likely injured during the procedure? A. Saphenous B. Obturator C. Lateral femoral cutaneous D. Tibial E. Femoral

A. The saphenous nerve runs with the great saphenous nerve which was being removed from patient. Sensory innervation to the areas of loss described is by the L4 root, which is carried by the saphenous nerve. The obturator nerve innervates the skin on the superior medial thigh. Lateral femoral cutaneous innervates the lateral aspect of the thigh. The tibial nerve supplies cutaneous innervation to the lateral aspect of the leg and if damaged would also produce muscular dysfunction. The femoral nerve is a motor and sensory nerve and is the origin of the saphenous nerve.

A popliteal arterial aneurysm can be very fragile, bursting with great loss of blood and the potential loss of the leg if it is not dealt with safely and effectively. In the 18th century, Dr. John Hunter (1728-1793) discovered that if a primary artery of the thigh is temporarily compressed, blood flow in the popliteal artery can be reduced long enough to treat the aneurysm in the popliteal fossa surgically, with safety. What structure is indicated in Fig. 5-3 that is related to his surgical procedure? A. Sartorius B. Femoral vein C. Femoral artery D. Gracilis E. Adductor brevis

A. The sartorius is indicated by the arrow in Figure 5-3. This muscle forms the roof of the subsartorial canal (Hunter's canal), with the adductor longus and vastus medialis forming other muscular borders. The femoral artery and vein, the saphenous nerve, the nerve to the vastus medialis, and the medial cutaneous nerve of the thigh all pass into this canal. The femoral artery leaves the canal by passing through the hiatus of the adductor magnus. The saphenous nerve emerges from the canal and from beneath the sartorius on the medial side of the lower limb proximally, thereafter providing sensory branches to the medial side of the lower limb and foot. Dr. Hunter mobilized the sartorius, thereby exposing the femoral artery (which continues as the popliteal artery beyond the adductor hiatus), which could be clamped while an aneurysmal popliteal artery was treated surgically.

A 23-year-old woman was taken to the emergency department after being involved in a head-on collision with a truck. On physical examination a hematoma was seen in the medial thigh. A CT scan revealed a fracture of the femur with a ruptured femoral artery. She was taken to the operating room for repair of the damaged structures. Two days postoperatively during physical examination the patient has loss of sensation to the anterior medial thigh and medial side of her leg and foot. Branches of which of the following nerves were most likely injured in the repair of the fracture? A. Femoral B. Saphenous C. Obturator D. Tibial E. Fibular (peroneal)

A. The skin of the anterior medial thigh and medial leg and foot is supplied by the femoral nerve. The saphenous nerve is a branch of the femoral and only supplies the medial leg and foot. The obturator supplies the medial and medial posterior aspect of the thigh. The tibial nerve supplies the skin of the posterolateral leg, lateral ankle and foot and sole of the foot. The fibular (peroneal) nerve supplies the skin over the lateral aspect of the leg and dorsal aspect of the foot.

In the process of meiosis, DNA replication of each chromosome occurs thereby forming a structure consisting of two sister chromatids attached to a single centromere. What is this structure? (A) A duplicated chromosome (B) Two chromosomes (C) A synapsed chromosome (D) A crossover chromosome (E) A homologous pair

A. The structure formed is a duplicated chromosome. DNA replication occurs so that the amount of DNA is doubled (2 x 2N = 4N). However, the chromatids remain attached to the centromere forming a duplicated chromosome.

A 42-year-old female patient has a malignancy involving the vestibule of her vagina. Which are the first lymph nodes to filter the lymph drainage from this area and therefore the most likely to become involved in tumor spread? A. Superficial inguinal B. Internal iliac C. Lumbar/lateral aortic D. Presacral lymph E. Axillary lymph

A. The superficial inguinal nodes drain superfi- cial perineal structures, including the superolateral uterine body near attachment of the round ligament, skin of the perineum (including the vulva), and the introitus of the vagina inferior to the hymen. The in- ternal iliac nodes drain the middle and upper vagina, cervix, and body of the uterus. The lumbar/lateral aortic lymph nodes drain the ovaries. The axillary lymph nodes drain the upper limb and chest wall, including the breast.

Three years following a 62-year-old's hip replacement, the man's CT scans indicated that two of his larger hip muscles had been replaced by adipose tissue. The opinion is offered that his superior gluteal nerve could have been injured during the replacement procedure, and the muscles supplied by that nerve had atrophied and been replaced by fat. Which of the following muscles receives its innervation from the superior gluteal nerve? A. Tensor fasciae latae B. Rectus femoris C. Gluteus maximus D. Piriformis E. Quadratus femoris

A. The superior gluteal nerve innervates the gluteus medius, gluteus minimus, and tensor fasciae latae muscles. The tensor fasciae latae arises from the iliac crest, inserts into the iliotibial tract of the lateral aspect of the thigh, and assists in flexion of the hip. The rectus femoris is innervated by the femoral nerve; it flexes the hip and extends the knee, thus acting upon two major joints. It arises in part from the anterior inferior iliac spine and the rim of the acetabulum and inserts into the quadriceps tendon. The gluteus maximus is supplied by the inferior gluteal nerve. The piriformis and quadratus femoris are both short lateral rotators of the hip and are supplied by branches of the sacral plexus.

A 37-year-old man is admitted to the hospital after an injury to his foot while playing flag football with friends on a Saturday morning. A series of radiographs demonstrates a fracture involving the talocrural (tibiotalar, ankle) joint. Which movements are the major ones to be affected by this injury? A. Plantar flexion and dorsiflexion B. Inversion and eversion C. Plantar flexion, dorsiflexion, inversion, and eversion D. Plantar flexion and inversion E. Dorsiflexion and eversion

A. The talocrural (tibiotalar, ankle) joint is a hinge-type synovial joint between the tibia and talus. It permits dorsiflexion and plantar flexion, and fracture of this joint would affect these movements

A 69-year-old woman, who fell down the stairs, presents to the emergency department. Radiologic imaging reveals a fracture of the talocrural (tibiotalar) joint. Which movements take place at this joint? A. Plantar flexion and dorsiflexion B. Inversion and eversion C. Plantar flexion, dorsiflexion, inversion, and eversion D. Plantar flexion and inversion E. Dorsiflexion and eversion

A. The talocrural joint is a synovial hinge joint that connects the distal end of the tibia and fibula with the proximal end of the talus. The articulation between the tibia and the talus bears more weight than other joints. Dorsiflexion (toes pointing upward) and plantar flexion (toes pointing downward) are possible. Dorsiflexion is performed by the tibialis anterior, extensor hallucis longus, extensor digitorum longus, and peroneus tertius. Plantar flexion is performed by the gastrocnemius, soleus, plantaris, peroneus longus, peroneus brevis, tibialis posterior, flexor digitorum longus, and flexor hallucis longus. The movements of inversion and eversion take place at the talocalcaneal joint.

A 37-year-old woman had been suffering for months from piriformis entrapment syndrome, which was not relieved by physical therapy. Part of the sciatic nerve passed through the piriformis, and a decision was made for surgical resection of the muscle. When the area of entrapment was identified and cleared, a tendon could be seen emerging through the lesser sciatic foramen, at first hidden by two smaller muscles and several nerves and vessels destined for the region of the perineum. The tendons of which of the following muscles pass through this opening? A. Obturator internus B. Obturator externus C. Quadratus femoris D. Gluteus minimus E. Gluteus medius

A. The tendon of the obturator internus leaves the pelvic cavity by passing through the lesser sciatic foramen, wrapping around the lesser sciatic notch, changing direction by about 90 degrees. It is joined there by the superior and inferior gemelli and inserts with them on the upper portion of the greater trochanter. The obturator externus arises on the external surface of the pubic bone and obturator membrane and inserts on the greater trochanter. The quadratus femoris arises from the ischial tuberosity and inserts on the intertrochanteric line of the femur. The gluteus medius and minimus insert together on the lateral aspect of the greater trochanter.

A bifid ureter or paired unilateral ureters result from partial or complete division of which of the following embryologic structures? A. Ureteric bud/metanephric diverticulum B. Mesonephric duct C. Paramesonephric duct D. Metanephric mesoderm E. Pronephros

A. The ureteric bud, or metanephric diverticu- lum, is an outgrowth from the mesonephric duct. It is the primordium of the ureter, renal pelvis, the calyces, and the collecting tubules. Incomplete division results in a divided kidney with a bifid ureter. Complete divi- sion results in a double kidney with a bifid ureter, or separate ureters.

A 35-year-old woman is admitted to the emergency department with severe left abdominal and back pain. Radiographic evaluation reveals that the left ureter is blocked with a kidney stone. Because the ureter is completely obstructed, an emergency surgical procedure must be performed. Which of the following landmarks is most reliable for the identification of the ureter? A. The left ureter is located anterior to the left common iliac artery. B. The left ureter is located medial to the left inferior epigastric artery. C. The left ureter is located anterior to the left gonadal artery. D. The left ureter is located anterior to the left renal vein. E. The left ureter is located anterior to the left inferior epigastric artery.

A. The ureters cross the pelvic brim anterior to the bifurcation of the common iliac artery bilaterally. Because of the proximity of this artery to the ureter, it is in danger of being damaged during surgery.

A 45-year-old woman is admitted to the hospital with lower abdominal and pelvic discomfort. Combined laparoscopic and MRI examinations reveal tears of the ligaments supporting the uterus with moderate uterine prolapse. Which of the following ligaments provides direct support to the uterus and thereby resists prolapse? A. Uterosacral B. Round ligament of the uterus C. Broad ligament D. Arcus tendineus fascia pelvis E. Levator ani muscle

A. The uterosacral ligaments and the transverse cervical (cardinal) ligaments are the two main liga- ments stabilizing the uterus. They help to inhibit the uterus from prolapsing into the vagina. The round liga- ment of the uterus is related to the descent of the ova- ries in embryologic development and continues into the inguinal canal. The broad ligament is the peritoneal covering over the uterine tubes, uterus, and vessels. The arcus tendineus fascia pelvis joins the muscle fas- cia of the levator ani to the pubocervical fascia on the vagina and is not directly associated with the uterus or its ligaments. The levator ani muscles contribute to the floor of the pelvis and support all of the pelvic viscera indirectly; it does not, however, prevent prolapse of the uterus into the vagina.

A 52-year-old woman visits the outpatient clinic due to a mass of tissue prolapsing through the vaginal introitus. Physical examination reveals that the patient has a cystocele. Loss of which of the following structures to the anterior part of the vagina is responsible for this problem? A. Pubovesical and vesicocervical fasciae B. Cardinal ligament C. Uterosacral ligament D. Levator ani muscle E. Median umbilical ligament

A. The uterus is stabilized and anchored to the bladder by the pubovesical and vesicocervical fasciae on its anterior surface. During childbirth this connec- tive tissue can be torn, allowing the bladder to herni- ate into the anterior vaginal wall, with prolapse pos- sible through the vaginal introitus. The transverse cervical (cardinal) ligament is located within the base of the broad ligament and is a major ligament of the uterus but would offer no support if the bladder her- niates through the vagina. The uterosacral ligament serves to anchor the uterus to the sacrum for support. Injury to the levator ani would not cause the bladder to herniate through the vagina. The median umbilical ligament contains the urachus and is located on the posterior aspect of the linea alba; the ligament is an embryologic remnant of the allantois.

A 23-year-old man is admitted to the emergency department with a deep, bleeding stab wound of the pelvis. After the bleeding has been arrested, a magnetic resonance imaging (MRI) examination gives evidence that the right ventral primary ramus of L4 has been transected. Which of the following problems will most likely be seen during physical examination? A. Reduction or loss of sensation from the medial aspect of the leg B. Loss of the Achilles tendon reflex C. Weakness of abduction of the thigh at the hip joint D. Inability to evert the foot E. Reduction or loss of sensation from the medial aspect of the leg and loss of Achilles tendon reflex

A. The ventral ramus of L4 contains both sensory and motor nerve fibers. Injury from a stab wound could result in loss of sensation from the dermatome supplied by this segment. A dermatome is an area of skin supplied by a single spinal nerve; L4 dermatome supplies the medial aspect of the leg and foot. Loss of the Achilles tendon reflex relates primarily to an S1 deficit. The Achilles tendon reflex is elicited by tapping the calcaneus tendon, which results in plantar flexion. The obturator internus and gluteus medius and minimus are responsible for abduction of the thigh and are innervated by nerves L4, L5, and S1 (with L5 usually dominant). Nerves L5, S1, and S2 are responsible for eversion of the foot (S1 dominant)

A 32-year-old woman is admitted to the hospital with a complaint of painful spasms of her vagina. Physical examination reveals several involuntary contractions of the vaginal musculature. The patient also complains of painful intercourse. Which of the following conditions will most likely describe the signs of this patient? A. Vaginismus B. Pudendal nerve compression in the pudendal (Alcock) canal C. Disruption of the perineal body D. Endometriosis E. Fibroma of the uterus

A. Vaginismus is a painful, psychosomatic gyne- cologic disorder; it is described as involving disten- sion of the cavernous tissues and the bulbospongio- sus and transverse perineal muscles, the stimulation of which triggers the involuntary spasms of the peri- vaginal and levator ani muscles. This can in turn lead to painful intercourse or dyspareunia.

A 54-year-old male is admitted to the hospital with severe back pain. Radiographic examination reveals carcinoma of the left kidney blocking the drain- age of the testicular vein. Which of the following conditions will be most likely associated with these signs? A. Varicocele B. Rectocele C. Cystocele D. Hydrocele E. Hypospadias

A. Varicose veins occur with loss of elasticity within the walls of the vessels. As the veins weaken, they simultaneously dilate under pressure. A varico- cele often occurs with a varicosity of the veins of the pampiniform venous plexus, resulting in a swelling of the veins. This condition can arise from a tumor in the left kidney, which occludes the testicular vein due to an anatomic constriction. A hydrocele is an accu- mulation of fluid within the cavity of the tunica vagi- nalis. Hypospadias occurs from failure of fusion of the urethral and labioscrotal folds, resulting in an exter- nal urethral opening on the ventral surface of the pe- nis or in the perineum.

A 50-year-old man is admitted to the emergency department after a car crash. An MRI examination reveals an injured anterior cruciate ligament. Physical examination reveals a positive drawer sign. Which of the following signs is expected to be present during physical examination? A. The tibia can be slightly displaced anteriorly B. The tibia can be slightly displaced posteriorly C. The fibula can be slightly displaced posteriorly D. The fibula can be slightly displaced anteriorly E. The tibia and fibula can be slightly displaced anteriorly

A. When the anterior cruciate ligament is torn, the tibia can be slightly displaced anteriorly from the area of the knee joint by pulling firmly with both hands upon the leg, with the patient in a seated position. This is a positive anterior drawer sign.

A 2-month-old infant has epispadias and the bladder mucosa is exposed to the outside. Which of the following is the most likely cause of this condition? A. Failure of the primitive streak mesoderm to migrate around the cloacal membrane B. Failure of urethral folds to fuse C. Insufficient androgen stimulation D. Klinefelter syndrome E. Persistent allantois

A. When the urinary bladder mucosa is open to the outside in the fetus or newborn, the condition is referred to as extrophy of the bladder. The extrophy results from failure of the primitive streak mesoderm to migrate around the cloacal membrane, and it oc- curs often in combination with epispadias. Penile hypospadias is characterized by a failure of fusion of the labioscrotal folds, with the external urethral ori- fice located between the two unfused halves of the scrotum. Androgens are responsible for development of the testes. Klinefelter syndrome is a condition in which the male has 47 XXY chromosomes. A persis- tent allantois is associated with a patent urachus and an allantoic cyst.

A 54-year-old male is admitted to the hospital with severe back pain. Upon radiographic examination the scrotum resembles a "bag of worms." Which of the following conditions will be most likely associated with this radiographic picture? A. Varicocele B. Rectocele C. Cystocele D. Hydrocele E. Hypospadias

A. When veins lose their elasticity, they can be- come weak and often dilate. This causes the veins to become swollen and oftentimes tortuous, as a result of incompetent valves. The appearance of a "bag of worms" on the radiograph is characteristic of a vari- cosity of the pampiniform venous plexus. A hydrocele is an accumulation of fluid within the tunica vaginalis cavity. Hypospadias occurs from failure of fusion of the urethral and labioscrotal folds, resulting in an external urethral opening on the ventral surface of the penis or in the perineum.

A 19-year-old patient is admitted to the orthopedic service with a complaint of severe pain in his very swollen and discolored foot. He states that he hurt the foot when jumping from his girlfriend's bedroom window to the concrete driveway below. Plain film radiographic studies reveal that the head of the talus has become displaced inferiorly, thereby causing the medial longitudinal arch of the foot to fall. What would be the most likely, serious problem in such a case? A. Tearing of the plantar calcaneonavicular (spring) ligament B. Fracture of the cuboid bone C. Interruption of the plantar aponeurosis D. Sprain of the anterior talofibular ligament E. Disruption of the distal tibiofibular ligament

A. With sufficient downward force, the head of the talus can break through the plantar calcaneonavicular (spring) ligament, causing the medial longitudinal arch of the foot to fall, forcing the anterior part of the foot into abduction. The plantar calcaneonavicular ligament is attached between the sustentaculum tali of the calcaneus and the medial surface of the navicular bone, with the head of the talus lying directly upon the inner surface of the ligament. The cuboid bone is located lateral and anterior to the talus bone and would not be fractured. The plantar aponeurosis, a dense, wide band of tissue beneath the fascia of the sole, attaches to the calcaneus and ends distally in longitudinal bands to each of the toes. It stretches very little, even under very heavy loads, and would not rupture in this case. The anterior talofibular ligament is very often injured in "sprained ankle" but would not be directly involved here. The distal tibiofibular joint is a fibrous (and usually nonsynovial) type of joint (called a syndesmosis) between the tibia and fibula, not involved in the displacement of the talus bone.

Muscles of the abdominal wall

An 18-year-old girl is thrust into the steering wheel while driving and experiences difficulty in expiration. Which of the following muscles is most likely damaged? -Levator costarum -Innermost intercostal muscle -External intercostal muscle -Diaphragm -Muscles of the abdominal wall

Anterior cardiac vein

An 8-year-old boy with ASD presents to a pediatrician. This congenital heart defect shunts blood from the left atrium to the right atrium and causes hypertrophy of the right atrium, right ventricle, and pulmonary trunk. Which of the following veins opens into the hypertrophied atrium? -Middle cardiac vein -Small cardiac vein -Oblique cardiac vein -Anterior cardiac vein -Right pulmonary vein

Posterior part of the left ventricle

An 83-year-old man with a typical coronary circulation has been suffering from an embolism of the circumflex branch of the left coronary artery. This condition would result in ischemia of which of the following areas of the heart? -Anterior part of the left ventricle -Anterior interventricular region -Posterior interventricular region -Posterior part of the left ventricle -Anterior part of the right ventricle

101 A 22-year-old pregnant woman was admitted emergently to the hospital after the baby had begun to appear at the introitus. The baby had presented in the breech position, and it had been necessary to exert considerable traction to complete the delivery. The newborn is shown in Fig. 6-6 . Which of the following structures was most likely injured by the trauma of childbirth? A. Radial nerve B. Upper trunk of the brachial plexus C. Lower trunk of the brachial plexus D. Median, ulnar, and radial nerves E. Upper and lower trunks of the brachial plexus

B

110 A 54-year-old woman presents with pain in her right wrist that resulted when she fell forcefully on her outstretched hand. Radiographic studies indicate an anterior dislocation of a carpal bone of the proximal row (see F ig. 6-3 on page 180). Which of the following bones is most commonly dislocated? ⃣ A. Capitate ⃣ B. Lunate ⃣ C. Scaphoid ⃣ D. Pisiform ⃣ E. Triquetrum

B

113 While working out with weights, a 28-year-old woman experiences a severe pain in her chest. The pain is referred to the anterior chest wall and radiating to the mandible and her left arm. The woman felt dizzy and after 10 minutes she collapsed and was unconscious. A physician happened to be near the woman and immediately tried to feel her radial pulse. The radial artery lies between two tendons near the wrist, which are useful landmarks. Which of the following is the correct pair of tendons? ⃣ A. Flexor carpi radialis and palmaris longus ⃣ B. Flexor carpi radialis and brachioradialis ⃣ C. Brachioradialis and fl exor pollicis longus ⃣ D. Flexor pollicis longus and fl exor digitorum superfi cialis ⃣ E. Flexor pollicis longus and fl exor digitorum profundus

B

114 A 59-year-old woman is admitted to the hospital in a state of shock. During physical examination, several lacerations are noted in her forearm and her radial pulse is absent. Where is the most typical place to identify the radial artery immediately after crossing the radiocarpal joint? ⃣ A. Between the two heads of the fi rst dorsal interosseous muscles ⃣ B. At the anatomic snuffbox ⃣ C. Below the tendon of the fl exor pollicis longus ⃣ D. Between the fi rst and second interosseous muscle ⃣ E. Between the fi rst interosseous muscle and the adductor pollicis longus

B

116 A 32-year-old man is admitted to the emergency department after a severe car crash. Radiographic examination reveals multiple fractures of his right upper limb. A surgical procedure is performed and metallic plates are attached to various bony fragments to restore the anatomy. Five months postoperatively the patient visits the outpatient clinic. Upon physical examination the patient can abduct his arm and extend the forearm, but the sensation of the forearm and hand is intact; however, the hand grasp is very weak, and he cannot extend his wrist against gravity. Which of the following nerves was most likely injured during the surgical procedure? ⃣ A. Posterior cord of the brachial plexus ⃣ B. Radial nerve at the distal third of the humerus ⃣ C. Radial and ulnar ⃣ D. Radial, ulnar, and median ⃣ E. Radial and musculocutaneous

B

117 A 52-year-old man is admitted to the emergency department after falling on wet pavement. Radiographic examination reveals fracture of the radius. An MRI study reveals a hematoma between the fractured radius and supinator muscle. Upon physical examination the patient has weakened abduction of the thumb and extension of the metacarpophalangeal joints of the fingers. Which of the following nerves is most likely affected? A. Anterior interosseous B. Posterior interosseous C. Radial nerve D. Deep branch of ulnar nerve E. Median nerve

B

127 A 42-year-old woman is admitted to the hospital with injury to the upper (superior) trunk of the brachial plexus. The diagnosis is Erb-Duchenne palsy. Which of the following conditions is expected to be present during physical examination? A. Winged scapula B. Inability to laterally rotate the arm C. Paralysis of intrinsic muscles of the hand D. Paraesthesia in the medial aspect of the arm E. Loss of sensation in the dorsum of the hand

B

13. A 24-year-old woman comes to a hospital to deliver her baby. Her obstetrician uses a caudal anesthesia during labor and childbirth to block the spinal nerves in the epidural space. Local anesthetic agents are most likely injected via which of the following openings? (A) Intervertebral foramen (B) Sacral hiatus (C) Vertebral canal (D) Dorsal sacral foramen (E) Ventral sacral foramen

B

136 A 54-year-old woman is found unconscious in her car. She is admitted to the hospital, and during physical examination she has absent biceps brachii refl ex. What is the spinal level of the afferent component of this refl ex? ⃣ A. C5 ⃣ B. C6 ⃣ C. C7 ⃣ D. C8 ⃣ E. T1

B

137 A 54-year-old woman is found unconscious in her bed. She is admitted to the hospital, and during physical examination she has absence of her brachioradialis refl ex. The ventral ramus of which spinal nerve is responsible for this refl ex? ⃣ A. C5 ⃣ B. C6 ⃣ C. C7 ⃣ D. C8 ⃣ E. T1

B

140 A 32-year-old woman visits the outpatient clinic after injuring her elbow falling from her bicycle. Physical examination reveals a "benediction attitude" of the hand with the index and long fi ngers extended and the ring and little fi ngers fl exed. Which of the following is the most likely diagnosis? ⃣ A. Injury to median and radial nerves ⃣ B. Injury to median nerve ⃣ C. Injury to radial and ulnar nerves ⃣ D. Injury to ulnar nerve ⃣ E. Injury to median ulnar and radial nerves

B

22. A 25-year-old man with congenital abnormalities at birth has a lesion of the dorsal scapular nerve, making him unable to adduct his scapula. Which of the following muscles is most likely paralyzed? (A) Semispinalis capitis (B) Rhomboid major (C) Multifidus (D) Rotator longus (E) Iliocostalis

B

28 A 54-year-old female marathon runner presents with pain in her right wrist that resulted when she fell with force on her outstretched hand. Radiographic studies indicate an anterior dislocation of a carpal bone ( Fig. 6-3 ). Which of the following bones is most likely dislocated? ⃣ A. Capitate ⃣ B. Lunate ⃣ C. Scaphoid ⃣ D. Trapezoid ⃣ E. Triquetrum

B

28. The spinal cord is crushed at the level of the upper part of the first lumbar vertebra. Which structure is most likely damaged?

B

32 A 15-year-old girl was brought to the emergency department with a tear of the tendons in the fi rst dorsal compartment of the wrist from a severe bite by a pit bull dog. The injured tendons in this compartment would include which of the following muscles? A. Extensor carpi radialis longus and brevis B. Abductor pollicis longus and extensor pollicis brevis C. Extensor digitorum D. Extensor indicis proprius E. Extensor carpi ulnaris

B

46 A 47-year-old female patient's right breast exhibited characteristics of peau d'orange; that is, the skin resembled orange peel. This condition is primarily a result of which of the following? ⃣ A. Shortening of the suspensory ligaments by cancer in the axillary tail ⃣ B. Blockage of cutaneous lymphatic vessels ⃣ C. Contraction of the retinacula cutis of the areola and nipple ⃣ D. Invasion of the pectoralis major by the cancer ⃣ E. Ipsilateral (same side) inversion of the periareolar skin from ductular cancer

B

49 A 34-year-old female skier was taken by ambulance to the hospital after she struck a tree on the ski slope. Imaging gives evidence of a shoulder separation. Which of the following typically occurs in this kind of injury? ⃣ A. Displacement of the head of the humerus from the glenoid cavity ⃣ B. Partial or complete tearing of the coracoclavicular ligament ⃣ C. Partial or complete tearing of the coracoacromial ligament ⃣ D. Rupture of the transverse scapular ligament ⃣ E. Disruption of the glenoid labrum

B

58 A 22-year-old male football player suffered a wrist injury while falling with force on his outstretched hand. When the anatomic snuffbox is exposed in surgery, an artery is visualized crossing the fractured bone that provides a floor for this space. What artery was visualized? A. Ulnar B. Radial C. Anterior interosseous D. Posterior interosseous E. Deep palmar arch

B

6. After a 26-year-old man's car was broadsided by a large truck, he is brought to the emergency department with multiple fractures of the transverse processes of the cervical and upper thoracic vertebrae. Which of the following muscles might be affected? (A) Trapezius (B) Levator scapulae (C) Rhomboid major (D) Serratus posterior superior (E) Rectus capitis posterior major

B

64 A 43-year-old woman visits the outpatient clinic with a neurologic problem. Diagnostically, she cannot hold a piece of paper between her thumb and the lateral side of her index finger without flexing the distal joint of her thumb. This is a positive Froment sign and a diagnosis of ulnar neuropathy. Weakness of which specific muscle causes this sign to appear? A. Flexor pollicis longus B. Adductor pollicis C. Flexor digiti minimi D. Flexor carpi radialis E. Extensor indicis

B

69 A 74-year-old woman is admitted to the emergency department after stumbling over her pet dog. Radiographic examination reveals a fracture of the upper third of the right radius, with the distal fragment of the radius and hand pronated. The proximal end of the fractured radius deviates laterally. Which of the following muscles is primarily responsible for the lateral deviation? ⃣ A. Pronator teres ⃣ B. Supinator ⃣ C. Pronator quadratus ⃣ D. Brachioradialis ⃣ E. Brachialis

B

71 A 22-year-old male medical student was seen in the emergency department with a complaint of pain in his hand. He confessed that he had hit a vending machine in the hospital when he did not receive his soft drink after inserting money twice. The medial side of the dorsum of the hand was quite swollen, and one of his knuckles could not be seen when he "made a fi st." The physician made a diagnosis of a "boxer's fracture." What was the nature of the impatient student's injury? ⃣ A. Fracture of the styloid process of the ulna ⃣ B. Fracture of the neck of the fi fth metacarpal ⃣ C. Colles' fracture of the radius ⃣ D. Smith's fracture of the radius ⃣ E. Bennett's fracture of the thumb

B

84 A 54-year-old female was found unconscious on the floor, apparently after a fall. She was admitted to the hospital, and during physical examination it was observed that she had absence of her brachioradialis reflex. Which spinal nerve is primarily responsible for this refl ex in the majority of cases? ⃣ A. C5 ⃣ B. C6 ⃣ C. C7 ⃣ D. C8 ⃣ E. T1

B

86 A 29-year-old patient has a dislocated elbow in which the ulna and medial part of the distal humerus have become separated. What classifi cation of joint is normally formed between these two bones? A. Trochoid B. Ginglymus C. Enarthrodial D. Synarthrosis E. Sellar

B

91 A 45-year-old man visits the outpatient clinic after a digit of his left hand was injured when a door was slammed on his hand. A superfi cial cut on his middle fi nger has been sutured, but functional defi cits are observed in the fi nger: The proximal interphalangeal joint is pulled into constant fl exion, whereas the distal interphalangeal joint is held in a position of hyperextension. What is the most likely diagnosis? ⃣ A. Mallet fi nger ⃣ B. Boutonnière deformity ⃣ C. Dupuytren contracture ⃣ D. Swan-neck deformity ⃣ E. Silver fork wrist deformity

B

94 A 72-year-old man consulted his physician because he had noticed a thickening of the skin at the base of his left ring fi nger during the preceding 3 months. As he described it, "There appears to be some hard tissue that is pulling my little and ring fi ngers into my palm." On examination of the palms of both hands, localized and fi rm ridges are observed in the palmar skin that extend from the middle part of the palm to the base of the ring and little fi ngers. What is the medical term for this sign? ⃣ A. Ape hand ⃣ B. Dupuytren ⃣ C. Clawhand ⃣ D. Wrist drop ⃣ E. Mallet fi nger

B

A 25-year-old male athlete is admitted to the emergency department after a bad landing in the pole vault. Radiographic examination of his hand reveals a fractured carpal bone in the floor of the anatomic snuffbox ( Fig. 6-1 ). Which bone has most likely been fractured? ⃣ A. Triquetral ⃣ B. Scaphoid C. Capitate D. Hamate E. Trapezoid

B

A 32-year-old woman is admitted to the emergency department after an automobile collision. Radiographic examination reveals multiple fractures of the humerus. Flexion and supination of the forearm are severely weakened. She also has loss of sensation on the lateral surface of the forearm. Which of the following nerves has most likely been injured? ⃣ A. Radial ⃣ B. Musculocutaneous ⃣ C. Median ⃣ D. Lateral cord of brachial plexus ⃣ E. Lateral cutaneous nerve of the forearm

B

A 55-year-old male is examined in a neighborhood clinic after receiving blunt trauma to his right axilla in a fall. He has difficulty elevating the right arm above the level of his shoulder. Physical examination shows the inferior angle of his right scapula protrudes more than the lower part of the left scapula. The right scapula protrudes far more when the patient pushes against resistance. Which of the following neural structures has most likely been injured? ⃣ A. The posterior cord of the brachial plexus ⃣ B. The long thoracic nerve ⃣ C. The upper trunk of the brachial plexus ⃣ D. The site of origin of the middle and lower subscapular nerves ⃣ E. Spinal nerve roots C7, C8, and T1

B

An 18-year-old male is brought to the emergency department after an injury while playing rugby. Imaging reveals a transverse fracture of the humerus about 1 inch proximal to the epicondyles. Which nerve is most frequently injured by the jagged edges of the broken bone at this location? ⃣ A. Axillary ⃣ B. Median ⃣ C. Musculocutaneous ⃣ D. Radial ⃣ E. Ulnar

B

A newborn baby was diagnosed with eventration of the diaphragm. In this condition, half of the diaphragm ascends into the thorax during inspiration, while the other half contracts normally. What is the cause of this condition? A. Absence of a pleuropericardial fold B. Absence of musculature in one half of the diaphragm C. Failure of migration of diaphragm D. Failure of the septum transversum to develop E. Absence of a pleuroperitoneal fold

B. An absence of musculature in one half of the diaphragm causes it to protrude into the thoracic cavity forming a pouch into which the abdominal viscera protrude. Pleuropericardial folds are responsible for separating the pericardial cavity from the pleural cavity. Typically, the diaphragm migrates to its position with the fibrous pericardium. The septum transversum is the primordial central tendon of the diaphragm that separates the heart from the liver. The pleuroperitoneal folds form the pleuroperitoneal membranes that separate the pleural cavity from the peritoneal cavity. Absence of any of these would not have anything to do with eventration of theiaphragm.

A 2-day-old newborn male develops mild cyanosis. An ultrasound examination reveals a patent ductus arteriosus. Which of the following infections will most likely lead to this congenital anomaly? A. Toxoplasmosis B. Rubella C. Cytomegalovirus D. Varicella virus E. Treponema pallidum

B. Congenital heart defects are common problems that can be caused by teratogens, such as the rubella virus, or single-gene factors or chromosomal abnormalities.

A 5-year-old boy is admitted to the hospital with severe dyspnea. During physical examination a loud systolic murmur and a wide, fixed, split S2 sound is noted. What is the most likely diagnosis? A. Ventricular septal defect B. Atrial septal defect C. Tetralogy of Fallot D. Transposition of the great arteries E. Aortic stenosis

B. The murmur at S2 localizes the defect at an atrioventricular valve. An atrial septal defect causes a diastolic murmur in the tricuspid valve, whereas a ventricular septal defect would cause a pansystolic murmur. Transposition of the great arteries and aortic stenosis will cause a murmur at S1, and tetralogy of Fallot does not cause a murmur at S1 or S2 .

A 28-year-old woman in her third trimester of pregnancy with a complaint of dizziness for several days is admitted to the hospital. Physical examination reveals that she has diabetes mellitus. Which of the following cardiac malformations is most likely to affect the fetus when the mother has this disease? A. Tetralogy of Fallot B. Transposition of the great arteries C. Atrial septal and ventricular septal defects D. Truncus arteriosus E. Coarctation of the aorta

B. Transposition of the great arteries is associated with maternal diabetes. Tetralogy of Fallot and truncus arteriosus are associated with DiGeorge syndrome (22q11). ASDs and VSDs are present in individuals with Down syndrome. Coarctation of the aorta is related to Turner syndrome. Marfan syndrome is present in individuals with aortic atresia.

A 26-year-old pregnant woman visits her gynecologist for a routine checkup. Ultrasound examination reveals that the patient has a normal pregnancy but that she also has two uteri. What is the most likely embryologic explanation of this condition? A. A complete fusion of the paramesonephric ducts B. An incomplete fusion of the paramesonephric ducts C. Hydronephros D. Cryptorchidism E. Regression of the pronephros

B. A double uterus is caused by failure of inferior parts of the paramesonephric ducts. A complete fu- sion results in abnormal development of the uterine tubes because the uterine tubes form from the un- fused portions of the cranial parts of the parameso- nephric ducts. Hydronephros, swelling of the renal pelvis and calyces with urine, can result from the obstruction of the ureter by a renal stone. Cryptorchi- dism is a condition characterized by an undescended testis, in which the testis can be localized in the ab- dominal cavity or in any place along the path of tes- ticular descent. The pronephros is part of the primor- dial urinary system and generally degenerates in the first four weeks of development.

A 35-year-old woman is undergoing a tension-free vaginal tape procedure to repair an inguinal hernia. Two days postoperatively the patient has a high fever and shows signs of hypovolemic shock. A radiographic examination reveals that a vessel crossing the pectineal (Cooper's) ligament as it descends into the pelvis was injured by a staple, confirming the presence of the so- called "arterial circle of death." Which of the following arteries is most likely injured? A. Obturator artery B. Aberrant obturator artery C. Superior vesicle artery D. Middle rectal E. Inferior vesicle

B. An aberrant obturator artery arising from the inferior epigastric arteries can be found in 20% to 30% of the population. Patients with this variation are more susceptible to inadvertent damage during certain surgeries if the surgeon is not aware of presence of the aberrant artery. The other arteries listed would be less likely to be injured because the surgeon would assume they are present and will thus take great care in making sure not to staple them.

A 55-year-old woman is admitted to the emergency department after an automobile crash. Physical examination reveals that the patient's foot is everted and she cannot invert it. A weakness in dorsiflexion and inversion of the foot is noted. Her ipsilateral patellar reflex is reduced in quality, although the Achilles tendon reflex is brisk. Knee extension is almost normal, as are all hip movements and knee flexion. Sensation is greatly reduced on the medial side of the leg. Which of the following nerves is most likely injured? A. Femoral nerve B. L4 spinal nerve C. L4 and L5 spinal nerves D. Common fibular (peroneal) nerve E. Tibial nerve

B. An injury to L4 would cause weakness in the patellar reflex and loss of cutaneous innervation to the medial side of the leg. The patellar reflex is used to test L2 to L4 nerve integrity. The motor side of the reflex is primarily derived from spinal nerves L2 and L3, whereas the sensory side of the arc is said to be principally from L4. The L4 spinal nerve supplies the L4 dermatome on the medial side of the leg and foot, by way of the saphenous nerve. It also supplies foot inversion, a function of the tibialis anterior and tibialis posterior muscles; the first is supplied by the deep fibular (peroneal) nerve, and the second supplied by the tibial nerve. Foot dorsiflexion is weakened because of partial denervation of the extensor digitorum longus, but L5 is still contributing to that function. The foot is everted because the S1-supplied (by the superficial fibular nerve) fibularis (peroneus) longus and brevis are unopposed. The Achilles reflex is also primarily supplied by S1. Hip movements are produced primarily by L5- and S1-supplied muscles, as is knee flexion.

A 49-year-old male worker fell from a ladder, with his weight impacting on the heels of his feet. Radiologic examination reveals comminuted calcaneal fractures. After the injury the contraction of which one of the following muscles would most likely increase the pain in the injured foot? A. Flexor digitorum profundus B. Gastrocnemius C. Tibialis posterior D. Tibialis anterior E. Fibularis (peroneus) longus

B. Contraction of the gastrocnemius on the fractured calcaneus would increase the pain because the gastrocnemius inserts with the soleus upon that bone, via the calcaneal tendon, or tendo Achilles. The flexor digitorum profundus passes the ankle medially to enter the sole of the foot, where it inserts upon the distal phalanges. The tibialis posterior, likewise, passes under the medial malleolus, with complex insertions upon the navicular bone, cuneiform bones, metatarsal bones, and the cuboid bone. The tibialis anterior, a muscle of the anterior leg compartment, inserts upon the navicular bone and, with the tibialis posterior, is a strong invertor of the foot. The fibularis (peroneus) longus is a muscle of the lateral compartment of the leg. It passes under the lateral malleolus, entering the sole of the foot by crossing the lateral surface of the calcaneus, and inserts primarily into the medial cuneiform and base of the first metatarsal bone.

A 16-year-old boy visits the urologist with a lump in his left testis. Diagnosis of testicular teratocarcinoma is made. This tumor can be loosely referred to as "male pregnancy" because at an early stage the carcinoma contains all three primary germ layers: ectoderm, mesoderm, and endoderm. In normal embryologic development, which of the following processes give rise to these three primary germ layers? A. Morulation B. Gastrulation C. Cranio-caudal folding D. Cleavage E. Induction

B. Gastrulation occurs in early embryogenesis and is the process by which the blastula is reorganized into a three-layered structure, that is, ectoderm, mesoderm, and endoderm. Morulation involves the cleavage or division of the fertilized ovum usually into a 16-cell structure that resembles a ball.

A 67-year-old woman has been suffering from osteoporosis for the past year. During her annual checkup, radiologic examination reveals an angle of 160 degrees made by the axis of the femoral neck to the axis of the femoral shaft. Which of the following conditions is associated with these examination findings? A. Coxa vara B. Coxa valga C. Genu valgum D. Genu varum E. Hallux valgus

B. Generally, the angle of inclination between the neck and shaft of the femur in older age decreases to around 120 degrees. However, in pathologic conditions it can either increase or decrease from the predicted value. When the angle of inclination increases, it is referred to as coxa valga. Coxa vara on the other hand is a condition characterized by a decreased angle of inclination. Genu varum and genu valgum are deformities characterized by a decreased Q-angle and increased Q-angle, respectively. The Q-angle refers to the angle between the femur and tibia. Hallux valgus is a condition that presents with a lateral deviation of the large toe.

While performing a voiding cystourethrogram on a 45-year-old male, the urologist was too forceful when he inserted the catheter and accidentally damaged the wall of the membranous portion of the urethra in the deep perineal compartment (urogenital diaphragm). Which of the following structures would most likely be traumatized at this location? A. Bulbospongiosus muscle B. Sphincter urethra (compressor urethra) C. Corpus cavernosus penis (crus) D. Ischiocavernosus muscle E. Opening of the bulbourethral duct

B. If the membranous portion of the urethra is injured, urine and blood can leak upward into the retropubic space (of Retzius) limited inferiorly by the urogenital diaphragm and the muscle within (compressor urethra), which would be injured. The bulbospongiosus muscle and other perineal muscles, the corpus cavernosa penis, and the openings of the bulbourethral ducts are inferior and anterior to the region of injury.

A 5-year-old boy is admitted to the emergency department after a car collision. Radiologic examination reveals a fracture of the head of the femur. An MRI examination reveals a large hematoma. Which of the following arteries is most likely injured? A. Deep circumflex iliac B. Acetabular branch of obturator C. Descending branch of lateral circumflex femoral D. Medial circumflex femoral E. Radicular branches of circumflex artery

B. In infants and children up to about 8 years of age, the head of the femur gets its arterial supply by a direct branch of the obturator artery (variably, the medial circumflex femoral). The arterial supply reaches the head of the femur at the fovea capitis by traveling along the ligament of the head of the femur. Probably due to repeated torsion on the ligament, and therefore on the artery, this artery occludes early in life. In turn, this source of supply is replaced by branches of the gluteal and femoral circumflex vessels.

A 30-year-old man suffered a superior gluteal nerve injury in a motorcycle crash in which his right lower limb was caught beneath the bike. He is stabilized in the emergency department. Later he is examined and he exhibits a waddling gait and a positive Trendelenburg sign. Which of the following would be the most likely physical finding in this patient? A. Difficulty in standing from a sitting position B. The left side of the pelvis droops or sags when he attempts to stand with his weight supported just by the right lower limb C. The right side of the pelvis droops or sags when he attempts to stand with his weight supported just by the left lower limb D. Weakened flexion of the right hip E. Difficulty in sitting from a standing position

B. Injury to the superior gluteal nerve results in a characteristic motor loss, with paralysis of the gluteus medius and minimus. In addition to their role in abducting the thigh, the gluteus medius and minimus function to stabilize the pelvis. When the patient is asked to stand on the limb of the injured side, the pelvis descends on the opposite side, indicating a positive Trendelenburg test. The gluteal, or lurching, gait that results from this injury is characterized by the pelvis drooping to the unaffected side when the opposite leg is raised. In stepping forward, the affected individual leans over the injured side when lifting the good limb off the ground. The uninjured limb is then swung forward. The gluteus maximus, supplied by the inferior gluteal nerve, is the main muscle responsible for allowing a person to rise to a standing position (extending the flexed hip). Spinal nerve roots L1 and L2 and the femoral nerve are responsible for hip flexion. Injury to the left superior gluteal nerve would result in sagging of the right side of the pelvis when the affected individual stands on the left limb. The hamstring muscles, mainly responsible for flexing the knees to allow a person to sit down from a standing position, are innervated by the tibial branch of the sciatic nerve.

A 16-year-old female is brought to the emergency department with severe abdominal pain and fever. Laboratory examination is remarkable for an elevated white blood cell count and a positive test for pregnancy. Colpocentesis is performed to ascertain the presence of blood in the pelvis from the ruptured ectopic pregnancy. Through which of the following structures does the needle need to be inserted? A. Through the perineal body into the vesicouterine space B. Through the posterior fornix into the rectouterine pouch C. Through the anterior fornix into the endocervical canal D. Through the introitus into the vestibular gland E. Though the perineal membrane into the urogenital diaphragm

B. It is very likely that the ectopic pregnancy ruptured into the rectouterine pouch, also known as the pouch of Douglas. The most direct route to the rectouterine pouch is through the vaginal wall at the posterior vaginal fornix. It is unlikely that the preg- nancy would have occurred in the vesicouterine space because the transfer of ova from the ovary to the fim- briae occurs on the posterior side of the broad liga- ment. Therefore, it would not be advisable to attempt initially to insert a needle into the vesicouterine space. Inserting a needle through the anterior fornix into the endocervical canal would lead one into the uterine cavity, with the probability of other undesir- able consequences. The urogenital diaphragm is a closed space in the perineum. Entering a vestibular gland with a needle would not be near the location of ectopic pregnancy.

A 16-year-old teenage girl suffered an inversion sprain of her ankle during dance class. Physical examination in the clinic most likely reveals severe tenderness over which ligament? A. Calcaneonavicular (spring) B. Calcaneofibular C. Long plantar D. Short plantar E. Deltoid

B. Ligaments act to prevent excessive movement of joints. When a joint is forced into a position, that ligament is stretched and will be tender or rupture if the force is severe enough. Inversion is when the sole of the foot is turned medially and therefore will stretch ligaments that oppose this action. The calcaneofibular ligament is on the lateral side and stretches between the fibula and the calcaneous. It is the only ligament that would be damaged during such an action. The calcaneonavicular and long and short plantar ligaments are located on the plantar surface of the foot and will not be damaged during inversion injuries. The deltoid ligament is located medially and will not be affected.

A 2-day-old female infant is diagnosed with tracheoesophageal fistula. In addition, physical examination reveals an imperforate anus. An MRI examination reveals that the rectum, vagina, and colon are joined into a single channel. Which of the following structures is directly involved in this malformation? A. Labioscrotal folds B. Persistent cloaca C. Urogenital folds D. Genital tubercle E. Urogenital membrane

B. Most anorectal anomalies result from abnor- mal development of the urorectal septum, ultimately resulting in nondivision of the cloaca into urogenital and anorectal parts. The common outlet of the intes- tinal, urinary, and reproductive tracts is specifically associated with a persistent cloaca. The labioscrotal folds are involved in forming the external urethral orifice only. The urogenital folds normally fuse along the ventral side of the penis to form the spongy ure- thra. Epispadias is an anomaly in the development of the genital tubercle and involves the urethral orifice. The urogenital membrane is bounded by the urogeni- tal folds and ruptures to form the urogenital orifice.

A 32-year-old male basketball player comes down hard on his ankle. He is admitted to the outpatient clinic, and radiologic examination reveals a Pott's fracture. What ligament is most likely injured? A. Calcaneofibular ligament B. Deltoid ligament C. Spring ligament D. Plantar ligament E. Long plantar ligament

B. Pott's fracture is a rather archaic term for a fracture of the fibula at the ankle. The term is often used to indicate a bimalleolar fracture of fibula and tibia, perhaps with a tear in the medial collateral ligament, allowing the foot to be deviated laterally. (The medial malleolus will often break before the deltoid ligament tears.) This fracture is also known as Dupuytren's fracture. The fracture results from abduction and lateral rotation of the foot in extreme eversion. There can also be fracture of the posterior aspect of the distal tibia. The spring ligament, also known as the plantar calcaneonavicular ligament, extends from the calcaneus to the navicular bone and is a part of the medial longitudinal arch. This ligament would not be affected in eversion or inversion of the ankle. The plantar ligament, which is composed of the long and short plantar ligaments, supports the lateral longitudinal arch of the foot and would therefore not be affected by inversion or eversion of the foot. The calcaneofibular ligament runs from the calcaneus to the fibula. It would be injured during inversion of the foot, not in eversion, as is the case in a Pott's fracture

In the production of female gametes, which of the following cells can remain dormant for 12 to 40 years? (A) Primordial germ cell (B) Primary oocyte (C) Secondary oocyte (D) First polar body (E) Second polar body

B. Primary oocytes are formed by month 5 of fetal life and remain dormant until puberty, when hormonal changes in the young woman stimulate the ovarian and menstrual cycles. From 5 to 15, oocytes then begin maturation with each ovarian cycle throughout the woman's reproductive life.

A 29-year-old, woman in her 27th week of pregnancy visits her obstetrician for a follow-up appointment. Ultrasound from a previous visit revealed a 4-cm mass growing at the base of the spine. What is the most likely diagnosis? A. Incomplete closure of the embryonic neural tube (lumbosacral myelomeningocele) B. Remnants of the primitive streak (sacrococcygeal teratoma) C. Swelling or growth of the endothelial cells that line blood vessels (hemangioma) D. Neuroendocrine tumor arising from any neural crest element of the sympathetic nervous system (neuroblastoma) E. Benign nerve sheath tumor of the peripheral nervous system (neurofibroma)

B. Remnants of the primitive streak (a thick linear band of epiblast at the beginning of week 3) may persist and give rise to a large tumor known as a sacrococcygeal teratoma. Because it is derived from pluripotent primitive streak cells, the tumor contains tissues derived from all three germ layers in incomplete stages of differentiation. Sacrococcygeal teratomas are the most common tumors in newborn infants and have an incidence of approximately 1 in 27,000. These tumors are usually surgically excised promptly and the prognosis is good.

Which structure is most likely to be removed by a pulmonary surgeon in a surgical resection of a lobe (lobectomy) to remove lung cancer in the apex of the right lung?

B. Right superior secondary (eparterial) bronchus

A 32-year-woman is admitted to the emergency department as a rape case. Fluids from her vagina are collected for DNA and fructose examination. Which of the following male organs is responsible for fructose production? A. Prostate gland B. Seminal vesicles C. Kidneys D. Testis E. Bulbourethral (Cowper) glands

B. Seminal vesicles produce the alkaline constituent of the ejaculate. This includes fructose and choline. The prostate gland secretes prostaglandins, citric acid, and acid phosphatase. The kidneys are the site of urine production. The testes produce spermatozoa and sex hormones. The bulbourethral glands (Cowper's glands) produce mucous secretions that enter the penile bulb.

The news reported that the 58-year-old ambassador received a slashing wound to the medial thigh and died from exsanguination in less than 2 minutes. What was the most likely nature of his injury? A. The femoral artery was cut at the inguinal ligament B. A vessel or vessels were injured at the apex of the femoral triangle C. The femoral vein was transected at its junction with the saphenous vein D. The medial circumflex femoral was severed at its origin E. The deep femoral artery was divided at its origin

B. The apex of the femoral triangle occurs at the junction of the adductor longus and sartorius muscles. The subsartorial (Hunter's) canal begins at this location. Immediately deep to this anatomic point lie the femoral artery, femoral vein, deep femoral artery, and deep femoral vein, often overlying one another in that sequence. This has historically been a site of injuries caused by slipping while handling a very sharp butcher's knife. For this reason, injuries at this location are referred to as the "butcher's block" injury. Fatal loss of blood can occur in just a few minutes if pressure, or a tourniquet, is not applied immediately. The common iliac artery becomes the femoral artery at the inguinal ligament. The saphenous vein joins the femoral vein at the saphenous hiatus, or fossa ovalis. The medial circumflex femoral usually arises from the deep femoral artery about 3 to 5 inches inferior to the inguinal ligament, near the origin of the deep femoral artery from the common femoral. Serious blood loss can occur with injury to any of these vessels, although injury to them is not often fatal.

Where does the blastocyst normally implant? (A) Functional layer of the cervix (B) Functional layer of the endometrium (C) Basal layer of the endometrium (D) Myometrium (E) Perimetrium

B. The blastocyst implants in the functional layer of the uterine endometrium. The uterus is composed of the perimetrium, myometrium, and endometrium. Two layers are identified within the endometrium: (1) the functional layer, which is sloughed off at menstruation, and (2) the basal layer, which is retained at menstruation and serves as the source of regeneration of the functional layer. During the progestational phase of the menstrual cycle, the functional layer undergoes dramatic changes; uterine glands enlarge and vascularity increases in preparation for blastocyst implantation.

A 22-year-old football player is admitted to the hospital with pain and swelling over the lateral aspect of the ankle. The emergency department doctor diagnoses an inversion sprain. Which ligament was most likely injured? A. Plantar calcaneonavicular (spring) B. Calcaneofibular C. Long plantar D. Short plantar E. Deltoid

B. The calcaneofibular ligament is a round cord that passes posteroinferiorly from the tip of the lateral malleolus to the lateral surface of the calcaneus. A forced inversion of the foot can result in tearing of the calcaneofibular ligament and sometimes the anterior talofibular ligament as well. Both of these ligaments act to stabilize the foot and prevent an inversion injury. The plantar calcaneonavicular ligament supports the head of the talus. The long plantar ligament passes from the planter surface of the calcaneus to the groove on the cuboid and is important in maintaining the longitudinal arch of the foot. The short plantar ligament is located deep (superior) to the long plantar ligament and extends from the calcaneus to the cuboid and is also involved in maintaining the longitudinal arch of the foot. The deltoid (medial ligament of the ankle) attaches proximally to the medial malleolus and fans out to reinforce the joint capsule of the ankle.

A 5-month-old baby boy is admitted to the pediatric orthopedic clinic. During physical examination it is noted that the baby has inversion and adduction of the forefoot relative to the hindfoot and plantar flexion. Which of the following terms is diagnostic for the signs observed on physical examination? A. Coxa vara B. Talipes equinovarus C. Hallux valgus D. Hallux varus E. Plantar fasciitis

B. The child has the problem of talipes equinovarus, or clubfoot. Clubfoot is a congenital malformation observed in about 1 in 1000 pediatric patients and first appears in the first trimester of pregnancy. This syndrome combines plantar flexion, inversion, and adduction of the foot. The heel is drawn upward by the tendo calcaneus and turned inward; the forefoot is also adducted, or turned inward. The foot usually is smaller than normal. In coxa vara, the angle between the femoral shaft and neck is reduced to less than 120 degrees, often due to excessive activity of the adductor musculature. Hallux valgus is also known as bunion, in which the big toe points laterally. Hallux varus involves a medial deviation of the first metatarsal or big toe, sometimes the result of attempted correction of bunions. It can also result from arthritis or muscular problems.

Upon removal of a knee-high leg cast, a 15-year-old boy complains of numbness of the dorsum of his right foot and inability to dorsiflex and evert his foot. Which is the most probable site of the nerve compression that resulted in these symptoms? A. Popliteal fossa B. Neck of the fibula C. Lateral compartment of the leg D. Anterior compartment of the leg E. Medial malleolus

B. The common fibular (peroneal) nerve winds around the neck of the fibula before dividing into superficial and deep branches that go on to innervate the lateral and anterior compartments of the leg, respectively. These compartments are responsible for dorsiflexion and eversion of the foot, and injury to these nerves would result in deficits in these movements. The tibial nerve lies superficially in the popliteal fossa. This nerve innervates the posterior compartment of the leg, so compression in this area would result in a loss of plantar flexion and weakness of inversion. The lateral compartment of the leg is innervated by the superficial fibular (peroneal) nerve and is mainly involved in eversion of the foot. The cutaneous branches of the superficial fibular (peroneal) nerve emerge through the deep fascia in the anterolateral aspect of the leg and supply the dorsum of the foot. The anterior compartment of the leg is innervated by the deep fibular (peroneal) nerve and is mainly involved in dorsiflexion of the foot. The medial malleolus is an inferiorly directed projection from the medial side of the distal end of the tibia. The tibial nerve runs near the groove behind the medial malleolus, and compression at this location would result in loss of toe flexion, adduction, abduction, and abduction of the great toe

A 35-year-old man is admitted to the surgery ward for correction of an inguinal hernia. During physical examination an indirect hernia is noted, together with an absent cremasteric reflex. Which of the following nerves carries the efferent component of the cremasteric reflex? A. Ilioinguinal nerve B. Genital branch of genitofemoral C. Iliohypogastric nerve D. Pudendal nerve E. Obturator nerve

B. The cremasteric reflex afferents are carried by the iliolinguinal nerve; the motor (efferent) out- put is by the genitofemoral nerve. The sensory fibers of the genitofemoral nerve are to skin over the femoral triangle. The ilioinguinal nerve is sensory to parts of the suprapubic region, anterior perineum, and in- ner thigh. The iliohypogastric nerve provides sensa- tion for the abdominal wall and suprapubic area. The pudendal and obturator nerves do not travel through the inguinal canal and would not be dam- aged from the hernia. In addition, they play no role in the cremasteric reflex.

A 16-year-old boy presents to the emergency department with a fracture of the first and second toes of his right foot. He received an anesthetic injection in the first web space of his foot, to permit easy manipulation and correction. Which nerve was blocked by the anesthesia? A. Saphenous B. Cutaneous branch of deep fibular (peroneal) C. Cutaneous branch of superficial fibular (peroneal) D. Sural E. Common fibular (peroneal)

B. The deep fibular (peroneal) nerve is a branch of the common fibular (peroneal) nerve. It is mainly a motor nerve that innervates the anterior compartment of the leg. Its only cutaneous innervation is to the skin of the first web space. The saphenous nerve innervates the medial side of the leg and foot. The cutaneous branch of superficial fibular (peroneal) nerve innervates the anterior part of the lower leg and the dorsum of the foot. The sural nerve innervates the lateral side of the leg and foot.

A 34-year-old male long-distance runner complained to the team physician of swelling and pain of his shin. Skin testing in a physical examination showed normal cutaneous sensation of the leg. Muscular strength tests showed marked weakness of dorsiflexion and impaired inversion of the foot. Which nerve serves the muscles involved in the painful swelling? A. Common fibular (peroneal) B. Deep fibular (peroneal) C. Sciatic D. Superficial fibular (peroneal) E. Tibial

B. The deep fibular (peroneal) nerve supplies the dorsiflexors of the foot, including the extensor hallucis longus and extensor digitorum longus. It also supplies the tibialis anterior, an invertor of the foot. This nerve has sensory distribution only to the skin between the first two toes. The common fibular (peroneal) nerve supplies not only the preceding muscles but also the evertors of the foot and provides sensation for most of the dorsum of the foot. The sciatic nerve innervates the muscles of the posterior thigh and all muscles of the leg and foot, in addition to providing sensory supply in those areas. The superficial fibular (peroneal) nerve innervates the evertors of the foot and provides sensation for the dorsum of the foot. The tibial nerve is the nerve for muscles of the posterior compartment of the leg and also of the plantar region and supplies sensation over the medial aspect of the leg posteriorly and the plantar surface of the foot and toes

A 29-year-old pregnant woman is admitted to the hospital for her delivery. During a vaginal delivery the obstetrician performs a median episiotomy in which the area of the perineal body is cut deeply. Two weeks after the delivery the woman complains that she has had fecal incontinence since the delivery. Which of the following structures was also most likely damaged during the episiotomy? A. Superficial and deep transverse perineal muscles B. External anal sphincter C. Ischiocavernosus muscle D. Sacrospinous ligament E. Sphincter urethra

B. The external anal sphincter is important for maintaining fecal continence. The external anal sphincter is located immediately posterior to the perineal body (central tendon) and would be susceptible to damage during a median episiotomy. The other structures listed play no role in maintaining fecal continence.

A 49-year-old man is admitted to the emergency department with a cold and pale foot. Physical examination reveals that the patient suffers from peripheral vascular disease; his popliteal artery is occluded and no pulse is felt upon palpation. What is the landmark to feel the pulse of the femoral artery? A. Adductor canal B. Femoral triangle C. Popliteal fossa D. Inguinal canal E. Pubic symphysis

B. The femoral triangle is the best place to palpate the femoral pulse. It is bounded by the sartorius muscle laterally, adductor longus medially, and the inguinal ligament superiorly. It contains the femoral vein, artery, and nerve (from medial to lateral, respectively). The adductor canal lies deep between the anterior and medial compartments of the thigh and therefore cannot be palpated. The popliteal fossa is the fossa at the back of the knee and contains the popliteal artery and vein, tibial nerve, and common fibular (peroneal) nerve. The femoral pulse cannot be palpated here. The inguinal canal is in the pelvis and is in communication with the anterior abdominal wall. It contains the spermatic cord in males and round ligament of the uterus in females.

A 24-year-old man is admitted to the emergency department after a car collision. Radiologic examination reveals a fracture at the junction of the middle and lower thirds of the femur. An MRI examination provides evidence that the popliteal vessels were injured when the distal fragment of the fracture was pulled posteriorly. Which of the following muscles is most likely to displace the distal fracture fragment? A. Soleus B. Gastrocnemius C. Semitendinosus D. Gracilis E. Tibialis anterior

B. The gastrocnemius muscle arises from the femur just proximal to the femoral condyles. This strong muscle could displace the distal fragment of the fractured femur posteriorly. In addition, the popliteal artery is the deepest structure in the popliteal fossa (right against the popliteal surface of the distal femur) and is susceptible to laceration in this scenario as the fractured end of the distal femoral fragment is pulled against the popliteal artery. Orthopedic surgeons always look for damage to the popliteal artery in a patient with a supracondylar fracture. The soleus arises from the tibia and would have no effect upon the femur. The semitendinosus arises from the ischial tuberosity and inserts medially on the proximal tibia, via the pes anserinus. The tibialis anterior arises from the tibia and inserts mostly onto the navicular bone.

During an interview, a 30-year-old man who is a psychiatric patient suddenly becomes aggressive. In order to calm him down, the patient is given an intramuscular injection in the upper lateral quadrant of the buttock. The injection is given at this specific location to prevent damage to which of the following nerves? A. Lateral femoral cutaneous B. Sciatic C. Superior gluteal D. Obturator E. Inferior gluteal

B. The gluteal region can be divided into quadrants by two lines positioned using palpable bony landmarks. One line runs inferiorly from the highest point of the iliac crest. The second line runs horizontally midway between the iliac crests and the ischial tuberosity. This divides the gluteal region into four quadrants. The sciatic nerve runs through the lower medial quadrant and must be avoided during intragluteal injections.

In preparing to isolate the proximal portion of the femoral artery, the vascular surgeon gently separated it from surrounding tissues. Posterior to the femoral sheath, what muscle forms the lateral portion of the floor of the femoral triangle? A. Adductor longus B. Iliopsoas C. Sartorius D. Pectineus E. Rectus femoris

B. The iliopsoas forms the lateral portion of the troughlike floor of the femoral triangle. The pectineus forms the medial portion of this floor. The adductor longus provides a medial border for the femoral triangle and meets the sartorius, the lateral border of the triangle, at the apex. The rectus femoris is a superficial contributor to the quadriceps femoris, lateral to the femoral triangle.

A 34-year-old woman is at her third stage of delivery. The obstetrician is concerned that the pelvic canal is too narrow for a vaginal delivery. Which of the following dimensions is the most reliable determinant of the capacity for a vaginal birth? A. Transverse diameter B. Interspinous distance C. True conjugate diameter D. Diagonal conjugate E. Oblique diameter

B. The interspinous distance is the distance be- tween the ischial spines. The interspinous distance is usually the shortest distance, therefore being the most restricted area along the birth canal. The true conju- gate diameter is the anteroposterior distance and does not change. The transverse diameter, oblique diame- ter, and diagonal conjugate diameter can change slightly during pregnancy, but the interspinous dis- tance changes the most during birth; plus, it is more easily measured.

A 72-year-old woman is admitted to the hospital with a painful right foot. A CT scan examination reveals a thrombotic occlusion of the femoral artery in the proximal part of the adductor canal. Which artery will most likely provide blood supply to the leg through the genicular anastomosis? A. Medial circumflex femoral B. Descending branch of the lateral circumflex femoral C. First perforating branch of the deep femoral D. Inferior gluteal E. Descending genicular branch of femoral

B. The lateral circumflex femoral artery arises from the deep femoral (profunda femoris) artery of the thigh and sends a descending branch down the length of the femur to anastomose with the superior medial genicular artery and the superior lateral genicular artery. The medial circumflex femoral artery is responsible for supplying blood to the head and neck of the femur, and it does not anastomose with distal vessels at the knee. The first perforating artery sends an ascending branch that anastomoses with the medial circumflex femoral and the inferior gluteal artery in the buttock. The inferior gluteal artery is a branch of the internal iliac; it has important anastomotic supply to the hip joint. The typically small descending genicular branch of the femoral artery is given off just proximal to the continuation of the femoral artery as the popliteal.

A 37-year-old unconscious man is rushed to the emergency department after being retrieved from a motor vehicle crash. On physical examination bruising and obvious deformity is seen over his left knee joint. Radiological studies showed a posteriorly dislocated supracondylar fracture with severe compression of the popliteal artery. Which of the following arteries would ensure adequate blood supply to the leg and foot in this patient? A. Medial femoral circumflex B. Lateral femoral circumflex C. Anterior tibial artery D. Posterior tibial artery E. Fibular (peroneal) artery

B. The lateral femoral circumflex artery is a branch of the femoral artery close to the hip joint. It gives a branch that runs down the lateral aspect of the thigh and joins the genicular anastomosis via the superior lateral genicular artery. The medial circumflex femoral artery does not provide any branches that descend toward the knee. The anterior and posterior tibial arteries are the terminal branches of the popliteal artery and would not receive any blood if the popliteal is damaged. The fibular (peroneal) artery is a branch of the posterior tibial artery.

A 35-year-old man is admitted to the hospital after being kicked in the groin while playing football. During physical examination it is noted that the left testicle of the patient is swollen. An MRI examination reveals coagulation of blood in the veins draining the testis. Into which of the following veins would a thrombus most likely pass first from the injured area? A. Inferior vena cava B. Left renal vein C. Left inferior epigastric D. Left internal pudendal E. Left iliac vein

B. The left testicular vein drains directly into the left renal vein, which then crosses over the midline to enter the inferior vena cava. The left inferior epigastric, left internal pudendal, and left iliac veins are not involved in the drainage of the testes.

A 34-year-old woman has a direct blow to the patella by the dashboard of the vehicle during an automobile crash. The woman is admitted to the emergency department and radiologic examination reveals patellofemoral syndrome. This type of syndrome is characterized by lateral dislocation of the patella. Which of the following muscles requires strengthening by physical rehabilitation to prevent future dislocation of the patella? A. Vastus lateralis B. Vastus medialis C. Vastus intermedius D. Rectus femoris E. Patellar ligament

B. The lower portion of the vastus medialis inserts upon the medial aspect of the patella and draws it medially, especially in the last quarter of extension—during which it is especially palpable in contraction. This lower portion of the muscle is referred to as the vastus medialis obliquus (VMO). Increasing the strength of this muscle can lessen the lateral dislocation of the patella. The rectus femoris arises from the anterior inferior iliac spine and lip of the acetabulum and draws the patella vertically upward, as does the vastus intermedius.

During the preparation of an evening meal a female medical student dropped a sharp, slender kitchen knife. The blade pierced the first web space of her foot, resulting in numbness along adjacent sides of the first and second toes. Which nerve was most likely injured? A. Saphenous B. Deep fibular (peroneal) C. Superficial fibular (peroneal) D. Sural E. Common fibular (peroneal)

B. The medial branch of the deep fibular (peroneal) nerve accompanies the dorsalis pedis artery and innervates the skin between the contiguous sides of the first and second toes. The saphenous nerve is responsible for cutaneous innervation of the anteromedial aspect of the leg and foot. The superficial fibular (peroneal) nerve innervates most of the dorsum of the foot, with the exception of the area where sensation was lost (medial branch of deep fibular nerve). The common fibular (peroneal) nerve gives off a cutaneous branch, the sural nerve, which innervates the lateral aspect of the leg and lateral side of the foot.

A 7-year-old girl accidentally stepped on a sharp snail shell while walking to the beach. She was admitted to the hospital, where she received a tetanus shot, and the wound was cleaned thoroughly and sutured. One week later, during a return visit to her physician, it is seen that she has great difficulty in flexing her big toe, even though there is no inflammation present in the sole of the foot. Which nerve was most likely damaged by the piercing of the shell? A. Lateral plantar nerve B. Medial plantar nerve C. Sural nerve D. Superficial fibular (peroneal) nerve E. Deep fibular (peroneal) nerve

B. The medial plantar nerve innervates the abductor hallucis and both flexor hallucis longus and brevis. This nerve also provides motor supply for the flexor digitorum brevis and the first lumbrical. The lateral plantar nerve innervates all other intrinsic muscles in the plantar region of the foot. The sural nerve is sensory to the lateral posterior leg and lateral side of the foot; it arises from a combination of branches of the tibial nerve and common fibular (peroneal) nerve. The deep fibular (peroneal) nerve supplies dorsiflexors, toe extensors, and invertors of the foot.

A 30-year-old woman was admitted to the emergency department after being involved in a motor vehicle crash. The patient complained of pain to the right hip and knee. During physical examination, there is no deformity of the lower limb, but there is tenderness over the right ischiopubic ramus. Pelvic radiographs revealed an inferiorly displaced fracture of the right superior and inferior pubic rami with dislocation of the right sacroiliac joint and pubic symphysis. The patient was referred to the orthopedics team, which had a high suspicion of rupture of the right obturator membrane. What clinical findings are most likely to be present in this patient? A. Urinary and fecal incontinence and diminished sensation over the perineum B. Weak adduction of the hip and diminished sensation over the upper medial thigh C. Weak abduction of the hip and positive Trendelenburg sign D. Weak flexion of the hip and diminished sensation over the anterior thigh and medial leg E. Weak extension of the hip and diminished sensation over the posterior thigh

B. The obturator membrane is a thin membrane that covers the obturator foramen except at its superior part. The obturator nerve exits the pelvis and enters into the medial compartment of the thigh by passing through the obturator canal alongside the obturator vessels. Traumatic injuries to the membrane will most likely lead to obturator nerve damage. The obturator nerve supplies motor innervations to the adductor muscles of the thigh (gracilis, obturator externus, adductor longus, adductor brevis and a portion of the adductor magnus). It also provides sensory innervation to the medial aspect of the thigh. Urinary and fecal incontinence is mediated by autonomic nerves and the pudendal nerve. Both nerves have no relationship with the obturator membrane. The gluteus medius and minimus muscles are the main hip abductors. They also stabilize the hip on the swing-side during motion. These muscles are supplied by the superior gluteal nerve, which leaves the pelvis through the greater sciatic foramen above the piriformis muscle. Flexors of the hip found in the anterior compartment of the thigh are innervated by the femoral nerve, which has no relationship with the obturator membrane. The sciatic nerve supplies the muscles in the posterior compartment of the thigh and also sends cutaneous innervations to the skin of the posterior thigh. It enters the posterior compartment of the thigh from the gluteal region.

A 56-year-old man with advanced bladder carcinoma suffers from difficulty while walking. Muscle testing reveals weakened adductors of the right thigh. Which nerve is most likely being compressed by the tumor to result in walking difficulty? A. Femoral B. Obturator C. Common fibular (peroneal) D. Tibial E. Sciatic

B. The obturator nerve arises from the lumbar plexus and enters the thigh through the obturator canal. This nerve is responsible for innervation of the medial compartment of the thigh (adductor compartment). Injury to this nerve can result in weakened adduction and difficulty walking. The femoral nerve innervates muscles of the anterior compartment of the thigh that are responsible for hip flexion and leg extension. The sciatic nerve branches into the common fibular (peroneal) and tibial nerves. The common fibular (peroneal) nerve branches into the deep and superficial branches of the fibular (peroneal) nerve responsible for innervation of the anterior and lateral compartments of the leg, respectively. The tibial nerve innervates the muscles of the posterior compartment of the thigh and leg, which are responsible for extension of the hip, flexion of the leg, and plantar flexion of the foot.

A 45-year-old intoxicated man was struck by a tour bus while walking in the middle of the street. The man was admitted to the emergency department and during physical examination was diagnosed with "adductor gait," in which an individual crosses one limb in front of the other, due to powerful hip adduction. Which of the following nerves was most likely involved in this condition? A. Tibial B. Obturator C. Inferior gluteal D. Superior gluteal E. Femoral

B. The obturator nerve innervates the adductor muscles, including the gracilis, pectineus, and obturator externus. The tibial nerve supplies the calf muscles and intrinsic muscles in the plantar portion of the foot. The inferior gluteal nerve innervates the gluteus maximus; the superior gluteal nerve supplies the gluteus medius and minimus and tensor fasciae latae. The femoral nerve provides motor supply to the quadriceps femoris, sartorius, and, in some cases, the pectineus. This gait pattern is characteristic of hypertonia in the lower limb. As a result these areas become flexed to various degrees, giving the appearance of crouching, while tight adductors produce extreme adduction

After a revascularization procedure involving the common iliac artery, a 68-year-old man has difficulty walking. Nerve conduction studies reveal decreased activity in the nerve that innervates the adductors of the thigh. Which nerve is this? A. Femoral B. Obturator C. Common fibular (peroneal) D. Tibial E. Sciatic

B. The obturator nerve is responsible for innervation of the thigh adductors which form the medial compartment of the thigh. The femoral nerve innervates the anterior compartment and is responsible for extension of the knee. Common fibular (peroneal) nerve supplies the anterior and lateral compartments of the leg while the tibial nerve supplies the posterior compartments of the leg and thigh. The common fibular (peroneal) and tibial nerves are branches of the sciatic nerve.

A 23-year-old man is admitted to the emergency department with pain and cyanosis of his right lower limb. Doppler ultrasound studies reveal deficiency in development of his femoral artery, which appears to terminate mid thigh. A thrombotic occlusion is seen in an unusual, rather tortuous, large vessel in the posterior compartment of the thigh, arising in the gluteal area and continuous inferiorly with a normal appearing popliteal artery. It is decided that a vascular graft should be placed from the femoral artery to the popliteal artery. What is the identity of the aberrant artery? A. A large, fifth perforating branch of the femoral B. An sciatic branch of the inferior gluteal artery C. Descending branch of the medial circumflex femoral D. Descending branch of the superior gluteal artery E. An enlarged descending lateral circumflex femoral artery

B. The original axial vessel of the lower limb is retained as the (usually tiny) sciatic branch of the inferior gluteal artery. In some cases this vessel is retained as the primary proximal vessel to the limb, wherein there is hypoplastic development of the femoral artery. Aneurysms of the enlarged sciatic artery in the gluteal region are relatively common, as is rupture of the vessel (with profuse bleeding) if they are exposed in the gluteal area. The profunda femoris or deep femoral branch of the femoral artery usually provides three perforating branches to the posterior compartment, but not a branch such as that described. The descending branch of the medial circumflex femoral anastomoses with the first perforator. The superior gluteal artery anastomoses with the inferior gluteal by a descending branch or branches. The descending branch of the lateral circumflex femoral is the descending genicular artery, which anastomoses with the superior lateral genicular branch of the popliteal artery

A 45-year-old is admitted to the hospital after his left leg impacted a fence post when he was thrown from a powerful four-wheel all-terrain vehicle. Radiologic examination reveals posterior displacement of the tibia upon the femur. Which of the following structures was most likely injured? A. Anterior cruciate ligament B. Posterior cruciate ligament C. Lateral collateral ligament D. Lateral meniscus ligament E. Patellar ligament

B. The posterior cruciate ligament tightens in flexion of the knee. It can be damaged by posterior displacement of the tibia upon the femur. With the patient seated, a rupture of the ligament can be demonstrated by the ability to push the tibia posteriorly under the femur. This is called the posterior drawer sign because it's similar to pushing in a desk drawer. The anterior cruciate ligament resists knee hyperextension. The lateral collateral ligament is a thick, cordlike band that passes from the lateral femoral condyle to the head of the fibula. It is located external to the capsule of the knee joint. The lateral meniscus is a nearly circular band of fibrocartilage that is located laterally within the knee joint. It is less frequently injured than the medial meniscus because it is not attached to the joint capsule or other ligaments. The patellar ligament is the heavy, ligamentous band of insertion of the quadriceps muscle to the tibial tuberosity. GAS 584-5

Following the insertion of a prosthetic hip joint in a 72-year-old man, it was observed that the patient had greatly diminished sensation in the region of distribution of the posterior femoral cutaneous nerve. Which of the following is characteristic of this nerve? A. Cutaneous supply of the superior aspect of the gluteal region B. Arises from sacral spinal nerve levels S1, S2, S3 C. Motor innervation of the obturator internus and gemelli muscles D. Injury results in meralgia paresthetica E. Provides origin of the sural nerve

B. The posterior femoral cutaneous nerve arises from nerves S1 to S3. It provides inferior cluneal branches to the lower portion of the gluteal region and a perineal branch to the perineum and supplies sensation to the posterior thigh to the level of the popliteal fossa. Superior gluteal innervation arises from dorsal rami of L1 to L3. Meralgia paresthetica is the occurrence of pain or burning sensations on the lateral thigh, from compression of the lateral femoral cutaneous nerve. The sural nerve, sensory to the lower calf and lateral foot, arises from contributions from the tibial nerve and common fibular (peroneal) nerve. The posterior femoral cutaneous is a sensory nerve and does not innervate muscles.

A female newborn was found to have a large midline tumor in the lower sacral area, which was diagnosed as a sacrococcygeal tumor. Which of the following courses of treatment is recommended for this child? (A) Immediate chemotherapy and radiation treatment (B) Surgical removal of the tumor by age 6 months (C) Surgical removal of the tumor at age 4-5 years (D) Surgical removal of the tumor at age 13-15 years (E) No treatment because this tumor normally regresses with age

B. The preponderance of sacrococcygeal tumors is found in female newborns. Since these tumors develop from pluripotent cells of primitive streak origin, malignancy is of great concern, and the tumor should be surgically removed by age 6 months. Occasionally, these tumors may recur after surgery, demonstrating malignant properties.

A 23-year-old man is taken to the emergency department because of anorexia, nausea, vomiting, and severe abdominal pain in the right lower quadrant. On examination, he has tenderness in the right lower quadrant with rebound tenderness. The physician suspects appendicitis. To confirm this diagnosis, the physician attempts to straighten the patient's flexed thigh. This causes the patient to wince with pain. Which of the following muscles most likely caused this symptom? A. Adductor magnus B. Psoas major C. Biceps femoris D. Obturator internus E. Gluteus maximus

B. The psoas muscle arises from the base of the transverse processes, the sides of the vertebral bodies, and the intervertebral discs, from the twelfth thoracic to the fifth lumbar vertebrae and inserted into the lesser trochanter of the femur. The psoas flexes the thigh at the hip joint on the trunk, or if the thigh is fixed, it flexes the trunk on the thigh, as in sitting up from a lying position. The inflamed appendix is pushed up against the peritoneum from the the contracted psoas. As a result it is in touch with the parietal peritoneum, producing acute pain. In some other cases it may retain the purulence of a psoas abscess, and spinal tuberculosis may present as a cold abscess in the groin. The psoas is enclosed in a fibrous sheath that is derived from the lumbar fascia. The sheath is not part of the lumbar fascia but the lateral edge blends with the anterior layer of that fascia.

A 51-year-old immigrant with tuberculosis is found to have large flocculent masses over the lateral lumbar spine. There is a similar mass located in the ipsilateral groin. Physical examination reveals increased tenderness just medial to the ipsilateral anterior superior iliac spine on palpation. This pattern of involvement most likely suggests an abscess tracking along which of the following muscles? A. Piriformis B. Psoas major C. Adductor longus D. Gluteus maximus E. Obturator internus

B. The psoas muscle arises from the base of the transverse processes, the sides of the vertebral bodies, and the intervertebral discs, from the twelfth thoracic to the fifth lumbar vertebrae and inserted into the lesser trochanter of the femur. The sheath of the psoas retains the pus of a psoas abscess, and spinal tuberculosis may present as a cold abscess in the groin (in the vicinity of the lesser trochanter). The psoas is enclosed in a fibrous sheath that is derived from the lumbar fascia. The sheath is not part of the lumbar fascia, but the lateral edge blends with the anterior layer of that fascia

A 53-year-old woman reports difficulty in walking. Physical examination showed a positive Trendelenburg sign when she is asked to stand on her right leg. Which nerve has been compromised to produce the positive sign? A. Sciatic B. Right superior gluteal C. Left inferior gluteal D. Left superior gluteal E. Right inferior gluteal

B. The right superior gluteal nerve is the correct choice. When a person stands on one leg or walks, the gluteus medius, gluteus minimus, and tensor muscles of the fascia latae act in synergy to stabilize the hip joint by abducting the hip (pelvic tilt). These muscles receive their innervation from the superior gluteal nerve. The abductors of the hip, as they contract to maintain the stability of the hip joint, draw the pelvis forcefully toward the weigh-bearing leg, causing the opposite side of the pelvis to tilt in that same direction. The right superior gluteal nerve innervates its ipsilateral medius, minimus, and tensor muscles of the fascia latae. Loss of these muscles results in a positive Trendelenburg sign with the pelvis dropping on the left side.

During a gymnastic session, a 24-year-old woman suddenly developed pain and swelling on the right buttock. This happened following a forceful thigh movement. There is severe weakness of right hip extension and knee flexion. Adduction of the thigh is also slightly weak. An avulsion fracture of the ischial tuberosity is found on a radiograph. Which of the following group of muscles has most likely involved in this process? A. Adductor brevis, adductor longus, adductor magnus, pectineus, and gracilis B. Biceps femoris, semimembranosus, semitendinosus, and adductor magnus C. Iliacus and psoas major D. Gluteus medius and gluteus minimus E. Gluteus maximus and adductor magnus F. Iliacus, psoas major, rectus femoris and sartorius

B. The rough bony projection at the junction of the inferior end of the body of the ischium and its ramus is the large ischial tuberosity. Much of the body's weight rests on these tuberosities when sitting, and it provides the proximal, tendinous attachment of the posterior thigh muscles (hamstring muscles and adductor magnus). The hamstring muscles are associated with hip extension and knee flexion. The adduction of the hip joint will be affected slightly because the adductor magnus is affected, although the rest of the adductor muscles are intact.

A 60-year-old man presents with pain on the medial aspect of his thigh. During physical examination he describes the pain to be constant, nonradiating and he also complains of numbness on the medial aspect of his leg and medial plantar arch. The nerve involved in this patient's numbness is closely associated with a structure with which of the following characteristics? A. Empties into the popliteal vein B. In its ascent in the medial aspect of the leg, it travels posterior to the medial condyle of the femur C. In its ascent in the medial aspect of the leg, it travels anterior to the medial condyle of the femur D. Arches posterior to the medial malleolus E. Is associated with nodes that drain to the horizontal group of inguinal nodes

B. The saphenous nerve is the longest and most widely distributed cutaneous branch of the femoral nerve; it is the only branch not from the sciatic nerve to extend beyond the knee. It gives sensory innervations to the medial aspect of the thigh, leg, and the medial planter arch. It accompanies the great saphenous vein over the medial side of the leg. The great saphenous vein is formed by the union of the dorsal vein of the great toe and the dorsal venous arch of the foot. It ascends anterior to the medial malleolus and passes posterior to the medial condyle of the femur and ends when it joins the femoral vein.

A 42-year-old man is bitten on his posterior thigh by a dog. The superficial wound is sutured in the emergency department. Four days later the patient returns to the hospital with high fever and swollen lymph nodes. Which group of nodes first receives lymph from the infected wound? A. External iliac B. Vertical group of superficial inguinal C. Deep inguinal D. Horizontal group of superficial inguinal E. Internal iliac

B. The superficial inguinal nodes are located near the saphenofemoral junction and drain the superior thigh region. The vertical group receives lymph from the superficial thigh, and the horizontal group receives lymph from the gluteal regions and the anterolateral abdominal wall. The deep inguinal lie deep to the fascia lata and receive lymph from deep lymph vessels (popliteal nodes). The external and internal iliac nodes first receive lymph from pelvic and perineal structures.

A 30-year-old man was brought to the emergency department after his involvement in a motor vehicle crash. He complained of pain in the right knee and an inability to bear weight. On examination, there were minor abrasions about his body, in addition to a deep, 5 cm oblique laceration over the anterior right knee, which exposed the patella. He was unable to extend the right knee. Radiographs revealed a displaced transverse fracture of the inferior pole of the patella. The superior fragment of the patella appeared to be "high riding" over the anterior surface of the femur. Which of the following most likely occurred? A. Blood and fat from the injury can enter the popliteus bursa B. Blood and fat from the injury can enter the suprapatellar bursa C. Joint fluid can enter the subcutaneous infrapatellar bursa D. The deep infrapatellar bursa will be affected E. The gastrocnemius bursa will not be affected

B. The suprapatellar bursa is found in the anterior surface of the inferior part of the femur under the quadriceps femoris muscle. A high-riding superior patellar fragment lead to the exposure of this bursa. In traumatic episodes following this condition, blood and fat from the knee can enter into the suprapatellar bursa. The popliteal bursa is located behind the knee, away from the site of injury. The superficial infrapatellar bursa is located below the patella, between the patella ligament and the skin. The deep infrapatellar bursa is located below the site of the injury, between the upper part of the tibia and the patella ligament. The gastrocnemius bursa is at the back of the knee, as well as below the level of the injury.

Which of the following components plays the most active role in invading the endometrium during blastocyst implantation? (A) Epiblast (B) Syncytiotrophoblast (C) Hypoblast (D) Extraembryonic somatic mesoderm (E) Extraembryonic visceral mesoderm

B. The syncytiotrophoblast plays the most active role in invading the endometrium of the mother's uterus. During the invasion, endometrial blood vessels and endometrial glands are eroded, and a lacunar network is formed.

A 27-year-old male triathlon competitor complained that he frequently experienced deep pains in one calf that almost caused him to drop out of a regional track-and-field event. Doppler ultrasound studies indicated, and surgical exposure confirmed, the existence of an accessory portion of the medial head of the gastrocnemius that was constricting the popliteal artery. Above the medial head of the gastrocnemius, the superior medial border of the popliteal fossa could be seen. Which of the following structures forms this border? A. Tendon of biceps femoris B. Tendons of semitendinosus and semimembranosus C. Tendon of plantaris D. Adductor hiatus E. Popliteus

B. The tendons of the semitendinosus and semimembranosus provide the superior medial border of the popliteal fossa. The semitendinosus inserts with the pes anserinus on the proximal, medial tibia. The semimembranosus inserts on the tibia posteriorly. The biceps femoris forms the superior lateral border of the fossa, as the tendon passes to insertion on the fibula. The plantaris arises from the femur just above the lateral head of the gastrocnemius, passing distally to insert on the calcaneus via the tendo Achilles. The popliteus arises from the tibia and passes superiorly and laterally to insert on the lateral condyle of the femur, with a connection to the lateral meniscus.

A 20-year-old man visits the family physician complaining of difficulty to flex and medially rotate his thigh while running and climbing. Which of the following muscles is most likely damaged in this individual? A. Rectus femoris B. Tensor fasciae latae C. Vastus intermedius D. Semimembranosus E. Sartorius

B. The tensor fasciae latae assists in flexion of the thigh, as well as medial rotation and abduction. Damage to this muscle would adversely affect these motions. The rectus femoris extends the hip. The vastus intermedius extends the knee. The semimembranosus extends the hip and flexes and medially rotates the knee. The sartorius assists in flexion and lateral rotation of the hip, as well as in medial rotation of the knee.

A 22-year-old man is admitted to the emergency department after falling from his bicycle. Radiologic examination reveals a fracture of the tibia above the ankle. MRI and physical examination reveal that the tibial nerve is severed on the posterior aspect of the tibia. Which of the following signs will most likely be present during physical examination? A. Sensory loss of the dorsum of the foot B. Sensory loss on the sole of the foot C. Foot drop D. Paralysis of the extensor digitorum brevis E. Sensory loss of the entire foot

B. The tibial nerve divides into the medial and lateral plantar nerves on the medial side of the ankle. These two nerves provide sensation for the sole of the foot. Sensory supply to the dorsum of the foot is provided mostly by the superficial fibular (peroneal) nerve, with the deep fibular (peroneal) nerve providing sensation for the skin between the first and second toes. Foot drop would be caused by interruption of the common fibular (peroneal) nerve. Sensory loss to the lateral side of the foot results from loss of the sural nerve. Paralysis of the extensor digitorum brevis would be attributed to injury to the terminal motor branch of the deep fibular (peroneal) nerve.

Several days after a 34-year-old woman was admitted to the hospital after her automobile collided with a lamppost, her urinary bladder gives evidence of paralysis. A CT scan reveals multiple fractures of her pelvis with a significant pelvic hematoma, either of which could have interrupted or injured the nerve sup- ply of the organ. Which of the following nerves was most likely traumatized? A. Superior hypogastric B. Pelvic splanchnic C. Sacral splanchnic D. Lumbar splanchnic E. Pudendal

B. The urinary bladder wall is formed by the detrusor muscle, and it receives both its motor and sensory innervation from parasympathetic nerve fi- bers transmitted by way of the pelvic splanchnic nerves from S2 to S4.

Which of the following structures must degenerate for blastocyst implantation to occur? (A) Endometrium in progestational phase (B) Zona pellucida (C) Syncytiotrophoblast (D) Cytotrophoblast (E) Functional layer of the endometrium

B. The zona pellucida must degenerate for implantation to occur. Early cleavage states of the blastula are surrounded by a zona pellucida, which prevents implantation in the uterine tube.

A 46-year-old male is admitted to the emergency department after a car crash. An MRI examination reveals a hematoma of the perineum spreading to his abdominal wall beneath the superficial fascia. Where should the initial extravasation be located? A. Between the superior aspect of the urogenital diaphragm and the pelvic diaphragm B. Between the perineal membrane and the fascia of Gallaudet C. Between Camper's fascia and Scarpa's fascia D. Between Colles' fascia and Gallaudet's fascia E. Between Buck's fascia and the tunica albuginea

B. This is a classic example of extravasation of blood and urine from the superficial perineal pouch. This usually is a result of rupture of the spongy ure- thra. The extravasation of the fluid (urine) will begin to invade the layer between the Buck's fascia and dartos layer. This extravasation example is evident due to the fluid invading up to the abdomen between the subcutaneous tissues and muscle fascia. If the fluid collects between the other layers of the perineum, the clinical evidence will present differently in the perineum and abdominal area.

During a football game a 21-year-old wide receiver was illegally blocked by a linebacker, who threw himself against the posterolateral aspect of the runner's left knee. As he lay on the ground, the wide receiver grasped his knee in obvious pain. Which of the following structures is frequently subject to injury from this type of force against the knee? A. Fibular collateral ligament B. Anterior cruciate ligament C. Lateral meniscus and posterior cruciate ligament D. Fibular collateral and posterior cruciate ligament E. All the ligaments of the knee will be affected

B. This type of injury can result in the "unhappy triad" (of O'Donoghue) injury, with damage to the medial collateral ligament (MCL), anterior cruciate ligament (ACL), and medial meniscus. A blow to the lateral side of the knee stretches and tears the MCL, which is attached to the medial meniscus. The ACL is tensed during knee extension and can tear subsequent to the rupture of the MCL. The remaining answer choices describe structures on the lateral surface of the knee, which are not usually injured by this type of trauma.

A 68-year-old male is admitted to the hospital with painful urination and nocturia (urination during the night). MRI examination reveals enlargement and irregularity of the uvula of the urethra. This enlargement resulted in difficulty with urinary voiding and inadequate emptying of the bladder. Which of the following lobes of the prostate gland will most likely be hypertrophied? A. Anterior B. Median C. Lateral D. Posterior E. Lateral and Posterior

B. When the internal urethral orifice is obstructed, it is most likely due to an enlargement of the median (or middle) lobe of the prostate gland. The prostate gland is located at the base of the urinary bladder and is often described as possessing five ill- defined lobes, although this is not accepted by most urologists. The middle lobe consists of glandular tis- sue dorsal to the uvula of the urethral meatus of the urinary bladder, adjacent to the beginning of the ure- thra. This glandular tissue is most frequently involved in benign hypertrophy.

A 24-year-old female motocross racer was involved in a crash that left her right leg pinned under her bike. After the accident, she could no longer extend her right knee. Which of the following nerves was most likely affected? A. Sciatic nerve B. Femoral nerve C. Obturator nerve D. Saphenous nerve E. Tibial nerve

B: Extensors of the knee are the quadriceps femoris muscle, which attaches to the tibia via the patellar ligament. This muscle group is supplied by the femoral nerve. The sciatic nerve supplies the posterior thigh muscles not the anterior compartment. The obturator supplies the medial compartment and the saphenous nerve does not supply motor innervation to any muscles but is a cutaneous branch of the femoral nerve

104 While sharpening his knife, a 23-year-old male soldier accidentally punctured the ventral side of the fi fth digit at the base of the distal phalanx. The wound became infected, and within a few days the infection has spread into the palm, within the sheath of the fl exor digitorum profundus tendons. If the infection were left untreated, into which of the following spaces could it most likely spread? ⃣ A. Central compartment ⃣ B. Hypothenar compartment ⃣ C. Midpalmar space ⃣ D. Thenar compartment ⃣ E. Thenar space

C

106 A 22-year-old woman had suffered a severe knife wound to the upper lateral portion of her pectoral region, with entry of the knife at the deltopectoral groove. Pressure applied to the wound had prevented further profuse bleeding. In the emergency department, vascular clamps were applied to the axillary artery, proximal and distal to the site of injury—which had occurred between the second and third parts of the axillary artery. The vascular surgeon knew there was time to repair the wound of the artery because of the rich collateral pathway provided by the anastomoses between which of the following of arteries? ⃣ A. Transverse cervical and suprascapular ⃣ B. Posterior humeral circumfl ex and profunda brachii ⃣ C. Suprascapular and circumfl ex scapular ⃣ D. Supreme (superior) thoracic and thoracoacromial ⃣ E. Lateral thoracic and suprascapular

C

108 A 55-year-old male fi refi ghter is admitted to the hospital after blunt trauma to his right axilla. Examination reveals winging of the scapula and partial paralysis of the right side of the diaphragm. Which of the following parts of the brachial plexus have been injured? ⃣ A. Cords ⃣ B. Divisions ⃣ C. Roots ⃣ D. Terminal branches ⃣ E. Trunks

C

119 A 22-year-old man is admitted to the hospital after a car collision. Radiographic examination reveals an oblique fracture of his humerus. Upon physical examination the patient is unable to extend his forearm. The damaged nerve was most likely composed of fi bers from which of the following spinal levels? ⃣ A. C5, C6 ⃣ B. C5, C6, C7 ⃣ C. C5, C6, C7, C8, T1 ⃣ D. C6, C7, C8, T1 ⃣ E. C7, C8, T1

C

12. A 27-year-old stuntman is thrown out of his vehicle prematurely when the car used for a particular scene speeds out of control. His spinal cord is crushed at the level of the fourth lumbar spinal segment. Which of the following structures would most likely be spared from destruction? (A) Dorsal horn (B) Ventral horn (C) Lateral horn (D) Gray matter (E) Pia mater

C

121 A 56-year-old woman visits the emergency department after falling on wet pavement. Radiographic examination reveals osteoporosis and a Colles' fracture. Which of the following carpal bones are often fractured or dislocated with a Colles' fracture? ⃣ A. Triquetrum and scaphoid ⃣ B. Triquetrum and lunate ⃣ C. Scaphoid and lunate ⃣ D. Triquetrum, lunate, and scaphoid ⃣ E. Triquetrum and pisiform

C

122 A 3-year-old girl is admitted to the emergency department with severe pain. History taking reveals that the girl was violently lifted by her raised arm by her mother to prevent the girl from walking in front of a moving car. Which of the following is most likely the cause of the pain? ⃣ A. Compression of median nerve ⃣ B. Separation of the head of radius from its articulation with trochlea of humerus ⃣ C. Separation of head of radius from its articulation with ulna and capitulum of humerus ⃣ D. Separation of ulna from its articulation with trochlea of humerus ⃣ E. Stretching of radial nerve as it passes behind medial epicondyle of humerus

C

125 A 45-year-old woman is admitted to the hospital with neck pain. An MRI examination reveals a herniated disk in the cervical region. Physical examination reveals weak triceps brachii muscles. Which of the following spinal nerves is most likely injured? ⃣ A. C5 ⃣ B. C6 ⃣ C. C7 ⃣ D. C8 ⃣ E. T1

C

134 A 62-year-old man is admitted to the emergency department after falling on wet pavement. Radiographic examination reveals a carpometacarpal fracture at the base of the thumb. What is the term applied to the described fracture? ⃣ A. Colles fracture ⃣ B. Scaphoid fracture ⃣ C. Bennett's fracture ⃣ D. Smith's fracture ⃣ E. Boxer's fracture

C

14. In a freak hunting accident, a 17-yearold boy was shot with an arrow that penetrated into his suboccipital triangle, injuring the suboccipital nerve between the vertebral artery and the posterior arch of the atlas. Which of the following muscles would be unaffected by such a lesion? (A) Rectus capitis posterior major (B) Semispinalis capitis (C) Splenius capitis (D) Obliquus capitis superior (E) Obliquus capitis inferior

C

16. A crush injury of the vertebral column can cause the spinal cord to swell. Which structure would be trapped between the dura and the vertebral body by the swelling spinal cord? (A) Anterior longitudinal ligament (B) Alar ligament (C) Posterior longitudinal ligament (D) Cruciform ligament (E) Ligamentum nuchae

C

21. A middle-aged coal miner injures his back after an accidental explosion. His magnetic resonance imaging (MRI) scan reveals that his spinal cord has shifted to the right because the lateral extensions of the pia mater were torn. Function of which of the following structures is most likely impaired? (A) Filum terminale internum (B) Coccygeal ligament (C) Denticulate ligament (D) Choroid plexus (E) Tectorial membrane

C

23. After an automobile accident, a back muscle that forms the boundaries of the triangle of auscultation and the lumbar triangle receives no blood. Which of the following muscles might be ischemic? (A) Levator scapulae (B) Rhomboid minor (C) Latissimus dorsi (D) Trapezius (E) Splenius capitis

C

24. A 38-year-old woman with a long history of shoulder pain is admitted to a hospital for surgery. Which of the following muscles becomes ischemic soon after ligation of the superficial or ascending branch of the transverse cervical artery? (A) Latissimus dorsi (B) Multifidus (C) Trapezius (D) Rhomboid major (E) Longissimus capitis

C

27. Dorsal and ventral roots of the lower lumbar and sacral nerves are lacerated. Which structure is most likely damaged?

C

30 A 23-year-old female maid was making a bed in a hotel bedroom. As she straightened the sheet by running her right hand over the surface with her fi ngers extended, she caught the end of the index fi nger in a fold. She experienced a sudden, severe pain over the base of the terminal phalanx. Several hours later when the pain had diminished, she noted that the end of her right index fi nger was swollen and she could not completely extend the terminal interphalangeal joint. Which one of the following structures within the digit was most likely injured? ⃣ A. The proper palmar digital branch of the median nerve B. The vinculum longa ⃣ C. The insertion of the tendon of the extensor digitorum onto the base of the distal phalanx ⃣ D. The insertion of the fl exor digitorum profundus tendon ⃣ E. The insertion of the fl exor digitorum superfi cialis tendon

C

36 The 58-year-old convenience store operator had received a superfi cial bullet wound to the soft tissues on the medial side of the elbow in an attempted robbery. A major nerve was repaired at the site where it passed behind the medial epicondyle. Bleeding was stopped from an artery that accompanied the nerve in its path toward the epicondyle. Vascular repair was performed on this small artery because of its important role in supplying blood to the nerve. Which of the following arteries was most likely repaired? ⃣ A. The profunda brachii ⃣ B. The radial collateral artery ⃣ C. The superior ulnar collateral artery ⃣ D. The inferior ulnar collateral artery ⃣ E. The anterior ulnar recurrent artery

C

37 A 60-year-old male butcher accidentally slashed his wrist with his butcher knife, partially dividing the ulnar nerve. Which of the following actions would most likely be lost as a result of this injury? ⃣ A. Flexion of the proximal interphalangeal joint of the fi fth digit (little fi nger) ⃣ B. Extension of the thumb ⃣ C. Adduction of the fi fth digit ⃣ D. Abduction of the thumb ⃣ E. Opposition of the thumb

C

38 A 23-year-old male medical student fell asleep in his chair with Netter's Atlas wedged into his axilla. When he awoke in the morning, he was unable to extend the forearm, wrist, or fi ngers. Movements of the ipsilateral shoulder joint appear to be normal. Which of the following nerves was most likely compressed, producing the symptoms described? ⃣ A. Lateral cord of the brachial plexus ⃣ B. Medial cord of the brachial plexus ⃣ C. Radial nerve ⃣ D. Median nerve ⃣ E. Lateral and medial pectoral nerves

C

4. A 39-year-old woman with headaches presents to her primary care physician with a possible herniated disk. Her magnetic resonance imaging (MRI) scan reveals that the posterolateral protrusion of the intervertebral disk between L4 and L5 vertebrae would most likely affect nerve roots of which of the following spinal nerves? (A) Third lumbar nerve (B) Fourth lumbar nerve (C) Fifth lumbar nerve (D) First sacral nerve (E) Second sacral nerve

C

41 A 35-year-old male wrestler is admitted to the emergency department with excruciating pain in his right shoulder and proximal arm. During physical examination the patient clutches the arm at the elbow with his opposite hand and is unable to move the injured limb. Radiographic studies show that the patient has a dislocation of the humerus at the glenohumeral joint. Which of the following conditions is the most likely? ⃣ A. The head of the humerus is displaced anteriorly. ⃣ B. The head of the humerus is displaced posteriorly. ⃣ C. The head of the humerus is displaced inferiorly. ⃣ D. The head of the humerus is displaced superiorly. ⃣ E. The head of the humerus is displaced medially.

C

44 A 44-year-old woman is diagnosed with radial nerve palsy. When muscle function is examined at the metacarpophalangeal (MCP), proximal interphalangeal (PIP), and distal interphalangeal (DIP) joints, what fi ndings are most likely to be present? ⃣ A. Inability to abduct the digits at the MCP joint ⃣ B. Inability to adduct the digits at the MCP joint ⃣ C. Inability to extend the MCP joints only ⃣ D. Inability to extend the MCP, PIP, and DIP joints ⃣ E. Inability to extend the PIP and DIP joints

C

5. A 57-year-old woman comes into her physician's office complaining of fever, nausea, vomiting, and the worst headache of her life. Tests and physical examination suggest hydrocephalus (widening ventricles) resulting from a decrease in the absorption of cerebrospinal fluid (CSF). A decrease of flow in the CSF through which of the following structures would be responsible for these findings? (A) Choroid plexus (B) Vertebral venous plexus (C) Arachnoid villi (D) Internal jugular vein (E) Subarachnoid trabeculae

C

51 The shoulder of a 44-year-old deer hunter had been penetrated by a bolt released from a crossbow. The bolt had transected the axillary artery just beyond the origin of the subscapular artery. A compress is placed on the wound with deep pressure. After a clamp is placed on the bleeding artery, thought is given to the anatomy of the vessel. What collateral arterial pathways are available to bypass the site of injury? ⃣ A. Suprascapular with circumfl ex scapular artery ⃣ B. Dorsal scapular with thoracodorsal artery ⃣ C. Posterior humeral circumfl ex artery with deep brachial artery ⃣ D. Lateral thoracic with brachial artery ⃣ E. Supreme thoracic artery with thoracoacromial artery

C

54 Endoscopic examination of the shoulder of a 62-year-old female clearly demonstrated erosion of the tendon within the glenohumeral joint. What tendon was this? ⃣ A. Glenohumeral ⃣ B. Long head of triceps ⃣ C. Long head of biceps ⃣ D. Infraspinatus ⃣ E. Coracobrachialis

C

57 A 45-year-old woman is admitted to the hospital with neck pain. An MRI examination reveals a herniated disk in the cervical region. Physical examination reveals weakness in wrist extension and paraesthesia on the back of her arm and forearm. Which of the following spinal nerves is most likely injured? A. C5 B. C6 C. C7 D. C8 E. T1

C

65 A 48-year-old female piano player visited the outpatient clinic with numbness and tingling in her left hand. A diagnosis was made of nerve compression in the carpal tunnel, and the patient underwent an endoscopic nerve release. Two weeks postoperatively the patient complained of a profound weakness in the thumb, with loss of thumb opposition. The sensation to the hand, however, was unaffected. Which of the following nerves was injured during the operation? A. The first common digital branch of the median nerve B. The second common digital branch of the median nerve C. Recurrent branch of median nerve D. Deep branch of the ulnar nerve E. Anterior interosseus nerve

C

68 A 31-year-old male hockey player fell on his elbow and is admitted to the emergency department. Radiographic examination reveals a fracture of the surgical neck of the humerus, producing an elevation and adduction of the distal fragment. Which of the following muscles would most likely cause the adduction of the distal fragment? A. Brachialis B. Teres minor C. Pectoralis major D. Supraspinatus E. Pectoralis minor

C

75 A 43-year-old female tennis player visits the outpatient clinic with pain over the right lateral epicondyle of her elbow. Physical examination reveals that the patient has lateral epicondylitis. Which of the following tests should be performed during physical examination to confirm the diagnosis? ⃣ A. Nerve conduction studies ⃣ B. Evaluation of pain experienced during fl exion and extension of the elbow joint ⃣ C. Observing the presence of pain when the wrist is extended against resistance ⃣ D. Observing the presence of numbness and tingling in the ring and little fi ngers when the wrist is fl exed against resistance ⃣ E. Evaluation of pain felt over the styloid process of radius during brachioradialis contraction

C

76 A male skier had a painful fall against a rocky ledge. Radiographic fi ndings revealed a hairline fracture of the surgical neck of the humerus. The thirdyear medical student assigned to this patient was asked to determine whether there was injury to the nerve associated with the area of injury. Which of the following tests would be best for checking the status of the nerve? ⃣ A. Have the patient abduct the limb while holding a 10-lb weight. ⃣ B. Have the patient shrug the shoulders. ⃣ C. Test for presence of skin sensation over the lateral side of the shoulder. ⃣ D. Test for normal sensation over the medial skin of the axilla. ⃣ E. Have the patient push against an immovable object like a wall and assess the position of the scapula.

C

78 A 15-year-old male received a shotgun wound to the ventral surface of the upper limb. Upon examination it is quickly observed that the patient exhibits a complete clawhand but can extend his wrist. What is the nature of this patient's injury? ⃣ A. The ulnar nerve has been severed at the wrist. ⃣ B. The median nerve has been injured in the carpal tunnel. ⃣ C. The median and ulnar nerves are damaged at the wrist. ⃣ D. The median and ulnar nerves have been injured at the elbow region. ⃣ E. The median, ulnar, and radial nerves have been injured at midhumerus.

C

8. A young toddler presents to her pediatrician with rather new onset of bowel and bladder dysfunction and loss of the lower limb function. Her mother had not taken enough folic acid (to the point of a defi- ciency) during her pregnancy. On examination, the child has protrusion of the spinal cord and meninges and is diagnosed with which of the following conditions? (A) Spina bifida occulta (B) Meningocele (C) Meningomyelocele (D) Myeloschisis (E) Syringomyelocele

C

83 The mastectomy procedure on a 52-year-old female involved excision of the tumor and a removal of lymph nodes, including the pectoral, central axillary, and infraclavicular groups. Six months after her mastectomy, the patient complains to her personal physician of an unsightly deep hollow area inferior to the medial half of the clavicle, indicating a significant area of muscle atrophy and loss. She states that the disfigurement has taken place quite gradually since her mastectomy. Physical examination reveals no obvious motor or sensory deficits. What was the most likely cause of the patient's cosmetic problem? A. Part of the pectoralis major muscle was cut and removed in the mastectomy. B. The pectoralis minor muscle was removed entirely in the surgery. C. A branch of the lateral pectoral nerve was cut. D. The medial pectoral nerve was cut. E. The lateral cord of the brachial plexus was injured.

C

85 A 43-year-old man is admitted to the hospital, having suffered a whiplash injury when his compact auto was struck from behind by a sports utility vehicle. MRI examination reveals some herniation of a disk in the cervical region. Physical examination reveals that the patient has lost elbow extension; there is absence of his triceps refl ex and loss of extension of the metacarpophalangeal joints on the ipsilateral side. Which of the following spinal nerves is most likely affected? ⃣ A. C5 ⃣ B. C6 ⃣ C. C7 D. C8 ⃣ E. T1

C

87 A 45-year-old woman motorcyclist, propelled over the handlebars of her bike by an encounter with a rut in the road, lands on the point of one shoulder. The woman is taken by ambulance to the emergency department. During physical examination the arm appears swollen, pale, and cool. Any movement of the arm causes severe pain. Radiographic examination reveals a fracture and a large hematoma, leading to diagnosis of Volkmann's ischemic contracture. At which of the following locations has the fracture most likely occurred? ⃣ A. Surgical neck of humerus ⃣ B. Radial groove of humerus ⃣ C. Supracondylar line of humerus ⃣ D. Olecranon ⃣ E. Lateral epicondyle

C

93 Several weeks after surgical dissection of her left axilla for the removal of lymph nodes for staging and treatment of her breast cancer, a 32-year-old woman was told by her general physician that she had "winging" of her left scapula when she pushed against resistance during her physical examination. She told the physician that she had also experienced diffi culty lately in raising her right arm above her head when she was combing her hair. In a subsequent consult visit with her surgeon, she was told that a nerve was accidentally injured during the diagnostic surgical procedure and that this produced her scapular abnormality and inability to raise her arm normally. What was the origin of this nerve? ⃣ A. The upper trunk of her brachial plexus ⃣ B. The posterior division of the middle trunk ⃣ C. Roots of the brachial plexus ⃣ D. The posterior cord of the brachial plexus ⃣ E. The lateral cord of the brachial plexus

C

97 After the orthopedic surgeon examined the MRI of the shoulder of a 42-year-old female he informed her that the supraspinatus muscle was injured and needed to be repaired surgically. Which of the following is true of the supraspinatus muscle? ⃣ A. It inserts on the lesser tubercle of the humerus. ⃣ B. It initiates adduction of the shoulder. ⃣ C. It is innervated chiefl y by the C5 spinal nerve. ⃣ D. It is supplied by the upper subscapular nerve. ⃣ E. It originates from the lateral border of the scapula.

C

A 13-year-old boy is brought to the emergency department after losing control during a motorbike race in which he was run over by several of the other racers. Physical examination reveals several cuts and bruises. He is unable to extend the left wrist, fi ngers, and thumb, although he can extend the elbow. Sensation is lost in the lateral half of the dorsum of the left hand. Which of the following nerves has most likely been injured to result in these signs, and in what part of the arm is the injury located? ⃣ A. Median nerve, anterior wrist ⃣ B. Median nerve, arm ⃣ C. Radial nerve, midhumerus ⃣ D. Ulnar nerve, midlateral forearm ⃣ E. Ulnar nerve, midpalmar region

C

A 22-year-old woman visits the outpatient clinic with pain in her left upper limb. She has a long history of pain in this limb and diffi culty with fi ne motor tasks of the hand. Physical examination reveals paraesthesia along the medial surface of the forearm and palm and weakness and atrophy of gripping muscles (long fl exors) and the intrinsic muscles of the hand. The radial pulse is diminished when her neck is rotated to the ipsilateral side (positive Adson test). What is the most likely diagnosis? ⃣ A. Erb-Duchenne paralysis ⃣ B. Aneurysm of the brachiocephalic artery, with plexus compression ⃣ C. Thoracic outlet syndrome ⃣ D. Carpal tunnel syndrome ⃣ E. Injury to the medial cord of the brachial plexus

C

A 24-year-old medical student was bitten at the base of her thumb by her dog. The wound became infected and the infection spread into the radial bursa. The tendon(s) of which muscle will most likely be affected? ⃣ A. Flexor digitorum profundus ⃣ B. Flexor digitorum superfi cialis ⃣ C. Flexor pollicis longus ⃣ D. Flexor carpi radialis ⃣ E. Flexor pollicis brevis

C

A 25-year-old woman is admitted to the emergency department after a car collision. Radiographic examination reveals a fracture at the spiral groove of the humerus. A cast is placed, and 3 days later the patient complains of severe pain over the length of her arm. During physical examination the arm appears swollen, pale, and cool. Radial pulse is absent, and any movement of the arm causes severe pain. Which of the following conditions will most likely characterize the fi ndings of the physical examination? A. Venous thrombosis B. Thoracic outlet syndrome C. Compartment syndrome D. Raynaud's disease E. Injury of the radial nerve

C

A 27-year-old male painter is admitted to the hospital after falling from a ladder. Physical examination reveals that the patient is unable to abduct his arm more than 15 ° and cannot rotate the arm laterally. A radiographic examination reveals an oblique fracture of the humerus. He has associated sensory loss over the shoulder area. Which of the following injuries will most likely correspond to the symptoms of the physical examination? ⃣ A. Fracture of the medial epicondyle ⃣ B. Fracture of the glenoid fossa ⃣ C. Fracture of the surgical neck of the humerus ⃣ D. Fracture of the anatomic neck of the humerus ⃣ E. Fracture of the middle third of the humerus

C

A 35-year-old patient has a small but painful tumor under the nail of the little finger. Which of the following nerves would have to be anesthetized for a painless removal of the tumor? ⃣ A. Superficial radial ⃣ B. Common palmar digital of median ⃣ C. Common palmar digital of ulnar ⃣ D. Deep radial ⃣ E. Recurrent branch of median

C

A 45-year-old woman is admitted to the hospital with neck pain. A CT scan reveals a tumor in the left side of her oral cavity. The tumor and related tissues are removed with a radical neck surgical procedure. Two months postoperatively the patient's left shoulder droops quite noticeably. Physical examination reveals distinct weakness in turning her head to the right and impairment of abduction of her left upper limb to the level of the shoulder. Which of the following structures was most likely injured during the radical neck surgery? ⃣ A. Suprascapular nerve ⃣ B. Long thoracic nerve ⃣ C. Spinal accessory nerve ⃣ D. The junction of spinal nerves C5 and C6 of the brachial plexus ⃣ E. Radial nerve

C

A 49-year-old female who had suffered a myocardial infarction must undergo a bypass graft procedure using the internal thoracic artery. Which vessels will most likely continue to supply blood to the anterior part of the upper intercostal spaces? ⃣ A. Musculophrenic ⃣ B. Superior epigastric ⃣ C. Posterior intercostal ⃣ D. Lateral thoracic E. Thoracodorsal

C

A 52-year-old band director suffered problems in her right arm several days after strenuous fi eld exercises for a major athletic tournament. Examination in the orthopedic clinic reveals wrist drop and weakness of grasp but normal extension of the elbow joint. There is no loss of sensation in the affected limb. Which nerve was most likely affected? ⃣ A. Ulnar ⃣ B. Anterior interosseous ⃣ C. Posterior interosseous ⃣ D. Median ⃣ E. Superfi cial radial

C

A 61-year-old man was hit by a cricket bat in the midhumeral region of his left arm. Physical examination reveals normal elbow motion; however, he could not extend his wrist or his metacarpophalangeal joints and he reported a loss of sensation on a small area of skin on the dorsum of the hand proximal to the first two digits. Radiographic examination reveals a hairline fracture of the shaft of the humerus just distal to its midpoint. Which of the following nerves is most likely injured? A. Median B. Ulnar C. Radial D. Musculocutaneous E. Axillary

C

While walking to his classroom building, a fi rstyear medical student slipped on the wet pavement and fell against the curb, injuring his right arm. Radiographic images showed a midshaft fracture of the humerus. Which pair of structures was most likely injured at the fracture site? A. Median nerve and brachial artery B. Axillary nerve and posterior humeral circumfl ex artery C. Radial nerve and deep brachial artery D. Suprascapular nerve and artery E. Long thoracic nerve and lateral thoracic artery

C

A 48-year-old female court stenographer is admitted to the orthopedic clinic with symptoms of carpal tunnel syndrome, with which she has suffered for almost a year. Which muscles most typically become weakened in this condition? A. Dorsal interossei B. Lumbricals III and IV C. Thenar D. Palmar interossei E. Hypothenar

C, and lumbricals I and II

During cardiac catheterization of a 6-year-old child, the radiologist notes that the contrast medium released into the arch of the aorta is visible immediately in the left pulmonary artery. What is the most likely explanation for this finding? A. Atrial septal defect B. Mitral stenosis C. Patent ductus arteriosus D. Patent ductus venosus E. Ventricular septal defect

C. The ductus arteriosus is an embryologic structure that acts as a communication between the pulmonary trunk and the aorta. If it remains patent, the injected contrast medium would flow from the aorta through this communication and into the pulmonary artery. An atrial septal defect is a communication between the atria. Mitral stenosis is a narrowing of the AV valve between the left atrium and left ventricle. The ductus venosus transports blood from the left umbilical vein to the inferior vena cava, bypassing the liver. Ventricular septal defect is a communication between the ventricles.

A 3-month-old infant is diagnosed with Down syndrome. A routine cardiovascular examination reveals that the infant suffers from arrhythmias. What other cardiac conditions are most likely to occur with Down syndrome? A. Tetralogy of Fallot B. Transposition of the great arteries C. Atrial septal and ventricular septal defects D. Truncus arteriosus E. Coarctation of the aorta

C. Down syndrome (more properly called "trisomy 21") is associated with cardiovascular abnormalities such as arrhythmias and atrial and ventricular septal defects. It is also characterized by mental retardation, brachycephaly, flat nasal bridge, upward slant of the palpebral fissure, protruding tongue, simian crease, and clinodactyly of the fifth digit.

After a 2-day-old newborn male swallows milk he becomes cyanotic. After 3 days he develops pneumonia. A tracheoesophageal fistula is suspected. Which of the following conditions is most likely to be associated with a tracheoesophageal fistula? A. Oligohydramnios B. Rubella C. Polyhydramnios D. Thalidomide E. Toxoplasmosis

C. Polyhydramnios is an excess of amniotic fluid, often associated with esophageal atresia or a tracheoesophageal fistula. This abnormality affects fetal ability to swallow the normal amount of amniotic fluid; therefore, excess fluid remains in the amniotic sac. None of the other factors listed has an association with this type of fistula.

A 35-year-old male is admit ed to the emergency department because of a severe nosebleed and a headache that had become worse during the weekend. On physical examination his upper body appears much bet er developed than his lower body, a loud midsystolic murmur is present on his anterior chest wall and back, his lower extremities are cold, and femoral pulses are absent. Which of the fol owing embryologic structure(s) has been most likely af ected to produce such symptoms? A. Bulbus cordis B. Ductus arteriosus C. Third, fourth, and sixth pharyngeal arches D. Right and left horns of sinus venosus E. Right cardinal vein

C. The anomalies present in this individual are all caused by a coarctation of the aorta. The portion of the aortic arch that is constricted arises from the third, fourth, and sixth pharyngeal arches. The bulbus cordis becomes part of the ventricular system. The ductus arteriosus becomes the ligamentum arteriosum.

A 24-year-old woman received a small-caliber bullet wound to the popliteal fossa from a drive-by assailant. The patient was admitted to the emergency department, where the surgeons recognized that the bullet had severed the tibial nerve. Such an injury would most likely result in which of the following? A. Inability to extend the leg at the knee B. Foot drop C. A dorsiflexed and everted foot D. A plantar flexed and inverted foot E. Total inability to flex the leg at the knee joint

C. A severe injury of the tibial nerve in the popliteal fossa would result in a dorsiflexed and everted foot because of the intact muscles of the extensor (anterior) and evertor (lateral) compartments of the leg. It would result also in some weakening of knee flexion because of loss of the gastrocnemius muscle, which flexes the knee and plantar flexes the foot. The hamstrings also flex the knee, so this function would not be lost. Plantar flexion at the ankle would be paralyzed with the loss of the gastrocnemius and soleus, in addition to the flexors of the toes, and inversion by the tibialis posterior. Foot drop results from loss of the anterior compartment, innervated by the deep fibular (peroneal) nerve.

A 42 year-old woman is admitted to the emergency department with a complaint of dull, poorly localized pain in the deep pelvis. An MRI examination reveals a prolapsing of abdominal viscera, probably due to a tear of the rectovaginal septum. Which of the following conditions will most likely result from a defect in the rectovaginal septum? A. Cystocele B. Urethrocele C. Enterocele D. Urinary incontinence E. Prolapsed uterus

C. An enterocele (herniation of small intestine into the posterior wall of the vagina) is caused by a tear of the rectovaginal septum, which weakens the pelvic floor. A urethrocele is characterized by prolapse of the urethra into the vagina. It is usually associated with a cystocele (prolapse of the bladder into the urethra). Cystocele or urethrocele are associated with defects in the pubocervical fascia that covers the anterior wall of the vagina and assists in supporting the bladder. Uri- nary incontinence can result from weakening of the muscles that surround the urethra but would not be caused by a tear of the rectovaginal septum. Prolapse of the uterus is caused by weakening or tearing of the ligaments that support the uterus (especially the cardi- nal and/or uterosacral ligaments).

A 75-year-old man is transported to the emergency department with severe pain of his right hip and thigh. A radiologic examination reveals avascular necrosis of the femoral head (Fig. 5-2). Which of the following conditions most likely occurred to produce avascular necrosis in this patient? A. Dislocation of the hip with tearing of the ligament of the head of the femur B. Intertrochanteric fracture of the femur C. Intracapsular femoral neck fracture D. Thrombosis of the obturator artery E. Comminuted fracture of the extracapsular femoral neck

C. An intracapsular femoral neck fracture causes avascular necrosis of the femoral head because the fracture damages the radicular branches of the medial and lateral circumflex arteries that pass beneath the ischiofemoral ligament and pierce the femoral neck. Until an individual reaches about 6 to 10 years of age, blood supply to the head of the femur is provided by a branch of the obturator artery that runs with the ligament of the head of the femur. Thereafter, the artery of the ligament of the head of the femur is insignificant. Intertrochanteric fracture of the femur would not damage the blood supply to the head of the femur but would cause complications because the greater trochanter is an attachment site for several gluteal muscles. During childhood the obturator artery provides the artery of the ligament of the head of the femur. Thrombosis of the obturator artery could result in muscular symptoms, although there are several collateral sources of blood supply in the thigh. Comminuted fracture of the extracapsular femoral neck would not ordinarily imperil the vascular supply.

Stenosis of which structure may produce left ventricular hypertrophy

C. Ascending aorta

The left coronary artery arises from which structure?

C. Ascending aorta

When does a secondary oocyte complete its second meiotic division to become a mature ovum? (A) At ovulation (B) Before ovulation (C) At fertilization (D) At puberty (E) Before birth

C. At ovulation, a secondary oocyte begins meiosis II, but this division is arrested at metaphase. The secondary oocyte remains arrested in metaphase until a sperm penetrates it at fertilization. Therefore, the term "mature ovum" is somewhat of a misnomer, because it is a secondary oocyte that is fertilized and, once fertilized, the new diploid cell is known as a zygote. If fertilization does not occur, the secondary oocyte degenerates.

A 43-year-old woman is examined by a neurologist, to whom she complains of pain in her lower limb of 6 months' duration. She has pain in the gluteal area, thigh, and leg. The neurologist observes reduced sensation over the dorsum and lateral side of the involved foot and some weakness in foot dorsiflexion and eversion. A diagnosis of a piriformis entrapment syndrome is made, with compression of the fibular (peroneal) division of the sciatic nerve. Which of the following conditions did the neurologist also most likely find during her physical examination of the patient? A. Paralysis of plantar flexion B. Instability of the knee, due to paralysis of the quadriceps femoris C. Foot drop D. Spasm or clonic contractures of the adductor musculature of the thigh E. Loss of sensation in the gluteal area, by paralysis of anterior cluneal nerves

C. Entrapment compression of all or part of the sciatic nerve by the piriformis can mimic disc herniation, most commonly resembling compression of spinal nerve S1. This results in pain down the posterior aspect of the thigh and leg and the lateral side of the foot. In this case, loss of sensation over the dorsum of the foot and weakness of foot extension, in addition to eversion, indicate that more than S1 is involved. Foot drop would be anticipated with fibular (peroneal) nerve involvement. As noted also in a previous question, compression of the common fibular (peroneal) division of the sciatic nerve by the piriformis gives rise to the clinical condition known as piriformis entrapment. This condition is associated with point pain in the gluteal area, pain in the posterior part of the limb, and possible weakness of muscles in the lateral and anterior compartments of the leg. It can be confused with herniated disc (L5) compression of S1 and sciatica. Paralysis of plantar flexion occurs with a lesion of the tibial division of the sciatic nerve or the tibial nerve. Paralysis of the quadriceps is associated with pathology of the femoral nerve. Clonic contraction of the adductors could result from obturator nerve problems

A 32-year-old man was brought to the emergency with complaints of pain to the left ankle and leg. During a game of football, his left foot landed in a hole as he was running on an uneven dirt field. He was unable subsequently bear weight on his left limb. On examination, the right ankle was swollen, with exquisite tenderness over the right medial malleolus and over the distal third of the lateral right leg. Radiographs of the right lower limb revealed an inferiorly displaced fracture of the right medial malleolus and a spiral fracture of the distal third of the right fibula. Which of the following describes the most likely mechanism of this injury? A. Forceful inversion of the ankle B. Direct upward force from the talus into the tibial plafond C. Forceful external rotation and eversion of the ankle D. Forceful dorsiflexion of the foot E. Extreme plantar flexion of the foot

C. Forceful external rotation and eversion of the ankle often leads to this type of injury as the bony components are pushed apart forcefully. It is commonly referred to as a Pott fracture (the medial malleolus is pulled forcefully by the strong deltoid ligament as the talus moves laterally, causing a fracture of the lateral malleolus). Inversion, extreme plantar flexion, and forceful dorsiflexion cause a rupture of the ligaments. Direct upward force of the talus is usually due to a fall from great height and will damage the spine and calcaneus.

A 13-year-old female is brought to the emergency department with a complaint of severe, deep pelvic discomfort. Physical examination reveals that the patient has an intact hymen. Incision of the hymen reveals hematocolpos. Which of the following conditions is associated with hematocolpos? A. Cyst of Bartholin gland B. Bleeding from an ectopic pregnancy C. Imperforate hymen D. Indirect inguinal hernia with cremasteric arterial bleeding E. Iatrogenic bleeding from the uterine veins

C. Hematocolpos is characterized by filling of the vagina with menstrual blood. This commonly occurs due to the presence of an imperforate hymen. Bartholin gland ducts open into the vestibule of the vagina; therefore, a cyst in Bartholin gland would not cause hematocolpos. Blood from a ruptured ectopic preg- nancy most often drains into the rectouterine pouch (of Douglas). Females often have a diminutive cremaster muscle and cremasteric artery and vein, but none of these is associated with hematocolpos. The cremasteric artery provides a small branch to the round ligament of the uterus (sometimes called "Samson's artery"), which must be kept in mind during a hysterectomy, with division of the round ligament. Bleeding from the uterine veins would not flow into the vagina.

When he attempted to lift one side of his new electric automobile from the ground to demonstrate his strength, a 51-year-old man felt a sharp pain in his back and quickly dropped the vehicle. Upon examination, it is observed that the patient has deficits in sensation on the dorsum and sole of his foot and marked weakness in abduction and lateral rotation of the lower limb. What was the nature of his injury? A. Piriformis syndrome, with entrapment of the sciatic nerve B. Disc lesion at L3-4 C. Disc lesion at L4-5 D. Disc lesion at L5-S1 E. Posterior hip dislocation

C. Herniation of the intervertebral disc at L4-5 results typically in compression of the L5 spinal nerve. The L4 spinal nerve exits at the L4-5 intervertebral foramen, but the L5 spinal nerve is put under tension as it passes the herniation to reach the L5-1 foramen. Piriformis entrapment of the fibular (peroneal) division of the sciatic nerve is relatively common, but the dermatome affected here appears to be confined to the L5 distribution to the skin of the foot and also includes the superior gluteal nerve, which supplies the large hip abductors. S1 would involve loss of sensation on the lateral side of the foot and potential weakness in hip extension and plantar flexion. A posterior dislocation of the hip would be unlikely in this injury but, even so, would not result in these deficits.

A 16-year-old girl presents on May 10 in obvious emotional distress. On questioning, she relates that on May 1 she experienced sexual intercourse for the first time, without any means of birth control. Most of her anxiety stems from her fear of pregnancy. What should the physician do to alleviate her fear? (A) Prescribe diazepam and wait to see if she misses her next menstrual period (B) Use ultrasonography to document pregnancy (C) Order a laboratory assay for serum hCG (D) Order a laboratory assay for serum progesterone (E) Prescribe diethylstilbestrol ("morningafter pill")

C. Human chorionic gonadotropin (hCG) can be assayed in maternal serum at day 8 of development and in urine at day 10. If this teenager is pregnant, the blastocyst would be in week 2 of development (day 10). Laboratory assay of hCG in either the serum or urine can be completed; however, serum hCG might be more reliable. It is important to note that if she is pregnant, she will not miss a menstrual period until May 15 at which time the embryo will be entering week 3 of development.

A 1 year-old is admitted to the pediatric clinic because he is passing urine on the underside of the penis. Which embryologic structure failed to fuse? A. Spongy urethra B. Labioscrotal folds C. Urethral folds D. Urogenital folds E. Genital tubercle

C. Hypospadias is a developmental defect in the urethra resulting in urine being expelled from the ventral side of the penis. This ectopic malformation may present when the urethral folds fail to completely fuse. Failure of fusion of the spongy urethra would result in epispadias. A failure of the labioscrotal folds to fuse will cause the external urethral orifice to be situated between the two scrotal halves. This is re- ferred to as penile hypospadias. Failure of the uro- genital folds to fuse would lead to agenesis of the external urethral folds.

A 32-year-old man is admitted to the emergency department after a car collision. Radiologic examination reveals a distal fracture of the femur. The patient is in severe pain, and a femoral nerve block is administered. What landmark is accurate for localizing the nerve for injection of anesthetics? A. 1.5 cm superolateral to the pubic tubercle B. 1.5 cm medial to the anterior superior iliac spine C. 1.5 cm lateral to the femoral pulse D. 1.5 cm medial to the femoral pulse E. Midway between the anterior superior iliac spine and pubic symphysis

C. If the needle is inserted about 1.5 cm lateral to the maximal femoral pulse, it will intersect the femoral nerve in most cases. (Fluoroscopic or ultrasound guidance is advisable to avoid iatrogenic errors.) The deep inguinal ring is located about 4 cm superolateral to the pubic tubercle and very close to the origin of the inferior epigastric vessels from the external iliac artery and vein. The approximate site of exit of the lateral femoral cutaneous nerve from the abdomen is 1.5 cm medial to the anterior superior iliac spine. Injections 1.5 cm medial to the femoral artery pulse will enter the femoral vein. Midway between the anterior superior iliac spine and the pubic symphysis can vary approximately 1.5 cm either medial or lateral from the femoral artery.

A 27-year-old female tennis pro injured her ankle during the quarterfinal match. A physical examination at the outpatient clinic revealed a severe inversion sprain of the ankle. Which of the following structures is most commonly damaged in such injuries? A. Medial plantar nerve B. Tibial nerve C. Anterior talofibular ligament D. Posterior talofibular ligament E. Deltoid ligament

C. In an inversion injury the most common ligament involvement comes from the anterior talofibular and calcaneofibular ligaments. The medial plantar nerve is medially located within the sole of the foot and might be injured by traction in an eversion injury, not an inversion injury. The posterior talofibular ligament is located posteriorly and is not usually injured in an inversion injury. The deltoid ligament is located medially and would be injured with an eversion injury; it is so strong, however, that eversion is more likely to fracture the medial malleolus rather than tear the deltoid ligament.

A 1-year-old male infant is admitted to the pediatric clinic because his parents could not palpate his testicles in the scrotum. The pediatrician examines the infant and palpates the testes in the inguinal canal. Which of the following best describes this condition? A. Pseudohermaphroditism B. True hermaphroditism C. Cryptorchidism D. Congenital adrenal hyperplasia E. Chordee

C. In cryptorchidism, the testis has failed to descend into its proper location in the scrotum and may be found within the abdomen.

A 58-year-old man is admitted to the hospital with pain in his lower limb for the past 2 months. Physical examination reveals point tenderness in the region of his greater sciatic foramen, with pain radiating down the posterior aspect of his thigh. An MRI examination reveals that the patient suffers from piriformis entrapment syndrome. He is directed to treatment by a physical therapist for stretching and relaxation of the muscle. Entrapment of which of the following nerves can mimic piriformis entrapment syndrome? A. L4 B. L5 C. S1 D. S2 E. S3

C. In piriformis entrapment, the sciatic nerve can be compressed when the piriformis is contracted, leading to painful sensations in the lower limb. These usually involve pain in the gluteal area, posterior thigh, and leg, most frequently resembling a disc lesion at L5-S1, with compression of the S1 spinal nerve. L4 compression would be rather unusual but would involve the quadriceps femoris knee extension, foot inversion, and sensory loss on the medial side of the leg. L5 compression would be indicated by weakness in hip abduction, knee flexion, and sensory loss on dorsal and plantar surfaces of the foot. S1 compression would weaken plantar flexion and foot eversion. Pudendal nerve entrapment would affect the perineal region. The fibular (peroneal) division of the sciatic nerve passes through the piriformis in some individuals, leading to L5, S1-S3 nerve compression.

A 22-year-old woman is found in a comatose condition, having lain for an unknown length of time on the tile floor of the courtyard. She is found in possession of cocaine. The patient is transported to the hospital while EMT personnel receive instructions for treatment of drug overdose. During the physical examination the patient's gluteal region shows signs of ischemia. After regaining consciousness, she exhibits paralysis of knee flexion and dorsal and plantar flexion and sensory loss in the limb. What is the most likely diagnosis? A. Tibial nerve loss B. S1-2 nerve compression C. Gluteal crush injury D. Piriformis entrapment syndrome E. Femoral nerve entrapment

C. Incapacitation and unconsciousness from use of cocaine and other powerful narcotics have led to numerous cases of the "gluteal crush syndrome." Compression of the gluteal region while supine for extended periods of time can lead to gluteal crush injury, in which the nerves and vessels of the gluteal area are compressed. This can result in loss of gluteal muscles and other soft tissues and sciatic nerve compression. The nerve compression can cause paralysis of knee flexors and muscles of the anterior and lateral compartments of the leg, with sensory loss in the posterior thigh and leg and sensory loss in the foot. Tibial nerve loss would not result in loss of dorsiflexion of the foot nor generalized sensory loss. Neither piriformis entrapment nor femoral nerve entrapment is associated with loss of gluteal musculature, nor loss of knee flexion or plantar flexion of the foot, nor do they lead to general sensory loss in the limb.

Intermediate mesoderm gives rise to the (A) neural tube (B) heart (C) kidneys and gonads (D) somites (E) notochord

C. Intermediate mesoderm is a subdivision of intraembryonic mesoderm that forms a longitudinal dorsal ridge called the urogenital ridge from which the kidneys and gonads develop.

A 23-year-old woman delivered a live male infant at 37 weeks' gestation after an uneventful pregnancy. Examination of the infant revealed the right second and third toes were fused. Radiographs of the right foot indicated 14 phalanges in their correct position. After review by the pediatric orthopedic surgeon, corrective surgery for the deformity was scheduled. Which of the following embryologic conditions explains the infant's condition? A. The digital ray for the third toe did not develop B. Excessive neural crest cell migration into the foot C. Incomplete apoptosis of tissue between digital rays D. Lack of signal from the zone of polarizing activity (ZPA) E. Faulty development of chondrification centers

C. Mesenchyme between digital rays undergoes apoptosis for the digits to form. Failure or incomplete apoptosis usually results in fused digits (syndactyly). This may involve the skin and soft tissues alone or may include the bone. Digital rays form from the hand plate. Failure of development of any digital ray results in underdevelopment of a finger or toe. Neural crest cells do not contribute to the formation of the foot. The zone of polarizing activity modulates the patterning of the limb in the anteroposterior diameter. The abnormality described did not involve the phalanges, as shown by radiograph, and thus could not have been caused by faulty chondrification.

On digital examination of the vagina, the portion of the uterus that one anticipates palpating with the examining finger is the cervix and its external os. Which of the following is the most common position of the uterus? A. Anteflexed and retroverted B. Retroflexed and anteverted C. Anteflexed and anteverted D. Retroflexed and retroverted

C. Normally the uterus is anteflexed at the junction of the cervix and the body and anteverted at the junction of the vagina and the cervical canal.

A young woman enters puberty with approximately 40,000 primary oocytes in her ovary. About how many of these primary oocytes will be ovulated over the entire reproductive life of the woman? (A) 40,000 (B) 35,000 (C) 480 (D) 48 (E) 12

C. Over her reproductive life, a woman ovulates approximately 480 oocytes. A woman ovulates 12 primary oocytes per year provided that she is not using oral contraceptives, does not become pregnant, or does not have any anovulatory cycles. Assuming a 40-year reproductive period, 40 x 12 = 480.

A 19-year-old woman is admitted to the hospital with low blood pressure and intense pelvic pain. Physical examination reveals heavy blood loss during this menstrual period. Speculum examination reveals irritation of the cervix of the uterus. Which of the following nerves conveys sensory fibers from the cervix of the uterus? A. Pudendal B. Superior hypogastric C. Pelvic splanchnic D. Sacral splanchnic E. Lesser splanchnic

C. Pain from the cervix is transmitted via the pelvic splanchnic nerves because the cervix is below the pelvic pain line. Pain above the pelvic pain line is carried via nerves that are primarily sympathetic in function. The superior hypogastric nerves carry pain fibers from the upper portions of the uterus. Sacral splanchnic nerves are principally sympathetic in func- tion. The pudendal nerve contains skeletal motor, sensory, and sympathetic fibers and provides primary sensory innervation to external genitalia, including the lower third of the vagina.

A 68-year-old male underwent a radical prostatectomy. Six months postoperatively the patient complains of being incapable of penile erection without the use of sildenafil (Viagra). Which of the following nerves was most probably damaged during the operation? A. Pudendal B. Perineal C. Pelvic splanchnic D. Sacral splanchnic E. Dorsal nerve to the penis

C. Penile erection is a parasympathetic mediated response that is delivered via the pelvic splanchnic nerves that pass through nerve bundles on the postero- lateral aspect of the prostate gland. (In prostatectomy, these bundles should be left intact, if at all possible, to avoid impotence.) The pudendal nerve and its terminal branch, the dorsal nerve of the penis, carry the primary skeletal motor and sensory innervation to the external genitalia, and also sympathetic fibers. Sacral splanch- nic nerves contain sympathetic fibers.

Contains the septomarginal trabecula?

C. Right ventricle

A 45-year-old man is admitted to the emergency department after experiencing a sharp pain while lifting a box of books. He told the physician that he "felt the pain in my backside, the back of my thigh, my leg, and the side of my foot." During physical examination it is observed that his Achilles tendon jerk is weakened on the affected side. Which is the most likely cause of injury? A. Disc lesion at L3-4 B. Disc lesion at L4-5 C. Disc lesion at L5-S1 D. Disc lesion at S1-2 E. Gluteal crush syndrome of the sciatic nerve or piriformis syndrome

C. The Achilles tendon reflex is a function of the triceps surae muscle, composed of the gastrocnemius and soleus muscles that insert on the calcaneus. The innervation is provided primarily by spinal nerve S1. The S1 root leaves the vertebral column at the S1 foramen of the sacrum, but a herniated disc at the L5-S1 intervertebral space puts the S1 root under tension, resulting in pain and possible weakness or paralysis of S1 supplied muscles, especially the plantar flexors. A disc lesion at L3-4 would affect the L4 spinal nerve (affecting foot inversion and extension); a lesion at L4-5 would cause problems with L5 (hip abduction and knee flexion). A disc lesion at S1-2 in the sacrum is improbable, unless there was lumbarization of the S1 vertebra. The gluteal crush syndrome usually occurs when a patient has been lying unconscious and unmoving on a hard surface for an extended period of time.

A 23-year-old woman in her seventh month of pregnancy visits her gynecologist for a routine checkup. The patient is informed that a hormone called "relaxin" is responsible for the relaxation of the sacroiliac joint and pubic symphysis. Which of the following pelvic distances will most likely remain unaffected? A. Transverse diameter B. Interspinous distance C. True conjugate diameter D. Diagonal conjugate E. Oblique diameter

C. The conjugate diameter of the pelvis (anteroposterior) is not altered by relaxation of the pelvic joints. The transverse diameter is the longest distance extending from the middle of one pelvic brim to the other. The interspinous distance is the distance be- tween the ischial spines and changes dramatically during pregnancy due to relaxation of the joints. The diagonal conjugate and oblique diameters are slightly increased during pregnancy due to the effects of the hormone relaxin.

A 32-year-old man is brought to the emergency department with complaints of pain to the left ankle and knee. The patient recalls that during a football game, his left foot landed in a hole as he was running on an uneven dirt field. The ankle was externally rotated and everted while the knee twisted medially. He was unable to bear weight subsequently. During physical examination, the right ankle is swollen and there is exquisite tenderness over the right medial malleolus and the proximal lateral leg. Radiologic examination of the right lower limb reveals a displaced fracture of the neck of right fibula and a comminuted fracture of the tibial plafond and medial malleolus. Which of the following describes the most likely consequences of this injury? A. Weak "push-off" while walking and numbness over the posteromedial leg B. Weak ankle eversion and numbness over the dorsum of the foot C. High stepping gait and numbness over the dorsum and first web space of the foot D. Waddling gait and inability to feel a pin prick over the anterolateral leg E. Swing-out gait and numbness over the medial leg

C. The deep fibular (peroneal) nerve is a branch of the common fibular (peroneal) nerve and begins at about the level of the neck of the fibula, between it and the fibularis (peroneus) longus. This nerve supplies the extensors of the foot (extensor digitorum longus, fibularis [peroneus] tertius, extensor hallucis longus, tibialis anterior, extensor digitorum brevis, and extensor hallucis brevis). It innervates the first web space of the foot. Fracture of the head of the fibula can damage this nerve, resulting in a high stepping gait and numbness over the dorsum and first web space of the foot. Muscles in the posterior compartment of the leg are involved in planter flexion. These muscles are innervated by the tibial nerve, which is a continuation of the sciatic nerve; its medial cutaneous branch supplies sensation to the posteromedial side of the leg. Waddling gait and numbness at the anterolateral side of the leg are associated with the superficial fibular and the lateral sural cutaneous nerves.

A 56-year-old diabetic man complains of repeated injury and ulcers to his right big toe. He also complains that he finds it difficult maintaining his shoes because the tips of the shoes around the toe area easily wear down. He also complains that for a while now, his first two toes "feel funny." He used to enjoy playing soccer on weekends but has found it difficult to be involved. Which of the following nerves is most likely affected? A. Superior gluteal nerve injury B. Inferior gluteal nerve injury C. Deep fibular (peroneal) nerve injury D. Superficial fibular (peroneal) nerve injury E. Common fibular (peroneal) nerve injury

C. The deep fibular (peroneal) nerve passes deep to the extensor retinaculum and supplies the intrinsic muscles on the dorsum of the foot (extensors digitorum and hallucis longus) and the tarsal and tarsometatarsal joints. When it finally emerges as a cutaneous nerve, it is so far distal in the foot that only a small area of skin remains available for innervation: the web of skin between and contiguous sides of the first and second toes. The superficial fibular (peroneal) nerve supplies the skin on the anterolateral aspect of the leg and divides into the medial and intermediate dorsal cutaneous nerves, which continue across the ankle to supply most of the skin on the dorsum of the foot.

Young parents were concerned that their 14-month-old daughter had not yet begun walking. Their pediatrician reassured them, saying that one of the muscles of the leg, the fibularis (peroneus) tertius, had to complete its central neurologic development before the child could lift the outer corner of the foot and walk without stumbling over her toes. What is the most common nerve supply of this muscle? A. Sural B. Lateral plantar C. Deep fibular (peroneal) D. Superficial fibular (peroneal) E. Tibial

C. The deep fibular (peroneal) nerve supplies the fibularis (peroneus) tertius muscle. Although its name might lead one to think that this muscle is in the lateral compartment with the other two fibularis (peroneus) muscles, it is in the anterior (extensor) compartment of the leg. It is named for its origin from the fibula. It inserts upon the dorsum of the base of the fifth (or fourth) metatarsal bone and assists in extension and eversion of the foot. The sural nerve is a cutaneous nerve, formed by contributions from the tibial and common fibular (peroneal) nerves; it supplies the posterior lateral leg and the lateral side of the foot. The lateral plantar nerve is a branch of the tibial nerve; it innervates the quadratus plantae, muscles of the little toe, the adductor hallucis, lumbricals 2 to 4, and all of the interossei. It is sensory to the lateral side of the sole and the lateral three and a half digits. The superficial fibular (peroneal) nerve supplies the fibularis (peroneus) longus and brevis and innervates the skin on most of the dorsum of the foot. The tibial nerve supplies the calf muscles and divides into the medial and lateral plantar nerves.

After suffering a deep stab wound to her posterior thigh, a 22-year-old woman presents to the emergency department. The wound is closed but the patient develops a subsequent wound infection. Which group of lymph nodes first receives drainage from this deep wound area, and would most likely be enlarged in this patient? A. External iliac B. Superficial inguinal C. Deep inguinal D. Common iliac E. Internal iliac

C. The deep nodes are located beneath the deep fascia (fascia cribrosa) and lie along the medial side of the femoral vein. The presence of swollen inguinal lymph nodes is an important clinical sign because swelling may indicate an infection in the lower extremities. They then drain superiorly to the external iliac lymph nodes. The superficial nodes lie in the superficial fascia below the inguinal ligament and can be divided into horizontal and vertical groups. External iliac lies along the external iliac vessels; they are arranged in groups of three (anteriorly, medially, and lateral to vessels).

An 18-year-old professional tennis player fell when she leaped for an overhead shot and landed with her foot inverted. Radiologic examination in the hospital revealed an avulsion fracture of the tuberosity of the fifth metatarsal. Part of the tuberosity is pulled off, producing pain and edema. Which of the following muscles is pulling on the fractured fragment? A. Fibularis (peroneus) longus B. Tibialis posterior C. Fibularis (peroneus) brevis D. Extensor digitorum brevis E. Adductor hallucis

C. The fibularis (peroneus) brevis arises from the fibula and inserts upon the tuberosity at the base of the fifth metatarsal bone. Its attachment is often involved in an inversion fracture of the foot. This common fracture can often be overlooked when it is combined with an inversion sprain of the ankle. The fibularis (peroneus) longus arises from the fibula passes under the lateral malleolus, and then turns medially into the plantar surface of the foot, where it inserts upon the medial cuneiform and first metatarsal bones. The tibialis posterior arises from the tibia in the posterior compartment of the leg; it passes under the medial malleolus and inserts upon the navicular and metatarsal bones. The extensor digitorum brevis arises dorsally from the calcaneus and inserts upon the proximal phalanges of the lateral toes. The adductor hallucis arises from the lateral metatarsals and transverse tarsal ligament and inserts upon the proximal phalanx and lateral sesamoid bone of the big toe.

A 72-year-old woman suffered a hip dislocation when she fell down the steps to her garage. Which of the following is most significant in resisting hyperextension of the hip joint? A. Pubofemoral ligament B. Ischiofemoral ligament C. Iliofemoral ligament D. Negative pressure in the acetabular fossa E. Gluteus maximus muscle

C. The iliofemoral ligament ("inverted Y ligament of Bigelow") is the most important ligament reinforcing the joint anteriorly that would resist both hyperextension and lateral rotation at the hip joint. The pubofemoral ligament reinforces the joint inferiorly and limits extension and abduction. The ischiofemoral ligament reinforces the joint posteriorly and limits extension and medial rotation. Negative pressure in the acetabular fossa has nothing to do with resisting hyperextension of the hip joint but does help resist dislocation of the head of the femur. The gluteus maximus muscle extends and laterally rotates the thigh and does not particularly resist hyperextension.

A 46-year-old woman stepped on a broken wine bottle on the sidewalk and the sharp glass entered the posterior part of her foot. The patient was admitted to the hospital, and a physical examination concluded that her lateral plantar nerve had been transected (cut through). Which of the following conditions will most likely be confirmed by further physical examination? A. Loss of sensation over the plantar surface of the third toe B. Paralysis of the abductor hallucis C. Paralysis of the interossei and adductor hallucis D. Flexor hallucis brevis paralysis E. Flexor digitorum brevis paralysis

C. The lateral plantar nerve innervates the interossei and adductor hallucis. These losses would be obvious when the patient attempts to abduct and adduct the toes. Sensation would be absent over the lateral side of the sole, the fifth and fourth toes, and half of the third toe. The medial plantar nerve provides sensation over the plantar surface of the first and second toes and half of the third toe as well as function of the so-called LAFF muscles: first lumbrical, abductor hallucis, flexor hallucis brevis, and flexor digitorum brevis

A 12-year-old boy is brought to the physician by his father because of redness and swelling of his left foot for 24 hours. Three days earlier he had scraped his foot while wading in a drainage ditch. Examination of the foot shows a purulent abrasion with edema, erythema, and tenderness on the lateral side. Infection will most likely spread from the lateral side of the foot to the regional lymph nodes in which area? A. Lateral surface of the thigh B. Medial malleolus, posteriorly C. Popliteal fossa D. Sole of the foot E. Superficial inguinal area

C. The lymphatic drainage of the foot follows its venous drainage. The small saphenous vein drains the lateral side of the foot and the posterolateral leg. It drains into the popliteal vein in the popliteal fossa. Therefore a lesion on the lateral side of the foot will drain to the popliteal nodes in the popliteal fossa.

A 50-year-old woman is admitted to the emergency department complaining of painful swelling to the left leg, fever, and malaise for 2 days. The patient has a history of type 2 diabetes mellitus, and she was bitten on the left leg by an insect a week before presentation. She scratched the pruritic area and applied alcohol to the site when the swelling increased; a purulent fluid began to drain from it 2 days later. During physical examination, the patient was febrile (40° C) with 5 × 5 cm tender, fluctuant swelling over the anterolateral aspect of the middle third of the left leg, which drained copious amounts of purulent fluid. Which of the following findings is most likely to be also present during physical examination of this patient? A. Tender vertical group of superficial inguinal lymph nodes B. Enlarged horizontal group of superficial inguinal lymph nodes C. Enlarged group of deep inguinal lymph nodes D. Enlarged popliteal lymph nodes E. Enlarged iliac nodes

C. The lymphatic drainage of the leg is such that superficial lymphatics on the anterolateral side of the foot and leg and all the deep lymphatics in the foot and leg first drain into the popliteal nodes and then to the deep inguinal nodes. This patient has an infected anterolateral mid leg injury, which will first drain into the popliteal nodes. The vertical group of superficial inguinal nodes receives superficial lymphatics from the medial side of the foot, leg, and all the superficial lymph from the thigh. The horizontal group of superficial lymphatics receives lymph from the anterior abdominal wall below the umbilicus, perineum (except the glans penis in men and clitoris in women), and lower third of the anal canal. The popliteal nodes eventually drain to the deep inguinal nodes, but are usually not palpable. The iliac nodes are deep structures and are not be palpable during physical examination.

A 34-year-old woman is admitted to the hospital due to severe lower abdominal pain. Radiographic examination reveals an ovarian tumor. Which of the following lymph nodes will most likely become invaded by cancerous cells? A. Superficial inguinal B. External iliac C. Lumbar/lateral aortic D. Deep inguinal E. Internal iliac

C. The lymphatic vessels of the ovaries join with lymphatics from the uterine tubes and the fundus of the uterus. These ascend to the right and left lumbar (caval/aortic) lymph nodes. These lymph nodes are the first to receive cancerous cells from the ovaries. Superficial inguinal nodes drain the lower limb, the anterior abdominal wall inferior to the umbilicus, and superficial perineal structures. The external iliac nodes drain the deep inguinal nodes that drain the clitoris and superficial inguinal nodes. The internal iliac nodes drain inferior pelvic structures and deep perineal structures.

A 75-year-old woman is admitted to the hospital after falling in her bathroom. Radiologic examination reveals an extracapsular fracture of the femoral neck. Which artery is most likely at risk for injury? A. Inferior gluteal B. First perforating branch of deep femoral C. Medial circumflex femoral D. Obturator E. Superior gluteal

C. The medial circumflex femoral artery is responsible for supplying blood to the head and neck of the femur by a number of branches that pass under the edge of the ischiofemoral ligament. This artery is most likely at risk for injury in an extracapsular fracture of the femoral neck. The inferior gluteal artery arises from the internal iliac and enters the gluteal region through the greater sciatic foramen, below the piriformis. The first perforating artery sends an ascending branch that anastomoses with the inferior gluteal artery in the buttock. The obturator artery arises from the internal iliac artery and passes through the obturator foramen. It commonly supplies the artery within the ligament of the head of the femur but is not likely to be patent in a person of this age. The superior gluteal artery arises from the internal iliac artery and enters through the greater sciatic foramen above the piriformis.

A 42-year-old woman visits the outpatient clinic due to painful urination. A dipstick test reveals leukocytosis, which confirms the diagnosis of urinary tract infection. Which of the following is the best anatomic explanation for the fact that women are more susceptible to urinary tract infections than men? A. The vagina contains less bacterial flora than the penis. B. The prostate gland produces antibacterial prostatic fluids. C. The urethra is much shorter in females. D. The urethra is located within the vagina. E. The seminal vesicles produce fluids resistant to bacteria.

C. The membranous urethra is shorter in women than in men. Because of its close proximity to the vestibule in women, it commonly leads to infections of the urinary tract. The vagina contains more bacte- rial flora than the penis. The prostate in the male produces a clear, alkaline fluid, but it has not been proved that it protects against bacterial infections. The uterus has no known antibacterial functions. The seminal vesicles produce a fructose-characterized fluid that provides nutrients to the sperm for the jour- ney through the female genital tract.

A 32-year-old woman is admitted to the hospital with intense lower abdominal pain and an elevated temperature. Upon physical examination it is readily observed that the abdominal wall is tender to the touch. Blood tests reveal leukoytosis and a diagnosis is made of peritonitis. An exploratory laparotomy reveals a ruptured ectopic pregnancy. Which of the following is the most common site of an ectopic pregnancy? A. Over the internal cervical os B. Wall of the bowel C. Uterine tube D. Mesentery of the bowel E. Surface of the ovary

C. The most common site of ectopic pregnancy is the uterine tube. Implantation at the internal cervical os would be within the uterus and lead to placenta previa. The other choices are less common sites of ectopic pregnancies.

An 82-year-old grandmother slipped on the polished floor in her front hall and was transported to the emergency department and admitted for examination with a complaint of great pain in her right lower limb. During physical examination it is observed by the resident that the right lower limb is laterally rotated and noticeably shorter than her left limb. Radiologic examination reveals an intracapsular fracture of the femoral neck. Which of the following arteries supplies the head of the femur in early childhood but no longer in a patient of this age? A. Superior gluteal B. Lateral circumflex femoral C. A branch of the obturator artery D. Inferior gluteal E. Internal pudendal

C. The obturator artery provides the artery within the ligament of the head of the femur (in about 60% of cases), the artery that supplies the head of the femur, primarily during childhood, later becoming atretic. In the adult this artery supplies only the area of the fovea of the head of the femur. The ligament of the head of the femur arises from the acetabular notch, thereafter receiving the little artery. In some individuals the medial circumflex femoral gives origin to the artery of the head. In the adult the arterial supply of the neck and head is provided by intracapsular branches of the medial circumflex femoral and lateral circumflex femoral arteries that pierce the neck of the femur, with some supply also from the gluteal arteries. The lateral circumflex femoral artery arises from the deep femoral and supplies the vastus lateralis. The pudendal artery arises from the internal iliac and provides blood supply for the structures of the perineum. Quite often, when an older patient with osteoporosis has a hip fracture, the femoral neck may have fractured, precipitating a fall, rather than the fall resulting in the hip fracture.

A 37-year-old woman is admitted to the hospital with pelvic pain. Radiographic examination reveals a benign tumor on the left ovary. An ovariectomy is per- formed and the ovarian vessels are ligated. Which of the following structures is most at risk of injury when the ovarian vessels are ligated? A. Uterine artery B. Vaginal artery C. Ureter D. Internal pudendal artery E. Pudendal nerve

C. The ovarian vessels lie anterior to the ureter just proximal to the bifurcation of the aorta. The ure- ter is the structure that is at the most risk when ligat- ing the ovarian vessels. The vaginal artery is a branch of the uterine artery. The uterine artery does anasto- mose with the ovarian vessels via the ascending uter- ine artery; however, it lies too far distally to be at risk during ligation of the ovarian vessels. The internal pudendal artery and pudendal nerve mostly lie in the perineum and are not at risk.

A 63-year-old woman visits the outpatient orthopedic clinic with the complaint of pain in her foot for more than a year. Radiologic and physical examinations give evidence of constant extension at the metatarsophalangeal joints, hyperflexion at the proximal interphalangeal joints, and extension of distal interphalangeal joints (Fig. 5-6). Which of the following terms is most accurate to describe the signs of physical examination? A. Pes planus B. Pes cavus C. Hammertoes D. Claw toes E. Hallux valgus

C. The patient's complaint is due to her case of hammertoes. Hammer toe can affect any toe but most commonly the second toe, then the third or fourth toes. It results most commonly from wearing shoes that are too short or shoes with heels that are too high. In hammertoe, the metatarsophalangeal joint is extended, the proximal interphalangeal joint is flexed, and the distal phalanx points downward, looking like a hammer. Hammertoe can occur as a result of a bunion. Calluses, or painful corns, can form on the dorsal surface of the joints. In claw toe, both the proximal and distal interphalangeal joints are strongly flexed, the result of muscle imbalance in the foot. Either hammertoe or claw toe can occur from arthritic changes. Pes cavus is the opposite of flat foot. In this case the patient has a high, flexed plantar arch; it occurs as a result of hereditary motor and sensory neural problems. It is painful because of metatarsal compression.

A 34-year-old man is admitted to the emergency department after a traumatic landing into a swimming pool from a high diving platform. The patient has multiple traumas in his abdominal cavity. After a reconstructive operation of his abdominal organs the patient develops a high fever. Radiographic examination re- veals that the lower portion of the descending colon and rectum has become septic and must be excised. Six months postoperatively the patient complains of impotence. Which of the following structures was most likely injured during the second operation? A. Pudendal nerve B. Sacral splanchnic nerves C. Pelvic splanchnic nerves D. Sympathetic chain E. Vagus nerve

C. The pelvic splanchnic nerves contain parasympathetic efferent fibers that mediate erection. These same nerves innervate the hindgut, the portions of the large intestine that were removed in this patient.

A 68-year-old man complains of pain upon urination. A CT scan and a biopsy provide evidence of an enlarged, cancerous prostate gland. Subsequently, he undergoes a radical prostatectomy. Postoperatively, he suffers from urinary incontinence because of paralysis of the external urethral sphincter. Which of the following nerves was injured during the operation? A. Pelvic splanchnic B. Sacral splanchnic C. Pudendal D. Superior gluteal E. Inferior gluteal

C. The perineal branch of the pudendal nerve is responsible for the innervation of the external urethral sphincter, and injury to this nerve can result in paralysis of the sphincter and urinary incontinence. Pelvic splanchnic and sacral splanchnic nerves are autonomic nerves that do not supply skeletal muscles in the urogenital region. The gluteal nerves innervate gluteal muscles.

A 34-year-old woman is admitted to the hospital with perineal pain. Laboratory blood tests reveal marked leukocytosis. Physical examination reveals abscesses in the anterior recess of the ischioanal fossa. A surgical procedure is performed to debride and drain the abscesses. Which of the following nerves will most likely need to be anesthetized to specifically numb the surgical area? A. Dorsal nerve to the clitoris B. Superficial perineal branch of perineal nerve C. Perineal nerve D. Inferior rectal nerve E. Pudendal nerve

C. The perineal nerve would need to be anesthetized because it supplies the area described. The dorsal nerve to the clitoris pierces the perineal membrane and innervates the clitoris and not the anterior recess of the ischiorectal fossa. The superficial perineal branch of the perineal nerve supplies only the labia majora. The inferior rectal nerve innervates the skin around the anus and the external anal sphincter muscle. The pudendal nerve is the main nerve of the perineum and gives rise to all of the aforementioned nerves; therefore, anesthetizing it would result in widespread effects that would be superfluous to what is actually needed for drainage of the abscess.

A 62-year-old man is admitted to the emergency department due to increasing difficulty in urinating over a period of several months. Physical examination reveals prostatic hypertrophy. After several unsuccess- ful attempts to catheterize the penile urethra, the urologist orders drainage of the urinary bladder by the least invasive procedure, avoiding entry into the perito- neal cavity or the injury of any major vessels or organs. Which of the following spaces needs to be traversed by the needle to reach the bladder? A. Ischioanal fossa B. Perineal body C. Retropubic space (of Retzius) D. Superficial perineal cleft E. Deep perineal pouch

C. The retropubic space (of Retzius) is the extra- peritoneal space between the pubic symphysis and the bladder. A needle placed over the pubic bone, through the body wall, and into the space of Retzius will enter the full bladder but avoids entry into the peritoneum and there is little risk of damaging major organs or vessels. Entry through the ischioanal fossa would not provide a direct route to the bladder. With entry through the superficial perineal cleft, perineal body, and deep perineal pouch there is a high risk of damaging important structures.

A 45-year-old man is admitted to the emergency department after a violent car crash. Physical examination reveals that the patient suffers from a "straddle" injury to the perineum. An MRI examination reveals that extravasating urine and blood from a torn bulbar urethra are present in the superficial perineal cleft. Which of the following fasciae provide boundaries for this space? A. Camper's fascia and Scarpa's fascia B. Perineal membrane and external perineal fascia of Gallaudet C. Colles' fascia and external perineal fascia of Gallaudet D. Perineal membrane and the superior fascia of the urogenital diaphragm E. The urogenital diaphragm and the apex of the prostate gland

C. The superficial perineal space or cleft lies be- tween the external perineal fascia of Gallaudet (fascia of inferior perineal muscles in the superficial perineal compartment) and the membranous layer of Colles' fascia. Camper's fascia is the superficial fatty layer of the anterior abdominal wall and the perineum; Scar- pa's fascia is the deep membranous layer of the abdominal wall. The perineal membrane is the inferior fascia of the urogenital diaphragm that forms the inferior boundary of the deep perineal compartment. The superior fascia of the urogenital diaphragm bounds the inferior border of the anterior recess of the ischioanal fossa. There is no space between the urogenital diaphragm and the apex of the prostate gland.

After having given birth to five children, a 41- year-old woman seeks correction of chronic urinary incontinence. While relating her history the patient reveals that she has leakage of urine with increased intraabdominal pressure. An MRI examination reveals injury to the pelvic floor that has altered the position of the neck of the bladder and the urethra. Which of the following structures has most probably been injured during the multiple deliveries? A. Tendinous arch of levator ani B. Coccygeus C. Tendinous arch of fascia pelvis D. Obturator internus E. Rectovaginal septum

C. The tendinous arch of fascia pelvis is a dense band of connective tissue that joins the fascia of the levator ani to the feltlike pubocervical fascia that cov- ers the anterior wall of the vagina. If this fascial band is torn, the ipsilateral side of the vagina falls, carrying with it the bladder and urethra, often leading to uri- nary incontinence. The tendinous arch of the levator ani is a thickened portion of the fascia of the obtura- tor internus and provides part of the origin of the le- vator ani muscle, but it plays no direct role in incon- tinence. The coccygeus muscle supports and raises the pelvic floor but is not directly associated with urinary incontinence. The obturator internus is in- volved with lateral rotation of the thigh. If the recto- vaginal septum is torn, the patient can be subject to the occurrence of rectocele or enterocele, as the lower portion of the GI tract prolapses into the posterior wall of the vagina.

The neurosurgeon had removed a portion of the dense tissue (dura mater) covering the brain of the patient when she removed the tumor that had invaded the skull. To replace this important tissue covering of the brain, she took a band of the aponeurotic tissue of the lateral aspect of the thigh, covering the vastus lateralis muscle. What muscle, supplied by the inferior gluteal nerve, inserts into this band of dense tissue as part of its insertion? A. Gluteus medius B. Gluteus minimus C. Gluteus maximus D. Tensor fasciae latae E. Rectus femoris

C. The tensor fasciae latae (which is innervated by the superior gluteal nerve) and the iliotibial tract are dense, wide aponeurosis that receives the insertion of the tensor fasciae latae and about 75% of the gluteus maximus. The gluteus maximus is the only one of the muscles listed that is supplied by the inferior gluteal nerve; in fact, it is the only muscle innervated by the inferior gluteal nerve. Gluteus medius and minimus insert on the greater trochanter and are innervated by the superior gluteal nerve. The rectus femoris, supplied by the femoral nerve, inserts via the quadriceps tendon on the patella and tibial tuberosity.

Individual blastomeres were isolated from a blastula at the 4-cell stage. Each blastomere was cultured in vitro to the blastocyst stage and individually implanted into four pseudopregnant foster mothers. Which of the following would you expect to observe 9 months later? (A) Birth of one baby (B) Birth of four genetically different babies (C) Birth of four genetically identical babies (D) Birth of four grotesquely deformed babies (E) No births

C. This scenario would result in four genetically identical children. Blastomeres at the 4-cell to 8-cell stage are totipotent, that is, capable of forming an entire embryo. Since blastomeres arise by mitosis of the same cell (zygote), they are genetically identical. This phenomenon is important in explaining monozygotic (identical) twins. About 30% of monozygotic twins arise by early separation of blastomeres. The remaining 70% originate at the end of week 1 of development by a splitting of the inner cell mass.

A 42-year-old woman is admitted to the hospital with severe uterine bleeding. Radiographic examination reveals uterine fibroids. A uterine artery embolization is performed. Which of the following arteries will supply collateral supply to the uterus? A. External iliac B. Inferior mesenteric C. Ovarian D. Internal pudendal E. Superior mesenteric

C. Uterine artery embolization is performed to starve uterine fibroids of their blood supply resulting in a decrease in size of these benign tumors. Follow- ing the procedure, the uterus receives collateral blood supply from the ovarian artery (a direct branch of the abdominal aorta). The external iliac, inferior mesen- teric, internal pudendal, and superior mesenteric ar- teries do not provide adequate collateral blood supply to the uterus.

A 34-year-old male power lifter visits the outpatient clinic because he has difficulty walking. During physical examination it is observed that the patient has a problem unlocking the knee joint to permit flexion of the leg. Which of the following muscles is most likely damaged? A. Biceps femoris B. Gastrocnemius C. Popliteus D. Semimembranosus E. Rectus femoris

C. When the popliteus contracts, it rotates the distal portion of the femur in a lateral direction. It also draws the lateral meniscus posteriorly, thereby protecting this cartilage as the distal femoral condyle glides and rolls backward, as the knee is flexed. This allows the knee to flex and therefore serves in unlocking the knee. The biceps femoris is a strong flexor of the leg and laterally rotates the knee when it is in a position of flexion. The gastrocnemius is a powerful plantar flexor of the foot. The semimembranosus, similar to the biceps femoris, is a component of the hamstring muscles and is involved in extending the thigh and flexing the leg at the knee joint. The rectus femoris is the strongest quadriceps muscle in extending the leg at the knee.

Left atrium and ventricle

Coronary angiographs of a 44-year-old male patient reveal an occlusion of the circumflex branch of the left coronary artery. This patient has been suffering from myocardial infarction in which of the following areas? -Right and left ventricles -Right and left atria -Interventricular septum -Apex of the heart -Left atrium and ventricle

11. An elderly man at a nursing home is known to have degenerative brain disease. When cerebrospinal fluid (CSF) is withdrawn by lumbar puncture for further examination, which of the following structures is most likely penetrated by the needle? (A) Pia mater (B) Filum terminale externum (C) Posterior longitudinal ligament (D) Ligamentum flavum (E) Annulus fibrosus

D

111 A 32-year-old male who is an expert target shooter reports pain in his right upper limb and slight tingling and numbness of all digits of the ipsilateral hand. However, the tingling and numbness of the fourth and fi fth digits is the most severe. The man states that the problem usually occurs when he is fi ring his gun with his hand overhead. Radiographic studies reveal the presence of a cervical rib and accessory scalene musculature. Which of the following structures is most likely being compressed? ⃣ A. Axillary artery ⃣ B. Upper trunk of brachial plexus ⃣ C. Subclavian artery ⃣ D. Lower trunk of brachial plexus ⃣ E. Brachiocephalic artery and lower trunk of brachial plexus

D

115 A 69-year-old woman visits the outpatient clinic with a complaint of numbness and tingling of her hand for the past 3 months. Physical examination reveals she has numbness and pain in the lateral three digits of her right hand that are relieved by vigorous shaking of the wrist. In addition, the abductor pollicis brevis, opponens pollicis, and the first two lumbrical muscles are weakened. Sensation was decreased over the lateral palm and the volar aspect of the fi rst three digits. Which of the following nerves is most likely compressed? A. Ulnar B. Radial C. Recurrent median D. Median E. Posterior interosseous

D

126 A 34-year-old woman is admitted to the hospital after a car collision. Physical examination reveals a mallet fi nger. Which of the following conditions is expected to be present during radiographic examination? ⃣ A. A lesion of the ulnar nerve at the distal fl exor crease of the wrist ⃣ B. A separation of the extension expansion over the middle interphalangeal joint ⃣ C. Compression of the deep ulnar nerve by dislocation of the lunate bone ⃣ D. Avulsion fracture of the dorsum of the distal phalanx ⃣ E. Fracture of the fourth or fi fth metacarpal bone

D

128 A 41-year-old woman is admitted to the hospital after a car crash. Radiographic examination reveals a transverse fracture of the radius proximal to the attachment of the pronator teres muscle. The proximal portion of the radius is deviated laterally. Which of the following muscles will most likely be responsible for this deviation? ⃣ A. Pronator teres ⃣ B. Pronator quadratus ⃣ C. Brachialis ⃣ D. Supinator ⃣ E. Brachioradialis

D

138 A 55-year-old woman is admitted to the emergency department after a car crash. Physical examination reveals severe pain in the flexor muscles of the forearm; fixed flexion position of the finger; and swelling, cyanosis, and anesthesia of the fingers. Which of the following is the most likely diagnosis? A. Colles' fracture B. Scaphoid fracture C. Bennett's fracture D. Volkmann's ischemic contracture E. Boxer's fracture

D

139 A 62-year-old man visits the outpatient clinic with pain in his hand after falling on the outstretched hand. Radiographic examination reveals a fracture of the pisiform bone and hematoma of the surrounding area. Which of the following nerves will most likely be affected? A. Ulnar B. Radial C. Median D. Deep ulnar E. Deep radial

D

17. A 44-year-old woman comes to her physician and complains of headache and backache. On examination, she is found to have fluid accumulated in the spinal epidural space because of damage to blood vessels or meninges. Which of the following structures is most likely ruptured? (A) Vertebral artery (B) Vertebral vein (C) External vertebral venous plexus (D) Internal vertebral venous plexus (E) Lumbar cistern

D

19. During a snowstorm, a 52-year-old man is brought to the emergency department after a multiple car accident. Which of the following conditions is produced by a force that drives the trunk forward while the head lags behind in a rear-end automobile collision? (A) Scoliosis (B) Hangman's fracture (C) Meningomyelocele (D) Whiplash injury (E) Herniated disk

D

2. A 23-year-old jockey falls from her horse and complains of headache, backache, and weakness. Radiologic examination would reveal blood in which of the following spaces if the internal vertebral venous plexus was ruptured? (A) Space deep to the pia mater (B) Space between the arachnoid and dura maters (C) Subdural space (D) Epidural space (E) Subarachnoid space

D

20. A 37-year-old man is brought to the emergency department with a crushed second cervical vertebra (axis) that he suffered after a stack of pallets fell on him at work. Which of the following structures would be intact after the accident? (A) Alar ligament (B) Apical ligament (C) Semispinalis cervicis muscle (D) Rectus capitis posterior minor (E) Obliquus capitis inferior

D

25. A 25-year-old soldier suffers a gunshot wound on the lower part of his back and is unable to move his legs. A neurologic examination and magnetic resonance imaging (MRI) scan reveal injury of the cauda equina. Which of the following is most likely damaged? (A) Dorsal primary rami (B) Ventral primary rami (C) Dorsal roots of the thoracic spinal nerves (D) Ventral roots of the sacral spinal nerves (E) Lumbar spinal nerves

D

26 A 17-year-old male has weakness of elbow fl exion and supination of the left hand after sustaining a knife wound in that arm in a street fi ght. Examination in the emergency department indicates that a nerve has been severed. Which of the following conditions will also most likely be seen during physical examination? ⃣ A. Inability to adduct and abduct his fi ngers ⃣ B. Inability to fl ex his fi ngers ⃣ C. Inability to fl ex his thumb ⃣ D. Sensory loss over the lateral surface of his forearm ⃣ E. Sensory loss over the medial surface of his forearm

D

3. A 42-year-old woman with metastatic breast cancer is known to have tumors in the intervertebral foramina between the fourth and fifth cervical vertebrae and between the fourth and fifth thoracic vertebrae. Which of the following spinal nerves may be damaged? (A) Fourth cervical and fourth thoracic nerves (B) Fifth cervical and fifth thoracic nerves (C) Fourth cervical and fifth thoracic nerves (D) Fifth cervical and fourth thoracic nerves (E) Third cervical and fourth thoracic nerves

D

30. Cerebrospinal fluid (CSF) is produced by vascular choroid plexuses in the ventricles of the brain and accumulated in which space?

D

43 During a fi ght in a tavern, a 45-year-old male construction worker received a shallow stab wound from a broken beer bottle at a point near the middle of the left posterior triangle of his neck. Upon physical examination it is observed that the left shoulder is drooping lower than the right shoulder, and the superior angle of the scapula juts out slightly. Strength in turning the head to the right or left appears to be symmetric. Which of the following nerves is most likely injured? ⃣ A. Suprascapular nerve in the supraspinous fossa ⃣ B. The terminal segment of the dorsal scapular nerve ⃣ C. The upper trunk of the brachial plexus ⃣ D. The spinal accessory nerve in the posterior cervical triangle ⃣ E. The thoracodorsal nerve in the axilla

D

47 A 29-year-old female is examined in the emergency department after falling from her balcony. Radiographic examination reveals that she has suffered a broken clavicle, with associated internal bleeding. Which of the following vessels is most likely to be injured in clavicular fractures? ⃣ A. Subclavian artery ⃣ B. Cephalic vein ⃣ C. Lateral thoracic artery ⃣ D. Subclavian vein ⃣ E. Internal thoracic artery

D

59 The right shoulder of a 78-year-old female had become increasingly painful over the past year. Abduction of the right arm caused her to wince from the discomfort. Palpation of the deltoid muscle by the physician produced exquisite pain. Imaging studies reveal intermuscular infl ammation extending over the head of the humerus. What structure was infl amed? ⃣ A. Subscapular bursa ⃣ B. Infraspinatus muscle ⃣ C. Glenohumeral joint cavity ⃣ D. Subacromial bursa ⃣ E. Teres minor muscle

D

62 A 23-year-old male basketball player is admitted to the hospital after injuring his shoulder during a game. Physical and radiographic examinations reveal total separation of the shoulder (F ig. 6-4 ). Which of the following structures has most likely been torn? ⃣ A. Glenohumeral ligament ⃣ B. Coracoacromial ligament ⃣ C. Tendon of long head of biceps brachii ⃣ D. Acromioclavicular ligament ⃣ E. Transverse scapular ligament

D

66 A 19-year-old male had suffered a deep laceration to an upper limb when he stumbled and fell on a broken bottle. On examination of hand function it is observed that he is able to extend the metacarpophalangeal joints of all his fingers in the affected limb. He cannot extend the interphalangeal joints of the fourth and fifth digits, and extension of the interphalangeal joints of the second and third digits is very weak. There is no apparent sensory defi cit in the hand. Which of the following nerves has most likely been injured? A. Radial nerve at the elbow B. Median nerve at the wrist C. Ulnar nerve in midforearm D. Deep branch of ulnar nerve E. Recurrent branch of the median nerve

D

7. A 27-year-old mountain climber falls from a steep rock wall and is brought to the emergency department. His physical examination and computed tomography (CT) scan reveal dislocation fracture of the upper thoracic vertebrae. The fractured body of the T4 vertebra articulates with which of the following parts of the ribs? (A) Head of the third rib (B) Neck of the fourth rib (C) Tubercle of the fourth rib (D) Head of the fifth rib (E) Tubercle of the fifth rib

D

73 A 23-year-old female had a painful injury to her hand in a dry ski-slope competition, in which she fell and caught her thumb in the matting. Radiographic and physical examinations reveal rupture of the ulnar collateral ligament of the metacarpophalangeal joint of the thumb. Lidocaine is injected into the area to relieve the pain, and she is scheduled for a surgical repair. From which of the following clinical problems is she suffering? ⃣ A. De Quervain's syndrome ⃣ B. Navicular bone fracture ⃣ C. Boxer's thumb ⃣ D. Gamekeeper's thumb ⃣ E. Bennett's thumb

D

79 A 68-year-old woman fell when she missed the last step from her motor home. Radiographic examination at the local medical care center reveals a fracture of the distal radius. The distal fragment of the radius is angled forward. What name is commonly applied to this type of injury? ⃣ A. Colles' fracture ⃣ B. Scaphoid fracture ⃣ C. Bennett's fracture ⃣ D. Smith's fracture ⃣ E. Boxer's fracture

D

82 A 19-year-old fell from a cliff when he was hiking in the mountains. He broke his fall by grasping a tree branch, but he suffered injury to the C8-T1 spinal nerve roots. Sensory tests would thereafter confi rm the nature of his neurologic injury by the sensory loss in the part of the limb supplied by which of the following? ⃣ A. Lower lateral brachial cutaneous nerve ⃣ B. Musculocutaneous nerve ⃣ C. Intercostobrachial nerve ⃣ D. Medial antebrachial cutaneous nerve ⃣ E. Median nerve

D

88 A 55-year-old female choreographer had been treated in the emergency department after she fell from the stage into the orchestra pit. Radiographs revealed fracture of the styloid process of the ulna. Disruption of the triangular fi brocartilage complex is suspected. With which of the following bones does the ulna normally articulate at the wrist? ⃣ A. Triquetrum ⃣ B. Hamate ⃣ C. Radius and lunate ⃣ D. Radius ⃣ E. Pisiform and triquetrum

D

9. A 34-year-old woman crashes into a tree during a skiing lesson and is brought to a hospital with multiple injuries that impinge the dorsal primary rami of several spinal nerves. Such lesions could affect which of the following muscles? (A) Rhomboid major (B) Levator scapulae (C) Serratus posterior superior (D) Iliocostalis (E) Latissimus dorsi

D

90 An 18-year-old male suffered a signifi cant laceration through the skin and underlying tissues at the distal crease of the wrist. The medical student rotating through the emergency department suspected (correctly) that the ulnar nerve was cut completely through at this location. Which of the following would most likely occur? ⃣ A. The patient could not touch the tip of the thumb to the tips of the other digits. ⃣ B. There would be loss of sensation on the dorsum of the medial side of the hand. ⃣ C. The patient would be unable to fl ex the interphalangeal joints. ⃣ D. There would be decreased ability to extend the interphalangeal joints. ⃣ E. There would be no serious functional problem at all to the patient.

D

92 A 67-year-old housepainter visits the outpatient clinic complaining that his hands are getting progressively worse, becoming more and more painful and losing their function. On physical examination of the hands, there is fl exion of the metacarpophalangeal joints, extension of the proximal interphalangeal joints, and slight fl exion of the distal interphalangeal joints. What is the most likely diagnosis? ⃣ A. Mallet fi nger ⃣ B. Boutonnière deformit C. Dupuytren contracture ⃣ D. Swan-neck deformity ⃣ E. Silver fork wrist deformity

D

A 19-year-old man is brought to the emergency department after dislocating his shoulder while playing soccer. Following reduction of the dislocation, he has pain over the dorsal region of the shoulder and cannot abduct the arm normally. An MRI of the shoulder shows a torn muscle. Which of the following muscles is most likely to have been damaged by this injury? ⃣ A. Coracobrachialis ⃣ B. Long head of the triceps ⃣ C. Pectoralis minor ⃣ D. Supraspinatus ⃣ E. Teres major

D

A 22-year-old male is diagnosed with metastatic malignant melanoma of the skin over the xiphoid process. Which nodes receive most of the lymph from this area and are therefore most likely to be involved in metastasis of the tumor? ⃣ A. Deep inguinal ⃣ B. Vertical group of superfi cial inguinal ⃣ C. Horizontal group of superfi cial inguinal ⃣ D. Axillary ⃣ E. Deep and superfi cial inguinal

D

A 22-year-old woman is admitted to the emergency department in an unconscious state. The nurse takes a radial pulse to determine the heart rate of the patient. This pulse is felt lateral to which tendon? ⃣ A. Palmaris longus ⃣ B. Flexor pollicis longus ⃣ C. Flexor digitorum profundus ⃣ D. Flexor carpi radialis ⃣ E. Flexor digitorum superfi cialis

D

A 24-year-old man is admitted with a wound to the palm of his hand. He cannot touch the pads of his fingers with his thumb but can grip a sheet of paper between all fingers and has no loss of sensation on the skin of his hand. Which of the following nerves has most likely been injured? ⃣ A. Deep branch of ulnar ⃣ B. Anterior interosseous ⃣ C. Median ⃣ D. Recurrent branch of median ⃣ E. Deep branch of radial

D

A 45-year-old male arrived at the emergency department with injuries to his left elbow after he fell in a bicycle race. Radiographic and MRI examinations show a fracture of the medial epicondyle and a torn ulnar nerve. Which of the following muscles would be most likely to be paralyzed? A. Flexor digitorum superfi cialis B. Biceps brachii C. Brachioradialis D. Flexor carpi ulnaris E. Supinaror

D

A 45-year-old woman is being examined as a candidate for cosmetic breast surgery. The surgeon notes that both of her breasts sag considerably. Which structure has most likely become stretched to result in this condition? A. Scarpa's fascia B. Pectoralis major muscle C. Pectoralis minor muscle D. Suspensory (Cooper's) ligaments E. Serratus Anterior

D

A 2-day-old newborn is diagnosed with transposition of the great arteries. Which structure is responsible for the division of the truncus arteriosus into the great arteries? A. Septum secundum B. Septum primum C. Bulbar septum D. Aorticopulmonary septum E. Endocardial cushions

D. The aorticopulmonary septum functions to divide the truncus arteriosus and bulbus cordis into the aorta and pulmonary trunk. The septum secundum forms an incomplete separation between the two atria. The septum primum divides the atrium into right and left halves. The bulbar septum is derived from the bulbus cordis and will give rise to the interventricular septum inferior to the aorticopulmonary septum, eventually fusing with it. The endocardial cushions play a role in the division of the AV canal into right and left halves, by causing the AV cushions to approach each other.

After a 2-day-old newborn male swallows milk he becomes cyanotic. After 3 days he develops pneumonia. A tracheoesophageal fistula is suspected. Failure of development has occurred most specifically in which of the following structures? A. Esophagus B. Trachea C. Tongue D. Tracheoesophageal septum E. Pharynx

D. The tracheoesophageal septum is a primordial structure that separates the trachea from the esophagus. If this structure fails to develop, a tracheoesophageal fistula will result, in which event the two structures will not separate completely. When the infant attempts to swallow milk, it spills into the esophageal pouch and is regurgitated. The child becomes cyanotic because an insufficient amount of oxygen is reaching the lungs as a result of the malformed trachea.

A 40-year-old man was brought to the emergency department after being struck by a car. He complained of pain to the left knee and leg and inability to bear weight on the affected limb. On examination, there was a joint effusion of the left knee, and tenderness over the medial and lateral side of the joint. A valgus stress test was positive while the varus stress test was negative. An MRI of the left knee showed complete disruption of multiple ligament support structures of the knee. What other symptoms are most likely possible in this patient? A. Inability to extend the knee B. Inability to flex the knee C. Instability of the knee when walking down a flight of stairs D. Instability of the knee when walking up a flight of stairs E. Excessive extension of the knee and difficulty walking down stairs

D. A positive valgus stress test indicates injury to the medial collateral ligament. Injuries to this ligament usually involve the anterior cruciate ligament. The femur is usually pushed posteriorly during stair climbing, an action that is opposed by a normal anterior cruciate ligament. Injury to the anterior cruciate ligament results in posterior displacement of the femur in relation to the tibia with difficulty climbing stairs. Extension of the knee is done mainly by the quadriceps femoris muscle. The posterior thigh muscles provide flexion of the knee. Gravity pushes the femur forward while walking down a flight, which is stabilized by the posterior cruciate ligament, which is not damaged in this case. The patient has no difficulty walking down the stairs; the posterior cruciate ligament stabilizes the knee during this action.

A 42-year-old woman presents with complaints of severe headaches, blurred vision, slurred speech, and loss of muscle coordination. Her last pregnancy 5 years ago resulted in a hydatidiform mole. Laboratory results show a high hCG level. Which of the following conditions is a probable diagnosis? (A) Vasa previa (B) Placenta previa (C) Succenturiate placenta (D) Choriocarcinoma (E) Membranous placenta

D. After a hydatidiform mole, it is very important to ensure that all the invasive trophoblastic tissue is removed. High levels of hCG are a good indicator of retained trophoblastic tissue because such tissue produces this hormone. In this case, the trophoblastic tissue has developed into a malignant choriocarcinoma and metastasized to the brain, causing her symptoms of headache, blurred vision, and so on.

A 2-year-old male is admitted to the hospital with testicular pain. Physical examination reveals an enlarged scrotum. An otoscope is placed beneath the lateral side of the scrotum and the testis is transilluminated through the scrotal sac. Which of the following best describes the signs observed in this patient? A. Varicocele B. Rectocele C. Cystocele D. Hydrocele E. Hypospadias

D. As seen in the photograph, the swollen scrotum contains mostly a clear fluid. Since hydrocele is the accumulation of fluid between the visceral and parietal layers of the tunica vaginalis, this condition best accounts for the findings in this patient.

Takes its origin from the left ventricle and ends at the sternal angle?

D. Ascending aorta

A 34-year-old man visits the outpatient clinic for an annual checkup. A radiographic examination of his knees is shown in Fig. 5-11. Physical examination reveals no pathology or pain to his knees. The patient has no past history of any knee problems. What is the most likely diagnosis? A. Enlarged prepatellar bursa B. Osgood-Schlatter disease C. Normal intercondylar eminence D. Bipartite patella E. Injury to lateral meniscus

D. Bipartite patella is a normal variant of an unfused superolateral secondary ossification center, which can easily be mistaken for a fracture on a radiograph. The subcutaneous prepatellar bursa can become painfully enlarged with acute or chronic compression, as in crawling about on the knees. Osgood-Schlatter disease is painful involvement of the patellar ligament on the tibial tuberosity, commonly in children 10 to 14 years of age. The medial retinaculum is an expanded portion of the vastus medialis tendon toward the patella.

A 23-year-old man is admitted to the emergency department after injuring his knee while playing football. During physical examination there is pain and swelling of the knee, in addition to locking of the knee in full extension. Radiologic examination reveals a bucket handle meniscal tear (Fig. 5-12). Which of the following ligaments is most likely injured? A. Posterior cruciate B. Medial collateral C. Lateral collateral D. Anterior cruciate E. Coronary

D. Both the medial and lateral menisci are subject to rotational injuries and may be torn. The medial meniscus is much more liable to injury because it is attached to the fused deep layer of the medial collateral ligament and joint capsule. The lateral meniscus is separated from the fibular collateral ligament and is external to the capsule of the knee joint. Commonly seen in football players' knees, meniscal tears are usually diagnosed by MRI or by arthroscopy. The presenting symptoms of tearing may be pain and swelling, or locking of the knee. Locking of the knee suggests a bucket handle tear, in which a partly detached cartilage wedges between the tibia and femur, inhibiting further movement. A bucket handle tear is often associated with rupture of the anterior cruciate ligament. Sometimes a momentary click can be heard in flexion/extension movements of the knee. Meniscectomy is a successful operation, but currently there is greater emphasis on repairing small tears. Meniscal cysts can form secondary to meniscal tears and some of these can also be treated arthroscopically.

A 38-year-old pregnant woman is admitted to the emergency department with severe vaginal bleeding. Ultrasound examination confirms the initial diagnosis of placenta previa. What is the site of implantation in placenta previa? A. Uterine (fallopian) tubes B. Cervix C. Mesentery of the abdominal wall D. Lower part of uterine body, overlapping the internal cervical os E. Fundus of the uterus

D. By definition, the site of implantation in placenta previa overlaps the internal cervical os. Ectopic pregnancy in the uterine (Fallopian) tubes results in tubal pregnancy. The fundus of the uterus is the nor- mal site of implantation. Implantation within the mesenteries of the abdomen will result in an abdomi- nal pregnancy. The cervix is not a notable site of ec- topic implantation.

Nuchal translucency is used to screen for congenital defects during the 11th through 14th weeks of pregnancy. This procedure is recommended by an obstetrician for a 35-year-old pregnant woman who previously had a child with Down's syndrome (trisomy 21). Which of the following would be common to see in addition to a larger than normal accumulation of fluid at the back of the baby's head, indicating the possibility of another baby with Down's syndrome? A. Meromelia B. Hydrocephalus C. Anencephaly D. Atrioventricular septal defect E. Plagiocephaly

D. Down's syndrome is a congenital abnormality involving a triplication of chromosome 21. It is more common in children of women who become pregnant at 35 years or older (advanced maternal age is a risk factor). Down's syndrome is associated with various abnormalities including congenital heart defects. The most common congenital heart defect in Down's syndrome is endocardiac cushion type of atrial septal defect (40%) followed by ventricular septal defects (35%).

A 6-year-old boy with a family history of muscular disease leading to wheelchair dependency in his maternal uncles presents with difficulty in standing from the seated position. He bends forward, uses his hands to help him push up from the floor, and then straightens his knees to stand. Which of the following muscles is most likely involved by this disease process? A. Tibialis posterior and gastrocnemius B. Quadratus femoris C. Gluteus medius and gluteus minimus D. Gluteus maximus E. Hamstrings F. Iliopsoas

D. Duchenne muscular dystrophy is a condition that causes muscle weakness. It starts in childhood and may be noticed when a child has difficulty standing up, climbing, or running, which requires extension of the hip. This patient has the classic Gower's sign. The gluteus maximus functions primarily between the flexed and standing (straight) positions of the thigh, as when rising from the sitting position, straightening from the bending position, walking uphill and up stairs, and running.

A 48-year-old woman is admitted to the hospital with severe abdominal pain. Several imaging methods reveal that the patient suffers from intestinal ischemia. An abdominopelvic catheterization is ordered for antegrade angiography. A femoral puncture is performed. What is the landmark for femoral artery puncture? A. Halfway between anterior superior iliac spine and pubic symphysis B. 4.5 cm lateral to the pubic tubercle C. Midpoint of the inguinal skin crease D. Medial aspect of femoral head E. Lateral to the fossa ovalis

D. Femoral artery puncture is one of the most common vascular procedures. The femoral artery can be localized often by simply feeling for the strongest point of the femoral pulse just inferior to the inguinal ligament. The femoral artery can be accessed with fluoroscopic assistance at the medial edge of the upper portion of the head of the femur. It is easily localized by Doppler ultrasound if the pulse is difficult to detect, such as in an obese patient. It is here that catheters are passed into the femoral artery for catheterization of abdominopelvic and thoracic structures and for antegrade angiography. It is also a site where arterial blood can be obtained for gas analysis. The mid inguinal point, halfway between the anterior superior iliac spine and the pubic symphysis, can be either medial or lateral to the femoral artery and is not a dependable landmark. A needle inserted at the level of the inguinal crease, or inferior to the femoral head, can enter the femoral artery distal to the origin of the deep femoral artery, presenting more risk for accidental vascular injury. Four centimeters lateral to the pubic tubercle overlies the deep inguinal ring, with potential entry to spermatic cord, femoral vein, or artery. The fossa ovalis is the opening in the deep fascia of the thigh for the termination of the great saphenous vein in the femoral vein.

A 55-year-old man complains of pain at his anus. Examination reveals external hemorrhoids. Which of the following nerves carries pain sensation from the anus? A. Sacral splanchnic B. Superior hypogastric C. Pelvic splanchnic D. Pudendal E. Ilioinguinal

D. Hemorrhoids are divided into two categories: internal and external. Pain due to external hemorrhoids is mediated by the pudendal nerve (somatosensory), which serves the majority of the perineum. The sacral splanchnic nerves are postganglionic sympathetic fi- bers from the sacral sympathetic chain, and the pelvic splanchnic nerves carry parasympathetic fibers and sensory fibers from within the pelvis. Superior hypo- gastric nerves are mixed nerves located anterior to the sacral promontory and do not mediate pain informa- tion from the perineum. The ilioinguinal nerve pro- vides sensory innervation to the skin at the base of the penis; the scrotum; and upper, inner thigh.

A 25-year-old male is admitted to the hospital with testicular pain. Physical examination reveals a swollen and inflamed right testis. CT scan examination reveals abnormal accumulation of fluid in the cavity of the tunica vaginalis. Which of the following conditions will most accurately describe the signs observed in the patient? A. Varicocele B. Rectocele C. Cystocele D. Hydrocele E. Hypospadias

D. Hydrocele results from an excess amount of fluid within a persistent processus vaginalis. Hydro- cele can result from injury to the testis or by retention of a processus that fills with fluid in infants. The tu- nica vaginalis consists of parietal and visceral layers, the latter of which is closely attached to the testis and epididymis. The fluid buildup occurs within the cav- ity between these layers. A varicocele consists of varicosed veins of the pampiniform plexus and is as- sociated with increased venous pressure in the tes- ticular vein, followed by the accumulation and coagu- lation of venous blood.

A 71-year-old man is admitted to the orthopedic clinic with difficulties walking. The patient has a past history of polio. Physical and radiologic examinations reveal extension at the metatarsophalangeal joints with flexion of both the proximal and distal interphalangeal joints. Which of the following descriptions is most appropriate for this patient's condition? A. Hallux valgus B. Pes planus C. Hammertoes D. Claw toes E. Pes cavus

D. In claw toe, both the proximal and distal interphalangeal joints are strongly flexed, the result of muscle imbalance in the foot. With muscular imbalance, the extensors of the interphalangeal joints are overpowered by the long flexors. The metatarsophalangeal joint is extended, whereas in hammertoe it can be in a neutral position. Either hammer toe or claw toe can occur from arthritic changes. Hammertoe can affect any toe, but it most commonly affects the second toe, then the third or fourth toes. It results most commonly from wearing shoes that are too short or shoes with heels that are too high. In hammertoe, the metatarsophalangeal joint is extended, the proximal interphalangeal joint is flexed, and the distal phalanx may be dorsiflexed, or it may point downward, looking like a hammer. Hammertoe can occur as a result of a bunion. Calluses, or painful corns, can form on the dorsal surface of the joints. Hallux valgus is more commonly referred to as a bunion. The big toe is angulated toward the little toe and may override the second toe. The base of the first metatarsal bone is directed medially and is subject, painfully, to compression. Pes cavus is the opposite of flat foot; the patient has a high, flexed plantar arch. Pes cavus occurs as a result of hereditary motor and sensory neural problems. It is painful because of metatarsal compression.

A 34-year-old male distance runner visits the outpatient clinic with a complaint of pain he has suffered in his foot for the past week. The clinical examination indicates that the patient has an inflammation of the tough band of tissue stretching from the calcaneus to the ball of the foot. Which of the following conditions is most characteristic of these symptoms? A. Morton's neuroma B. Ankle eversion sprain C. Tarsal tunnel syndrome D. Plantar fasciitis E. Inversion sprain of the ankle

D. Inflammation of the plantar aponeurosis is referred to as plantar fasciitis. Plantar fasciitis is a common clinical condition that results from tearing or inflammation of the tough band of tissue stretching from the calcaneus to the ball of the foot (the plantar aponeurosis). It happens frequently to people who are on their feet all day, such as mail carriers, or engaged in athletics, especially in running and jumping. The pain of plantar fasciitis is usually most significant in the morning, just after you get up from bed and begin to walk. Rest, orthotics, night splints, and antiinflammatory medications are employed in treatment. A Morton's neuroma is a painful lesion of the neural interconnection of the medial and lateral plantar nerves between the third and fourth toes. An eversion sprain of the ankle can break the medial malleolus or tear the deltoid ligament. An inversion sprain commonly injures the fibulocalcaneal ligament or anterior talofibular ligament.

A 55-year-old woman is bitten by a dog in the dorsum of the foot and is admitted to the emergency department. The wound is cleaned thoroughly, during which it is seen that no tendons have been cut, but the dorsalis pedis artery and the accompanying nerve have been injured. Which of the following conditions would be expected during physical examination? A. Clubfoot B. Foot drop C. Inability to extend the big toe D. Numbness between the first and second toes E. Weakness in inversion of the foot

D. Injury to the dorsalis pedis artery on the dorsum of the foot can also cause trauma to the terminal portion of the deep fibular (peroneal) nerve. In the proximal part of the foot, this could result in loss of sensation between the first and second toes and paralysis of the extensor digitorum brevis and the extensor hallucis brevis muscles. In the distal part of the foot, only the sensory loss might be apparent. Clubfoot is a congenital malformation observed in pediatric patients. This syndrome combines plantar flexion, inversion, and adduction of the foot. Neither extension of the big toe by the extensor hallucis longus nor paralysis of the tibialis anterior (weakness of foot inversion) would occur by this injury because both of these muscles are innervated by the deep fibular (peroneal) nerve much more proximally in the leg

A 63-year-old male is admitted to the hospital with ascites, rectal bleeding, and an enlarged cirrhotic liver. He is overweight and has a history of alcohol abuse. Upon clinical examination it is found he is suffering from internal hemorrhoids. Which of the following best describes the nerves containing the pain fibers from his hemorrhoids? A. The pain fibers are carried by the inferior rectal nerve. B. The pain fibers are carried by the perineal nerve. C. The pain fibers are carried by the obturator nerve. D. The patient would not experience pain because this area is innervated by visceral afferent fibers. E. The patient would not experience well-localized pain because this area is innervated by pelvic splanchnic nerves.

D. Internal hemorrhoids are located above the pectinate line. This tissue is derived from the hindgut and innervated by visceral nerves. Pain is usually not a symptom of internal hemorrhoids.

A 32-year-old man visits the outpatient clinic and says he has had a painless mass at his right scrotum for several months. Ultrasonographic examination reveals a homogeneous hypoechoic intratesticular mass. Biopsy reveals a seminoma. Cancer of the testis metastasizes first to which of the following lymph nodes? A. Deep inguinal B. External iliac C. Internal iliac D. Lumbar E. Superficial inguinal

D. Lymph vessels from the testicles follow the path of the testicular blood supply (abdominal aorta), and therefore lymph from the testicles drains into the lumbar nodes. The superficial inguinal lymph nodes drain lymph from the lower limb. The deep inguinal nodes drain lymph from the superficial inguinal nodes, the glans of the penis, and spongy urethra. The external iliac nodes drain lymph from anterosu- perior pelvic structures and receive lymph from the deep inguinal nodes. Internal iliac nodes drain lymph from inferior pelvic structures and receive lymph from the sacral nodes.

During meiosis, pairing of homologous chromosomes occurs, which permits large segments of DNA to be exchanged. What is this process called? (A) Synapsis (B) Nondisjunction (C) Alignment (D) Crossing over (E) Disjunction

D. Synapsis (pairing of homologous chromosomes) is a unique event that occurs only during meiosis I in the production of gametes. Synapsis is necessary so that crossing over, in which large segments of DNA are exchanged, can occur.

A 48-year-old man comes to the physician because of severe back pain for 2 days. The pain radiates down to the buttock, posterior thigh, and posterolateral leg. He also has numbness on the lateral side of his left foot. On physical examination sensation to pain is decreased over the lateral side of the left foot. Deep tendon reflexes are absent at the left ankle, and there is a weakness of dorsiflexion of the left foot. Compression of which of the following nerve roots is the most likely cause of these findings? A. T12 B. L2 C. L4 D. S1 E. S3

D. The S1 nerve root provides cutaneous innervation to the lateral aspect of the ankle, the lateral sides of the dorsum and sole of foot, and motor innervation to the gastrocnemius muscle, which plantar flexes the foot and contracts during the ankle jerk reflex. It receives its innervation from the S1, S2 nerve roots via the tibial nerve, making D the correct choice. T12 roots do not reach the foot; L2 roots will reach the hip region and thigh; L4 innervates the invertors of the foot and skin over medial leg, ankle, and side of foot; and S3 innervates the sitting area of the buttocks, posterior scrotum or labia, and the small muscles of the foot.

A 62-year-old man is admitted to the emergency department. Radiologic examination and the available data indicate the likelihood of a transient ischemic attack. During physical examination the ankle jerk reflex is absent. Which of the following nerves is most likely responsible for the reflex arc? A. Common fibular (peroneal) B. Superficial fibular (peroneal) C. Deep fibular (peroneal) D. Tibial E. Superficial and deep fibular (peroneal)

D. The ankle jerk reflex, elicited by tapping the tendo Achilles with the reflex hammer, is mediated by the tibial nerve. The superficial fibular (peroneal) nerve supplies the foot evertor muscles of the lateral compartment of the leg and provides sensory supply for the dorsum of the foot. The deep fibular (peroneal) nerve innervates the foot extensor and invertor muscles in the anterior compartment of the leg and supplies skin between the first and second toes. The common fibular (peroneal) nerve combines the functions of the superficial and deep branches. The medial plantar nerve innervates the abductor and flexor muscles of the big toe, the first lumbrical muscle, and flexor digitorum brevis muscle and provides sensation for the medial plantar surface and three and a half toes. GAS

A 22-year-old male professional football player is admitted to the emergency department with acute right knee pain after sustaining a kick injury to an extended leg. A radiograph and a subsequent MRI revealed that the trauma caused anterior displacement of the tibia with respect to her femur. Which of the following ligaments was most likely injured? A. Fibular (lateral) collateral B. Tibial (medial) collateral C. Patellar D. Anterior cruciate E. Posterior cruciate F. Oblique popliteal

D. The anterior cruciate ligament is attached to the anterior intercondylar area and the posterior part of the medial surface of the lateral femoral condyle. Posterior displacement of the femur on the tibia is prevented by the ACL. With the knee joint flexed, the ACL prevents the tibia from being pulled anteriorly. The posterior cruciate ligament is stronger, shorter, and broader, less oblique, and prevents anterior displacement of the femur on the tibia. Lateral collateral ligaments are cordlike and are attached proximally to the lateral side of the head of the fibula overlapped by the tendon of biceps femoris. The medial collateral ligament is a flat band and is attached above to the medial condyle of the femur and below to the medial surface of the shaft of the tibia. It is firmly attached to the edge of the medial meniscus and consequently is more prone to be injured. The oblique popliteal ligament is a tendinous expansion derived from the semimembranosus muscle. It strengthens the posterior aspect of the knee joint capsule. The patellar ligament (tendon) connects the lower border of the patella with the smooth convexity on the tuberosity of the tibia. It represents the continuation of the quadriceps tendon.

A 42-year-old woman is admitted to the emergency department because of pelvic discomfort. During physical examination the gynecologist discovers that the patient has suffered complete uterine prolapse. Which of the following ligaments provides direct support to the uterus? A. Mesosalpinx and mesometrium B. Infundibulopelvic ligament C. Round ligament of the uterus D. Lateral cervical (cardinal) ligament E. Broad ligament of the uterus

D. The cardinal ligament, also known as Mackenrodt's ligament or transverse cervical ligament, is composed of condensations of fibromuscular tissues that accompany the uterine vessels. These bands of pelvic fascia provide direct support to the uterus. The other ligaments listed do not play a direct role in uterine stability

A 27-year-old man has had increasing difficulty walking and complained of an area of numbness on the dorsum of his right foot. Examination reveals a hard mass at the anterolateral aspect of his right leg just below the knee. Imaging studies reveal a large bone tumor between the fibula and tibia that is compressing a nerve, accounting for his neurological symptoms. Which of the following is the most likely description of abnormalities on neurological examination? A. Decreased/absent knee jerk reflex and decreased sensation on the medial aspect of the leg B. Weakness of flexion at the knee and decreased sensation of the plantar aspect of the foot C. Weakness of eversion at the ankle and decreased sensation between the first and second toes D. Weakness of inversion, dorsiflexion at the ankle, and decreased sensation between the first and second toes E. Weakness of plantar flexion at the ankle, weakness of toe flexion, decreased sensation of the plantar aspect of the foot

D. The deep fibular (peroneal) nerve is responsible for sensation over the first web space of the foot. Dorsiflexion and inversion of the ankle is produced by the muscles supplied by the deep fibular (peroneal) nerve. The nerves responsible for the knee jerk reflex, knee flexion, eversion, and plantar flexion are all located superior to the location of the tumor and will not be damaged. The nerve located in the space between the tibia and fibula is the deep fibular (peroneal) nerve.

Which of the following are components of the definitive chorion? (A) Extraembryonic somatic mesoderm and epiblast (B) Extraembryonic somatic mesoderm and cytotrophoblast (C) Extraembryonic somatic mesoderm and syncytiotrophoblast (D) Extraembryonic somatic mesoderm, cytotrophoblast, and syncytiotrophoblast (E) Extraembryonic visceral mesoderm, cytotrophoblast, and syncytiotrophoblast

D. The definitive chorion consists of three components: extraembryonic somatic mesoderm, cytotrophoblast, and syncytiotrophoblast. The chorion defines the chorionic cavity in which the embryoblast is suspended and is vital in the formation of the placenta.

A 55-year-old man is admitted to the hospital for an iliofemoral bypass. The operation is performed successfully and the blood flow between the iliac and femoral arteries is restored. During rehabilitation which of the following arteries should be palpated to monitor good circulation of the lower limb? A. Anterior tibial B. Deep fibular (peroneal) C. Deep plantar D. Dorsalis pedis E. Dorsal metatarsal

D. The dorsalis pedis is the continuation of the anterior tibial artery into the foot, as it passes the distal end of the tibia and the ankle joint. The pulse of the dorsalis pedis can be felt between the tendon of the extensor hallucis longus and the tendon of the extensor digitorum longus to the second toe. A strong pulse is a positive indicator of circulation through the limb. The fibular (peroneal) artery is a branch of the posterior tibial artery and passes in the calf between the flexor hallucis longus and tibialis posterior, making it difficult to palpate. The deep plantar artery, the extension of the first dorsal interosseous or lateral plantar arteries, passes deep to the aponeurotic tissues and central muscles of the foot, making palpation unlikely. The dorsal metatarsal branches of the dorsalis pedis pass under cover of the extensor digitorum longus and brevis tendons. Palpable pulses of the first or other dorsal metatarsal arteries can therefore be difficult to detect

A 58-year-old male farmer was accidentally struck with a scythe (a long, curved cutting blade) by another worker while they were cutting wheat. He was admitted to the county hospital with severe bleeding. During physical examination the doctor noted that the patient had a foot drop; sensation was present over the dorsum of the foot and the skin of the posterior calf. Which of the following nerves was injured? A. Femoral nerve B. Sciatic nerve C. Superficial fibular (peroneal) nerve D. Deep fibular (peroneal) nerve E. Common fibular (peroneal) nerve

D. The farm instrument has injured the deep fibular (peroneal) branch of the common fibular (peroneal) nerve. It is vulnerable to injury as it arises from the common fibular (peroneal) at the neck of the fibula. The muscles denervated are largely dorsiflexors of the foot; hence, foot drop and a high stepping gait can occur. Sensation on the dorsum of the foot is still present; therefore, the superficial branch is mostly or entirely intact, although sensation between the first and second toes would be absent. Femoral nerve injury would result in loss of knee extension. Loss of the sciatic nerve would result in loss of both the tibial and common fibular (peroneal) nerves. Because plantar flexion is still functional, the tibial nerve has not been cut.

During a battle, a 19-year-old soldier is shot in the lateral aspect of the right foot by a bullet that ricocheted off a building. The soldier is taken to a field hospital. A radiograph of the foot reveals that the base of the fifth metatarsal was completely obliterated. Which of the following muscles is most likely affected by this injury? A. Tibialis anterior B. Fibularis (peroneus) longus C. Gastrocnemius D. Fibularis (peroneus) brevis E. Extensor hallucis longus

D. The fibularis (peroneal) brevis muscle originates from the lateral lower two thirds of the shaft of the fibula and inserts on the tubercle at the base of the fifth metatarsal. Any injury to this area will affect this muscle. Patients will present with a weakness in the eversion of the foot. Fibularis (peroneus) longus, extensor hallucis longus and tibialis anterior all insert on the medial side of the foot and will not be affected in this patient. The gastrocnemius inserts via the Achilles tendon to the posterior surface of the calcaneus.

A 22-year-old woman is admitted to the emergency department after another vehicle collided with the passenger side of the convertible in which she was riding. Radiologic examination reveals an avulsion fracture of the greater trochanter. Which of the following muscles would continue to function normally if such an injury was incurred? A. Piriformis B. Obturator internus C. Gluteus medius D. Gluteus maximus E. Gluteus minimus

D. The gluteus maximus inserts into the gluteal tuberosity and the iliotibial tract. Although the gluteus maximus would continue to contract at the regions of insertion, their orientation would be displaced by the fracture. The gluteus medius, gluteus minimus, obturator internus, and piriformis all insert on some aspect of the greater trochanter of the femur

A 49-year-old man underwent a coronary bypass graft procedure using the great saphenous vein. Postoperatively, the patient complains of pain and general lack of normal sensation on the medial surface of the leg and foot on the limb from which the graft was harvested. Which nerve was most likely injured during surgery? A. Common fibular (peroneal) B. Superficial fibular (peroneal) C. Lateral sural D. Saphenous E. Tibial

D. The great saphenous vein is commonly used in coronary artery bypass grafts. Because branches of the saphenous nerve cross the vein in the distal part of the leg, the nerve can be damaged if the vein is stripped from the ankle to the knee. Stripping the vein in the opposite direction can protect the nerve and lessen the postoperative discomfort of patients. The saphenous nerve is responsible for cutaneous innervations on the medial surface of the leg and the medial side of the foot. Injury to this nerve will result in a loss of sensation and also can create chronic dysesthesias in the area. The common fibular (peroneal) nerve bifurcates at the neck of the fibula into the superficial and deep fibular (peroneal) nerves, which continue on to innervate the lateral and anterior compartments of the leg, respectively. These nerves are lateral and therefore not associated with the great saphenous vein. The lateral sural nerve is a cutaneous nerve that arises from the junction of branches from the common fibular (peroneal) nerve and tibial nerve and innervates the skin on the posterior aspect of the leg and lateral side of the foot. This nerve is often harvested for nerve grafts elsewhere in the body. The tibial nerve is a terminal branch of the sciatic nerve that continues deep in the posterior compartment of the leg.

A 29-year-old construction worker falls onto some rusty wire mesh and suffers a deep laceration to his right buttock. When the ambulance arrives to transport him to the emergency department and it is noted that he has difficulty stepping up into the ambulance with his right leg. Which nerve has probably been damaged? A. Superior gluteal B. Tibial C. Common fibular (peroneal) D. Inferior gluteal E. Nerve to piriformis

D. The inferior gluteal nerve supplies the gluteus maximus muscle, which extends and laterally rotates the hip joint; through the iliotibial tract, it also extends the knee joint. The superior gluteal nerve supplies the gluteus medius and minimus and tensor fasciae latae muscles, which work together as medial rotators of lower limb. The nerve to the piriformis supplies the piriformis muscle, which laterally rotates the femur with hip extension and abducts the femur with hip flexion.

A 34-year-old woman is admitted to the hospital with perineal pain. Laboratory blood tests reveal marked leukocytosis. Physical examination reveals perforation of the wall of the anal canal at the level of the anal valves. A horseshoe abscess extends from one ischioanal fossa to the other in the posterior recess. Which of the following nerves will most likely need to be anesthetized? A. Dorsal nerve to the clitoris B. Superficial perineal branch of perineal nerve C. Perineal nerve D. Inferior rectal nerve E. Pudendal nerve

D. The inferior rectal nerve supplies the external anal sphincter muscle and the skin around the anus. Therefore this would be the best nerve to anesthetize for abscess drainage in this area. The dorsal nerve to the clitoris does not innervate the posterior recess of the perineum. The superficial perineal branch of the perineal nerve supplies the labia majora and would not need to be anesthetized in the event of a horse- shoe anal abscess. The perineal nerve supplies all the perineal muscles and the labia majora, but for the area in question it does not have as direct a supply as the inferior rectal nerve. The pudendal nerve gives off all the branches above and thus anesthetizing it would result in additional unwanted effects.

A 65-year-old male with a history of heavy smoking and hypercholesterolemia is diagnosed with severe atherosclerosis, affecting most of the arteries of his body. During the taking of the patient's history he com- plains also of impotence. Occlusion of which of the following arteries would most likely be the cause of this condition? A. External iliac B. Inferior epigastric C. Umbilical D. Internal pudendal E. Superficial and deep circumflex

D. The internal pudendal artery gives rise to both the dorsal artery and deep artery of the penis. The deep artery is the main supply for erectile tissue; therefore, significant atherosclerosis of the internal pudendal ar- tery may result in impotence (erectile dysfunction).

A 27-year-old woman had suffered a penetrating injury in the popliteal region by an object thrown from a riding lawnmower. She was admitted to the emergency department for removal of the foreign object. After making a midline incision in the skin of the popliteal fossa, the surgical resident observed a vein of moderate size in the superficial tissues. What vein would be expected at this location? A. Popliteal vein B. Perforating tributary to the deep femoral vein C. Great saphenous vein D. Lesser (short) saphenous vein E. Superior medial genicular vein

D. The lesser (short) saphenous vein ascends up the middle of the calf from beneath the lateral malleolus, most commonly terminating at the popliteal fossa by piercing the deep fascia and joining the popliteal vein. The popliteal vein is the most superficial of major structures deep to the deep popliteal fascia. The perforating tributaries of the deep femoral vein drain to the deep femoral vein of the posterior compartment of the thigh, thereafter into the femoral vein. The superior medial genicular vein is a tributary to the popliteal vein.

A 42-year-old mother of three children visits the outpatient clinic complaining that her youngest son cannot walk yet even though he is 4 years old. Radiologic and physical examinations reveal an unstable hip joint. Which of the following ligaments is responsible for stabilization of the hip joint in childhood? A. Iliofemoral B. Pubofemoral C. Ischiofemoral D. Ligament of the head of the femur E. Transverse acetabular ligament

D. The ligament of the head of the femur conveys a small blood vessel for supply of the head of the femur (primarily in childhood). The ligament is stretched during abduction and lateral rotation of the hip joint and has an important role in stabilizing an infant's hip joint before walking. It has the potential to increase stability of the joint in hip reconstruction in developmental hip dysplasia in the pediatric population. The strength of this ligament is comparable to the anterior cruciate ligament of the knee. The iliofemoral ligament (the inverted "Y-shaped ligament of Bigelow") on the anterior aspect of the hip bone resists hyperextension of the hip joint. The pubofemoral ligament arises from the pubic bone and is located on the inferior side of the hip joint; it resists abduction of the joint. The ischiofemoral ligament is a triangular band of strong fibers that arises from the ischium and winds upward and laterally over the femoral neck, strengthening the capsule posteriorly. The transverse acetabular ligament attaches to the margins of the acetabular notch and provides origin for the ligament of the head of the femur. The transverse acetabular ligament is fibrous, not cartilaginous, but is regarded as part of the acetabular labrum.

A 42-year-old woman is admitted to the hospital with a mass on her right ovary. An ovariectomy is performed and the lymphatics of the lateral pelvic wall are also removed. Four days postoperatively the patient complains of painful spasms of the adductor muscles of the thigh and sensory deficit in the distal medial thigh. Which of the following nerves is most likely injured? A. Genitofemoral B. Ilioinguinal C. Iliohypogastric D. Obturator E. Lumbosacral trunk

D. The obturator nerve innervates the major adductors of the thigh and the skin on the medial aspect of the distal thigh. Damage to the obturator nerve is the most likely cause for the sensory and motor deficit experienced by the patient. The genitofemoral nerve is motor to the cremaster muscle and sensory to the skin over the femoral triangle. The ilioinguinal nerve innervates the skin over the labium majus and upper, inner thigh. The iliohypogastric nerve supplies skin over the anterolateral gluteal region and a strip to the area above the pubis. The lumbosacral trunk contains motor and sensory fibers from L4 and L5 and is the lumbar contribution to the lumbosacral plexus.

After dividing the overlying superficial tissues and gluteal musculature in a 68-year-old female patient, the orthopedic surgeon carefully identified the underlying structures while performing a total hip arthroplasty. The key landmark in the gluteal region, relied upon in surgical explorations of this area, is provided by which of the following structures? A. Gluteus medius B. Obturator internus tendon C. Sciatic nerve D. Piriformis muscle E. Spine of the ischium

D. The piriformis muscle arises from the pelvic surface of the sacrum, passes through the greater sciatic notch, and inserts at the greater trochanter. It is considered the "anatomical key" to gluteal anatomy; the greater sciatic foramen is the "door." The gluteus medius lies posterior to the piriformis. The sciatic nerve emerges from the greater sciatic foramen, normally through the infrapiriform's space. The spine of the ischium separates the greater and lesser sciatic foramina.

A 49-year-old man is admitted to the emergency department with a cold and pale foot. Physical examination reveals that the patient suffers from peripheral vascular disease; duplex ultrasound studies indicate possible occlusion of his popliteal artery, and the pulse of the posterior tibial artery is absent. What is the most common location for palpation of the pulse of the posterior tibial artery? A. Lateral to the muscular belly of the abductor hallucis B. Posteroinferior to the medial femoral condyle C. Groove midway between the lateral malleolus and the calcaneus D. Groove midway between the medial malleolus and the calcaneus E. Medially, between the two heads of the gastrocnemius

D. The posterior tibial artery passes under the medial malleolus, about halfway between that bony landmark and the heel, or the calcaneus. The medial edge of the plantar aponeurosis can be palpated just medial to the muscular belly of the abductor hallucis. The sural nerve and the short (lesser) saphenous vein pass around the lateral side of the foot, about halfway between the lateral malleolus and the calcaneus. The sartorius passes behind the medial femoral condyle to insert on the proximal, medial aspect of the tibia via the pes anserinus; usually no pulse can be felt clearly there. The popliteal artery passes between the two heads of the gastrocnemius, where the arterial pulse may be felt very deeply, medial to the midline.

A 31-year-old woman presents to the department of surgery with a complaint of facial paralysis (Bell's palsy), which had appeared a year earlier and had resulted in paralysis of muscles of one side of her face. The chief of plastic surgery recommends a nerve graft, taking a cutaneous nerve from the lower limb to replace the defective facial nerve. The surgery is successful. Six months after the procedure, there is restoration of function of previously paralyzed facial muscles. There is an area of skin on the back of the leg laterally and also on the lateral side of the foot that has no sensation. What nerve was used in the grafting procedure? A. Superficial fibular (peroneal) B. Tibial C. Common fibular (peroneal) D. Sural E. Saphenous

D. The sural nerve is formed by contributions from the tibial nerve and a branch from the common fibular (peroneal) nerve. It provides sensation for the lower lateral portion of the calf and continues beneath the lateral malleolus as the lateral cutaneous nerve of the foot. It is often used for nerve grafting procedures as well as biopsied for diagnostic purposes. When it is grafted to the "living end" of a cut motor or sensory nerve, the severed nerve processes within the "living" nerve grow into the sural nerve sheath, using it as a guide to the distal, surgically anastomosed nerve. Thus, axons from a branch of a functional motor nerve can grow to reinnervate paralyzed muscles. In this case, the surgeon would connect portions of the sural nerve to the functional facial nerve, tunnel it to the opposite side of the face, and join it surgically to the branches of the paralyzed nerve, where it would grow through the now empty nerve sheaths (due to Wallerian degeneration) to the muscles. Growth and reinnervation usually occur at a rate of 1 mm/day (or 1 inch/month) so the time estimated before reinnervation is based on the distance the regenerating fibers need to traverse. The tibial nerve supplies muscles and sensation to the calf and plantar surface of the foot. The common fibular (peroneal) nerve innervates the lateral and anterior compartment muscles and sensation to the dorsum of the foot. The saphenous nerve accompanies the great saphenous vein on the medial side of the leg and foot.

In the radiographs of the knee of a male 28-year-old basketball player, who had apparently suffered a tear in a medial ligament of the knee, the tubercle on the superior aspect of the medial femoral condyle could be seen more clearly than in most individuals. What muscle attaches to this tubercle? A. Semimembranosus B. Gracilis C. Popliteus D. Adductor magnus E. Vastus medialis

D. The tendinous distal portion of the adductor magnus inserts on the adductor tubercle on the upper border of the medial condyle of the femur. The femoral artery passes through the adductor hiatus proximal to this tendinous band, continuing as the popliteal artery. The semimembranosus inserts on the proximal, posterior portion of the tibia. The gracilis inserts with the pes anserinus on the proximal, medial aspect of the tibia. The popliteus inserts on the distal lateral portion of the femur, just above the origin of the lateral head of gastrocnemius. The vastus medialis inserts with other quadriceps muscle components on the patella and then on to the tibial tuberosity.

The swollen and painful left foot of a 23-year-old female long distance runner is examined in the university orthopedic clinic. She states that she stepped on an unseen sharp object while running through the park several days earlier. Emergency surgery is ordered to deal with her tarsal tunnel syndrome. The tarsal tunnel is occupied normally by tendons, vessels, and nerves that pass beneath a very strong band of tissue (flexor retinaculum) on the medial side of the ankle. What is the most anterior of the structures that pass through this tunnel? A. Flexor hallucis longus tendon B. Plantaris tendon C. Tibialis anterior tendon D. Tibialis posterior tendon E. Tibial nerve

D. The tibialis posterior tendon is the most anterior of the structures that pass under the laciniate ligament (flexor retinaculum) on the medial side of the ankle to enter the sole of the foot. Increases of pressure within the tissues of the plantar aspect of the foot, usually due to increased fluid from hemorrhage, inflammatory processes, or infections, cause tarsal tunnel syndrome, comparable to carpal tunnel syndrome of the hand. The plantar aponeurosis and other fibrous and osseous tissues of the plantar surface cause this area to be relatively nondistensible; therefore, it takes little increase of fluid content to result in pressures adequate to restrict venous drainage and, thereafter, arterial inflow to the region. Fasciotomy of the medial skin and fascia of the foot and the posterior compartment of the leg can be required to reduce the pressure and allow healing to take place. The structures that pass beneath the flexor retinaculum are, from anterior to posterior: Tendon of tibialis posterior; tendon of flexor Digitorum longus; posterior tibial Vessels and Nerve; tendon of flexor Hallucis longus. (This is the basis of the mnemonic: "Tom, Dick, and a Very Nervous Harry.") Neither the plantaris tendon nor the tibialis anterior tendon pass through this canal.

A 24-year-old female motocross racer was involved in a crash that left her right leg pinned under her bike. After the accident, she could no longer extend her right knee. The affected nerve gives rise to which of the following cutaneous nerves? A. Middle clunial nerve B. Lateral femoral cutaneous branch C. Superficial fibular (peroneal) nerve D. Saphenous nerve E. Deep fibular (peroneal)

D: The saphenous nerve is a cutaneous branch of the femoral nerve and supplies the skin on the medial leg along the great saphenous vein. Middle clunial is a posterior division of S1 to S3. Lateral femoral cutaneous is from L2 to L3 and the lumbar plexus but is not a femoral nerve branch. Superficial fibular (peroneal) nerve is a branch from the common fibular (peroneal) nerve.

Tracheoesophageal septum

During early development of the respiratory system, the laryngotracheal tube maintains communication with the primitive foregut. Which of the following embryonic structures is most likely responsible for partitioning these two embryonic structures? -Tracheoesophageal folds -Tracheoesophageal fistula -Tracheoesophageal septum -Laryngotracheal diverticulum -Laryngotracheal septum

102 A 17-year-old female student of martial arts entered the emergency department with a complaint of pain in her hand. Patient history reveals that she had been breaking concrete blocks with her hand. Examination reveals that the patient has weak abduction and adduction of her fi ngers but has no diffi culty in fl exing them. The patient also has decreased sensation over the palmar surfaces of the fourth and fi fth digits. Which of the following best describes the nature of her injury? ⃣ A. Compression of the median nerve in the carpal tunnel ⃣ B. Fracture of the triquetrum, with injury to the dorsal ulnar nerve ⃣ C. Dislocation of a bone in the proximal row of the carpus ⃣ D. Fracture of the shaft of the fi fth metacarpal ⃣ E. Injury of the ulnar nerve in Guyon's canal

E

103 A 10-year-old male suffered a dog bite that entered the common flexor synovial sheath of his forearm. He was admitted to the hospital, where the wound was cleaned and dressed and he was treated further with rabies antiserum. Two days later the boy was suffering from an elevated temperature, and his palm and one digit were obviously swollen, causing him to cry with pain. Into which of the digits could the infection spread most easily, following the anatomy of the typical common flexor sheath? A. First B. Second C. Third D. Fourth E. Fifth

E

131 A 22-year-old woman is admitted to the hospital after falling from a tree. Radiographic examination reveals fractured pisiform and hamate bones. Which of the following nerves will most likely be injured? ⃣ A. Median ⃣ B. Recurrent median ⃣ C. Radial ⃣ D. Anterior interosseous ⃣ E. Deep ulnar

E

142 A 34-year-old man visits the outpatient clinic with a painful upper limb after a fall onto a concrete fl oor. Physical examination reveals that the patient has weak abduction and adduction of his fi ngers but has no diffi culty in fl exing them. The patient also has decreased sensation over the palmar surface of the fourth and fi fth fi ngers. Which of the following diagnoses is most likely? ⃣ A. Compression of the median nerve in the carpal tunnel ⃣ B. Injury of the radial nerve from fractured humerus in the radial tuberosity ⃣ C. Compression of the median nerve as it passes between the two heads of the pronator teres ⃣ D. Compression of the radial nerve from the supinator ⃣ E. Injury of the ulnar nerve by a fractured pisiform

E

15. A 26-year-old heavyweight boxer was punched on his mandible, resulting in a slight subluxation (dislocation) of the atlantoaxial joint. The consequence of the injury was decreased range of motion at that joint. What movement would be most affected? (A) Extension (B) Flexion (C) Abduction (D) Adduction (E) Rotation

E

18. A 69-year-old man has an abnormally increased curvature of the thoracic vertebral column. Which of the following conditions is the most likely diagnosis? (A) Lordosis (B) Spina bifida occulta (C) Meningocele (D) Meningomyelocele (E) Kyphosis

E

26. When the internal vertebral venous plexus is ruptured, venous blood may spread into which tissue and space?

E

27 Following several days of 12-hour daily rehearsals of the symphony orchestra for a performance of a Wagnerian opera, the 52-year-old male conductor experienced such excruciating pain in the posterior aspect of his right forearm that he could no longer direct the musicians. When the maestro's forearm was palpated 2 cm distal to, and posteromedial to, the lateral epicondyle, the resulting excruciating pain caused the conductor to weep. Injections of steroids and rest were recommended to ease the pain. Which of the following injuries is most likely? ⃣ A. Compression of the ulnar nerve by the fl exor carpi ulnaris ⃣ B. Compression of the median nerve by the pronator teres ⃣ C. Compression of the median nerve by the fl exor digitorum superfi cialis ⃣ D. Compression of the superfi cial radial nerve by the brachioradialis ⃣ E. Compression of the deep radial nerve by the supinator

E

31 A 45-year-old patient had fallen upon his outstretched hand, resulting in a Smith fracture of the distal end of the radius. The fractured bone displaced a carpal bone in the palmar direction, resulting in nerve compression within the carpal tunnel. Which of the following carpal bones will most likely be dislocated? ⃣ A. Scaphoid ⃣ B. Trapezium ⃣ C. Capitate ⃣ D. Hamate ⃣ E. Lunate

E

33 As she fell from the uneven parallel bars, the 17year-old female gymnast grasped the lower bar briefl y with one hand but then fell painfully to the fl oor. An MRI examination reveals an injury to the medial cord of the brachial plexus. Which of the following spinal nerve levels would most likely be affected? ⃣ A. C5, C6 ⃣ B. C6, C7 ⃣ C. C7, C8 ⃣ D. C7, C8, T1 ⃣ E. C8, T1

E

52 A 17-year-old male suffered the most common of fractures of the carpal bones when he fell on his outstretched hand. Which bone would this be? ⃣ A. Trapezium ⃣ B. Lunate ⃣ C. Pisiform ⃣ D. Hamate ⃣ E. Scaphoid

E

53 A 54-year-old male cotton farmer visits the outpatient clinic because of a penetrating injury to his forearm with a baling hook. After the limb is anesthetized, the site of the wound is opened and fl ushed thoroughly to remove all debris. The patient is not able to oppose the tip of the thumb to the tip of the index fi nger, as in making the OK sign. He is able to touch the tips of the ring and little fi ngers to the pad of his thumb. What nerve has most likely been injured? ⃣ A. Median ⃣ B. Posterior interosseous ⃣ C. Radial ⃣ D. Recurrent median ⃣ E. Anterior interosseous

E

55 The orthopedic surgeon exposed the muscle in the supraspinous fossa so that she could move it laterally, in repair of an injured rotator cuff. As she refl ected the muscle from its bed, an artery was exposed crossing the ligament that bridges the notch in the superior border of the scapula. What artery was this? ⃣ A. Subscapular ⃣ B. Transverse cervical ⃣ C. Dorsal scapular ⃣ D. Posterior humeral circumfl ex ⃣ E. Suprascapular

E

60 A 55-year-old male metallurgist had been diagnosed with carpal tunnel syndrome. To begin the operation, an anesthetic injection into his axillary sheath was used instead of a general anesthesia. From which of the following structures does the axillary sheath take origin? ⃣ A. Superfi cial fascia of the neck ⃣ B. Superfi cial cervical investing fascia ⃣ C. Buccopharyngeal fascia ⃣ D. Clavipectoral fascia ⃣ E. Prevertebral fascia

E

72 Fine motor function in the right hand of a 14year-old female with scoliosis since birth appeared to be quite reduced, including thumb opposition, abduction and adduction of the digits, and interphalangeal joint extension. Radiography confirmed that her severe scoliosis was causing marked elevation of the right first rib. Long flexor muscles of the hand and long extensors of the wrist appear to be functioning within normal limits. There is notable anesthesia of the skin on the medial side of the forearm; otherwise, sensory function in the limb is intact. Which of the following neural structures is most likely impaired? A. Median nerve B. Middle trunk of the brachial plexus C. Radial nerve D. Lower trunk of the brachial plexus E. T1 nerve root

E

77 A 27-year-old male had lost much of the soft tissue on the dorsum of his left hand in a motorcycle crash. Imaging studies show no other upper limb injuries. Because the left extensor carpi radialis longus and brevis tendons were lost, it was decided to replace those tendons with the palmaris longus tendons from both forearms because of those tendons' convenient location and relative unimportance. Postoperatively it is found that sensation is absent in both hands on the lateral palm and palmar surfaces of the fi rst three digits; there is also paralysis of thumb opposition. What is the most likely cause of the sensory defi cit and motor loss in both thumbs? ⃣ A. Bilateral loss of spinal nerve T1 with fractures of fi rst rib bilaterally ⃣ B. Lower plexus (lower trunk) trauma ⃣ C. Dupuytren contracture ⃣ D. Left radial nerve injury in the posterior compartment of the forearm ⃣ E. The palmaris longus was absent bilaterally; the nerve beneath it looked like a tendon.

E

80 It was reported by the sports media that the outstanding 27-year-old shortstop for the New York team would miss a number of baseball games. He was hit on a fingertip while attempting to catch a ball barehanded. A tendon had been torn. The team doctor commented that the ballplayer could not straighten the last joint of the long finger of his right hand, and the finger would require surgery. From what injury did the ballplayer suffer? ⃣ A. Clawhand deformity ⃣ B. Boutonnière deformity ⃣ C. Swan-neck deformity ⃣ D. Dupuytren contracture ⃣ E. Mallet finger

E

98 A 5-year-old boy is admitted to the emergency department after falling from a tree. The parents are informed by the radiologist that their son's fracture is the most common fracture that occurs in children. Which of the following bones was broken? A. Humerus B. Radius C. Ulna D. Scaphoid E. Clavicle

E

A 36-year-old man is brought to the emergency department because of a deep knife wound on the medial side of his distal forearm. He is unable to hold a piece of paper between his fingers and has sensory loss on the medial side of his hand and little finger. Which nerve is most likely injured? A. Axillary B. Median C. Musculocutaneous D. Radial E. Ulnar

E

A 47-year-old female tennis professional is informed by her physician that she has a rotator cuff injury that will require surgery. Her physician explains that over the years of play a shoulder ligament has gradually caused severe damage to the underlying muscle. To which of the following ligaments is the physician most likely referring? ⃣ A. Acromioclavicular ligament ⃣ B. Coracohumeral ligament ⃣ C. Transverse scapular ligament ⃣ D. Glenohumeral ligament ⃣ E. Coracoacromial ligament

E

After a forceps delivery of a male infant, the baby presents with his left upper limb adducted, internally rotated, and flexed at the wrist. The startle reflex is absent on the ipsilateral side. Which part of the brachial plexus was most likely injured during this delivery? A. Lateral cord B. Medial cord C. Roots of the lower trunk D. Root of the middle trunk E. Roots of the upper trunk

E

A premature infant has progressive difficulty in breathing and is diagnosed with respiratory distress syndrome. Which cells are deficient in synthesizing surfactant in this syndrome? A. Alveolar capillary endothelial B. Bronchial mucous C. Bronchial respiratory epithelium D. Type I alveolar E. Type II alveolar

E. Type II alveolar cells are the only cells that produce surfactant.

A 5-year-old boy has frequent episodes of fatigability and dyspnea. An ultrasound examination reveals an atrial septal defect, located at the opening of the superior vena cava. Which of the following types of atrial septal defects are characteristic for this description? A. Ostium secundum B. Ostium primum C. Atrioventricular (AV) canal D. Common atrium E. Sinus venosus

E. Sinus venosus atrial septal defects occur close to the entry of the superior vena cava in the superior portion of the interatrial septum. Ostium secundum atrial septal defects are located near the fossa ovale and encompass both septum primum and septum secundum defects. An ostium primum defect is a less common form of atrial septal defect and is associated with endocardial cushion defects because the septum primum fails to fuse with the endocardial cushions, resulting in a patent foramen primum. An AV canal defect is not a clinically significant type of atrial septal defect. A common atrium is an uncommon type of atrial septal defect in which the interatrial septum is absent.

During a 100-meter sprint a 25-year-old male Olympic athlete suddenly pulls up in discomfort and is seen to be clutching the back of his left thigh in agony. Upon further examination the athlete describes the pain as a "tearing" sensation and is unable to flex his knee. Based on these symptoms which of the following actions are affected due to this injury? A. Flexion of the hip and extension of the knee B. Extension of the hip and dorsiflexion C. Medial rotation of the hip D. Lateral rotation of the hip E. Hip extension and knee flexion

E. Because the hamstrings cross two joints and are very crucial during all phases of running, but especially during the late swing through midstance phase of running, are easily injured. Their normal action includes hip extension and knee flexion. The do not rotate the hip.

A 36-year-old male rodeo rider is admitted to the hospital after being thrown violently from a Brahma bull. An MRI scan reveals rupture of the penile urethra and deep (Bucks) fascia. Where is the most likely place that extravasated urine will flow? A. Ischioanal fossa B. Rectovesical pouch C. Deep perineal pouch D. Retropubic space E. Superficial perineal cleft

E. Because the penile urethra and deep (Bucks) fascia are both located in the superficial perineal pouch, rupture will occur here, with extravasation of fluids into the superficial perineal cleft. The ischio- anal fossa is located posterior to the urogenital trian- gle, behind the area of injury. The other listed spaces are deep to the superficial compartment or within the pelvis and are not associated with the area of injury.

A 72-year-old man visits the outpatient clinic with a complaint of severe pain when walking. Physical examination reveals the problems in his feet as shown in Fig. 5-5. What is the most likely diagnosis? A. Coxa varus B. Coxa valgus C. Genu valgus D. Genu vara E. Hallux valgus

E. Hallux valgus, or lateral displacement of the great toe, usually presents as pain over the prominent metatarsal head, due to rubbing from shoes, and it can be associated with deformity of the second toe, which then tends to override the great toe. Hallux valgus is commonly known as bunion. In this deformity the big toe points toward the little toe; the base of the first metatarsal points medially, with a swollen bursal sac at the metatarsophalangeal joint. Excess bony growth of the distal protruding part of the metatarsal bone can also occur. Bunions occur only rarely in people who do not routinely wear shoes. Genu varus is also referred to as bowlegs, or bandy legs, in which the knees are bowed outward. The opposite of this is genu valgus, or knock knee. The normal angle between the femoral shaft and femoral neck is between 120 and 135 degrees. In coxa vara the angle between the shaft and neck of the femur is less than 120 degrees. This can result from fractures, other injuries, or congenital softness of the bone of the femoral neck. This defect results in limb shortening and limping. In coxa valga there is an increase in femoral shaft neck angulation, which can lead to hip subluxation or dislocation. Coxa valga results from weakness of the adductor musculature.

A 75-year-old man is admitted to the emergency department with severe pain at his right hip and thigh. An MRI examination reveals avascular necrosis of the femoral head. Which of the following arteries is most likely injured, resulting in avascular necrosis? A. Deep circumflex iliac B. Acetabular branch of obturator C. Descending branch of lateral circumflex femoral D. First perforating branch of profunda femoris E. Ascending branch of medial circumflex femoral

E. In infants and children until about the age of 8 years, the head of the femur gets its arterial supply by a direct branch of the obturator artery (variably, the medial circumflex femoral). The arterial supply reaches the head of the femur at the fovea capitis by traveling along the ligament of the head of the femur. This source of supply is replaced later by vessels such as branches of the ascending branch of the medial circumflex femoral that pass into foramina of the neck of the femur within the capsule of the hip joint. Similar branches can arise from the lateral circumflex femoral and gluteal arteries. The deep circumflex iliac artery arises from the external iliac artery and supplies branches to the ilium, the iliacus muscle, and lower portions of the abdominal wall. The acetabular branch of the obturator artery often provides the branch to the head of the femur, an artery that normally regresses early in life, so that it supplies only the immediate area of the fovea capitis. The descending branch of the lateral circumflex femoral supplies the vastus lateralis muscle and participates in anastomoses at the knee. The second perforating branch of the deep femoral artery often supplies the nutrient artery of the shaft of the femur.

Receives oxygenated blood via pulmonary veins?

E. Left atrium

A 23-year-old male basketball player injured his foot during training and is admitted to the emergency department. An MRI examination reveals a hematoma around the medial malleolus. Upon physical examination the patient shows excessive eversion of his foot. Which of the following ligaments most likely has a tear? A. Plantar calcaneonavicular (spring) B. Calcaneofibular C. Long plantar D. Short plantar E. Deltoid

E. One important function of the deltoid ligament is the prevention of excessive extension of the ankle. The ligament is so strong that excessive eversion can cause the medial malleolus to be pulled off (an avulsion fracture) rather than tearing the deltoid ligament.

After being struck from behind by a motor vehicle, a 55-year-old man presents to the hospital with a swelling of his right knee. Imaging reveals a large hematoma of the popliteal artery compressing his tibial nerve. Upon neurologic examination which movement would likely be diminished in strength? A. Dorsiflexion of the foot B. Flexion of the thigh C. Extension of the digits D. Extension of the leg E. Plantar flexion of the foot

E. Plantar flexion is mostly due to the gastrocnemius and soleus muscles, which are supplied by the tibial nerve. The tibial nerve leaves the popliteal fossa by passing deep to the gastrocnemius and soleus muscles and lies posterior to the popliteal artery. Therefore a hematoma of the popliteal artery will also compress the nerve. Dorsiflexion of the foot is due to contraction of the muscles in the anterior compartment of the leg

A 60-year-old retired male marathon runner complains to his primary care physician that during his daily morning jog he experiences bouts of numbness and tingling on the medial aspect of his heel. Upon further examination the doctor discovers the patient has trouble tiptoeing and shows a positive Tinel's sign. Which of the following conditions is most characteristic of these symptoms? A. Plantar fasciitis B. Ankle inversion sprain C. Morton's neuroma D. Lateral ligament E. Tarsal tunnel syndrome

E. Tarsal tunnel syndrome is a compression neuropathy resulting from the compression of the tibial nerve in the tarsal tunnel. The tarsal tunnel is located between the medial malleolus, the inferomedial surface of the calcaneus, and the flexor retinaculum. The contents are the tibial nerve and its plantar branches, the tendons of the tibialis posterior, flexor digitorum longus, and the flexor hallucis longus muscles together with the posterior tibial vessels. Any inflammation or swelling in the area will compress on these structures, most significantly the tibial nerve. The posterior tibial vein will be most easily compressed but the nerve is most clinically significant. Clinically, this syndrome is diagnosed with the patient's history and physical examination findings including a positive Tinel's sign (lightly tapping over the flexor retinaculum elicits numbness and tingling in the skin over the calcaneus and the sole of the foot).

A 41-year-old male visits the outpatient urology clinic for a vasectomy procedure. Two months later the patient revisits the outpatient clinic complaining of pain in his testis. The diagnosis was made for postvasectomy pain syndrome. Which of the following nerves was most likely injured? A. Sympathetic fibers to ductus deferens B. Ilioinguinal C. Iliohypogastric D. Genital branch of genitofemoral E. Visceral afferent T10-L2

E. The afferents of the testis and most of the ductus accompany sympathetics to enter the chain at T10- L2, with cell bodies in the dorsal root ganglia of those spinal nerves. (That's why a kick in the testis seems to hurt so terribly in the upper belly.) The more proximal portion of the ductus has sensory fibers in the pelvic splanchnics. The ilioinguinal nerve is somatic and in- nervates the upper and medial thigh as well as the anterior scrotum and skin at the root of the penis, not the ductus deferens. The iliohypogastric nerve is an anterior abdominal wall nerve that innervates trans- verse and oblique abdominal muscles, supplies skin above the pubis, and has cutaneous supply to the lat- eral buttocks. The genital branch of the genitofemoral nerve supplies the cremaster muscle and the scrotum.

A 72-year-old woman is admitted to the emergency department after an episode of stroke. During neurologic examination the patient shows no response to the ankle reflex test. Which of the following nerve roots is responsible for this reflex? A. L2 B. L3 C. L4 D. L5 E. S1

E. The ankle jerk reflex involves S1 and S2 levels. L2 to L4 are involved in the patellar reflex. L5 is not a component of a deep tendon reflex.

A 19-year-old woman is admitted to the emergency department with complaints of pain and swelling to the right ankle. In a recent volleyball game she jumped to spike the ball then landed on the opponent's shoe with her right foot. She recalls hearing a loud "pop" and felt immediate pain to the ankle. She was unable to bear weight subsequently. On examination, the right ankle was swollen, with maximal tenderness inferior and anterior to the lateral malleolus. Radiographs of the ankle showed no fractures. Which of the following structures were most likely injured? A. Posterior talofibular ligament B. Interosseous ligament between the tibia and fibula C. Tibionavicular ligament D. Anterior tibiofibular ligament E. Calcaneofibular ligament

E. The calcaneofibular ligament is located inferior and just anterior to the lateral malleolus and connects the lateral malleolus to the calcaneus. The interosseous ligament between the tibia and the fibula is located medially and superior to the lateral malleolus. The tibionavicular ligaments are located on the medial side of the ankle joint, and the point of injury and tenderness is at the lateral side. The anterior tibiofibular ligament is located anterior to the ankle joint, away from the point of injury.

A 45-year-old man is admitted to the emergency department after a fall and subsequent leg injury. On physical examination the patient has a foot drop but eversion is unaffected. Which nerve is most likely injured? A. Tibial B. Common fibular (peroneal) C. Superficial fibular (peroneal) D. Saphenous E. Deep fibular (peroneal)

E. The deep fibular (peroneal) nerve along with the superficial fibular (peroneal) nerve are branches of the common fibular nerve. The deep fibular (peroneal) nerve innervates muscles of the anterior compartment of the leg dorsiflexors of the foot and the skin between the great toe and second toes, while the superficial fibular nerve innervates the lateral compartment muscles of the leg, which are evertors of the foot and the skin on most of the dorsum of the foot. If the common fibular nerve were damaged all the structures that receive innervation via this nerve will be compromising dorsiflexion and eversion. Damage to the superficial fibular nerve affects the ability to evert the foot but does not result in foot drop, making the deep fibular nerve, which innervates the dorsiflexors of the foot, the best choice as the injury describes foot drop with the ability to evert the foot conserved. The saphenous nerve is a cutaneous nerve, while the tibial nerve innervates posterior compartment muscles.

A 45-year-old man is treated at the hospital after he fell from his bicycle. Radiologic examination reveals fractures both of the tibia and the fibula. On physical examination the patient has a foot drop, but normal eversion (Fig. 5-1). Which of the following nerves is most likely injured? A. Tibial B. Common fibular (peroneal) C. Superficial fibular (peroneal) D. Saphenous E. Deep fibular (peroneal)

E. The deep fibular (peroneal) nerve is responsible for innervating the muscles of the anterior compartment of the leg, which are responsible for toe extension, foot dorsiflexion, and inversion. Injury to this nerve will result in foot drop and also loss of sensation between the first and second toes. Injury to the tibial nerve affects the posterior compartment muscles of the leg, which are responsible for plantar flexion and toe flexion, as well as the intrinsic muscles of the sole of the foot. The common fibular (peroneal) nerve splits into the superficial and deep fibular (peroneal) nerves, and these supply both the lateral and anterior compartments. The superficial fibular (peroneal) nerve innervates the fibularis (peroneus) longus and brevis muscles, which provide eversion of the foot. If the common fibular (peroneal) nerve were injured, eversion of the foot and plantar flexion would be lost in addition to dorsiflexion and inversion. The saphenous nerve, a continuation of the femoral nerve, is a cutaneous nerve that supplies the medial side of the leg and foot and provides no motor innervation.

The developing embryo has a distinct human appearance by the end of (A) week 4 (B) week 5 (C) week 6 (D) week 7 (E) week 8

E. The embryo starts the embryonic period as a two-dimensional disk and ends as a three-dimensional cylinder. This dramatic change in geometry is caused by formation of all the major organ systems. As the organ systems gradually develop during the embryonic period, the embryo appears more and more human-like; it has a distinct human appearance at the end of week 8.

Between which two layers is the extraembryonic mesoderm located? (A) Epiblast and hypoblast (B) Syncytiotrophoblast and cytotrophoblast (C) Syncytiotrophoblast and endometrium (D) Exocoelomic membrane and syncytiotrophoblast (E) Exocoelomic membrane and cytotrophoblast

E. The extraembryonic mesoderm is derived from the epiblast and is located between the exocoelomic membrane and the cytotrophoblast. The overall effect is to completely separate the embryoblast from the trophoblast with the extraembryonic mesoderm serving as a conduit (connection) between them.

A 45-year-old obese woman was admitted to the hospital because of "pains in her leg." Physical examination led to a diagnosis of "meralgia paresthetica." In her condition, excessive adipose tissue bulging over the inguinal ligament exerted traction upon it, compressing a nerve that passed through, or beneath, the ligament, just medial to the anterior superior iliac spine. Which of the following nerves was most likely affected in this patient? A. Femoral branch of the genitofemoral nerve B. Femoral nerve C. Iliohypogastric nerve D. Ilioinguinal nerve E. Lateral femoral cutaneous nerve

E. The lateral femoral cutaneous nerve (L2, L3) emerges from the lateral side of the psoas muscle and runs in front of the iliacus and through, or behind, the inguinal ligament and innervates the skin of the lat- eral aspect of the thigh to the level of the knee. This nerve has been constricted in this case of "Calvin Klein syndrome" (in this case from the patient's obe- sity, not too-tight jeans) causing pain, tingling, or burning sensations in the lateral thigh. The femoral branch of the genitofemoral nerve (L1, L2) supplies a small area of skin (over the femoral triangle), just inferior to the midpoint of the inguinal ligament. The femoral nerve (L2 to L4) is motor to the quadriceps and sartorius muscles and sensory to the anterior thigh and the medial thigh and leg. The ilioinguinal supplies the suprapubic region; part of the genitalia and anterior perineum; and the upper, medial thigh. Cutaneous branches of the iliohypogastric nerve in- nervate skin of the anterolateral gluteal area and su- prapubic region.

A 29-year-old male police officer is examined in a neighborhood clinic, with a complaint of discomfort in the lateral thigh. The physician observes that the policeman is rather overweight and that he is wearing a heavy leather belt, to which numerous objects are attached, including his empty holster. After a thorough physical examination a diagnosis of meralgia paresthetica is confirmed. Which of the following nerves is most likely involved? A. Superior gluteal B. Femoral C. Obturator D. Fibular (peroneal) division of sciatic E. Lateral femoral cutaneous

E. The lateral femoral cutaneous nerve leaves the pelvis laterally, about 2 cm medial to the anterior superior iliac spine, passing beneath, or through, the inguinal ligament. As a consequence of its site of exit, any tension upon or compression of the inguinal ligament can affect the nerve. If it is thus affected, the individual may feel burning sensations or pain along the lateral aspect of the thigh, which is the region of distribution of the nerve. Obesity, sudden weight loss, wearing a heavy gun belt, wearing trousers that are too tight (Calvin Klein syndrome), or having someone sitting on another's lap for an extended period of time can lead to meralgia paresthetica, the painful lateral thigh. The femoral nerve emerges from beneath the middle of the inguinal ligament and is not usually affected by similar traction or compression. The obturator nerve leaves the pelvis through the obturator canal and enters the thigh deeply in a protected location. It innervates the adductor muscles and supplies sensation on the medial aspect of the thigh. The fibular (peroneal) division of the sciatic nerve supplies the muscles of the anterior and lateral compartments of the leg and provides sensory fibers for the dorsum and lateral side of the foot. The superior gluteal nerve provides motor supply to the gluteus medius and minimus muscles.

A 58-year-old diabetic patient is admitted to the hospital with a painful foot. Physical examination reveals that the patient suffers from peripheral vascular disease. There is no detectable dorsalis pedis arterial pulse, but the posterior tibial pulse is strong. Which of the following arteries will most likely provide adequate collateral supply from the plantar surface to the toes and dorsum of the foot? A. Anterior tibial B. Fibular (peroneal) C. Arcuate D. Medial plantar E. Lateral plantar

E. The lateral plantar artery provides origin to the deep plantar arterial arch. Medially, the vascular arch anastomoses with the distal portion of the dorsalis pedis by way of the deep plantar artery. The anterior tibial artery continues as the dorsalis pedis at the ankle joint. The fibular (peroneal) artery, by way of a perforating branch in some individuals, replaces the dorsal pedis. The arcuate artery, a branch of the dorsalis pedis, provides origin for the dorsal metatarsal arteries to the lateral toes.

A 30-year-old woman is admitted to the emergency department with complaints of pain to the anterior left thigh. While participating in a 100-meter race, she felt a sudden onset of pain in the anterior mid thigh area and could only limp to the finish line. Physical examination revealed a swollen, tender right thigh anteriorly. Extension of the knee was limited due to pain. Ultrasonography of the area revealed a defect in the fibers of the quadriceps muscle, confirmed by CT scan of the limb. Which of the following is the embryologic origin of the affected structure? A. Lateral plate mesoderm B. Dorsolateral migration of neural crest cells C. Preceded the development of chondrification centers D. Intermediate mesoderm E. Migration of cells from paraxial mesoderm

E. The muscles of the limbs develop from the myotome component of the somites. The somites are condensations of paraxial mesoderm that form after the formation of the trilaminar disc. The lateral plate mesoderm gives rise to the serous membranes, coverings of organs and the heart. The intermediate mesoderm gives rise to the urogenital system and its accessory glands. The chondrification centers precede the formation of the muscles, as it gives the skeletal framework. Neural crest cells give rise to dorsal root ganglia, leptomeninges, Schwann cells, sympathetic ganglia, and the chromaffin cells of the adrenal medulla.

A 30-year-old man who is a bodybuilder presents to the physician's office complaining of pain and tingling sensation radiating down the inside of his thigh that was exacerbated upon thigh movement. A hernia through which opening would most likely cause this presentation? A. Femoral ring B. Superficial inguinal ring C. Deep inguinal ring D. Fossa ovalis E. Obturator canal

E. The obturator membrane is a fibrous sheet that almost completely closes the obturator foramen, leaving a small gap, the obturator canal, for the passage of the obturator nerve and vessels as they leave the pelvis to enter the medial thigh. The femoral canal is the small medial compartment for the lymph vessels. It is about 0.5 in (1.3 cm) long, and its upper opening is called the femoral ring. It has following borders: anteriorly the inguinal ligament; posteriorly the superior ramus of the pubis; medially the lacunar ligament; and laterally the femoral vein. A triangular shaped defect in the external oblique aponeurosis lies immediately above and medial to the pubic tubercle. This is known as the superficial inguinal ring. The deep ring is an oval opening in the transversalis fascia and lies about 0.5 in (1.3 cm) above the inguinal ligament, midway between the anterior superior iliac spine and the pubic symphysis. Fossa ovalis, which refers to an oval opening in the superomedial part of the fascia lata of the thigh, lies 3 to 4 cm inferolateral to the pubic tubercle.

A 22-year-old male is admitted to the hospital with groin pain and blood in the semen. An MRI examination and biopsy reveal testicular cancer. Which of the following lymph nodes will be first involved in case of metastasis? A. Internal iliac B. External iliac C. Superficial inguinal D. Deep inguinal E. Paraaortic and lumbar

E. The paraaortic and lumbar nodes at the level of the kidneys will most likely be infiltrated by metas- tasis of testicular cancer because testicular lymphatics run in close association with the testicular vessels and drain the testicles and epididymis. Testicular cancer is said to be a disease that is especially dangerous for young men. The internal iliac nodes drain the inferior pelvis and deep perineal structures. The external iliac nodes drain all anterosuperior pelvic structures. The superficial inguinal nodes drain all of the superficial structures below the umbilicus. Finally, the deep in- guinal nodes receive more superficial vessels and drain the glans penis in males.

A 30-year-old woman is admitted to the emergency department with complaints of pain to the anterior left thigh. While participating in a 100-meter race, she felt a sudden onset of pain in the anterior mid thigh area and could only limp to the finish line. Physical examination revealed a swollen, tender right thigh anteriorly. Extension of the knee was limited due to pain. Ultrasonography of the area revealed a defect in the fibers of the quadriceps muscle, confirmed by CT scan of the limb. Which of the following is the embryologic origin of the affected structure? A. Lateral plate mesoderm B. Dorsolateral migration of neural crest cells C. Preceded the development of chondrification centers D. Intermediate mesoderm E. Migration of cells from paraxial mesoderm

E. The paraxial mesoderm develops into somites. Limb muscles develop from the ventral myotome of the somites in response to molecular signals. Embryological derivatives of the lateral plate mesoderm include the circulatory and gut wall, body wall lining, and dermis. Derivative of the neural crest cells does not include the limb muscles. Chondrification is associated with cartilage formation and not muscles. The intermediate mesoderm eventually thins out laterally and becomes the mesoderm, which gives the circulatory and gut walls, plus the lining of the body wall and dermis.

A 6-year-old boy has a large intraabdominal mass in the midline just above the pubic symphysis. During surgery a cystic mass is found attached to the umbilicus and to the apex of the bladder. Which of the following is the most likely diagnosis? A. Hydrocele B. Meckel cyst C. Meckel diverticulum D. Omphalocele E. Urachal cyst

E. The persistence of the epithelial lining of the urachus can give rise to a urachal cyst. This swelling is found in the midline in the umbilical region. Hydro- cele is fluid accumulation between the visceral and parietal layers of the tunica vaginalis of the testis. A Meckel diverticulum is located in the ileum of the small intestine. When it becomes inflamed, it can cause symptoms of appendicitis. A diverticulum can form a cyst (Meckel cyst). An omphalocele is the per- sistence of the herniation of the abdominal contents into the umbilical cord.

Lower limb angiography of an 82-year-old woman reveals a possible cause for her limb pain during her workout routines in the health spa. The artery that was occluded is one that should have been demonstrable passing between the proximal part of the space between the tibia and fibula. Which of the following arteries is most likely affected? A. Deep femoral B. Popliteal C. Posterior tibial D. Fibular (peroneal) E. Anterior tibial

E. The popliteal artery is the continuation of the femoral artery after it passes through the hiatus of the adductor magnus. The popliteal artery divides into the anterior and posterior tibial arteries. The anterior tibial artery passes between the tibia and fibula proximally in the posterior compartment of the leg, whereas the posterior tibial artery continues in the posterior compartment of the leg, to its division into medial and lateral plantar arteries. The posterior tibial artery provides origin for the fibular (peroneal) artery, which supplies the lateral compartment of the leg. The deep femoral artery provides origin for the three or four perforating branches that supply the posterior compartment of the thigh. GAS 600, 617; N 509;

Following an injury suffered in a soccer match, a 32-year-old woman is examined in a seated position in the orthopedic clinic. Holding the right tibia with both hands, the clinician can press the tibia backward under the distal part of her femur. The left tibia cannot be displaced in this way. Which structure was most likely damaged in the right knee? A. Anterior cruciate ligament B. Lateral collateral ligament C. Medial collateral ligament D. Medial meniscus E. Posterior cruciate ligament

E. The posterior cruciate ligament is responsible for preventing the forward sliding of the femur on the tibia. The anterior cruciate ligament prevents posterior displacement of the femur on the tibia. The lateral collateral ligament limits extension and adduction of the leg. The medial meniscus acts as a shock absorber and cushions the articular surfaces of the knee joint.

A 22-year-old soccer player collides with one of her teammates. During examination on the field, the posterior drawer test was performed and the tibia moved backward in relation to her femur. Injury to which structure is confirmed by performing this test? A. Anterior cruciate ligament B. Lateral collateral ligament C. Medial collateral ligament D. Medial meniscus E. Posterior cruciate ligament

E. The posterior cruciate ligament runs from the posterior aspect of the intercondylar area of the tibia to the medial wall of the intercondylar fossa. It prevents posterior displacement of the tibia relative to the femur. This is usually tested with the posterior drawer test, in which the physician pushes the tibia backward while the knee is flexed in an attempt to displace it posteriorly. This is called the positive posterior drawer sign. The anterior cruciate ligament prevents anterior displacement of the tibia on the femur. The medial and lateral collateral ligaments stabilize the medial and lateral sides of the knee joint, respectively. The medial meniscus is an intracapsular fibrocartilage that improves the articulation of the femur and tibia.

A 65-year-old man is admitted to the hospital after falling from his roof while cleaning leaves and pine needles from the gutters. Among other injuries suffered in his fall, radiologic examination reveals a fracture of the talus bone in one foot. Much of the blood supply of this bone can be lost in such an injury and can result in osteonecrosis. From what artery does this bone receive its primary vascular supply? A. Medial plantar B. Lateral plantar C. Dorsalis pedis D. Anterior tibial E. Posterior tibial

E. The posterior tibial artery provides most of the arterial supply for the neck and body of the talus bone. The fibular (peroneal) artery provides a small amount of vascular supply. The medial plantar and lateral plantar branches of the posterior tibial artery are distributed to tissues in the plantar surface of the foot. The dorsalis pedis is the continuation of the anterior tibial artery on the dorsum of the foot.

A 52-year-old woman is admitted to the emergency department after severely injuring her right lower limb when she fell from a trampoline. Radiologic examination reveals a trimalleolar fracture of the ankle involving the lateral malleolus, medial malleolus, and the posterior process of the tibia. Which of the following bones will also most likely be affected? A. Navicular B. Calcaneus C. Cuneiform D. Cuboid E. Talus

E. The talus can be rotated externally when the ankle sustains a trimalleolar fracture, also called a Henderson fracture. The fracture may be caused by eversion and posterior displacement of the talus. This injury involves the fracture of the distal fibula (lateral malleolus); the medial malleolus of the tibia; and the posterior portion, or lip, of the tibial plafond (the distal articular portion of the tibia, sometimes referred to as the posterior malleolus). The posterior part of the plafond is not truly a malleolus but acts this way in this type of twisting fracture of the ankle. The talus can be forced from its normal position in this fracture, adding to the instability of the ankle. The other bones listed are relatively far from the site of the fractures

A 45-year-old man presents at the local emergency clinic with the complaint of a painful knee and difficulty in walking. A computed tomography (CT) scan examination reveals a very large cyst in the popliteal fossa compressing the tibial nerve. Which movement will most likely be affected? A. Dorsiflexion of the foot B. Flexion of the thigh C. Extension of the digits D. Extension of the leg E. Plantar flexion of the foot

E. The tibial nerve is responsible for innervating the posterior compartment of the leg. These muscles are responsible for knee flexion, plantar flexion, and intrinsic muscle functions of the foot. Compression of this nerve can affect plantar flexion of the foot. Dorsiflexion of the foot would be compromised if the deep fibular (peroneal) nerve were compressed by this Baker's cyst. Flexion of the thigh is a function of muscles supplied by lumbar nerves and the femoral nerve. The deep fibular (peroneal) nerve is also responsible for extension of the digits, whereas the femoral nerve is responsible for extension of the leg.

A 43-year-old man visits the outpatient clinic with a painful, swollen knee joint. The patient's history reveals chronic gonococcal arthritis. A knee aspiration is ordered for bacterial culture of the synovial fluid. A standard suprapatellar approach is used, and the needle passes from the lateral aspect of the thigh into the region immediately proximal to and deep to the patella. Through which of the following muscles would the needle pass? A. Adductor magnus B. Short head of biceps femoris C. Rectus femoris D. Sartorius E. Vastus lateralis

E. The vastus lateralis muscle is located on the lateral aspect of the thigh. The distal portion of this muscle lies superficial to the proximal part of the lateral aspect of the joint capsule of the knee. When a needle is inserted superiorly and laterally to the patella, it penetrates the vastus lateralis muscle on its course to the internal capsule. The short head of biceps femoris has its origin on the posterior aspect of the femur, merges with the long head of the biceps femoris, and inserts on the head of the fibula. The rectus femoris passes longitudinally on the medial aspect of the femur and inserts on the tibial tuberosity, via the patellar tendon, or quadriceps tendon. A needle inserted laterally to the patella would not penetrate this muscle. The sartorius originates on the anterior superior iliac spine and forms part of the pes anserinus, which inserts on the medial aspect of the proximal part of the tibia. A needle inserted laterally to the patella would not penetrate this muscle.

C ascending aorta

Stenosis of which structure may produce left ventricular hypertrophy

D ascending aorta

Takes its origin from the left ventricle and ends at the sternal angle?

An abscess is detected between the perineal membrane and the inferior fascia of the pelvic diaphragm in a 27-year-old man. Which of the following structures would most likely be affected by this abscess within the deep perineal space? (A) Bulbourethral glands (B) Perineal body (C) Superficial transverse perineal muscles (D) Bulbospongiosus muscles (E) Ischiocavernosus muscles

The answer is A: Bulbourethral glands. Bulbourethral (Cowper) glands secrete nonviscous material into the urethra before and during ejaculation to mix with sperm, seminal vesicle fluid, and prostatic fluid to form semen. The bulbourethral glands are found deep to the perineal membrane within the deep perineal compartment, so they are the most likely structures to be affected by the described abscess within the deep perineal compartment. Choice B (Perineal body) is incorrect. The perineal body is located at the central point of the perineum where the perineal muscles converge. This unique structure serves as an anchoring point for the perineum and contains collagen and elastic fibers as well as skeletal and smooth muscles. Due to its location between the anal and urogenital triangles of the perineum, it is generally not considered part of the superficial or deep perineal compartments. Therefore, it would not be the most likely structure on this list affected by the described abscess. Choice C (Superficial transverse perineal muscles) is incorrect. The superficial transverse perineal muscles attach to the ischial tuberosities and help to support and stabilize the superficial perineal region. The muscles are, however, located in the superficial perineal space, which is inferior to the perineal membrane, so they would most likely not be affected by this abscess in the deep perineal space. Choice D (Bulbospongiosus muscles) is incorrect. The bulbospongiosus muscles envelop and com- press the bulb of the penis and corpus spongiosum to aid in penile erection by increasing pressure on erectile tissue in the root of the penis. These muscles also support and fix the male pelvic floor and play an important role in emptying the penile (spongy) urethra of residual urine or semen. However, these muscles are located in the superficial perineal space, which is inferior to the perineal membrane, so the bulbospongiosus muscles would most likely not be affected by this abscess in the deep perineal space. Choice E (Ischiocavernosus muscles) is incorrect. The ischiocavernosus muscles are paired muscles covering the crura of the penis and the associated erectile tissue, the corpora cavernosa. These muscles force blood within the crura into the distal parts of the corpora cavernosa and compress venous outflow via the deep dorsal vein of the penis. Both of these actions increase the turgidity (firm dis- tension) of the penis during erection. However, these muscles are located in the superficial perineal space, which is inferior to the perineal membrane, so the ischiocavernosus muscles would most likely not be affected by this abscess in the deep perineal space.

Which structure is derived from the same embryonic primordium as the kidney? (A) Gonad (B) Epidermis (C) Pineal gland (D) Liver (E) Adrenal medulla

The answer is A: Gonad. -Both the kidneys and the gonads are derived from the intermediate mesoderm. This longitudinal dorsal ridge of mesoderm forms the urogenital ridge, which is involved with the formation of the future kidneys and gonads. -The liver is derived from the lateral plate mesoderm (specifically the splanchnic mesoderm), not the intermediate mesoderm. -The lateral plate mesoderm is a thin plate of mesoderm in which large spaces (intraembryonic coelom) form. These spaces coalesce and divide the lateral plate mesoderm into the intraembryonic somatic mesoderm and the intraembryonic splanchnic (visceral) mesoderm.

The given axial T2-weighted MRI of the prostate of a 50-year- old man was obtained via an endorectal coil. An image shows prostate cancer with extracapsular spread, which damages the neighboring hypogastric plexuses. Lesions of these nerves are most likely to result in which of the following deficits? (A) Inability to control the detrusor muscle (B) Inability to control the external anal sphincter (C) Chronic increased motility in the sigmoid colon (D) Reduced sensation on the glans penis (E) Loss of vasomotor control in the lower limb

The answer is A: Inability to control the detrusor muscle. The hypogastric plexuses are an extensive network of visceral motor (autonomic—both sympathetic and parasympathetic) and visceral sensory fibers across the true pelvis. The pelvic splanchnic nerves send presynaptic parasympathetic fibers through the inferior hypogastric plexus to the urinary bladder, where they supply the detrusor muscle in the wall of the bladder and the internal urethral sphincter in the neck of the bladder. The aortic (intermesenteric) plexus sends sympathetic fibers descending into the hypogastric plexuses and on to the bladder and internal urethral sphincter. Thus, damage within the lower part of the hypogastric plexuses will compromise autonomic control of urination. The axial T2-weighted MRI demonstrates a low signal intensity adenocarcinoma (indicated by the black arrow) arising in the normally high intensity peripheral zone of the prostate. The prostatic adenocarcinoma damages the hypogastric plexus and leads to an inability to control the detrusor muscle of the bladder. Choice B (Inability to control the external anal sphincter) is incorrect. The external anal sphincter is a skeletal muscle complex innervated by the inferior anal (rectal) branches of the pudendal nerve. The pudendal nerve originates within the pelvis from anterior primary rami of spinal nerves S2-4, and immediately exits the pelvis through the greater sciatic foramen. It is not a component of the hypogastric plexuses. Choice C (Chronic increased motility in the sigmoid colon) is incorrect. Parasympathetic fibers from the pelvic splanchnic nerves course through the inferior hypogastric plexus and ascend posterior to the peritoneum to reach the descending colon and sigmoid colon. Their normal function is to stimulate peristalsis in these organs. Thus, loss of parasympathetic input results in decreased motility here. Choice D (Reduced sensation on the glans penis) is incorrect. Sensation on the glans penis (and glans clitoris) is conveyed by general sensory fibers in the dorsal nerve of the penis (or clitoris), a terminal branch of the pudendal nerve. As noted above, the pudendal nerve is not related to the hypogastric plexuses. Choice E (Loss of vasomotor control in the lower limb) is incorrect. Sympathetic fibers that govern vasomotor function in the vessels of the lower limb originate in the sympathetic chain, pass into the sacral plexus, and run into the lower limb via the lower limb branches of the sacral plexus (e.g., sciatic nerve). This pathway is essentially separate from the hypogastric plexuses.

During micturition (urination), excitation of parasympathetic nerve fibers causes contraction of which of the following muscles? (A) Pubococcygeus muscle (B) Detrusor muscle (C) Internal urethral sphincter (D) Bulbospongiosus muscle (E) External urethral sphincter

The answer is B: Detrusor muscle. The smooth muscle walls of the urinary bladder are composed mainly of the detrusor muscle. Parasympathetic input during urination stimulates the detrusor to contract, thus emptying the bladder into the urethra. Choice A (Pubococcygeus muscle) is incorrect. The pubococcygeus muscle is a skeletal muscle that forms part of the levator ani in the pelvic diaphragm. It acts to support and lift the pelvic floor. Its contraction is evoked by somatic motor branches of the sacral plexus, including branches of the pudendal nerve. Choice C (Internal urethral sphincter) is incorrect. The internal urethral sphincter is a smooth muscle bundle located at the internal urethral orifice in the neck of the urinary bladder in males. During urination (in males), para- sympathetic stimulation causes the internal urethral sphincter to relax, allowing urine to flow into the urethra. An organized internal urethral sphincter is not present in females. Choice D (Bulbospongiosus muscle) is incorrect. The bulbospongiosus is a skeletal muscle enveloping the bulb of the penis in males and the individual vestibular bulbs in females. During urina- tion in males, contraction of the muscle squeezes the bulb, assisting in draining the penile urethra. However, the innerva- tion of the bulbospongiosus muscle is via somatic motor fibers of the perineal nerves. Choice E (External urethral sphincter) is incorrect. The external urethral sphincter is composed of skeletal muscle and acts as the voluntary urethral sphincter to constrict the urethra and prevent urination. The innerva- tion of the external urethral sphincter is controlled by somatic motor fibers in the perineal nerves.

A young couple hoping for a pregnancy buys an overthe-counter pregnancy kit. What substance will this test most likely detect? (A) Early pregnancy factor (EPF) (B) Human chorionic gonadotropin (C) Progesterone (D) Estrogen (E) Luteinizing hormone (LH)

The answer is B: Human chorionic gonadotropin. Early pregnancy detection kits most likely detect the level of human chorionic gonadotropin (hCG) in the female. hCG is a glycoprotein hormone produced by the syncytiotrophoblast to prevent the disintegration of the corpus luteum of the ovary. While hCG is a reliable marker of pregnancy, it cannot be detected until after implantation (6 to 12 days after fertilization), which results in false negatives if the test is performed during the very early stages of pregnancy.

The urinary system develops from which of the embryonic sources labeled in the given diagram?

The answer is B: Intermediate mesoderm. The intermediate mesoderm is a small bridge of mesoderm that connects the paraxial mesoderm with the lateral plate mesoderm. It differ- entiates into urogenital structures, including the kidney paren- chyma and nephrons, and the gonads. Choice A (Paraxial mesoderm) is incorrect. The paraxial mesoderm is the thick- ened portion of mesoderm closest to the midline and neural tube. It becomes segmented and forms somites, which further differentiate into myotome, dermatome, and sclerotome units that form skeletal muscles, dermis, and most of the vertebral column, respectively. Choice C (Parietal [somatic] meso- derm) is incorrect. This derivative of the lateral plate meso- derm becomes associated with the wall of the amniotic sac. Ultimately, it forms (along with the ectoderm) the lateral body wall folds and gives rise to the dermis plus the bones and con- nectives tissues in the body wall and limbs. Choice D (Visceral [splanchnic] mesoderm) is incorrect. The visceral (splanchnic) mesoderm is the other derivative of the splitting of the lateral plate mesoderm. It joins with the endodermal lining of the yolk sac and forms the wall of the gut tube. Choice E (Yolk sac endoderm) is incorrect. This germ layer becomes associated with the visceral mesoderm to form the wall of the gut tube. Specifically, it forms the lining of the gut tube.

A 70-year-old patient with a history of hypercholesterolemia complains of severe cramps in his gluteal region despite no history of physical exertion in the past few days. An arteriogram shows atherosclerotic blockage leading to insufficient perfusion to the gluteal region. What artery is most likely occluded leading to his complications? (A) External iliac (B) Internal iliac (C) Femoral (D) Internal pudendal (E) Abdominal aorta

The answer is B: Internal iliac. The blood supply to the gluteal region is supplied by the superior and inferior gluteal arteries, which arise off the internal iliac artery. The internal iliac artery is occluded in this patient, and his symptoms are due to lack of blood perfusion to the gluteal muscles and surrounding tissue, which causes ischemia and severe pain in the affected (gluteal) region. Remember that the internal iliac artery and its branches supply the gluteal region as well as contents of the pelvis and the medial thigh. Choice A (External iliac) is incorrect. The external iliac artery supplies blood to the iliac region and the anterior abdominal wall. This artery becomes the fem- oral artery, the major lower limb artery, after it passes under the inguinal ligament. Atherosclerotic blockage of the external iliac would affect arterial supply to the hip and lower limb, not the gluteal region. Choice C (Femoral artery) is incorrect. The femoral artery is the continuation of the external iliac artery, which changes its name to the femoral artery after passing under the inguinal ligament. Atherosclerotic blockage of the femoral artery would affect the blood supply to the lower limb, except for the gluteal region. Choice D (Internal pudendal) is incorrect. The internal pudendal artery is a branch of the anterior division of the internal iliac artery, which supplies the anal and urogenital triangles of the perineum with its three major branches: inferior anal, perineal, and dorsal artery of the penis. Blockage of this artery would not be responsible for poor blood perfusion to the gluteal region; however, block- age of the internal iliac artery would lead to similar signs and symptoms in the arterial distribution of the internal pudendal artery. Choice E (Abdominal aorta) is incorrect. Atherosclerotic blockage of the abdominal aorta would lead to more widespread complications than simply pain in the gluteal region.

A painful abscess is located in the ischioanal fossa of a 39-year-old man. A CT reveals the abscess contains air and displaces the rectum to the right. Which of the following locations should the surgeon avoid during drainage of the abscess to prevent damage to the pudendal nerve and internal pudendal vessels? (A) Roof of the ischioanal fossa (B) Lateral wall of the ischioanal fossa (C) Medial wall of the ischioanal fossa (D) Posterior recess of the ischioanal fossa (E) Anterior recess of the ischioanal fossa

The answer is B: Lateral wall of the ischioanal fossa. The lateral wall of the ischioanal fossa is formed by the ischium and the overlying inferior part of the obturator internus muscle. The pudendal (Alcock) canal is a shallow depression on the surface of the obturator internus muscle formed by the invest- ing fascia of this muscle. The pudendal canal contains the pudendal nerve and the internal pudendal vessels along their entry into the ischioanal fossa. When draining an ischioanal abscess, a surgeon must avoid damage to the pudendal canal and its contents by avoiding the lateral wall of the ischioanal fossa. Remember that the ischioanal fossa is a large wedge- shaped space on each side of the anal canal located between the skin of the anal region and the pelvic diaphragm. This fascia-filled fossa supports the anal canal while simultaneously providing a space that accommodates the expansion of the anal canal during defecation. Choice A (Roof of the ischioanal fossa) is incorrect. The roof of the ischioanal fossa is formed by the levator ani muscle, which ascends laterally to the point where this muscular component of the pelvic diaphragm arises from the investing fascia of the obturator internus. The surgeon draining the ischioanal abscess should avoid the lat- eral wall, not the roof, of the ischioanal fossa to avoid dam- age to the pudendal nerve and the internal pudendal vessels. Choice C (Medial wall of the ischioanal fossa) is incorrect. The medial wall of the ischioanal fossa is also formed by the levator ani muscle and the external anal sphincter, which merge to form this medial wall. An incision in the vicinity of the medial wall of the ischioanal wall would not damage the pudendal nerve and the internal pudendal vessels specifically. However, it may damage the inferior anal (rectal) nerve and inferior rectal artery, which branch off of this neurovascular bundle. Choice D (Posterior recess of the ischioanal fossa) is incorrect. The posterior recess of the ischioanal fossa extends posterior to the sacrotuberous ligament and gluteus maximus muscle. A surgeon can avoid damage to the pudendal nerve and the internal pudendal vessels by avoiding the medial wall of this posterior recess. Choice E (Anterior recess of the ischioanal fossa) is incorrect. The anterior recess of the ischioanal fossa extends into the urogenital triangle of the perineum superior to the perineal membrane. When draining an ischioanal recess, a surgeon can avoid damage to the pudendal nerve and the internal pudendal vessels by avoiding the medial wall of this anterior recess.

A medical resident is charged with catheterizing the urethra in a 68-year-old man in order to drain urine from the bladder. Which of the following sequence of structures correctly lists the order of the structures encountered when passing a catheter through the external urethral orifice to the urinary bladder? (A) Navicular fossa, spongy urethra, membranous urethra, ductus deferens, prostatic urethra (B) Navicular fossa, spongy urethra, membranous urethra, prostatic urethra, intramural urethra (C) Spongy urethra, membranous urethra, prostatic urethra, intramural urethra, ureter (D) Spongy urethra, membranous urethra, ejaculatory duct, ductus deferens, prostatic urethra (E) Intramural urethra, prostatic urethra, membranous urethra, spongy urethra, navicular fossa

The answer is B: Navicular fossa, spongy urethra, membranous urethra, prostatic urethra, intramural urethra. The urethra is a continuous tube that can be divided into four main parts: (1) spongy urethra, (2) membranous (intermediate) urethra, (3) prostatic urethra, (4) intramural urethra. The urethra terminates (or begins for catheter insertion) at the external urethral orifice. The spongy (penile) urethra is the longest part, running though the corpus spongio- sum in the penis. The distal end of the spongy urethra is the expanded navicular fossa within the glans penis. The membra- nous (intermediate) urethra passes through the deep perineal compartment and is the narrowest segment of the urethra. The prostatic urethra, which is the widest and most distensible part of the urethra, traverses the prostate gland. The final segment, the intramural (preprostatic) urethra, extends through the neck of the urinary bladder. Because the urethral wall is thin, it can be ruptured easily by inserted instruments. Thus, understand- ing and properly navigating the curved pathway of the ure- thra are critical. Choice A (Navicular fossa, spongy urethra, membranous urethra, ductus deferens, prostatic urethra) is incorrect. The ductus (vas) deferens is not part of the urethral pathway to the urinary bladder. The ductus deferens empties into the prostatic urethra after joining the duct of the seminal vesicle to form the ejaculatory duct. Choice C (Spongy urethra, membranous urethra, prostatic urethra, intramural urethra, ure- ter) is incorrect. The ureter is not part of the urethral pathway to the bladder. The paired ureters drain into the urinary bladder separately from the bladder drainage into the urethra. The two ureteric orifices plus the urethral orifice mark the angles of the trigone of the bladder. Choice D (Spongy urethra, membra- nous urethra, ejaculatory duct, ductus deferens, prostatic ure- thra) is incorrect. The ejaculatory duct and the ductus deferens are not parts of the urethral pathway to the urinary bladder. The ductus deferens joins with the duct of the seminal vesicle to form the ejaculatory duct, which empties into the prostatic urethra. Choice E (Intramural urethra, prostatic urethra, mem- branous urethra, spongy urethra, navicular fossa) is incorrect. This sequence is the correct order of urethral segments through which urine would leave the urinary bladder to exit the external urethral orifice, but the inverse order of structures encountered when inserting a catheter into the urinary bladder.

The notochord forms the initial axial skeletal element of the body and induces the formation of the neural plate. Which of the following structures is the sole postnatal remnant of the embryonic notochord? (A) Spinal cord (B) Nucleus pulposus (C) Rib cage (D) Anulus fibrosus (E) Spinal meninges

The answer is B: Nucleus pulposus. The only remnant of the notochord is the central core portion of each of the intervertebral discs, the nucleus pulposus. This hydrostatic structure is the component of the disc that herniates out of its normal position in cases of herniated ("slipped") discs.

22 A 72-year-old man is brought to the ER after being struck by a car. Radiographic imaging reveals a rotationally unstable pubic ramus fracture. Which of the following structures is most likely damaged specifically by the fracture of the superior pubic ramus? (A) Sympathetic chain (B) Superior gluteal artery (C) Ductus deferens (D) Lumbosacral trunk (E) Piriformis muscle

The answer is C: Ductus deferens. The ductus (vas) deferens separates from the spermatic cord as the cord emerges from the deep inguinal ring. The ductus crosses over the external iliac vessels and enters the true pelvis over the pelvic brim at the superior pubic ramus. It then runs along the lateral pelvic wall and on to the posterolateral side of the urinary bladder. Thus, fractures of the lateral wall of the true pelvis, especially those at the superior pubic ramus, endanger the ductus def- erens. Choice A (Sympathetic chain) is incorrect. The sympa- thetic chain crosses the pelvic brim at the ala of the sacrum to enter the true pelvis. It descends along the pelvic surface of the sacrum and finally terminates as the ganglion impar. There- fore, the sympathetic chain is situated in relation to the roof of the posterior wall of the true pelvis and is not in position to be affected by a lateral wall fracture. Choice B (Superior glu- teal artery) is incorrect. The superior gluteal artery branches from the posterior division of the internal iliac artery, within the true pelvis. It immediately leaves the pelvis through the greater sciatic foramen at the upper margin of the piriformis muscle. This vessel does not cross the pelvic brim and is not in position to be affected by a fracture of the brim. Choice D (Lumbosacral trunk) is incorrect. The lumbosacral trunk is formed by the union of the L4 and L5 ventral primary rami as they descend into the true pelvis to merge with the sacral plexus. These nerves cross the ala of the sacrum and typically form the lumbosacral trunk below the pelvic brim, within the true pelvis. However, the junction may occur at a higher level. In either case, the lumbosacral trunk is related to the roof of the posterior wall of the true pelvis rather than to the lateral wall. Choice E (Piriformis muscle) is incorrect. The piriformis muscle originates from the pelvic surface of the superior part of the sacrum, lateral to the anterior sacral foramina, within the true pelvis. It exits the pelvis through the greater sciatic foramen to reach the greater trochanter of the femur. The piriformis does not cross the pelvic brim. It forms the postero- lateral wall of the true pelvis and is not affected by a fracture of the lateral wall.

A 26-year-old medical student noticed a lump on his left testis, which was later diagnosed as testicular cancer. To rule out metastases of the testicular cancer, what group of lymph nodes should be checked in the first instance? (A) Superficial inguinal (B) Deep inguinal (C) Lumbar (D) External iliac (E) Internal iliac

The answer is C: Lumbar. The lymphatic drainage of the testes goes directly to the lumbar lymph nodes due to their develop- mental origin in the superoposterior abdominal wall. Lumbar lymph nodes, which are located anterior to the lumbar vertebrae and surrounding the inferior vena cava and abdominal aorta, also receive lymphatic drainage directly from the posterior abdominal wall, kidneys, ovaries, ureters, uterus, and uterine tubes. Because most lymphatic vessels supplying pel- vic viscera follow arteries retrograde, lymph from the testes, which receive arterial blood from testicular artery arising off the abdominal aorta at vertebral level L1, terminates primarily in the lumbar lymph nodes. Therefore, the lumbar lymph nodes should be checked initially to rule out metastases in the case of testicular cancer. Remember, when considering spread of cancer from pelvic visceral organs, it is crucial to remember the arterial supply of the organ in question. Choice A (Super- ficial inguinal) is incorrect. Superficial inguinal lymph nodes, which are located in the subcutaneous tissue near the termina- tion of the great saphenous vein, receive lymphatic drainage from the lower abdominal wall, buttock, lower limb, and all of the perineum and external genitalia, except for the glans penis and its distal spongy urethra. Despite the fact that the scrotum drains to the superficial inguinal lymph nodes, the testes drain to the lumbar nodes due to their descent from the posterior abdominal wall in development. Choice B (Deep inguinal) is incorrect. Deep inguinal lymph nodes, which are located deep to the fascia lata and medial to the femoral vein, receive lymphatic drainage directly from the glans penis, the distal spongy urethra of the penis, deep structures of the lower limb, and indirect lymphatic drainage from the superficial inguinal lymph nodes. Due to their descent from the posterior abdomi- nal wall in development, the lymphatic drainage of the testes goes directly to the lumbar lymph nodes. Choice D (External iliac) is incorrect. The external iliac lymph nodes, which are associated with the external iliac vein, receive lymphatic drain- age from pelvic organs in direct contact with the peritoneum, including the superior bladder and superior pelvic ureters. Even though the tunica vaginalis is a serous covering of the testes derived from the peritoneum, the lymphatic drainage of the testes goes directly to the lumbar lymph nodes due to their descent from the posterior abdominal wall in develop- ment. Choice E (Internal iliac) is incorrect. The internal iliac lymph nodes, which are located along the internal iliac artery and its branches, receive lymphatic drainage from the pelvic viscera, gluteal region, and deep parts of the perineum. The testes drain to the lumbar nodes due to their descent from the posterior abdominal wall in development and their arterial supply from the testicular artery arising off the abdominal aorta.

A married woman who is having diffi culty getting pregnant undergoes an endometrial function test (EFT), which determines that the endometrium of her uterus is not capable of implantation. Which of the following substances would likely increase the chances for implantation of the fertilized oocyte (or preimplantation embryo) into the uterine mucosa? (A) Follicle-stimulating hormone (B) Testosterone (C) Progesterone (D) Estrogen (E) Luteinizing hormone

The answer is C: Progesterone. Progesterone is a steroid hormone produced by the ovaries, brain, and placenta of pregnant females. In the menstrual cycle, progesterone production remains low until after ovulation. The corpus luteum (the remnant of the collapsed ovarian follicle after ovulation) produces progesterone, which halts endometrial proliferation and builds the endometrial lining of the uterus in preparation for implantation of the fertilized oocyte (zygote). Therefore, progesterone supplementation would increase the chances for implantation of the fertilized oocyte into the uterine mucosa of this patient. Because increased serum levels of progesterone and estrogen suppress its release, FSH levels remain low during the buildup of the endometrial lining of the uterus, which is crucial for implantation. Also, the production of FSH peaks approximately 3 days after menstruation because serum levels of progesterone and estrogen are low. Therefore, FSH supplementation would decrease the chances for implantation of the fertilized oocyte into the uterine mucosa.

A young boy has a chronic problem with urine dripping from abnormal openings on the underside (anterior or ventral side) of the glans and body of the penis. Which of the following structures most likely did not fuse normally during development? (A) Mesonephric ducts (B) Paramesonephric ducts (C) Urethral folds (D) Scrotal swellings (E) Genital tubercle

The answer is C: Urethral folds. In male fetuses, the urethral (urogenital) folds close over the urethral groove and fuse in the midline to form the ventral (anterior) aspect of the penis. This enclosure of the urethral groove forms the penile ure- thra. Incomplete fusion of the urethral folds results in abnor- mal openings of the penile urethra along the underside of the penis. This condition is termed hypospadias and may include one or more openings anywhere from the glans to the base of the penis. In females, the urethral folds do not fuse but form the separate labia minora. The urethral groove remains open and forms the vestibule. Choice A (Mesonephric ducts) is incorrect. In males, the mesonephric ducts are stimulated but do not fuse with one another. They form the testicular drain- age ducts, including the efferent ductules, ductus epididymis, ductus deferens, and ejaculatory duct. The seminal vesicle forms from a secondary bud off the ductus deferens. In females, the mesonephric ducts are normally suppressed, leaving only small vestigial structures in association with the reproductive tract. Choice B (Paramesonephric ducts) is incorrect. These ducts are normally suppressed in male fetuses. The sole rem- nant is the small, vestigial appendix of the testis. In females, the paramesonephric ducts are stimulated and partly fused. They form the uterine tubes, uterus, cervix, and superior third of the vagina. Choice D (Scrotal swellings) is incorrect. The scrotal swellings arise from the genital swellings on the lateral sides of the urethral folds. They enlarge and fuse in the mid- line to form the scrotum. Rarely, hypospadias extends poste- rior into the scrotal raphe. In females, the genital swellings enlarge, but do not fuse, forming the labia majora. Choice E (Genital tubercle) is incorrect. The genital tubercle forms at the cranial junction of the cloacal folds and elongates to form the phallus, which in turn forms the glans penis, corpora cav- ernosa, and corpus spongiosum. In females, the phallus forms the glans clitoris, corpora cavernosa, and vestibular bulbs.

A 24-year-old woman is the driver of a car in a head-on collision. She suffers bilateral fractures of the pubic rami and dislocation of the pubic symphysis as a result of anteroposterior compression of the pelvis . Which of the following structures is most likely damaged in this patient? (A) Anal canal (B) Piriformis muscles (C) Urinary bladder (D) Uterus (E) Vagina

The answer is C: Urinary bladder. When empty (i.e., in the "resting" anatomical position), the adult urinary bladder is located in the true (lesser) pelvis, lying partly on the pubic bones and pubic symphysis. It is separated from these bones by only the thin, potential retropubic space. The bladder may be ruptured by fractures of the pubic bones or, especially when distended, by injuries to the anteroinferior abdominal wall. Rupture of the bladder wall may allow extravasation (passing out) of urine into extraperitoneal spaces or into the peritoneal cavity. Choice A (Anal canal) is incorrect. The anal canal is far posterior of the pubic bones, with other organs intervening. It is not normally directly affected by fracture of those bones. Choice B (Piriformis muscles) is incorrect. The piriformis muscles run from the pelvic surface of the sacrum through the greater sciatic foramina to the femur. They form part of the posterolateral wall of the true pelvis and so are on the oppo- site side of the true pelvis from the pubic bones. Choice D (Uterus) is incorrect. The uterus normally lays anteverted and anteflexed in the reproductive tract. Its anterior surface lies on the posterosuperior aspect of the urinary bladder. Thus, the bladder intervenes between the pubic bones and the uterus. Choice E (Vagina) is incorrect. The vagina lies posterior to the base of the urinary bladder and the urethra. Therefore, those organs would be directly affected in pubic fractures before the vagina.

A 28-year-old woman who does not know that she is pregnant undergoes a chemotherapy treatment at the end of her 1st week of pregnancy. Chemotherapy is associated with slowing the rate of mitosis in exposed cells, which is good in cancer treatment. However, this treatment may also have a negative influence on the implantation and growth of an embryo. In which of the following layers would a lowered rate of cell division be most likely to hinder implantation of the blastocyst? (A) Amnioblast (B) Epiblast (C) Hypoblast (D) Cytotrophoblast (E) Syncytiotrophoblast

The answer is D: Cytotrophoblast. The trophoblast forms the fetal part of the placenta, so it is concerned with implantation. The cytotrophoblast is the mitotically active inner part of the trophoblast, and it forms the primary chorionic villi that extend into the syncytiotrophoblast. The cytotrophoblast also provides the cells that migrate into the syncytiotrophoblast and allow it to expand. Thus, chemotherapy may directly affect mitotic activity in the cytotrophoblast, causing stunted growth of both it and the syncytiotrophoblast and possibly hindering implantation. Choice A (Amnioblast) is incorrect. Amnioblasts are the epiblast cells that line the amniotic cavity adjacent to the cytotrophoblast. They are mitotically active in the growth of the amniotic membrane; however, they are not involved in implantation. Choice B (Epiblast) is incorrect. The epiblast is the dorsal cell layer of the bilaminar germ disc. It contributes to the formation of the embryo proper. While certainly mitotically active, it is not involved in implantation. Choice C (Hypoblast) is incorrect. The hypoblast is the ventral cell layer of the bilaminar germ disc. It also contributes to the formation of the embryo proper. Choice E (Syncytiotrophoblast) is incorrect. The syncytiotrophoblast is the outer, multinucleated part of the trophoblast that is mitotically inactive. The syncytiotrophoblast erodes the maternal endometrium and contributes to the formation of the primitive uteroplacental circulation. However, its growth depends on incorporation of new cells from the active cytotrophoblast. Thus, chemotherapy in this case would not affect cell division within the syncytiotrophoblast.

A married couple having diffi culty with conception keeps a daily diary of the female's basal body temperature (BBT) throughout the month. On Friday, the woman noted a slight elevation in her BBT of approximately one-half to one degree Fahrenheit (one-quarter to one-half degree Celsius), which may indicate that she is ovulating. If her ovum, depicted on the right side in the given figure, is expelled into the peritoneal cavity from the ovary (ovulation), the secondary oocyte resides at what specific stage of meiosis? (A) Prophase of meiosis I (B) Prophase of meiosis II (C) Metaphase of meiosis I (D) Metaphase of meiosis II (E) Meiosis is completed at the time of ovulation

The answer is D: Metaphase of meiosis II. The secondary is arrested in metaphase of meiosis II about 3 hours before ovulation, and it will remain in this meiotic stage until fertilization occurs. Oogonia are formed in month 5 of a woman's fetal life. Of these 7 million oogonia, 5 million degenerate or become atretic before birth, leaving 2 million oogonia to differentiate into primary oocytes before birth. No oogonia are present at birth. *The primary oocytes are dormant in prophase of meiosis I until puberty because they are surrounded by follicular cells, which secrete oocyte maturation inhibitor (OMI) that causes the arrest of meiosis I.*

Familial dysautonomia is a rare genetic disorder characterized by abnormal functioning of the autonomic nervous system. Problems include difficulty in feeding and respiration, vasomotor instability, insensitivity to pain, and ataxia (an unsteady gait). Patients with this syndrome have low numbers of autonomic (visceral motor) neurons, probably related to defects in production and/or survival of the neural crest precursors of these neurons. Such a disorder could directly affect the innervation of the muscle cells in which of the following structures? (A) Pelvic diaphragm (B) External anal sphincter (C) Urogenital diaphragm (D) Internal urethral sphincter (E) Levator ani

The answer is D: Internal urethral sphincter. Familial dysautonomia is an autonomic disorder that affects control of smooth muscle. The internal urethral sphincter is a thickening of the circular layer of smooth muscle at the internal urethral orifice. It is the involuntary sphincter that regulates urination (mictu- rition) and is the only smooth muscle structure of the choices provided. It is supplied by both sympathetic (for contraction) and parasympathetic (for relaxation) fibers through the hypo- gastric plexuses in the pelvis. Choice A (Pelvic diaphragm) is incorrect. The pelvic diaphragm is a musculofascial sheet that forms a large part of the pelvic floor. It is composed of two main parts: the levator ani and the coccygeus. Because it is composed of skeletal muscle, it is innervated by somatic motor fibers. These nerves branch directly off the sacral plexus and off the perineal branches of the pudendal nerve. Choice B (External anal sphincter) is incorrect. The exter- nal anal sphincter is the skeletal muscle complex surround- ing the lower anal canal that forms the voluntary sphincter, regulating defecation. It is innervated by somatic motor fibers of the inferior anal (rectal) nerves (branches of the pudendal nerves). Choice C (Urogenital diaphragm) is incorrect. Tra- ditionally, the urogenital diaphragm has been described as a three-layered, triangular structure that makes up the deep perineal pouch within the urogenital triangle. The middle layer is a skeletal muscle zone composed mainly of the exter- nal urethral sphincter (sphincter muscle of urethra) and the deep transverse perineal muscles. However, current concepts describe this area as a much more complex region, with the muscle and fascial components arranged in three-dimensional relations rather than in a flat sandwich-like manner. In either case, the muscles here receive somatic motor innervation from the perineal nerves, which are branches of the pudendal nerve. Readers are advised to reference the most current texts for a proper and better understanding of this region. Choice E (Levator ani) is incorrect. The levator ani is the larger of the two parts of the pelvic diaphragm. It is usually divided into three main subparts: puborectalis, pubococcygeus, and iliococcygeus. These muscles are supplied by somatic motor fibers of the perineal nerves.

A forensic anthropologist is called to the basement of an abandoned house, where an intact human skeleton has been discovered in a shallow grave. Without the benefit of soft tissue, the gender of the body can still be established based upon the characteristics of the two unearthed coxal (hip) bones. Which of the following characteristics is indicative of the female pelvis? (A) Subpubic angle less than 70 degrees (B) Heart-shaped (android) pelvic inlet (C) More inverted ischial tuberosities (D) More everted alae of the ilia (E) Ischial spines close together in the midline

The answer is D: More everted alae of the ilia. The greater pelvis (pelvis major; false pelvis) of a female is relatively shallow due to the everted wings (alae) of the iliac portions of the coxal (hip) bones. This shallow greater pelvis shortens the distance the head of a fetus needs to descend through the pelvic aper- ture during parturition. If the wings of the iliac portions of the coxal bones are more everted, then the unearthed coxal bones are most likely female. The rest of the characteristics listed within this question are more characteristic of a male pelvis. Choice A (Subpubic angle less than 70 degrees) is incorrect. A subpubic angle of less than 70 degrees is a characteristic of a male pelvis. This angle can be approximated by spreading the index and middle finger. In the female pelvis, this sub- pubic angle is wider (greater) than 80 degrees, which enables the head of a fetus more space to descend through the pelvic outlet (inferior pelvic aperture). The size of the female subpu- bic angle can be approximated by spreading the thumb and index finger. Choice B (Heart-shaped [android] pelvic inlet) is incorrect. A heart-shaped (android) pelvic inlet (superior pelvic aperture) is a characteristic of a male pelvis. In contrast, the female pelvis has an oval and rounded pelvic inlet, which is better adapted for parturition. Choice C (More inverted ischial tuberosities) is incorrect. Inverted ischial tuberosities are often seen in the male pelvis. This feature decreases the diameter of the pelvic outlet (inferior pelvic aperture), which is not conducive to parturition. In contrast, the ischial tuber- osities in the female pelvis are more everted, which increases the diameter of the pelvic outlet. Choice E (Ischial spines close together in the midline) is incorrect. A small interspinous dis- tance, due to inverted ischial spines, is often seen in the male pelvis. This characteristic decreases the diameter of the pelvic outlet (or inferior pelvic aperture), which is not conducive to parturition. In contrast, the distance between the ischial spines in the female pelvis is wider, which increases the diam- eter of the pelvic outlet.

In a 15-day embryo, the epiblast is capable of forming which of the following germ layers? (A) Ectoderm only (B) Ectoderm and mesoderm only (C) Ectoderm and endoderm only (D) Mesoderm and endoderm only (E) Ectoderm, mesoderm, and endoderm

The answer is E: Ectoderm, mesoderm, and endoderm. The epiblast is capable of forming all three germ layers (ectoderm, mesoderm, and endoderm) during gastrulation. Epiblast cells migrate to the primitive streak and invaginate into a space between the epiblast and the hypoblast

A 4-month-old male infant presents with a "growing sore" located posterior to his left ear. This sore is diagnosed as a postauricular hemangioma, as seen in the given photo with the auricle pulled anterior. The cells forming the hemangioma are derived from which of the following cell layers? (A) Endoderm (B) Neural crest (C) Neuroectoderm (D) Ectoderm (E) Mesoderm

The answer is E: Mesoderm. The mesoderm forms during gastrulation when invaginating epiblast cells form an additional germ layer between the endoderm and the ectoderm. The mesoderm has many notable derivatives, including muscle, connective tissue, bone, cartilage, blood cells, dermis of the skin, and organs, such as the kidney, spleen, and gonads.

A 16-year-old boy, who was riding his skateboard down a stair railing, falls and impacts his perineum on the railing, causing a straddle injury. He presents in the ER with discoloration and swelling in the scrotum, penis, and anterior abdominal wall, which is characteristic of extravasation of urine. Which of the following structures is most likely compromised? (A) Ureter (B) Urinary bladder (C) Prostatic urethra (D) Intermediate urethra (E) Spongy urethra

The answer is E: Spongy urethra. In men, a straddle injury may rupture the spongy urethra, which courses through the corpus spongiosum of the penis. Damage to the spongy urethra causes extravasation of urine, in which the urine leaves the spongy urethra and spills out into the surrounding tis- sue. This extravasation of urine would be confined by the superficial layer of perineal fascia, which is continuous with the superficial (dartos) fascia of the scrotum, the superficial fascia of the penis, and the membranous layer of subcutane- ous tissue of the lower abdomen (Scarpa fascia). Therefore, urine (and blood, due to the trauma and the discoloration noted in this presentation) will spread into the scrotum, penis, and lower anterior abdominal wall, as noted in this patient. Because the superficial layer of perineal fascia blends with the posterior edge of the perineal membrane and adheres onto the ischiopubic rami laterally, urine will not spread into the anal triangle of the perineum or the thighs, respectively. Damage to the spongy urethra can also occur following an incorrect insertion of a catheter to drain the urinary bladder. Choice A (Ureter) is incorrect. The ureters descend posterior to the peri- toneum of the abdomen to connect the kidneys to the urinary bladder. Due to their location, the ureters would not be at risk during a straddle injury to the perineum. Furthermore, rupture of the ureters would cause extravasation of urine in the abdomen or pelvis, but not the perineum, as noted in this patient. Choice B (Urinary bladder) is incorrect. The urinary bladder is located in the true pelvis (or pelvis minor), and due to its position above the perineal membrane and the pelvic diaphragm, it would not be at risk during a straddle injury to the perineum. Furthermore, rupture of the urinary bladder would cause extravasation of urine into extraperitoneal spaces or into the peritoneal cavity, which was not seen in this patient. Choice C (Prostatic urethra) is incorrect. The prostatic urethra is the portion of the male urethra that traverses the prostate gland. Because it is not located within the perineum, prostatic urethra damage would result in extravasation of urine in the true pelvis (or pelvis minor). Choice D (Intermediate urethra) is incorrect. The intermediate (membranous) urethra passes through the deep perineal compartment. This portion of the male urethra is the narrowest urethral segment. Due to its location within the deep perineal compartment, extravasation of urine from the intermediate urethra would reside between the inferior fascia of the pelvic diaphragm and the perineal membrane. So, damage to the intermediate urethra would not cause discoloration and swelling in the scrotum, penis, and anterior abdominal wall, which was noted in this patient.

Development of the primitive gonads into testes or ovaries is induced by invasion of the genital ridges by the primordial germ cells. These cells migrate from which of the following locations? (A) Paraxial mesoderm (B) Intermediate mesoderm (C) Lateral plate mesoderm (D) Neural crest (E) Yolk sac endoderm

The answer is E: Yolk sac endoderm. The primordial germ cells originate in the epiblast and migrate to the caudal endo- dermal wall of the yolk sac, near the allantois. From there, they migrate along the dorsal mesentery of the gut tube to the genital ridges. Once they invade the genital ridges, they induce the primitive gonads to develop into either testes or ovaries. Choice A (Paraxial mesoderm) is incorrect. The paraxial mesoderm is the thickened portion of mesoderm clos- est to the midline and neural tube. It becomes segmented and forms somites, which further differentiate into myotome, der- matome, and sclerotome units that form skeletal muscles, der- mis, and most of the vertebral column, respectively. Choice B (Intermediate mesoderm) is incorrect. The intermediate meso- derm is the small bridge of mesoderm that connects the parax- ial mesoderm with the lateral plate mesoderm. It differentiates into urogenital structures, including the kidney parenchyma and nephrons and the gonads. Thus, the intermediate meso- derm is the target of the migrating primordial germ cells rather than their source. Choice C (Lateral plate mesoderm) is incor- rect. The lateral plate mesoderm is a relatively flat, wide area, which divides into the parietal (somatic) mesoderm and vis- ceral (splanchnic) mesoderm. The parietal part becomes asso- ciated with the wall of the amniotic sac, ultimately forming (along with the ectoderm) the lateral body wall folds. The visceral part joins with the endodermal lining of the yolk sac and forms the wall of the gut tube. Choice D (Neural crest) is incorrect. The neural crest forms at the free lips of the neu- ral folds, dissociates from the neural tube, and spreads widely across the body. Crest cells form a diverse assortment of struc- tures, including melanocytes, neural ganglia, craniofacial skel- eton, and odontoblasts.

Tricuspid and mitral

The attending faculty in the coronary intensive care unit demonstrates to his students a normal heart examination. The first heart sound is produced by near-simultaneous closure of which of the following valves? -Aortic and tricuspid -Aortic and pulmonary -Tricuspid and mitral -Mitral and pulmonary -Tricuspid and pulmonar

Right superior bronchus

The bronchogram of a 45-year-old female smoker shows the presence of a tumor in the eparterial bronchus. Which airway is most likely blocked? -Left superior bronchus -Left inferior bronchus -Right superior bronchus -Right middle bronchus -Right inferior bronchus

A 17-year-old boy is injured in an automobile accident. He has a fracture of the shaft of the humerus. 58. Which of the following nerves is most likely damaged? (A) Axillary nerve (B) Radial nerve (C) Musculocutaneous nerve (D) Median nerve (E) Ulnar nerve

58. The answer is B. The radial nerve runs in the radial groove on the back of the shaft of the humerus with the profunda brachii artery. The axillary nerve passes around the surgical neck of the humerus. The ulnar nerve passes the back of the medial epicondyle. The musculocutaneous and median nerves are not in contact with the bone, but the median nerve can be damaged by supracondylar fracture.

65. Destruction of this area would most likely cause weakness of supination and flexion of the forearm.

B

An obstetrician performs a median episiotomy to expand the birth canal during a childbirth. Which of the following structures is typically incised during this procedure? (A) Bulbospongiosus muscle (B) Urethra (C) Analcanal (D) Ischiocavernosus muscle (E) Perineal body

The answer is E: Perineal body. An episiotomy is a surgical incision from the posterior wall of the vagina into the perineum posterior to the vagina. This procedure is used to gain a con- trolled enlargement of the vaginal canal during childbirth. In a median episiotomy, the incision is made in the midline and extends through the perineal body (see diagram). The advan- tage of this procedure is that it may decrease excessive reduc- tion of the perineal body and minimize trauma to the pelvic diaphragm and perineal muscles. An alternate procedure is the posterolateral episiotomy. In this protocol, the incision begins in the median line and then is directed laterally and posteriorly toward the ischial tuberosity. The advantage here is that the perineal body is spared. Choice A (Bulbospongio- sus muscle) is incorrect. The bulbospongiosus is the skeletal muscle sheath surrounding the individual vestibular bulbs. The bulbs are located deep to the labia minora, lateral to the vaginal opening. Thus, the bulbs are anterior to the incision line in either type of episiotomy and are not cut. Choice B (Urethra) is incorrect. The urethra is anterior to the vagina. It is not in the line of incision in an episiotomy and is not cut. Choice C (Anal canal) is incorrect. The anal canal is directly in the line of incision in a median episiotomy. However, the incision normally stops short of the anal canal. It is undesir- able to have a communication between the anal and vaginal canals because of the danger of anal material contacting the fetus during delivery. Choice D (Ischiocavernosus muscle) is incorrect. The ischiocavernosus is the thin skeletal muscle layer surrounding the crus of the clitoris. Each crus is attached to the ischiopubic ramus, lateral to the vagina and anterior to the ischial tuberosity. Thus, the crus of the clitoris would not be incised because it is located off the median sagittal plane.

2. A patient comes in with a gunshot wound and requires surgery in which his thoracoacromial trunk needs to be ligated. Which of the following arterial branches would maintain normal blood fl ow? (A) Acromial (B) Pectoral (C) Clavicular (D) Deltoid (E) Superior thoracic

2. The answer is E. The superior thoracic artery is a direct branch of the axillary artery. The thoracoacromial trunk has four branches: the pectoral, clavicular, acromial, and deltoid.

47. Which of the following conditions is most likely to occur secondary to the fractured clavicle? (A) A fatal hemorrhage from the brachiocephalic vein (B) Thrombosis of the subclavian vein, causing a pulmonary embolism (C) Thrombosis of the subclavian artery, causing an embolism in the ascending aorta (D) Damage to the upper trunk of the brachial plexus (E) Damage to the long thoracic nerve, causing the winged scapula

47. The answer is B. The fractured clavicle may damage the subclavian vein, resulting in a pulmonary embolism; cause thrombosis of the subclavian artery, resulting in embolism of the brachial artery; or damage the lower trunk of the brachial plexus.

A 21-year-old man injures his right arm in an automobile accident. Radiographic examination reveals a fracture of the medial epicondyle of the humerus. 48. Which of the following nerves is most likely injured as a result of this accident? (A) Axillary (B) Musculocutaneous (C) Radial (D) Median (E) Ulnar

48. The answer is E. The ulnar nerve runs down the medial aspect of the arm and behind the medial epicondyle in a groove, where it is vulnerable to damage by fracture of the medial epicondyle. Other nerves are not in contact with the medial epicondyle.

A 21-year-old man injures his right arm in an automobile accident. Radiographic examination reveals a fracture of the medial epicondyle of the humerus. 49. Which of the following muscles is most likely paralyzed as a result of this accident? (A) Extensor pollicis brevis (B) Abductor pollicis longus (C) Abductor pollicis brevis (D) Adductor pollicis (E) Opponens pollicis

49. The answer is D. The ulnar nerve innervates the adductor pollicis muscle. The radial nerve innervates the abductor pollicis longus and extensor pollicis brevis muscles, whereas the median nerve innervates the abductor pollicis brevis and opponens pollicis muscles.

A 21-year-old man injures his right arm in an automobile accident. Radiographic examination reveals a fracture of the medial epicondyle of the humerus. 50. After this injury, the patient is unable to do which of the following? (A) Flex the proximal interphalangeal joint of his ring fi nger (B) Flex the DIP joint of his index fi nger (C) Feel sensation on his middle fi nger (D) Abduct his thumb (E) Adduct his index fi nger

50. The answer is E. The fi ngers are adducted by the palmar interosseous muscles; abduction is performed by the dorsal interosseous muscles. The palmar and dorsal interosseous muscles are innervated by the ulnar nerve. The proximal interphalangeal joints are fl exed by the fl exor digitorum superfi cialis, which is innervated by the median nerve. However, the DIP joints of the index and middle fi ngers are fl exed by the fl exor digitorum profundus, which is innervated by the median nerve (except the medial half of the muscle, which is innervated by the ulnar nerve). The median nerve supplies sensory innervation on the palmar aspect of the middle fi nger. The abductor pollicis brevis is innervated by the median nerve; the abductor pollicis longus is innervated by the radial nerve.

A 10-year-old boy falls off his bike, has diffi culty in moving his shoulder, and is brought to an emergency department. His radiogram and angiogram reveal fracture of the surgical neck of his humerus and bleeding from the point of the fracture. 51. Which of the following nerves is most likely injured as a result of this accident? (A) Musculocutaneous (B) Axillary (C) Radial (D) Median (E) Ulnar

51. The answer is B. The axillary nerve runs posteriorly around the surgical neck of the humerus and is vulnerable to injury such as fracture of the surgical neck of the humerus or inferior dislocation of the humerus. The other nerves listed are not in contact with the surgical neck of the humerus.

A 10-year-old boy falls off his bike, has diffi culty in moving his shoulder, and is brought to an emergency department. His radiogram and angiogram reveal fracture of the surgical neck of his humerus and bleeding from the point of the fracture. 52. Following this accident, the damaged nerve causes diffi culty in abduction, extension, and lateral rotation of his arm. Cell bodies of the injured nerve involved in movement of his arm are located in which of the following structures? (A) Dorsal root ganglion (B) Sympathetic chain ganglion (C) Anterior horn of the spinal cord (D) Lateral horn of the spinal cord (E) Posterior horn of the spinal cord

52. The answer is C. The (injured) axillary nerve contains GSE fi bers whose cell bodies are located in the anterior horn of the spinal cord, and these GSE fi bers supply the deltoid and teres minor muscles. The axillary nerve also contains GSA and GVA fi bers, whose cell bodies are located in the dorsal root ganglia, and sympathetic postganglionic fi bers, whose cell bodies are located in sympathetic chain ganglia. The lateral horn of the spinal cord between T1 and L2 contains cell bodies of sympathetic preganglionic fi bers. The posterior horn of the spinal cord contains cell bodies of interneurons.

A 10-year-old boy falls off his bike, has diffi culty in moving his shoulder, and is brought to an emergency department. His radiogram and angiogram reveal fracture of the surgical neck of his humerus and bleeding from the point of the fracture. 53. The damaged nerve causes numbness of the lateral side of the arm. Cell bodies of the injured nerve fi bers involved in sensory loss are located in which of the following structures? (A) Anterior horn of the spinal cord (B) Posterior horn of the spinal cord (C) Lateral horn of the spinal cord (D) Dorsal root ganglia (E) Sympathetic chain ganglia

53. The answer is D. Axillary nerve contains GSE, GSA, GVA, and sympathetic postganglionic GVE fi bers. Cell bodies of GSA and GVA fi bers are located in the dorsal root ganglia. Cell bodies of GSE fi bers are located in the anterior horn of the spinal cord. Cell bodies of sympathetic postganglionic GVE fi bers are located in the sympathetic chain ganglia, but cell bodies of sympathetic preganglionic GVE fi bers lie in the lateral horn of the spinal cord.

A 10-year-old boy falls off his bike, has diffi culty in moving his shoulder, and is brought to an emergency department. His radiogram and angiogram reveal fracture of the surgical neck of his humerus and bleeding from the point of the fracture. 54. This accident most likely leads to damage of which of the following arteries? (A) Axillary (B) Deep brachial (C) Posterior humeral circumfl ex (D) Superior ulnar collateral (E) Scapular circumfl ex

54. The answer is C. The posterior humeral circumfl ex artery accompanies the axillary nerve that passes around the surgical neck of the humerus. None of the other arteries are involved.

A 10-year-old boy falls off his bike, has diffi culty in moving his shoulder, and is brought to an emergency department. His radiogram and angiogram reveal fracture of the surgical neck of his humerus and bleeding from the point of the fracture. 55. Following this accident, the boy has weakness in rotating his arm laterally. Which of the following muscles are paralyzed? (A) Teres major and teres minor (B) Teres minor and deltoid (C) Infraspinatus and deltoid (D) Supraspinatus and subscapularis (E) Teres minor and infraspinatus

55. The answer is B. The lateral rotators of the arm include the teres minor, deltoid, and infraspinatus muscles, but the infraspinatus muscle is innervated by the suprascapular nerve.

A 64-year-old man with a history of liver cirrhosis has been examined for hepatitis A, B, and C viruses. In an attempt to obtain a blood sample from the patient's median cubital vein, a registered nurse inadvertently procures arterial blood. 56. The blood most likely comes from which of the following arteries? (A) Brachial (B) Radial (C) Ulnar (D) Common interosseous (E) Superior ulnar collateral

56. The answer is A. The median cubital vein lies superfi cial to the bicipital aponeurosis and thus separates it from the brachial artery, which can be punctured during intravenous injections and blood transfusions.

A 64-year-old man with a history of liver cirrhosis has been examined for hepatitis A, B, and C viruses. In an attempt to obtain a blood sample from the patient's median cubital vein, a registered nurse inadvertently procures arterial blood. 57. During the procedure, the needle hits a nerve medial to the artery. Which of the following nerves is most likely damaged? (A) Radial (B) Median (C) Ulnar (D) Lateral antebrachial (E) Medial antebrachial

57. The answer is B. The median nerve is damaged because it lies medial to the brachial artery. The bicipital aponeurosis lies on the brachial artery and the median nerve. The V-shaped cubital fossa contains (from medial to lateral) the median nerve, brachial artery, biceps tendon, and radial nerve. The ulnar nerve runs behind the medial epicondyle; the lateral and medial antebrachial cutaneous nerves are not closely related to the brachial artery.

A 17-year-old boy is injured in an automobile accident. He has a fracture of the shaft of the humerus. 60. Following this accident, the patient has no cutaneous sensation in which of the following areas? (A) Medial aspect of the arm (B) Lateral aspect of the forearm (C) Palmar aspect of the second and third digits (D) Area of the anatomic snuffbox (E) Medial one and one-half fi ngers

60. The answer is D. The superfi cial branch of the radial nerve runs distally to the dorsum of the hand to innervate the radial side of the hand, including the area of the anatomic snuffbox and the radial two and one-half digits over the proximal phalanx. The medial aspect of the arm is innervated by the medial brachial cutaneous nerve; the lateral aspect of the forearm is innervated by the lateral antebrachial cutaneous nerve of the musculocutaneous nerve; the palmar aspect of the second and third digits is innervated by the median nerve; and the medial one and one-half fi ngers are innervated by the ulnar nerve.

A 17-year-old boy is injured in an automobile accident. He has a fracture of the shaft of the humerus. 61. Which of the following arteries may be damaged? (A) Brachial artery (B) Posterior humeral circumfl ex artery (C) Profunda brachii artery (D) Radial artery (E) Radial recurrent artery

61. The answer is C. The radial nerve accompanies the profunda brachii artery in the radial groove on the posterior aspect of the shaft of the humerus. The posterior humeral circumfl ex artery accompanies the axillary nerve around the surgical neck of the humerus. Other arteries are not associated with the radial groove of the humerus.

An 11-year-old boy falls down the stairs. A physician examines a radiograph of the boy's shoulder region 63. If the superior transverse scapular ligament is calcified, which of the following muscles is most likely paralyzed? (A) Deltoid (B) Teres major (C) Teres minor (D) Infraspinatus (E) Subscapularis

63. The answer is D. The scapular notch transmits the suprascapular nerve below the superior transverse ligament, whereas the suprascapular artery and vein run over the ligament. The suprascapular nerve supplies the supraspinatus and infraspinatus muscles. The axillary nerve innervates the deltoid and teres minor muscles. The subscapular nerves innervate the teres major and subscapularis muscles.

71. A lesion of the median nerve causes paralysis of this structure.

A

69. If the floor of the anatomic snuffbox and origin of the abductor pollicis brevis are damaged, which of the following bones is most likely to be involved? (A) A (B) B (C) C (D) D (E) E

B

68. Destruction of the structure indicated by the letter E most likely causes weakness of which of the following muscles? (A) Flexor carpi radialis (B) Palmaris longus (C) Flexor carpi ulnaris (D) Brachioradialis (E) Flexor digitorum superficialis

C

63. If the structure indicated by the letter A is calcified, which of the following muscles is most likely paralyzed? (A) Deltoid (B) Teres major (C) Teres minor (D) Infraspinatus (E) Subscapularis

D

64. If the structure indicated by the letter B is fractured, which of the following structures is most likely injured? (A) Musculocutaneous nerve (B) Radial nerve (C) Deep brachial artery (D) Posterior humeral circumflex artery (E) Scapular circumflex artery

D

69. A lesion of the radial nerve would most likely cause paralysis of muscles that are attached to this area.

D

70. The patient is unable to abduct her middle finger because of paralysis of this structure.

D

66. Destruction of this area would most likely cause weakness of pronation of the forearm and flexion of the wrist joints.

E

73. Atrophy of this structure impairs extension of both the metacarpophalangeal and interphalangeal joints.

E

41. A 17-year-old boy fell from his motorcycle and complains of numbness of the lateral part of the arm. Examination reveals that the axillary nerve is severed. Which of the following types of axons is most likely spared? (A) Postganglionic sympathetic axons (B) Somatic afferent axons (C) Preganglionic sympathetic axons (D) General somatic efferent axons (E) General visceral afferent axons

The answer is C. The axillary nerve contains no preganglionic sympathetic general visceral efferent (GVE) fi bers, but it contains postganglionic sympathetic GVE fi ber

After a blood test showed an elevated prostate-specific antigen level in a 65-year-old man, the given axial T2-weighted MRI of the prostate was obtained via an endorectal coil. The image shows abnormally low signal intensity within the right peripheral zone of the prostate, which is characteristic of an adenocarcinoma and is identified by the white arrowheads. A radical prostatectomy was scheduled. Before surgery, his physician wants to rule out possible metastases of the cancer. Which group of lymph nodes should be checked in the first instance to rule out metastases? (A) Lumbar (B) Deep inguinal (C) Superficial inguinal (D) Internal iliac (E) External iliac

The answer id D: Internal Iliac Cancer cells from an adenocarcinoma within the posterior part of the prostate would most likely metastasize initially to the internal iliac lymph nodes, which lie along the internal iliac artery and receive lymph from the pelvic visceral and gluteal region. Because most lym- phatic vessels supplying pelvic viscera follow arteries retro- grade, lymph from the prostate gland, which receives arterial blood from the inferior vesical artery arising off the anterior division of the internal iliac artery, would terminate primarily in the internal iliac lymph nodes. Therefore, cancer cells from an adenocarcinoma within the posterior part of the prostate would most likely metastasize to the internal iliac lymph nodes. The sacral lymph nodes may also receive lymphatic drainage from the prostate, so these nodes should also be checked for metastases. Remember, when considering spread of cancer from pelvic organs, it is crucial to remember the arterial supply of the organ in question. Choice A (Lumbar) is incorrect. Lumbar lymph nodes, which are located anterior to the lumbar vertebrae and surrounding the inferior vena cava and abdominal aorta, receive lymphatic drainage directly from the posterior abdominal wall, gonads (ovaries and testes), kid- neys, ureters, uterus, and uterine tubes. Unless metastasis is in advanced stages by spreading through several sets of lymph nodes, the lumbar lymph nodes would not be affected because they do not directly receive lymphatic drainage from a prostatic adenocarcinoma. Choice B (Deep inguinal) is incorrect. Deep inguinal lymph nodes, which are located deep to the fascia lata and medial to the femoral vein, receive lymphatic drain- age directly from the glans penis, the distal spongy urethra of the penis, and deep structures of the lower limb and indirect lymphatic drainage from the superficial inguinal lymph nodes. Since the prostate is a pelvic organ, it does not have lymphatic drainage to the deep inguinal lymph nodes. Choice C (Super- ficial inguinal) is incorrect. Superficial inguinal lymph nodes, which are located in the subcutaneous tissue near the termina- tion of the great saphenous vein, receive lymphatic drainage from the lower abdominal wall, buttock, lower limb, and all of the perineum and external genitalia, except for the glans penis and its distal spongy urethra. Since the prostate is a pelvic organ, it does not have lymphatic drainage to the superficial inguinal lymph nodes. Choice E (External iliac) is incorrect. The external iliac lymph nodes, which are associated with the external iliac vein, receive lymphatic drainage from pelvic organs in direct contact with the peritoneum, including the superior bladder and superior pelvic ureters. Since the pros- tate is situated well below the peritoneum in the pelvis, it does not drain to the external iliac lymph nodes. 30

7. A 16 year-old boy fell from a motorcycle, and his radial nerve was severely damaged because of a fracture of the midshaft of the humerus. Which of the following conditions would most likely result from this accident? (A) Loss of wrist extension leading to wrist drop (B) Weakness in pronating the forearm (C) Sensory loss over the ventral aspect of the base of the thumb (D) Inability to oppose the thumb (E) Inability to abduct the fi ngers

The answer is A. Injury to the radial nerve results in loss of wrist extension, leading to wrist drop. The median nerve innervates the pronator teres, pronator quadratus, and opponens pollicis muscles and the skin over the ventral aspect of the thumb. The ulnar nerve innervates the dorsal interosseous muscles, which act to abduct the fi ngers.

19. An 18-year-old boy involved in an automobile accident presents with arm that cannot abduct. His paralysis is caused by damage to which of the following nerves? (A) Suprascapular and axillary (B) Thoracodorsal and upper subscapular (C) Axillary and musculocutaneous (D) Radial and lower subscapular (E) Suprascapular and dorsal scapular

The answer is A. The abductors of the arm are the deltoid and supraspinatus muscles, which are innervated by the axillary and suprascapular nerves, respectively. The thoracodorsal nerve supplies the latissimus dorsi, which can adduct, extend, and rotate the arm medially. The upper and lower subscapular nerves supply the subscapularis, and the lower subscapular nerve also supplies the teres major; both of these structures can adduct and rotate the arm medially. The musculocutaneous nerve supplies the fl exors of the arm, and the radial nerve supplies the extensors of the arm. The dorsal scapular nerve supplies the levator scapulae and rhomboid muscles; these muscles elevate and adduct the scapula, respectively.

34. A patient bleeding from the shoulder secondary to a knife wound is in fair condition because there is vascular anastomosis around the shoulder. Which of the following arteries is most likely a direct branch of the subclavian artery that is involved in the anastomosis? (A) Dorsal scapular artery (B) Thoracoacromial artery (C) Circumfl ex scapular artery (D) Transverse cervical artery (E) Suprascapular artery

The answer is A. The dorsal scapular artery arises directly from the third part of the subclavian artery and replaces the deep (descending) branch of the transverse cervical artery. The suprascapular and transverse cervical arteries are branches of the thyrocervical trunk of the subclavian artery. The thoracoacromial artery is a short trunk from the fi rst or second part of the axillary artery and has pectoral, clavicular, acromial, and deltoid branches.

36. A patient with a stab wound receives a laceration of the musculocutaneous nerve. Which of the following conditions is most likely to have occurred? (A) Lack of sweating on the lateral side of the forearm (B) Inability to extend the forearm (C) Paralysis of brachioradialis muscle (D) Loss of tactile sensation on the arm (E) Constriction of blood vessels on the hand

The answer is A. The musculocutaneous nerve contains sympathetic postganglionic fi bers that supply sweat glands and blood vessels on the lateral side of the forearm as the lateral antebrachial cutaneous nerve. The musculocutaneous nerve does not supply the extensors of the forearm and the brachioradialis. This nerve also supplies tactile sensation on the lateral side of the forearm but not the arm and supplies blood vessels on the lateral side of the forearm but not the hand.

A physician examining a 16-year-old girl for the first time notes that she is in good overall health and appears outwardly to have reached puberty. However, she explains that she has not yet had a menstrual period. Full physical and radiological examinations reveal a short, closed vaginal pouch, and no upper vagina, cervix, or uterus. Which of the following descriptions is the most likely cause of this condition? (A) Atresia of the paramesonephric ducts (B) Atresia of the mesonephric ducts (C) Atresia of the urogenital sinus (D) Imperforate hymen (E) Agenesis of the ureteric bud

The answer is A: Atresia of the paramesonephric ducts. In females, the paired embryonic paramesonephric ducts form the uterine tubes, uterus, cervix, and superior third of the vagina. Thus, if the paramesonephric ducts are absent (atretic), these organs do not form. The lower 2/3 of the vagina develops from the sinovaginal bulbs, which are paired evagi- nations of the wall of the urogenital sinus. The two parts of the vagina (paramesonephric duct part and sinovaginal bulb part) canalize to form a single, complete vaginal canal. How- ever, without the paramesonephric duct components, only a short vaginal pouch forms. Because the ovaries form from the indifferent embryonic gonads, they can still develop nor- mally, providing the hormonal stimuli to puberty. Choice B (Atresia of the mesonephric ducts) is incorrect. In females, the mesonephric ducts are normally suppressed. Only small vestigial structures form in association with the reproduc- tive tract. However, because the ureteric bud develops from the mesonephric duct, the renal collecting ducts would not form in case of atresia there. Choice C (Atresia of the uro- genital sinus) is incorrect. The urogenital sinus gives rise to the urinary bladder and the urethra. In addition, in females, the sinovaginal bulbs evaginate from the wall of the urogenital sinus and form the lower 2/3 of the vagina. Thus, with atresia of the urogenital sinus, the lower part of the vagina would be absent or imperforate, whereas the upper vagina, cervix, uterus, and uterine tubes would be intact. Further, one would expect malformation of the urinary bladder and/or urethra. Choice D (Imperforate hymen) is incorrect. The hymen is the thin plate of tissue that separates the vestibule from the lumen of the vagina. Remember, the sinovaginal bulbs origi- nate as solid tissue masses off the posterior wall of the uro- genital sinus. When these sinovaginal bulbs canalize to form the vaginal lumen, the hymen remains at the junction of the urogenital sinus and vagina. It usually develops one or more small openings during postnatal life. Choice E (Agenesis of the ureteric bud) is incorrect. The ureteric bud is an outgrowth from the wall of the mesonephric duct. It forms the renal col- lecting ducts, including the ureter, renal pelvis, renal calyces, and collecting tubules. Additionally, it induces the formation of the metanephric cap that forms the kidney parenchyma and the nephrons (i.e., the excretory units in the kidney).

A 52-year-old man riding his bicycle is struck by a passing car and thrown into a wall alongside the road. The damage this man sustained was to his pubic symphysis, anterior sacroiliac ligament, and sacrospinous ligament. Which of the following muscles is most likely damaged in company with the torn sacrospinous ligament? (A) Coccygeus (B) External anal sphincter (C) Iliococcygeus (D) Pubococcygeus (E) Puborectalis

The answer is A: Coccygeus. The sacrospinous ligament runs from the pelvic surface of the lower sacrum to the ischial spine. It aids in resisting loading-induced sacral rotation that could disarticulate the sacroiliac joint. The coccygeus muscle, the most posterior component of the pelvic diaphragm, attaches to the ischial spine and sacrospinous ligament. Often, the muscle is so well intertwined with the ligament that it is difficult to differentiate the two. Thus, damage to the ligament typically also includes damage to the muscle. Developmentally, the epimysium of the coccygeus muscle modifies and thickens to form the sacrospinous ligament, which underscores the inti- mate relation between these two structures. Choice B (Exter- nal anal sphincter) is incorrect. The external anal sphincter is a multilayered skeletal muscle bundle encircling the lower anal canal. It is entirely subcutaneous, having no bony or liga- mentous attachments. Choice C (Iliococcygeus) is incorrect. This component of the pelvic diaphragm originates from the tendinous arch of the obturator fascia on the lateral wall of the true pelvis. In the fetus, the muscle originally attaches to the pelvic brim along the arcuate line of the ilium, explaining the ilio-part of its name. During development, it shifts down- ward onto the thickened obturator fascia to reach its final position. Choice D (Pubococcygeus) is incorrect. This ante- rior component of the pelvic diaphragm originates from the pelvic side of the pubic bone. It is at the opposite end of the pelvic diaphragm from the sacrospinous ligament. Choice E (Puborectalis) is incorrect. This part of the pelvic diaphragm originates from the pubic bone and forms a loop around the anorectal junction with its opposite side mate. It is not related to the sacrospinous ligament.

During the physical examination of a newborn male, a pediatrician noted a painless, tense, fluctuant scrotal mass on the left side. Shining a light toward the enlarged scrotal mass caused the left side of the scrotum to light up, as depicted in the given figure. What is the most likely diagnosis? (A) Hydrocele (B) Varicocele (C) Indirect inguinal hernia (D) Direct inguinal hernia (E) Spermatocele

The answer is A: Hydrocele. A hydrocele testis is an accumulation of serous fluids within the spermatic cord of the testis. This pathology may be due to serous secretions from a remnant piece of peritoneum (tunica vaginalis), or due to communication with the abdomen via a persistent processus vaginalis. Patients with a hydrocele testis present with a pain- less swollen testis that feels like a water balloon, which may be corrected via drainage with a needle (aspiration) or sur- gery. Verification of a hydrocele can be accomplished by the diagnostic technique of transillumination, in which a light is placed near the enlarged portion of the scrotum. If the scro- tum is full of clear fluid and a hydrocele testis is present, the scrotum will light up on the affected side, which was seen in this newborn. Choice B (Varicocele) is incorrect. A varicocele is an abnormal dilation of the pampiniform plexus of veins within the spermatic cord. This scrotal mass often resembles and feels like a "bag of worms" on physical examination (see photo in question 6). Idiopathic varicocele is usually caused by defective one-way valves within the pampiniform plexus, which cause dilatation of these veins near the testis. However, a secondary varicocele will be seen following compression of the venous drainage of the testis due to the presence of a pelvic or abdominal malignancy, and this possibility must be ruled out by the physician. Varicoceles are found in the left side in approximately 98% of cases, can develop in 15% to 20% of all males, and are most frequently diagnosed in males between 15 and 25 years of age. Due to the nature (tense and fluctuant) of this scrotal mass, the fact that the scrotum is transilluminated, and the age of this newborn, a hydrocele is the most likely diagnosis. Choice C (Indirect inguinal hernia) is incorrect. An indirect inguinal hernia is usually a congenital hernia that results when abdominal cavity contents herniate through a patent processus vaginalis (open communication between the abdomen and scrotum). The hernia traverses the deep and superficial inguinal rings to descend into the scro- tum in males. An indirect inguinal hernia often presents as a soft, nontender, reducible bulge in the inguinal canal that can extend into the scrotum. It is located off the midline, especially at times of increased intra-abdominal pressure, and exists as a visible or palpable lump underneath the skin in the groin region, which was seen in this patient. Due to the nature (tense and fluctuant) of this scrotal mass and the fact that the scrotum is transilluminated by the light used by the physician, a hydrocele is the most likely diagnosis. It should be noted that indirect inguinal hernias may present with a hydrocele, so further diagnosis with an ultrasound can be utilized to rule out a herniation of abdominal contents into the scrotum. Choice D (Direct inguinal hernia) is incorrect. A direct inguinal hernia is an acquired hernia that results when abdominal cavity contents herniate through a weakness in the anterior abdominal wall in the inguinal (Hesselbach) triangle. The hernia exits through the superficial inguinal ring and often presents as a soft, nontender, reducible bulge underneath the skin overlying the superficial inguinal ring. Given the age of this patient and the presence of the mass that has descended into the scrotum, a direct inguinal hernia is not likely. Choice E (Spermatocele) is incorrect. A sperma- tocele is a cyst-like mass within the scrotum that develops in a tubule of the rete testis or the head of the epididymis. It usually contains a milky fluid and spermatozoa. These reten- tion cysts are usually not painful and can be confirmed by an ultrasound of the scrotum. Given the size of this scrotal mass, the age of the patient, and the fact that the entire scro- tum is transilluminated with the light used by the physician, a hydrocele is the most likely diagnosis.

A 25-year-old woman suffers a severe back injury in an automobile accident. Thorough examination in the ER reveals a fracture dislocation of T12 on L1 with a complete spinal cord transection that severes the sacral segments of the spinal cord. This patient will likely experience which of the following outcomes? (A) Loss of voluntary control of defecation (B) Intact ability to achieve erection of the clitoris (C) Intact voluntary control of micturition (D) Loss of ovulation (E) Loss of sensation in the skin overlying the urinary bladder

The answer is A: Loss of voluntary control of defecation. The sacral part of the spinal cord (segments S2-4) gives rise to the pudendal nerve. This nerve leaves the pelvic cavity and passes into the perineum, where it has a wide distribution. The first major branch of the pudendal nerve is the inferior anal (rec- tal) nerve to the inferior end of the anal canal, external anal sphincter, and perianal skin. The external anal sphincter is the voluntary anal sphincter, controlled by somatic motor fibers. It acts to constrict the end of the anal canal, resisting defecation. Thus, loss of the sacral segments of the spinal cord results in loss of voluntary control of defecation, as well as loss of sensation in the anal canal inferior to the pectinate line and in the perianal skin. Involuntary control of defecation is governed by the internal anal sphincter, a thickening of the circular smooth muscle layer in the wall of the upper part of the anal canal, which is controlled by autonomic (visceral motor) fibers. Choice B (Intact ability to achieve erection of the clitoris) is incorrect. Erection of the clitoris (and penis) is controlled by parasympathetic outflow from the sacral segments of the spinal cord (S2-4). Thus, destruction of the sacral cord results in loss of erection rather than intact control. Choice C (Intact voluntary control of micturition) is incor- rect. The pudendal nerve also controls the external sphincter muscle of the urethra, which is the voluntary sphincter that regulates urination (micturition), via its perineal branches. Thus, loss of the sacral spinal cord segments will cause loss of voluntary control of urination rather than intact voluntary control. Choice D (Loss of ovulation) is incorrect. Ovulation is not controlled by direct neural input. Thus, loss of the sacral spinal cord will not affect ovulation. Choice E (Loss of sensa- tion in the skin overlying the urinary bladder) is incorrect. The skin overlying the urinary bladder is that of the pubic (suprapubic; hypogastric) region of the lower abdominal wall. Cutaneous sensation here is conveyed by the ilioinguinal and iliohypogastric nerves, both branches of the first lumbar seg- ment of the spinal cord. Because only the sacral segments of the spinal cord were destroyed by the transection of the spinal cord, this sensation in the pubic region of the lower abdomi- nal wall would remain intact. Remember, the medullary cone (conus medullaris) of the spinal cord typically lies within the T12-L3 vertebral levels, so only the sacral segments of the spi- nal cord were destroyed in this injury.

A 34-year-old woman presents with pain and discomfort related to an abscess in the posterior aspect of her left labium majus. The abscess is determined to be derived from a greater vestibular (Bartholin) gland cyst. What group of lymph nodes should be checked first for lymphadenitis? (A) Superficial inguinal (B) Deep inguinal (C) Lumbar (D) External iliac (E) Internal iliac

The answer is A: Superficial inguinal. Superficial inguinal lymph nodes, which are located in the subcutaneous tissue near the termination of the great saphenous vein, receive lymphatic drainage from the lower abdominal wall, buttock, lower limb, and all of the perineum and external genitalia, except for the glans clitoris. Therefore, the superficial inguinal lymph nodes should be checked initially for lymphadenitis in the case of a cyst located on the labium majus. Choice B (Deep inguinal) is incorrect. Deep inguinal lymph nodes, which are located deep to the fascia lata and medial to the femoral vein, receive lymphatic drainage directly from the glans clitoris, deep structures of the lower limb, and indirect lymphatic drainage from the superficial inguinal lymph nodes. Because the lymphatic drainage of the posterior labium majus drains to the superficial inguinal lymph nodes initially, these nodes should be checked for lymphadenitis before the deep inguinal nodes. Choice C (Lumbar) is incorrect. Lumbar lymph nodes, which are located anterior to the lumbar vertebrae and sur- rounding the inferior vena cava and abdominal aorta, receive lymphatic drainage directly from the posterior abdominal wall, gonads (ovaries and testes), kidneys, ureters, uterus, and uterine tubes. Because the lymphatic drainage of the posterior labium majus drains to the superficial inguinal lymph nodes initially, these nodes should be checked for lymphadenitis before the lumbar nodes. Choice D (External iliac) is incor- rect. The external iliac lymph nodes, which are associated with the external iliac vein, receive lymphatic drainage from pelvic organs in direct contact with the peritoneum, including the superior bladder and superior pelvic ureters. Because the lymphatic drainage of the posterior labium majus drains to the superficial inguinal lymph nodes initially, these nodes should be checked for lymphadenitis before the external iliac nodes. Choice E (Internal iliac) is incorrect. The internal iliac lymph nodes, which are located along the internal iliac artery and its branches, receive lymphatic drainage from the pelvic vis- cera, gluteal region, and deep parts of the perineum. Because the lymphatic drainage of the posterior labium majus drains to the superficial inguinal lymph nodes initially, these nodes should be checked for lymphadenitis before the internal iliac nodes.

The sagittal MRI of a 45-year-old woman revealed severe uterine fibroids (noncancerous tumors) within the walls of the uterus. She underwent a complete hysterectomy, and the uterine artery was ligated near the junction of the uterus and vagina during the procedure. What structure would most likely be at risk during the ligation of the uterine artery? (A) Ureter (B) Pudendal nerve (C) Round ligament of the uterus (D) Obturator artery (E) Ovarian artery

The answer is A: Ureter. As it descends into the pelvis, the ureter travels anterior to the external iliac artery just distal to the bifurcation of the common iliac vessels. It continues ante- rior and medial to pass inferior to the uterine artery, which is enveloped by the transverse cervical (cardinal) ligament, near the lateral aspect of the cervix. When ligating the uterine artery during a hysterectomy (surgical excision of the uterus), a surgeon must positively identify the ureter and exclude it from the ligature. To accomplish this task and avoid iatrogenic injury to the ureter, a surgeon will ligate the uterine artery as close to the cervix as possible. The relationship of the ureter to the uterine artery can be remembered with the mnemonic "The water runs under the bridge," in which the water equals urine traveling within the ureter and the bridge is the uter- ine artery as it travels horizontally toward the uterine cervix. Choice B (Pudendal nerve) is incorrect. The pudendal nerve is located inferior to the pelvic diaphragm where it inner- vates a majority of the perineum. It arises from the ventral primary rami of spinal levels S2-4 but quickly exits the true pelvic cavity via the greater sciatic foramen. It is not located in close proximity to the uterine artery, so it is unlikely the pudendal nerve would be damaged in this procedure. Choice C (Round ligament of the uterus) is incorrect. The round liga- ment of the uterus extends from the superior part of the body of the uterus (near the uterotubal junction) to the labium majus via the inguinal canal. It is not located in close proxim- ity to the uterine artery, so it is unlikely the round ligament of the uterus would be damaged in this procedure. Choice D (Obturator artery) is incorrect. The obturator artery emerges from the anterior division of the internal iliac artery near the uterine artery; however, the paths of these arteries diverge as the arteries travel distally. The uterine artery travels medially toward the uterine cervix, whereas the obturator artery trav- els along the lateral pelvic wall toward the obturator foramen. Because the ligation of the uterine artery was located near the junction of the uterus and vagina, it is unlikely the obturator artery would be damaged. Choice E (Ovarian artery) is incor- rect. The ovarian artery travels along the lateral wall of the pelvis within the infundibulopelvic (suspensory) ligament of the ovary. It splits into tubal and ovarian branches, which later coalesce and anastomose with the proper uterine artery at the junction of the uterine tube and the fundus of the uterus. This location is well above the ligated uterine artery, so it is unlikely the ovarian artery would be damaged.

22. A 27-year-old baseball player is hit on his forearm by a high-speed ball during the World Series, and the muscles that form the floor of the cubital fossa appear to be torn. Which of the following groups of muscles have lost their functions? (A) Brachioradialis and supinator (B) Brachialis and supinator (C) Pronator teres and supinator (D) Supinator and pronator quadratus (E) Brachialis and pronator teres

The answer is B. The brachialis and supinator muscles form the floor of the cubital fossa. The brachioradialis and pronator teres muscles form the lateral and medial boundaries, respectively. The pronator quadratus is attached to the distal ends of the radius and the ulna.

23. A 23-year-old man complains of numbness on the medial side of the arm following a stab wound in the axilla. On examination, he is diagnosed with an injury of his medial brachial cutaneous nerve. In which of the following structures are the cell bodies of the damaged nerve involved in numbness located? (A) Sympathetic chain ganglion (B) Dorsal root ganglion (C) Anterior horn of the spinal cord (D) Lateral horn of the spinal cord (E) Posterior horn of the spinal cord

The answer is B. The medial brachial cutaneous nerve contains sensory (general somatic afferent [GSA]) fi bers that have cell bodies in the dorsal root ganglia, and an injury of these GSA fi bers causes numbness of the medial side of the arm. It also contains sympathetic postganglionic fi bers that have cell bodies in the sympathetic chain ganglia. The anterior horn of the spinal cord contains cell bodies of skeletal motor (general somatic efferent [GSE]) fi bers, and the lateral horn contains cell bodies of sympathetic preganglionic fi bers. The posterior horn contains cell bodies of interneurons.

While performing a newborn checkup on a baby boy, a pediatrician discovers the external urethral orifice (urethral opening) is located on the ventral (bottom) surface of the penis. What is the most likely diagnosis in this newborn? (A) Epispadias (B) Hypospadias (C) Micropenis (D) Exstrophy of the bladder (E) Cryptorchidism

The answer is B: Hypospadias. Hypospadias is a congenital defect of the external genitalia, in which the external urethral orifice is incompletely developed and located along the ventral (bottom) surface of the penis. This common anomaly of the external genitalia is seen in approximately 1:300 newborn males. It may be easy to detect due to the presence of a hooded foreskin, i.e., a foreskin that is developed normally on the dorsal (top) and sides of the glans penis but not the ventral (bottom) surface, not appreciated in the figure. Though a hooded foreskin is common in newborns with hypospadias, the foreskin can be completely formed in approximately 5% of the cases. The severity of hypospadias can also vary greatly. The mildest form, glanular (first degree) hypospadias, occurs in approximately 60% of the cases. Penile (second degree) hypospadias occurs in approximately 25% of cases and is described as the external urethral orifice being located along the anterior shaft of the penis from the midshaft of the penis to the subcoronal region (ventral to the glans penis). Penile hypospadias is illustrated in the given drawing. Finally, perineal (third degree) hypospadias results in approximately 15% of cases. In this condition, the external urethral ori- fice is located in the perineum, midscrotum, or base of the penis. During development in males, the phallus elongates, and paired urogenital (urethral) folds come together in the midline and fuse to enclose the penile urethra. Hypospa- dias results when the urogenital folds fail to fuse completely along the midline. Choice A (Epispadias) is incorrect. Epis- padias is a rare congenital defect of the external genitalia, seen in 1:120,000 newborn males. In this malformation, the external urethral orifice is located along the dorsal (top) sur- face of the penis between the pubic bones and the dorsal surface of the glans penis. Epispadias can result in a short, widened penis with an abnormal curvature, reflux nephrop- athy (backward flow of urine into the kidney), and urinary tract infections. Epispadias results when the primordium of the genital tubercle develops in the region of the urorectal septum, so when the urogenital membrane ruptures, clefts are formed on the dorsal aspect of the phallus. Epispadias is often associated with exstrophy of the bladder, in which the bladder is exposed due to a defect in the anterior abdomi- nal wall. The etiology of these defects may be due to insuf- ficient migration of mesodermal tissue to the area cranial to the genital tubercle. Due to the external urethral orifice being located on the ventral (bottom) surface of the penis in this patient, hypospadias is the correct diagnosis. Choice C (Micropenis) is incorrect. Micropenis is a penis with normal developmental features that is well below the normal size range for an infant. It is caused by lack of growth during development, often due to reduced androgen production. Treatment of micropenis in infancy can be achieved with injections of human chorionic gonadotropin or testosterone. Choice D (Exstrophy of the bladder) is incorrect. Exstro- phy of the bladder is a rare congenital abnormality in which the bladder is everted through a midline defect in the lower anterior abdominal wall. It is thought to be caused by insuf- ficient migration of mesodermal tissue to the lower portion of the abdominal wall, and it often presents with epispadias, widening of the pubic symphysis, and maldescent of the tes- tes. Choice E (Cryptorchidism) is incorrect. Cryptorchidism is the absence of one or both of the testes within the scrotum due to maldescent of the testes. This finding was not noted in this patient.

A pelvic CT scan of a 65-year-old man who has been suffering from colon cancer reveals recurrent perirectal masses, which compress the pelvic splanchnic nerves. Damage to these nerves could affect the organs supplied by which of the following arteries? (A) Right colic artery (B) Left colic artery (C) Renal arteries (D) Testicular arteries (E) Lumbar arteries

The answer is B: Left colic artery. The pelvic splanchnic nerves provide the sacral parasympathetic outflow that sup- plies the gut tube below the left colic flexure, the pelvic viscera, and the perineum. Part of the distribution route for the para- sympathetic nerves to the descending colon, sigmoid colon, and upper rectum (i.e., to most of the hindgut) is to hitch-hike along the arterial branches that supply those organs. Because the left colic artery is the parent vessel to those structures, damage to the pelvic splanchnic nerves would affect the organs supplied by the left colic artery. Choice A (Right colic artery) is incorrect. The right colic artery carries parasympathetic fibers derived from the vagus nerve to the ascending colon. This vessel/organ pairing is outside the pelvic splanchnic territory. Choice C (Renal arteries) is incorrect. These vessels supply the kidneys and suprarenal glands. Again, any parasympathetic supply to these organs is derived from vagal branches that travel with the renal arteries. Choice D (Testicular arteries) is incorrect. The gonads do not receive autonomic innervation. Thus, it is questionable if the gonadal vessels convey any para- sympathetic fibers. Choice E (Lumbar arteries) is incorrect. The lumbar arteries supply structures in the posterior abdomi- nal wall (e.g., psoas muscles, vertebral column). Parasympa- thetic nerves do not supply the body walls and limbs. Thus, the lumbar arteries do not convey parasympathetic fibers.

A 20-year-old woman delivers a stillborn infant with bilateral agenesis of the kidneys. During the later stages of pregnancy, the fetus likely also had which of the following conditions? (A) Polyhydramnios (B) Oligohydramnios (C) Renal hypoplasia (D) Pelvic kidneys (E) Polycystic kidneys

The answer is B: Oligohydramnios. Oligohydramnios is a diminution in the volume of amniotic fluid as a result of the fetus' failure to excrete urine into the amniotic sac, for example, as resulting from renal agenesis or urethral obstruction. Renal agenesis occurs when the ureteric bud fails to develop, thus preventing formation of the kidney. Normally, the fetus pro- duces urine and excretes it into the amniotic cavity, where it mixes with the amniotic fluid. The fetus drinks amniotic fluid, which is absorbed into the bloodstream, filtered at the placenta, and excreted by the fetus. Thus, the prenatal kidneys are not necessary for waste exchange but are important in regulating the volume of amniotic fluid. In renal agenesis (no kidneys), urine is not produced and not secreted, thus reducing the vol- ume of fluid in the amniotic sac (oligohydramnios). The given obstetric sonographic image, an ultrasound-based diagnostic imaging technique, depicts severe oligohydramnios associated with bilateral renal agenesis. In this image, the spine (Sp) of the infant is noted, and the shadow, which is highlighted by the black arrowheads, corresponds to the adrenal gland. The kidneys are absent. Bilateral renal agenesis is relatively rare, with a usual ultimate outcome of stillbirth or death soon after birth. Unilateral renal agenesis is relatively more common, often asymptomatic, and compatible with life, as the remain- ing kidney typically hypertrophies and produces an appropri- ate volume of urine. Choice A (Polyhydramnios) is incorrect. Polyhydramnios is an increased volume of amniotic fluid in the amniotic sac. In this case, the kidneys are normal, and urine is produced and excreted into the amniotic sac. However, obstruction (atresia) of the gut tube prevents the fetus from drinking and/or absorbing that fluid, resulting in accumula- tion of greater volume of fluid in the amniotic sac. Choice C (Renal hypoplasia) is incorrect. Renal hypoplasia is a condition of abnormally small kidneys. In renal agenesis, the kidneys are not formed. Choice D (Pelvic kidneys) is incorrect. In the case of pelvic kidneys, the kidneys are formed; however, one or both fail to ascend from their origination point in the pelvis. Thus, they are ectopic, located in the pelvis or lower lumbar region, and fully functional. Choice E (Polycystic kidneys) is incorrect. Here, the kidneys are formed. However, the collect- ing ducts and tubules are dilated, forming multiple, large fluid- filled cysts within the kidney parenchyma.

Multiple tumors in the pelvic cavity cause widespread destruction through the hypogastric plexuses in a 67-year-old man. The nerve fibers that degenerate as a result of these lesions are mainly those of which of the following? (A) Presynaptic sympathetic neurons (B) Presynaptic parasympathetic neurons (C) Postsynaptic parasympathetic neurons (D) Somatic motor neurons (E) General sensory neurons

The answer is B: Presynaptic parasympathetic neurons. The hypogastric plexuses are an extensive network of visceral motor (autonomic—both sympathetic and parasympa- thetic) and visceral sensory fibers within the pelvic cavity that supply the pelvic viscera and the perineum. The para- sympathetic components are mainly presynaptic neurons derived from the pelvic splanchnic nerves, which synapse in the intramural ganglia of the target organs. The sympa- thetic components are mainly postsynaptic neurons derived from the aortic (intermesenteric) plexuses in the abdomen, where they synapsed with presynaptic elements. Choice A (Presynaptic sympathetic neurons) is incorrect. The pre- synaptic sympathetic neurons that signal to pelvic viscera originate in the low thoracic and upper lumbar segments of the spinal cord and send axonal processes through the lumbar splanchnic nerves to the aortic (intermesenteric) plexuses, where they synapse with postsynaptic elements. Choice C (Postsynaptic parasympathetic neurons) is incor- rect. The pelvic splanchnic nerves send presynaptic fibers through the hypogastric plexuses to the hindgut, pelvic viscera, and perineum. These neurons synapse with post- synaptic cells in the intramural (terminal) ganglia in the walls of the target organs. The postsynaptic neurons make up a large component of the enteric nervous system in the walls of the gut tube. Choice D (Somatic motor neurons) is incorrect. Somatic motor neurons supply skeletal muscles in the pelvic walls and floor and perineum. They are con- veyed via the branches of the sacral plexus (e.g., pudendal nerve) which, strictly speaking, are not components of the hypogastric plexuses. Choice E (General sensory neurons) is incorrect. General sensory (general somatic afferent) neurons from the pelvic walls and floor as well as the perineum travel through the branches of the sacral plexus, in company with the somatic motor fibers to those areas. However, the visceral sensory fibers from pelvic viscera travel with visceral motor (autonomic) fibers through the hypogastric plexuses.

To reduce pain during her childbirth, an obstetrician performs a local nerve block on his 25-year-old patient while she is in the lithotomy position, as depicted in the given figure. Which of the following structures will remain fully sensitive following administration of this local anesthetic? (A) Lower anal canal (B) Rectum (C) Perinealbody (D) Urogenitaldiaphragm (E) Vulva

The answer is B: Rectum. The illustration shows a pudendal nerve block, which anesthetizes the area innervated by the pudendal nerve, that is, the majority of the perineum, includ- ing the lower vagina and lower anal canal. The rectum lies above the perineum and is supplied by visceral sensory fibers from the hypogastric plexuses rather than by the pudendal nerve. Thus, it retains visceral sensation following a pudendal block. The block is accomplished by injecting a local anesthetic into the tissues around the pudendal nerve where it crosses the sacrospinous ligament near the ischial spine. Achieving total anesthesia of the perineum additionally requires blocking the ilioinguinal nerve (covering the anterior perineum) and perhaps the posterior cutaneous nerve of the thigh (for the extreme posterior perineum). Choice A (Lower anal canal) is incorrect. The lower anal canal (i.e., the area below the pec- tinate line) receives general sensory innervation through the inferior anal (rectal) nerve, a branch of the pudendal nerve. Thus, this area is anesthetized by a pudendal block. Choice C (Perineal body) is incorrect. The perineal body is a dense con- nective tissue mass in the center of the perineum. This area is innervated by branches of the pudendal nerve. Choice D (Urogenital diaphragm) is incorrect. The urogenital diaphragm is a musculofascial complex within the urogenital triangle of the perineum. Its major component is the external urethral sphincter. This area is supplied by the perineal branches of the pudendal nerve. Remember, the anatomical concept of the urogenital diaphragm has changed considerably in recent years. It is important to reference the most current texts for a proper understanding of the morphology here so as not to be confused by the traditional, inaccurate description. Choice E (Vulva) is incorrect. The vulva is the part of the perineum in females containing the external genitalia, which includes the mons pubis, labia majora and minora, vestibule of the vagina, clitoris, and bulbs of the vestibule. The vulva is innervated largely by the dorsal nerve of the clitoris and the perineal nerves, all branches of the pudendal nerve. However, of note, the anterior aspect of the vulva (especially the mons pubis and anterior labia majora) is mainly supplied by the ilioinguinal nerve. Thus, a pudendal nerve block greatly reduces sensation in the vulva but likely does not provide total anesthesia.

26. A patient with a deep stab wound in the middle of the forearm has impaired movement of the thumb. Examination indicates a lesion of the anterior interosseous nerve. Which of the following muscles is paralyzed? (A) Flexor pollicis longus and brevis (B) Flexor pollicis longus and opponens pollicis (C) Flexor digitorum profundus and pronator quadratus (D) Flexor digitorum profundus and superfi - cialis (E) Flexor pollicis brevis and pronator quadratus

The answer is C. The anterior interosseous nerve is a branch of the median nerve and supplies the flexor pollicis longus, half of the flexor digitorum profundus, and the pronator quadratus. The median nerve supplies the pronator teres, flexor digitorum superfi cialis, palmaris longus, and fl exor carpi radialis muscles. A muscular branch (the recurrent branch) of the median nerve innervates the thenar muscles.

A 65-year-old woman suffers chronic constipation. Which of the following events occurs when she strains heavily in attempting to pass her stool? (A) Relaxation of the pubococcygeus (B) Relaxation of the puborectalis (C) Relaxation of the coccygeus (D) Contraction of the external anal sphincter (E) Contraction of the internal anal sphincter

The answer is B: Relaxation of the puborectalis. The pub- orectalis is the component of the levator ani that joins with its opposite side mate to form a U-shaped sling around the anorectal junction, as shown in the given figure. Tension of this muscle produces the anorectal angle (perineal flexure), which aids significantly in resisting fecal movement. Relax- ation of the puborectalis sling causes the anorectal junction to straighten, thus allowing defecation to proceed. The other parts of the levator ani normally contract during defecation. This action raises the pelvic floor and assists the abdomi- nal wall muscles in increasing intra-abdominal pressure and compressing the abdominopelvic contents, thus assisting in expelling feces during defecation. Thus, the complementary actions of relaxation of the puborectalis plus contraction of the remainder of the levator ani are important in voluntary control of defecation. Choice A (Relaxation of the pubococcygeus) is incorrect. The pubococcygeus is the major component of the levator ani. It contracts to raise the pelvic floor during def- ecation and other actions requiring increased intra-abdominal pressure (e.g., forced expiration, sneezing, vomiting, and uri- nation). Choice C (Relaxation of the coccygeus) is incorrect. The coccygeus is the most posterior component of the pelvic diaphragm. However, it is not part of the levator ani portion of the pelvic diaphragm. Even so, it contracts in the same fashion as the levator ani during defecation to raise and tense the pel- vic floor. Choice D (Contraction of the external anal sphinc- ter) is incorrect. The external anal sphincter is the voluntary sphincter surrounding the lower anal canal. It must relax in order to allow dilation of the anal canal and passage of feces. Choice E (Contraction of the internal anal sphincter) is incor- rect. The internal anal sphincter is the involuntary sphincter in the lower rectal wall. It relaxes during defecation to allow passage of feces into the anal canal.

As part of a general pelvic examination, an obstetrician measures the diagonal conjugate diameter of the true pelvis in his 26-year-old pregnant patient. This measurement is taken between which of the following points? (A) Ischial spine to the opposite ischial spine (B) Sacral promontory to the inferior edge of the pubic symphysis (C) Sacral promontory to the posterosuperior edge of the pubic symphysis (D) Sacroiliac joint to the posterior side of the opposite side body of the pubis (E) Tip of the coccyx to the pubic symphysis

The answer is B: Sacral promontory to the inferior edge of the pubic symphysis. The true (obstetrical) conjugate diam- eter of the true pelvis (see Discussion point C below) can- not be measured directly in a manual pelvic exam because of the presence of the urinary bladder. Therefore, the next best approximation of that, the diagonal conjugate, is mea- sured instead (see the given figure). This measurement is taken by first palpating the sacral promontory with the tip of the middle finger of the gloved examining hand. Next, the other hand marks the position of the inferior edge of the pubic symphysis on the examining hand. Finally, after withdrawing the examining hand, the distance from the tip of the index finger to the marked point for the symphysis is measured to obtain an estimate of the true conjugate. This distance should be 11.5 cm or greater. The diameters of the true pelvis are important measurements in determining the capacity of the pelvic canal for childbirth. Choice A (Ischial spine to the opposite ischial spine) is incorrect. This mea- surement is the interspinous distance in the pelvis. Because the ischial spines project into the true pelvis and toward one another, this distance is normally the narrowest part of the pelvic (birth) canal. However, the interspinous length is not a fixed distance because of normal relaxation of pel- vic ligaments and added joint mobility during pregnancy. Choice C (Sacral promontory to the posterosuperior edge of the pubic symphysis) is incorrect. This measurement is the true (obstetrical) conjugate of the true pelvis. It is the nar- rowest fixed distance (diameter) of the true pelvis and indi- cates the smallest anteroposterior space available for the fetal head to pass during vaginal delivery. Choice D (Sacroiliac joint to the posterior side of the opposite side body of the pubis) is incorrect. This distance is the oblique diameter of the true pelvis. It is a diameter that usually is noted via radio- graphic imaging rather than manual palpation. Choice E (Tip of the coccyx to the pubic symphysis) is incorrect. This distance indicates the anteroposterior diameter of the pelvic outlet. However, it is not a fixed distance because of enhanced pelvic mobility during pregnancy.

A pregnant woman in active labor receives an epidural anesthetic to relieve pain from her uterine contractions and cervical dilation in preparation for the birth of her child. Pain derived from the labor contractions of the upper uterus is referred to what spinal cord level? (A) T6 (B) T12 (C) L4 (D) S1 (E) S3

The answer is B: T12. Visceral sensory fibers relaying pain from the labor contractions of the upper uterus are referred to the T11-L2 spinal cord levels because these organs are located above the pelvic pain line, as defined by their being in con- tact with or above the peritoneum. Because visceral sensory fibers follow the visceral motor (autonomic) fibers retrograde, visceral pain from the contractions of the upper uterus fol- low the presynaptic sympathetic nerves of the abdominopel- vic splanchnic nerves retrograde to spinal cord levels T11-L2. Conversely, pelvic organs located below the pelvic pain line are, by definition, located below, and not in contact with, the peri- toneum. The visceral sensory fibers from these organs follow the presynaptic parasympathetic nerves of the pelvic splanch- nic nerves retrograde to spinal cord levels S2-4. Therefore, labor pains derived from the contractions of the upper uterus will be referred by visceral sensory fibers to the T12 spinal cord level. Choice A (T6) is incorrect. Visceral sensory fibers associated with the T6 vertebral level would be involved with conveying visceral pain associated with the heart and lungs, due to traveling retrograde with cardiopulmonary splanch- nic nerves derived from postsynaptic sympathetic fibers from T1-6. Choice C (L4) is incorrect. Visceral sensory fibers car- rying pain from visceral organs and involuntary (smooth or cardiac) muscle follow retrograde the pathway of the visceral motor (or autonomic) nerve fibers. There are no presynaptic autonomic nerve fibers that arise from the L4 vertebral level, so there are no visceral sensory nerves associated with this ver- tebral level. Choice D (S1) is incorrect. Visceral sensory fibers carrying pain from visceral organs and involuntary (smooth or cardiac) muscle follow retrograde the pathway of the visceral motor (or autonomic) nerve fibers. There are no presynaptic autonomic nerve fibers that arise from the S1 vertebral level, so there are no visceral sensory nerves associated with this ver- tebral level. Choice E (S3) is incorrect. Pain derived from labor contractions of the upper uterus would not refer to vertebral level S3, because the upper uterus is in contact with the peri- toneum, which means it is located above the pelvic pain line. However, labor pains from the dilation of the uterine cervix, an important stage of labor, will follow the presynaptic para- sympathetic nerves of the pelvic splanchnic nerves retrograde to the vertebral levels of S2-4. Therefore, the uterine cervix is a pelvic organ located below the pelvic pain line, which means it is located below, and not in contact with, the peritoneum. Therefore, the vertebral level of S3 would receive visceral sen- sory fibers conveying pain from labor due to the dilation of the uterine cervix; however, this question asked specifically about labor pains derived from the contractions of the upper uterus, which would be referred to the T11-L2 vertebral levels.

During dissection of deep pelvic lymph nodes in a 60-year-old man, the surgeon accidentally severs the internal pudendal artery at its origin. Which of the following structures will maintain its normal blood supply immediately following this lesion? (A) Penis (B) Urinary bladder (C) Analcanal (D) Scrotum (E) Urogenital diaphragm

The answer is B: Urinary bladder. In males, the urinary bladder is supplied by the superior and inferior vesicle arteries, both of which usually are branches of the anterior division of the internal iliac artery. The internal pudendal artery is usually a terminal branch of the anterior division of the internal iliac artery. However, rather than supplying pelvic viscera, it exits the pelvis, enters the posterolateral aspect of the perineum, and provides the main supply to that entire region. Severing the internal pudendal artery usually does not alter blood flow to the bladder. However, remember that the internal iliac arterial tract is the most variable branching network of all the arteries. In this variation pattern, the internal pudendal artery may be closely related to the inferior vesicle artery, or, may be well removed from it. Choice A (Penis) is incorrect. The penis is supplied by terminal branches of the internal pudendal artery within the perineum. Thus, it will be deprived of blood in case of lesion at the origin of the internal pudendal artery. Choice C (Anal canal) is incorrect. The anal canal is supplied by the inferior anal (rectal) branches of the internal pudendal artery within the perineum. It will be deprived of blood in case of lesion at the origin of the internal pudendal artery. Choice D (Scrotum) is incorrect. The scrotum is supplied by posterior scrotal branches of the internal pudendal artery and anterior scrotal branches of the external pudendal artery (a branch of the femoral artery). Thus, lesion of the internal pudendal artery will reduce blood flow to the scrotum. Choice E (Urogenital diaphragm) is incorrect. The structures of the anatomically questionable urogenital diaphragm are supplied by the perineal branches of the internal pudendal artery. Thus, this area will be deprived of blood following lesion of the internal pudendal artery.

A 24-year-old man comes to his family physician with an irregular scrotal swelling on his left. He reports aching pain within the scrotum and a feeling of heaviness in his left testis. During the physical examination, the physician reports that the scrotal mass feels like a bag of worms. What is the most likely diagnosis? (A) Hydrocele (B) Varicocele (C) Testicular cancer (D) Indirect inguinal hernia (E) Spermatocele

The answer is B: Varicocele. A varicocele is an abnormal dila- tion of the pampiniform plexus of veins, which travels within the spermatic cord. This scrotal mass often resembles and feels like a "bag of worms" on physical examination, as seen in the photo. Idiopathic varicocele is usually caused by defec- tive one-way valves within the pampiniform plexus, which cause dilatation of these veins near the testis. However, a sec- ondary varicocele will be seen following compression of the venous drainage of the testis due to the presence of a pelvic or abdominal malignancy, and this possibility must be ruled out by the physician. Varicoceles are found in the left side in approximately 98% of cases, can develop in 15% to 20% of all males, and are most frequently diagnosed between 15 and 25 years of age. Choice A (Hydrocele) is incorrect. A hydrocele testis is a pathological accumulation of serous fluids within the spermatic cord due to serous secretions from a remnant piece of peritoneum, termed the tunica vaginalis. Patients with a hydrocele testis present with a painless swollen testis that feels like a water balloon, which may be corrected via drain- age with a needle (aspiration) or surgery. To identify a hydro- cele, a diagnostic technique called transillumination (shining a light through the enlarged portion of the scrotum) can be implemented. If the scrotum is full of clear fluid, the scrotum will light up. A hydrocele testis is not likely in this patient due to the irregular nature of the scrotal mass seen within the photo. Choice C (Testicular cancer) is incorrect. Testicular cancer presents with a lump on or hardening of the testis lead- ing to unilateral enlargement of the affected testis, abnormal sensitivity within the testes, dull pain within the lower back, abdomen, or groin, loss of libido, and general weakness and tiredness. The extent of the scrotal mass and its irregular nature make the diagnosis of testicular cancer unlikely. If testicular cancer were present, it would not grow to this size without the patient noticing its presence. Moreover, the cardinal sign for testicular cancer is unilateral enlargement of the testis; how- ever, in a varicocele, the testis is usually smaller on the affected side. Choice D (Indirect inguinal hernia) is incorrect. An indi- rect inguinal hernia is usually a congenital hernia that results when abdominal cavity contents herniate through a patent processus vaginalis, or in an adult, travel through the inguinal canal. The hernia traverses the deep and superficial inguinal rings to descend into the scrotum in males. The hernia often presents as a visible or palpable lump underneath the skin in the groin region, which was seen in this patient. However, given the irregular nature of this scrotal mass and the physi- cian's observation that the mass felt like a "bag of worms," a varicocele is more likely. Choice E (Spermatocele) is incorrect. A spermatocele is a cyst-like mass within the scrotum that develops in a tubule of the rete testis or the head of the epididymis. It usually contains a milky fluid and spermatozoa. These retention cysts are usually not painful and can be con- firmed by an ultrasound of the scrotum. Given the irregular nature, size, and "bag of worms" description of the scrotal mass in this patient, a varicocele is the most likely diagnosis.

1. A 21-year-old patient has a lesion of the upper trunk of the brachial plexus (Erb- Duchenne paralysis). Which of the following is the most likely diagnosis? (A) Paralysis of the rhomboid major (B) Inability to elevate the arm above the horizontal (C) Arm tending to lie in medial rotation (D) Loss of sensation on the medial side of the arm (E) Inability to adduct the thumb

The answer is C. A lesion of the upper trunk of the brachial plexus results in a condition called "waiter's tip hand," in which the arm tends to lie in medial rotation because of paralysis of lateral rotators and abductors of the arm. The long thoracic nerve, which arises from the root (C5-C7) of the brachial plexus, innervates the serratus anterior muscle that can elevate the arm above the horizontal. The dorsal scapular nerve, which arises from the root (C5), innervates the rhomboid major. The medial side of the arm receives cutaneous innervation from the medial brachial cutaneous nerve of the medial cord. The adductor pollicis is innervated by the ulnar nerve.

14. A 12-year-old boy walks in; he fell out of a tree and fractured the upper portion of his humerus. Which of the following nerves are intimately related to the humerus and are most likely to be injured by such a fracture? (A) Axillary and musculocutaneous (B) Radial and ulnar (C) Radial and axillary (D) Median and musculocutaneous (E) Median and ulnar

The answer is C. The axillary nerve passes posteriorly around the surgical neck of the humerus, and the radial nerve lies in the radial groove of the middle of the shaft of the humerus. The ulnar nerve passes behind the medial epicondyle, and the median nerve is vulnerable to injury by supracondylar fracture of the humerus, but these nerves lie close to or in contact with the lower portion of the humerus. The musculocutaneous nerve is not in direct contact with the humerus.

29. A secretary comes in to your offi ce complaining of pain in her wrists from typing all day. You determine that she likely has carpal tunnel syndrome. Which of the following conditions would help you determine the diagnosis? (A) Inability to adduct the little fi nger (B) Inability to fl ex the DIP joint of the ring fi nger (C) Flattened thenar eminence (D) Loss of skin sensation of the medial one and one-half fi ngers (E) Atrophied adductor pollicis muscle

The answer is C. The carpal tunnel contains the median nerve and the tendons of flexor pollicis longus, fl exor digitorum profundus, and fl exor digitorum superfi cialis muscles. Carpal tunnel syndrome results from injury to the median nerve, which supplies the thenar muscle. Thus, injury to this nerve causes the fl attened thenar eminence. The middle fi nger has no attachment for the adductors. The ulnar nerve innervates the medial half of the flexor digitorum profundus muscle, which allows flexion of the DIP joints of the ring and little fingers. The ulnar nerve supplies the skin over the medial one and one-half fingers and adductor pollicis muscle.

12. An automobile body shop worker has his middle finger crushed while working on a transmission. Which of the following muscles is most likely to retain function? (A) Extensor digitorum (B) Flexor digitorum profundus (C) Palmar interosseous (D) Dorsal interosseous (E) Lumbrical

The answer is C. The extensor digitorum, fl exor digitorum profundus, dorsal interosseous, and lumbrical muscles are attached to the middle digit, but no palmar interosseous muscle is attached to the middle digit.

27. A 29-year-old patient comes in; he cannot fl ex the distal interphalangeal (DIP) joint of the index fi nger. His physician determines that he has nerve damage from a supracondylar fracture. Which of the following conditions is also a symptom of this nerve damage? (A) Inability to fl ex the DIP joint of the ring fi nger (B) Atrophy of the hypothenar eminence (C) Loss of sensation over the distal part of the second digit (D) Paralysis of all the thumb muscles (E) Loss of supination

The answer is C. The fl exor digitorum profundus muscle fl exes the DIP joints of the index and middle fi ngers and is innervated by the median nerve, which also supplies sensation over the distal part of the second digit. The same muscle fl exes the DIP joints of the ring and little fi ngers but receives innervation from the ulnar nerve, which also innervates the hypothenar muscles. The median nerve innervates the thenar muscles. The radial nerve innervates the supinator, abductor pollicis longus, and extensor pollicis longus and brevis muscles. The ulnar nerve innervates the adductor pollicis. The musculocutaneous nerve supplies the biceps brachii that can supinate the arm.

9. A 21-year-old woman walks in with a shoulder and arm injury after falling during horseback riding. Examination indicates that she cannot adduct her arm because of paralysis of which of the following muscles? (A) Teres minor (B) Supraspinatus (C) Latissimus dorsi (D) Infraspinatus (E) Serratus anterior

The answer is C. The latissimus dorsi adducts the arm, and the supraspinatus muscle abducts the arm. The infraspinatus and the teres minor rotate the arm laterally. The serratus anterior rotates the glenoid cavity of the scapula upward, abducts the arm, and elevates it above a horizontal position.

17. A 27-year-old pianist with a known carpal tunnel syndrome experiences diffi culty in fi nger movements. Which of the following intrinsic muscles of her hand is paralyzed? (A) Palmar interossei and adductor pollicis (B) Dorsal interossei and lateral two lumbricals (C) Lateral two lumbricals and opponens pollicis (D) Abductor pollicis brevis and palmar interossei (E) Medial two and lateral two lumbricals

The answer is C. The median nerve innervates the abductor pollicis brevis, opponens pollicis, and two lateral lumbricals. The ulnar nerve innervates all interossei (palmar and dorsal), the adductor pollicis, and the two medial lumbricals.

16. A patient with Bennett's fracture (a fracture of the base of the fi rst metacarpal bone) experiences an impaired thumb movement. Which of the following intrinsic muscles of the thumb is most likely injured? (A) Abductor pollicis brevis (B) Flexor pollicis brevis (superfi cial head) (C) Opponens pollicis (D) Adductor pollicis (E) Flexor pollicis brevis (deep head)

The answer is C. The opponens pollicis inserts on the first metacarpal. All other intrinsic muscles of the thumb, including the abductor pollicis brevis, the fl exor pollicis brevis, and the adductor pollicis muscles, insert on the proximal phalanges.

5. A 27-year-old patient presents with an inability to draw the scapula forward and downward because of paralysis of the pectoralis minor. Which of the following would most likely be a cause of his condition? (A) Fracture of the clavicle (B) Injury to the posterior cord of the brachial plexus (C) Fracture of the coracoid process (D) Axillary nerve injury (E) Defects in the posterior wall of the axilla

The answer is C. The pectoralis minor inserts on the coracoid process, originates from the second to the fifth ribs, and is innervated by the medial and lateral pectoral nerves that arise from the medial and lateral cords of the brachial plexus. It depresses the shoulder and forms the anterior wall of the axilla. The pectoralis minor has no attachment on the clavicle.

20. A 17-year-old boy with a stab wound received multiple injuries on the upper part of the arm and required surgery. If the brachial artery were ligated at its origin, which of the following arteries would supply blood to the profunda brachii artery? (A) Lateral thoracic (B) Subscapular (C) Posterior humeral circumfl ex (D) Superior ulnar collateral (E) Radial recurrent

The answer is C. The posterior humeral circumflex artery anastomoses with an ascending branch of the profunda brachii artery, whereas the lateral thoracic and subscapular arteries do not. The superior ulnar collateral and radial recurrent arteries arise inferior to the origin of the profunda brachii artery.

32. A patient has a torn rotator cuff of the shoulder joint as the result of an automobile accident. Which of the following muscle tendons is intact and has normal function? (A) Supraspinatus (B) Subscapularis (C) Teres major (D) Teres minor (E) Infraspinatus

The answer is C. The rotator cuff consists of the tendons of the supraspinatus, infraspinatus, subscapularis, and teres minor muscles. It stabilizes the shoulder joint by holding the head of the humerus in the glenoid cavity during movement. The teres major inserts on the medial lip of the intertubercular groove of the humerus.

43. A rock climber falls on his shoulder, resulting in a chipping off of the lesser tubercle of the humerus. Which of the following structures would most likely have structural and functional damage? (A) Supraspinatus muscle (B) Infraspinatus muscle (C) Subscapularis muscle (D) Teres minor muscle (E) Coracohumeral ligament

The answer is C. The subscapularis muscle inserts on the lesser tubercle of the humerus. The supraspinatus, infraspinatus, and teres minor muscles insert on the greater tubercle of the humerus. The coracohumeral ligament attaches to the greater tubercle.

44. A 22-year-old female Macarena dancer fell from the stage and complains of elbow pain and inability to supinate her forearm. Which of the following nerves are most likely injured from this accident? (A) Median and ulnar nerves (B) Axillary and radial nerves (C) Radial and musculocutaneous nerves (D) Ulnar and axillary nerves (E) Musculocutaneous and median nerves

The answer is C. The supinator and biceps brachii muscles supinate the forearm. The supinator is innervated by the radial nerve, and the biceps brachii is innervated by the musculocutaneous nerve.

28. A 27-year-old man with cubital tunnel syndrome complains of numbness and tingling in the ring and little fi nger and back and sides of his hand because of damage to a nerve in the tunnel at the elbow. Which of the following muscles is most likely to be paralyzed? (A) Flexor digitorum superfi cialis (B) Opponens pollicis (C) Two medial lumbricals (D) Pronator teres (E) Supinator

The answer is C. The ulnar nerve innervates the two medial lumbricals. However, the median nerve innervates the two lateral lumbricals, the fl exor digitorum superfi cialis, the opponens pollicis, and the pronator teres muscles.

30. A man is unable to hold typing paper between his index and middle fi ngers. Which of the following nerves was likely injured? (A) Radial nerve (B) Median nerve (C) Ulnar nerve (D) Musculocutaneous nerve (E) Axillary nerve

The answer is C. To hold a typing paper, the index fi nger is adducted by the palmar interosseous muscle, and the middle fi nger is abducted by the dorsal interosseous muscle. Both muscles are innervated by the ulnar nerve.

A 72-year-old man comes to his urologist complaining of urinary frequency, urgency, weak urinary stream, hesitancy, and straining to void. A rectal examination and blood tests revealing normal prostate-specific antigen levels lead to a diagnosis of benign prostatic hyperplasia. The man undergoes a prostatectomy to remove the prostate gland. During the surgical procedure, the prostatic nerve plexus is damaged. What is the most likely complication of this surgery? (A) Loss of sensation from the anus (B) Fecalincontinence (C) Erectile dysfunction (D) Loss of sensation from the posterior scrotum (E) Vasomotor dysfunction in the rectum

The answer is C: Erectile dysfunction. The cavernous nerves of the penis supply parasympathetic (visceral motor; autonomic) fibers to erectile (cavernous) tissues of the penis, stimulating penile erection. These nerves arise from the pro- static portion of the pelvic nerve plexus, which surrounds the capsule of the prostate, and it is at this location that the cavernous nerves can be damaged during prostatectomy. After enveloping the prostatic capsule, the cavernous nerves descend to penetrate the perineal membrane to reach erectile tissues of the penis, providing parasympathetic stimulation. This innervation causes the helicine arteries of the penis, the coiled terminal branches of the deep and dorsal arteries of the penis, to uncoil, enabling blood at arterial pressure to fill the cavernous tissue causing penile erection. Thus, damage to these nerves would most likely result in erectile dysfunction. Choice A (Loss of sensation from the anus) is incorrect. The sensation from the skin surrounding the anus is conveyed by the inferior anal (rectal) nerves, which are branches of the pudendal nerve. The pudendal nerve exits the pelvis via the greater sciatic notch and enters the perineum through the lesser sciatic notch to supply the external anal sphincter and its surrounding skin, the superficial and deep perineal com- partments, and the external genitalia. Neither the inferior anal or pudendal nerves would be at risk during a prostatectomy due to their posterior locations. Choice B (Fecal incontinence) is incorrect. Fecal incontinence may be due to loss of inner- vation to the external anal sphincter, which is supplied by the inferior anal (rectal) nerves, which are branches from the pudendal nerve. Due to the posterior location of the pudendal nerve, it would not be at risk during a prostatectomy. Choice D (Loss of sensation from the posterior scrotum) is incorrect. The sensation from the posterior scrotum is supplied by the posterior scrotal nerve, which is a superficial perineal branch of the pudendal nerve. Due to the inferior and posterior loca- tions of the posterior scrotal and pudendal nerves, they would not be at risk during a prostatectomy. Choice E (Vasomotor dysfunction in the rectum) is incorrect. The vasomotor func- tions of the rectum are supplied by the rectal plexus, which is derived from the inferior hypogastric plexus. The rectal plexus contains visceral motor (parasympathetic and sympa- thetic) fibers, but due to its proximity to the lateral sides of the rectum, it would be unlikely for it to be at risk during a prostatectomy.

A 31-year-old woman delivers her first child via natural childbirth, without anesthetics. During the delivery, she experiences painful spasms in the muscles in the medial part of her right thigh and paresthesia in the skin in the medial thigh. Following delivery, these conditions gradually dissipate. Which of the following nerves may have been compressed during this childbirth? (A) Inferior gluteal nerve (B) Pudendal nerve (C) Obturator nerve (D) Sciatic nerve (E) Pelvic splanchnic nerves

The answer is C: Obturator nerve. The obturator nerve branches from the lumbar plexus in the false (greater) pelvis of the lower abdomen. It descends into the true pelvis, runs across the surface of the obturator internus muscle on the lateral pelvic wall, and passes through the obturator canal to enter the medial thigh. In the thigh, the obturator nerve supplies the adductor muscles in the medial compartment of the thigh and also a cutaneous area in the medial thigh. During childbirth, the nerve may be compressed against the lateral wall of the pelvis by the passing baby. Choice A (Inferior gluteal nerve) is incorrect. The inferior gluteal nerve arises from the sacral plexus and immediately exits the pelvis through the greater sciatic foramen to reach the gluteal region. In the buttock, it supplies the gluteus maximus muscle. Choice B (Pudendal nerve) is incorrect. This nerve arises from the sacral plexus, leaves the pelvis through the greater sciatic foramen, curls around the ischial spine, and enters the perineum. It is the pri- mary motor and sensory nerve to the perineum. Choice D (Sci- atic nerve) is incorrect. The sciatic nerve is the largest nerve in the body. It originates from the sacral plexus, leaves the pelvis through the greater sciatic foramen, runs through the buttock, and into the posterior thigh. The sciatic nerve supplies the hip joint, the muscles in the posterior compartment of the thigh, all muscles below the knee, and considerable cutaneous areas below the knee. Choice E (Pelvic splanchnic nerves) is incorrect. These nerves are presynaptic parasympathetic fibers arising off the sacral plexus. They distribute through the hypogastric plexuses to the hindgut organs and the pelvic and perineal viscera. Parasympathetic nerves do not supply the body wall or limbs.

39. A 7-year-old boy falls from a tree house and is brought to the emergency department of a local hospital. On examination, he has weakness in rotating his arm laterally because of an injury of a nerve. Which of the following conditions is most likely to cause a loss of this nerve function? (A) Injury to the lateral cord of the brachial plexus (B) Fracture of the anatomic neck of the humerus (C) Knife wound on the teres major muscle (D) Inferior dislocation of the head of the humerus (E) A tumor in the triangular space in the shoulder region

The answer is D. Inferior dislocation of the head of the humerus may damage the axillary nerve, which arises from the posterior cord of the brachial plexus, runs through the quadrangular space accompanied by the posterior humeral circumfl ex vessels around the surgical neck of the humerus, and supplies the deltoid and teres minor, which are lateral rotators of the arm.

11. The police bring in a murder suspect who has been in a gunfi ght with a police offi cer. The suspect was struck by a bullet in the arm; his median nerve has been damaged. Which of the following symptoms is likely produced by this nerve damage? (A) Waiter's tip hand (B) Claw hand (C) Wrist drop (D) Ape hand (E) Flattening of the hypothenar eminence

The answer is D. Injury to the median nerve produces the ape hand (a hand with the thumb permanently extended). Injury to the radial nerve results in loss of wrist extension, leading to wrist drop. Damage to the upper trunk of the brachial plexus produces waiter's tip hand. A claw hand and flattening of the hypothenar eminence or atrophy of the hypothenar muscles result from damage to the ulnar nerve.

15. A man injures his wrist on broken glass. Which of the following structures entering the palm superfi cial to the fl exor retinaculum may be damaged? (A) Ulnar nerve and median nerve (B) Median nerve and fl exor digitorum profundus (C) Median nerve and fl exor pollicis longus (D) Ulnar artery and ulnar nerve (E) Ulnar nerve and fl exor digitorum superfi cialis

The answer is D. Structures entering the palm superficial to the fl exor retinaculum include the ulnar nerve, ulnar artery, palmaris longus tendon, and palmar cutaneous branch of the median nerve. The median nerve, the fl exor pollicis longus, and the fl exor digitorum superfi cialis and profundus run deep to the fl exor retinaculum.

8. A patient comes in complaining that she cannot fl ex her proximal interphalangeal joints. Which of the following muscles appear(s) to be paralyzed on further examination of her fi nger? (A) Palmar interossei (B) Dorsal interossei (C) Flexor digitorum profundus (D) Flexor digitorum superfi cialis (E) Lumbricals

The answer is D. The flexor digitorum superficialis muscle fl exes the proximal interphalangeal joints. The fl exor digitorum profundus muscle fl exes the DIP joints. The palmar and dorsal interossei and lumbricals can fl ex metacarpophalangeal joints and extend the interphalangeal joints. The palmar interossei adduct the fi ngers, and the dorsal interossei abduct the fi ngers.

21. A 23-year-old woman who receives a deep cut to her ring fi nger by a kitchen knife is unable to move the metacarpophalangeal joint. Which of the following pairs of nerves was damaged? (A) Median and ulnar (B) Radial and median (C) Musculocutaneous and ulnar (D) Ulnar and radial (E) Radial and axillary

The answer is D. The metacarpophalangeal joint of the ring fi nger is fl exed by the lumbrical, palmar, and dorsal interosseous muscles, which are innervated by the ulnar nerve. The extensor digitorum, which is innervated by the radial nerve, extends this joint. The musculocutaneous and axillary nerves do not supply muscles of the hand. The median nerve supplies the lateral two lumbricals, which can fl ex metacarpophalangeal joints of the index and middle fingers.

31. The victim of an automobile accident has a destructive injury of the proximal row of carpal bones. Which of the following bones is most likely damaged? (A) Capitate (B) Hamate (C) Trapezium (D) Triquetrum (E) Trapezoid

The answer is D. The proximal row of carpal bones consists of the scaphoid, lunate, triquetrum, and pisiform bones, whereas the distal row consists of the trapezium, trapezoid, capitate, and hamate bones.

13. A 14-year-old boy falls on his outstretched hand and has a fracture of the scaphoid bone. The fracture is most likely accompanied by a rupture of which of the following arteries? (A) Brachial artery (B) Ulnar artery (C) Deep palmar arterial arch (D) Radial artery (E) Princeps pollicis artery

The answer is D. The scaphoid bone forms the fl oor of the anatomic snuffbox, through which the radial artery passes to enter the palm. The radial artery divides into the princeps pollicis artery and the deep palmar arch.

46. Which of the following muscles causes upward displacement of the medial fragment of a fractured clavicle? (A) Pectoralis major (B) Deltoid (C) Trapezius (D) Sternocleidomastoid (E) Scalenus anterior

The answer is D. The sternocleidomastoid muscle is attached to the superior border of the medial third of the clavicle, and the medial fragment of a fractured clavicle is displaced upward by the pull of the muscle.

24. A 38-year-old homebuilder was involved in an accident and is unable to supinate his forearm. Which of the following nerves are most likely damaged? (A) Suprascapular and axillary (B) Musculocutaneous and median (C) Axillary and radial (D) Radial and musculocutaneous (E) Median and ulnar

The answer is D. The supinator and biceps brachii muscles, which are innervated by the radial and musculocutaneous nerves, respectively, produce supination of the forearm. This is a question of two muscles that can supinate the forearm.

25. A 31-year-old patient complains of sensory loss over the anterior and posterior surfaces of the medial third of the hand and the medial one and one-half fi ngers. He is diagnosed by a physician as having "funny bone" symptoms. Which of the following nerves is injured? (A) Axillary (B) Radial (C) Median (D) Ulnar (E) Musculocutaneous

The answer is D. The ulnar nerve supplies sensory fi bers to the skin over the palmar and dorsal surfaces of the medial third of the hand and the medial one and one-half fingers. The median nerve innervates the skin of the lateral side of the palm; the palmar side of the lateral three and one-half fi ngers; and the dorsal side of the index fi nger, the middle fi nger, and one-half of the ring fi nger. The radial nerve innervates the skin of the radial side of the hand and the radial two and one-half digits over the proximal phalanx.

A 15-year-old girl delivering her first child at home with only her mother's help experiences a difficult birthing. As the baby's head passes through the birth canal, the perineum begins to tear. Which of the following muscles is the most likely to tear in this event? (A) Coccygeus (B) Ischiocavernosus (C) Obturator internus (D) Pubococcygeus (E) External anal sphincter

The answer is D: Pubococcygeus. The pubococcygeus, the major component of the levator ani, is the muscle most often torn during childbirth. The pelvic diaphragm supports the fetal head during delivery. The pubococcygeus plays a key mechanical role in that it encircles the urethra, vagina, and anal canal and provides the main support for these organs during birthing. However, because of its proximity to the vagina, it also is subject to tearing if the perineum ruptures. Subsequent weakening of the muscle and associated pelvic fascia may cause changes in the positioning of the bladder and urethra that may lead to urinary stress incontinence. Choice A (Coccygeus) is incorrect. The coccygeus is the most posterior component of the pelvic diaphragm. However, it is not part of the levator ani portion of the pelvic diaphragm. Because of its relative distance from the birth canal, the coccygeus is less subject to the stresses applied to the pubococcygeus during delivery, therefore less subject to tearing. Choice B (Ischio- cavernosus) is incorrect. The ischiocavernosus is the thin skeletal muscle layer surrounding the crus of the clitoris. The crura are attached to the ischiopubic rami, well lateral to the birth canal. Thus, the levator ani will normally tear before that opening reaches the crus of the clitoris and the ischiocaverno- sus muscle, which envelops it. Choice C (Obturator internus) is incorrect. The obturator internus is applied to the internal lateral wall of the true pelvis and sends its tendon of insertion out of the lesser sciatic foramen into the gluteal region. This muscle is far removed from the birth canal, and it is highly unlikely to be damaged during childbirth. Choice E (Exter- nal anal sphincter) is incorrect. The external anal sphincter surrounds the lower anal canal. It is not part of the pelvic diaphragm. Because of its proximity to the vagina, it may be damaged during extensive tearing of the perineum. However, the pubococcygeus normally tears before the external anal sphincter becomes involved.

A 58-year-old postmenopausal mother (of four children) goes to her gynecologist complaining of discomfort and problems during micturition (emptying of the bladder). Physical examination reveals a herniation in the anterior aspect of the vaginal orifice, as illustrated below. What is the most likely diagnosis? (A) Urinary tract infection (B) Hydrocele (C) Urethrocele (D) Cystocele (E) Rectocele

The answer is D: Cystocele. A cystocele is a herniation of the urinary bladder into the anterior aspect of the vagina through a tear in the pubocervical fascia, which separates the bladder from the vagina. This fascia is often torn during childbirth (parturition), creating a passage for the hernia- tion of the bladder. A cystocele often causes discomfort or problems during micturition due to the change in the nor- mal position of the bladder. Because estrogen strengthens the elastic tissues around the vagina, a cystocele may not occur until menopause, when levels of estrogen decrease. Therefore, a cystocele is the most likely diagnosis due to the patient being a postmenopausal mother (of four children) and the location of the herniated bladder in the anterior aspect of the vaginal orifice. Choice A (Urinary tract infec- tion) is incorrect. Patients with urinary tract infections often present with dysuria, pain during voiding of urine, which is reported in this patient. However, this diagnosis does not account for the herniation in the vaginal orifice. Choice B (Hydrocele) is incorrect. A hydrocele testis is a pathological accumulation of serous fluids within the spermatic cord of the testis due to serous secretions from a remnant piece of peritoneum, termed the tunica vaginalis. This diagnosis can be easily eliminated in this female patient. Choice C (Ureth- rocele) is incorrect. A urethrocele is a prolapse of the urethra into the vestibule of the vagina, in which the inner lining of the urethra sticks out through the external urethral orifice. A urethrocele may occur with a cystocele, but in these cases, a distinct groove is usually seen between the border of the prolapsed urethra and bladder, not seen in this illustration. Interestingly, a urethrocele may occur in prepubertal girls and presents as a small pink donut-shaped mass (with the external urethra orifice making the hole of the donut) in the vestibule of the vagina. This condition is often detected by a parent who finds a small amount of blood in the child's underwear, and it can be treated with an estrogen cream to strengthen the elastic fibers around the urethra. Anatomical knowledge of the location of the female urethra being ante- rior to the vagina rules out this diagnosis because the hernia- tion was located in the anterior aspect of the vaginal orifice. Choice E (Rectocele) is incorrect. A rectocele is a herniation of rectal tissue through a tear in the rectovaginal septum, which separates the rectum from the vagina. It is detected in the posterior aspect of the vaginal orifice. A rectocele may be present in women following childbirth or hysterectomy, but this diagnosis can be ruled out by the location of the hernia- tion in the anterior aspect of the vaginal orifice.

A 25-year-old man is admitted to the emergency room after a fall, complaining of lower back pain and paresthesia into the lower extremities. The CT reveals a L2 burst fracture with a fracture fragment that is displaced posterior into the vertebral canal. A comprehensive neurologic examination indicates destruction of the sacral segments of the spinal cord. Which of the following functional outcomes is expected? (A) Lowered sperm count (B) Paralysis of the arrector pili muscles in the lower limbs (C) Reduced sweat gland secretion in the lower limbs (D) Inability to achieve erection (E) Increased motility in the descending colon and rectum

The answer is D: Inability to achieve erection. Destruction of the sacral segments of the spinal cord by a posterior frac- ture fragment of L2 would lead to pain within the lower back and paresthesia in the lower limbs. It would also eliminate the sacral parasympathetic outflow due to damage to the pre- synaptic parasympathetic cell bodies, which are located in the grey matter of spinal cord segments S2-4. The sacral parasym- pathetic outflow affects the gut tube below the left (splenic) flexure of the colon, the pelvic viscera, and the external geni- talia. Parasympathetic stimulation produces genital erection, whereas sympathetic stimulation governs ejaculation. This pattern of innervation can be remembered by the mnemonic "Point" and "Shoot." Thus, this injury will result in inability to achieve erection. However, ejaculation can still occur, and such patients can impregnate a woman. Choice A (Lowered sperm count) is incorrect. Sperm production is not governed by the autonomic nervous system. This patient will continue to produce sperm and, as noted above, can still ejaculate and father a child. Choice B (Paralysis of the arrector pili muscles in the lower limbs) is incorrect. Parasympathetic fibers distrib- ute to the body cavities and the external genitalia but not to the body wall and limbs. The innervation of arrector pili mus- cles in the lower limbs is governed by the sympathetic nervous system, which was not damaged in this patient. Choice C (Reduced sweat gland secretion in the lower limbs) is incor- rect. Parasympathetic fibers distribute to the body cavities and the external genitalia but not to the body wall and limbs. The innervation of sweat glands in the lower limbs is governed by the sympathetic nervous system, which was not damaged in this patient. Choice E (Increased motility in the descend- ing colon and rectum) is incorrect. The sacral parasympathetic outflow does govern the descending colon and rectum, with one normal effect being increased motility (peristalsis) in these regions. Therefore, loss of parasympathetic supply, combined with unopposed sympathetic input, would result in decreased motility here.

A 67-year-old man falls off his roof while cleaning his gutters. The impact of the landing causes a dislocation of his right hip joint and fractures his right superior and inferior pubic rami. Which of the following structures is most likely damaged in company with this extensive trauma to the right lateral wall of the true pelvis? (A) Pubic symphysis (B) Piriformis muscle (C) Sacral promontory (D) Obturator internus muscle (E) Ganglion impar

The answer is D: Obturator internus muscle. The lateral wall of the true (lesser) pelvis is composed of the iliopubic ramus, ischiopubic ramus, body of the ischium, and the obturator foramen (mostly closed by the obturator membrane). The obturator internus muscle lies on the lateral wall of the true pelvis, surrounding and covering the obturator foramen. Frac- tures of the pubic and obturator areas are relatively common. In the case of a lateral wall fracture, the obturator internus muscle can easily be torn in association with disruption of the bony elements and/or the obturator membrane. Choice A (Pubic symphysis) is incorrect. Many students believe that the pubic symphysis forms the anterior border of the pelvis. However, the pelvis is tilted in its normal anatomical posi- tion such that the anterior superior iliac spines and the pubic tubercles are aligned in the same vertical plane. This orien- tation causes the bodies of the pubic bones and the sym- physis to actually occupy the floor of the true pelvis, where they take on a weight-bearing function. Choice B (Piriformis muscle) is incorrect. The piriformis muscle originates from the pelvic surface of the superior part of the sacrum, lateral to the anterior sacral foramina. It exits the pelvis through the greater sciatic foramen to reach the greater trochanter of the femur. Because the piriformis fills a large extent of the greater sciatic foramen, it forms the posterolateral wall of the true pelvis. Choice C (Sacral promontory) is incorrect. The sacral promontory is the anterosuperior lip of the S1 element of the sacrum. It forms the posterior midline part of the bony edge that defines the pelvic inlet (pelvic brim; superior pel- vic aperture) and separates the false (greater) pelvis from the true (lesser) pelvis. Due to the orientation of the pelvis in the anatomical position, the sacrum and coccyx form the roof of the posterior wall of the true pelvis. Choice E (Ganglion impar) is incorrect. The ganglion impar is the single, midline coccygeal ganglion that forms the inferior end of the sympa- thetic chain. It lies on the pelvic surface of the sacrum. Thus, it may be damaged in fractures of the posterior wall of the true pelvis.

As part of a complete physical examination, an obstetrician conducts a digital vaginal examination of her 25-year-old patient. Which of the following structures is normally palpable through the posterior fornix? (A) Urethra (B) Ovaries (C) Ischial spines (D) Perineal body (E) Ureters

The answer is D: Perineal body. The vaginal fornix is the recess that surrounds the cervix. It has anterior, lateral, and posterior parts. Because of the postural angles between the vagina, cer- vix, and uterine body, the posterior fornix is the deepest part, affording a wide palpation area plus direct access to the rec- touterine pouch. The perineal body lies in the midline directly posterior to the vagina and can be readily palpated. Addition- ally, the rectum, anal canal, sacrum, and sacral promontory are available to palpation. Choice A (Urethra) is incorrect. The urethra and the base of the bladder lie against the anterior wall and fornix of the vagina. They can be palpated in that direction. Choice B (Ovaries) is incorrect. The ovaries, ischial spines, ureters, uterine tubes, and the uterine arteries can be palpated through the lateral fornices of the vagina. Choice C (Ischial spines) is incorrect. The ischial spines, ovaries, ure- ters, suterine tubes, and the uterine arteries can be palpated through the lateral fornices of the vagina. Choice E (Ureters) is incorrect. The ureters, ovaries, ischial spines, uterine tubes, and the uterine arteries can be palpated through the lateral fornices of the vagina.

A male infant is born with undescended testes that remain intra-abdominal 5 months later (cryptorchidism). Each testis possesses an elongate, distinctive gubernaculum testis running into the scrotum. Which of the following structures in female anatomy is homologous to the gubernaculum testis? (A) Uterine tube (B) Ureter (C) Labiumminus (D) Round ligament of the uterus (E) Vestibular bulb

The answer is D: Round ligament of the uterus. Both the round ligament of the uterus and the ovarian ligament are the homologues of the gubernaculum testis. In males, the testes descend along a path formed by the caudal genital ligament and gubernaculum testis. In females, the same structures guide the ovaries in their descent. However, the upward growth of the reproductive tract and different hormonal influences disrupt full ovarian descent. When the ovaries settle into their final position in the lateral aspect of the true pelvis, the remnants of the caudal genital ligament and gubernaculum are retained as the ovarian ligament and round ligament of the uterus. The round ligament extends through the inguinal canal and into the labium majus, marking what would be the full descent pathway in males. Choice A (Uterine tube) is incorrect. The uterine tubes (and uterus, cervix, and superior third of the vagina) are derived from the embryonic paramesonephric ducts. In males, the paramesonephric ducts are suppressed, and the only remnant is the vestigial appendix of the testis. Choice B (Ureter) is incorrect. The ureter is formed from the ureteric bud, an outgrowth of the mesonephric duct. Over- all, the ureteric bud gives rise to the renal collecting ducts, including the ureter, renal pelvis, renal calyces, and collect- ing tubules. Choice C (Labium minus) is incorrect. The labia minora are formed from the embryonic urethral (urogenital) folds. In males, these folds fuse in the midline to form the ventral (anterior) aspect of the penis enclosing the penile ure- thra. In females, the folds do not fuse, forming the separate labia minora. Choice E (Vestibular bulb) is incorrect. The vestibular bulbs are formed from the more posterior part of the embryonic phallus, which is continuous with the ure- thral folds. In males, this area fuses in the midline to form the corpus spongiosum and bulb of the penis. In females, the structures remain separate and form the bulbs of the vestibule, underlying the labia minora

10. A 35-year-old man walks in with a stab wound to the most medial side of the proximal portion of the cubital fossa. Which of the following structures would most likely be damaged? (A) Biceps brachii tendon (B) Radial nerve (C) Brachial artery (D) Radial recurrent artery (E) Median nerve

The answer is E. The contents of the cubital fossa from medial to lateral side are the median nerve, the brachial artery, the biceps brachii tendon, and the radial nerve. Thus, the median nerve is damaged. The radial recurrent artery ascends medial to the radial nerve.

4. A 16-year-old patient has weakness fl exing the metacarpophalangeal joint of the ring fi nger and is unable to adduct the same fi nger. Which of the following muscles is most likely paralyzed? (A) Flexor digitorum profundus (B) Extensor digitorum (C) Lumbrical (D) Dorsal interosseous (E) Palmar interosseous

The answer is E. The dorsal and palmar interosseous and lumbrical muscles can flex the metacarpophalangeal joints and extend the interphalangeal joints. The palmar interosseous muscles adduct the fingers, while the dorsal interosseous muscles abduct the fingers. The flexor digitorum profundus flexes the distal interphalangeal (DIP) joints.

45. The lateral portion of the fractured clavicle is displaced downward by which of the following? (A) Deltoid and trapezius muscles (B) Pectoralis major and deltoid muscles (C) Pectoralis minor muscle and gravity (D) Trapezius and pectoralis minor muscles (E) Deltoid muscle and gravity

The answer is E. The lateral fragment of the clavicle is displaced downward by the pull of the deltoid muscle and gravity. The medial fragment is displaced upward by the pull of the sternocleidomastoid muscle. None of the other muscles are involved.

37. A 20-year-old man fell from the parallel bar during the Olympic trial. A neurologic examination reveals that he has a lesion of the lateral cord of the brachial plexus. Which of the following muscles is most likely weakened by this injury? (A) Subscapularis (B) Teres major (C) Latissimus dorsi (D) Teres minor (E) Pectoralis major

The answer is E. The pectoralis major is innervated by the lateral and medial pectoral nerves originating from the lateral and medial cords of the brachial plexus, respectively. The subscapularis, teres major, latissimus dorsi, and teres minor muscles are innervated by nerves originating from the posterior cord of the brachial plexus.

42. A construction worker suffers a destructive injury of the structures related to the anatomic snuffbox. Which of the following structures would most likely be damaged? (A) Triquetral bone (B) Trapezoid bone (C) Extensor indicis tendon (D) Abductor pollicis brevis tendon (E) Radial artery

The answer is E. The radial artery lies on the fl oor of the anatomic snuffbox. Other structures are not related to the snuffbox. The tendons of the extensor pollicis longus, extensor pollicis brevis, and abductor pollicis longus muscles form the boundaries of the anatomic snuffbox. The scaphoid and trapezium bones form its fl oor.

A 72-year-old woman, who lost 50 lb (22.7 kg) of weight in the past year, visited her physician complaining of abdominal cramps, pain, and vomiting. Physical examination detected a distended abdomen with no visible or palpable masses in the groin and upper thigh area. She also described a sharp cutting pain in the medial aspect of her left thigh, which was exacerbated by the physician extending and medially rotating her left thigh at the hip joint. An abdominal CT revealed small bowel obstruction. What is the most likely cause of the obstruction? (A) Direct inguinal hernia (B) Indirect inguinal hernia (C) Umbilical hernia (D) Femoral hernia (E) Obturator hernia

The answer is E: Obturator hernia. An obturator hernia can present with small bowel obstruction and lancinating (sharp cutting) pain in the medial thigh region. This type of hernia is most often seen in elderly females (over the age of 70), espe- cially women who have recently lost a large amount of weight. It occurs when a portion of the small bowel is entrapped within the confines of the obturator canal, which compresses the obturator nerve and leads to sharp cutting pain within its distribution pattern. When a physician is able to exacer- bate this pain by extension, medial rotation, and adduction of the thigh at the hip joint, it is called a positive Howship- Romberg sign, which is further suggestive of an obturator hernia. Remember that an obturator hernia does not present with visible lumps because the herniation is trapped under the muscles of the medial compartment of the thigh. Choice A (Direct inguinal hernia) is incorrect. A direct inguinal hernia is an acquired hernia that results when abdominal cavity con- tents herniate through a weakness in the anterior abdominal wall in the inguinal (Hesselbach) triangle. The hernia traverses the superficial inguinal ring and often presents as a visible or palpable lump underneath the skin overlying the superficial inguinal ring, which was not present in this patient. More- over, direct inguinal hernias are rare in women; in fact, some references cite groin hernias are 25 times more likely to occur in men than women. Choice B (Indirect inguinal hernia) is incorrect. An indirect inguinal hernia is usually a congenital hernia that results when abdominal cavity contents herniate through a patent processus vaginalis, or in an adult, the ingui- nal canal. The hernia exits through the deep and superficial inguinal rings and descends into the labium majus of females or the scrotum of males. An indirect inguinal hernia often pres- ents as a visible or palpable lump underneath the skin in the groin region, and this finding was not reported in this patient. Choice C (Umbilical hernia) is incorrect. Umbilical hernias are protrusions in the umbilical region that can be congenital (seen in newborns) or acquired (occurring later in life). These hernias present as a visible or palpable lump underneath the skin in the umbilical region, and this finding was not reported in this patient. Choice D (Femoral hernia) is incorrect. A femoral hernia is a protrusion through the femoral ring that extends into the femoral canal and compresses its contents (lymphatic vessels and connective tissue). Initially the hernia is contained in the femoral canal; however, it may extend infe- riorly to the saphenous hiatus where it creates a bulge within the femoral triangle. The femoral ring is a weakened aspect in the anterior abdominal wall through which femoral hernias enter the femoral canal. These hernias are more common in females due to their wider pelves and often contain abdominal viscera. Moreover, femoral hernias are more susceptible to strangulation, wherein the blood supply to the herniated vis- cera can be interrupted due to the sharp boundaries of the femoral ring, particularly the lacunar ligament. Strangulation can lead to ischemia, sharp pain, and necrosis of the impinged tissue. However, a patient with a femoral hernia would not present with sharp cutting pain in the medial aspect of the thigh or a positive Howship-Romberg sign (exacerbation of this pain after extending and medially rotating the thigh at the hip joint).

A pregnant woman in active labor receives an epidural anesthetic to relieve pain from her uterine contractions and cervical dilation in preparation for the birth of her child. Visceral sensory fibers project the pain derived from the dilation of the uterine cervix to what spinal cord level? (A) T4 (B) L1 (C) L4 (D) S1 (E) S3

The answer is E: S3. Visceral sensory fibers relaying pain from the lower part of the uterus (cervix) and upper part of the vagina are referred to the S2-4 spinal cord levels, as these organs are located below the pelvic pain line, which generally corresponds to the lower limit of the peritoneum. Because visceral sensory fibers follow the visceral motor (autonomic) fibers retrograde, visceral pain from the uterine cervix will follow the presynaptic parasympathetic nerves of the pelvic splanchnic nerves retrograde to spinal cord levels S2-4. Conversely, pelvic and lower abdominal organs located above the pelvic pain line are, by definition, in contact with or located above the peritoneum, and their visceral sensory fibers follow the visceral motor fibers retrograde to refer pain to the lowest limits of the sympathetic (thoracolumbar) nervous system located at spinal cord levels T11-L2. Therefore, labor pains derived from the dilation of the uterine cervix will be referred by visceral sensory fibers to the S3 spinal cord level. Choice A (T4) is incorrect. Visceral sensory fibers associated with the T4 spinal cord level would be involved in convey- ing visceral pain associated with the heart and lungs, due to traveling retrograde with cardiopulmonary splanchnic nerves derived from postsynaptic sympathetic fibers from T1 to T6. Choice B (L1) is incorrect. The uterine cervix is located below the peritoneum, which implies it is below the pelvic pain line. Because visceral sensory fibers follow the visceral motor (autonomic) fibers retrograde, visceral pain from the uterine cervix will follow the presynaptic parasympathetic nerves of the pelvic splanchnic nerves retrograde to the spinal cord levels S2-4. However, pain derived from the labor contractions of the upper uterus, which is in contact with the peritoneum and thus located above the pelvic pain line, travel via visceral sensory fibers that follow the visceral motor fibers retrograde to refer pain to the lowest limits of the sympathetic (thoracolumbar) nervous system located at spinal cord levels T11-L2. Though pain derived from the labor contractions of the upper uterus refer to the L1 vertebral level, this question asked specifically about labor pains derived from the dilation of the uterine cervix, which would be referred to the S2-4 vertebral levels. Choice C (L4) is incorrect. Visceral sensory fibers carrying pain from visceral organs and involuntary (smooth or cardiac) muscle follow retrograde to the pathway of the visceral motor (autonomic) nerve fibers. There are no pre- synaptic autonomic nerve fibers that arise from L4, so there are no visceral sensory nerves associated with this spinal cord level. Choice D (S1) is incorrect. Visceral sensory fibers carrying pain from visceral organs and involuntary (smooth or cardiac) muscle follow retrograde to the pathway of the visceral motor (autonomic) nerve fibers. Because there are no presynaptic autonomic nerve fibers that arise from S1, there are no visceral sensory nerves associated with this spinal cord level.

A genetic coding error during the 5th week of development causes a unilateral failure of development of the ureteric bud. This condition would directly affect the formation of which of the following structures? (A) Proximal convoluted tubules (B) Renal glomeruli (C) Urinary bladder (D) Uterine tube (E) Ureter

The answer is E: Ureter. The permanent kidneys develop from two sources: (1) the ureteric bud and (2) the meta- nephric mesoderm. The ureteric bud is an outgrowth of the mesonephric duct near its junction with the cloaca. It forms the renal collecting ducts, including the ureter, renal pelvis, renal calyces, and collecting tubules. The ureteric bud invades the metanephric mesoderm, inducing it to form a meta- nephric cap over its distal end. The cap differentiates into the kidney parenchyma and the nephrons (i.e., the excretory and functional units in the kidney). Choice A (Proximal convo- luted tubules) is incorrect. The metanephric mesoderm forms the nephrons in the kidney, including Bowman capsule, the proximal convoluted tubule, loop of Henle, and distal con- voluted tubule. Nephrons are formed throughout prenatal development. Choice B (Renal glomeruli) is incorrect. Renal glomeruli are small capillary bundles that form in association with Bowman capsule. They are not derived from either the ureteric bud or the metanephric mesoderm. Choice C (Uri- nary bladder) is incorrect. The urinary bladder is derived from the cloaca. The cloaca divides into a urogenital sinus and an anal canal, which are located anterior and posterior, respec- tively. The urogenital sinus subsequently differentiates into the urinary bladder and urethra. Choice D (Uterine tube) is incor- rect. The uterine tube is formed from the paramesonephric duct. More caudally, the paired paramesonephric ducts join together to form the uterus, cervix, and cranial third of the vagina.


Set pelajaran terkait

Pharm ATI CANCER & IMMUNOSUPPRESANT Medications

View Set

Chapter 25 Submersion Incidents: Drowning & Diving Emergencies

View Set

Test 3: Chapter 7-9 Contemporary Human Geography

View Set

Finance Test 2 Practice Questions

View Set

FInal Exam 3, Econ 1040 final exam Ohio University

View Set

Marketing 409 (Services Marketing) Exam 2

View Set